You are on page 1of 459

1.

APPLICATIONS OF MATRICES
AND DETERMINANTS
1.1. Introduction :
The students are already familiar with the basic definitions, the elementary
operations and some basic properties of matrices. The concept of division is not
defined for matrices. In its place and to serve similar purposes, the notion of the
inverse of a matrix is introduced. In this section, we are going to study about the
inverse of a matrix. To define the inverse of a matrix, we need the concept of
adjoint of a matrix.

1.2 Adjoint :
Let A = [aij] be a square matrix of order n. Let Aij be the cofactor of aij.
Then the nth order matrix [Aij]T is called the adjoint of A. It is denoted by adjA.
Thus the adjA is nothing but the transpose of the cofactor matrix [Aij] of A.
Result : If A is a square matrix of order n, then A (adjA) = | A | In = (adj A) A,
where In is the identity matrix of order n.
Proof : Let us prove this result for a square matrix A of order 3.
a11 a12 a13

a31
A
11
Then adj A = A12
A13

a33
A31

A32
A33

Let A = a21 a22 a23


a32
A21
A22
A23

The (i, j)th


= ai1 Aj1 + ai2 Aj2 + ai3 Aj3 = = | A | if i = j
element of A (adj A)
= 0 if i j
0
| A | 0
1 0 0

A (adj A) = 0 | A | 0 = | A | 0 1 0 = | A | I3

0
0 0 1
0 | A |
Similarly we can prove that (adj A)A = | A | I3

A (adj A) = | A | I3 = (adj A) A
In general we can prove that A (adj A) = | A | In = (adj A) A.

a b
Example 1.1 : Find the adjoint of the matrix A =

c d
Solution: The cofactor of a is d, the cofactor of b is c, the cofactor of c is b
and the cofactor of d is a. The matrix formed by the cofactors taken in order is
the cofactor matrix of A.
d c
The cofactor matrix of A is =
.
b a
Taking transpose of the cofactor matrix, we get the adjoint of A.
d b
The adjoint of A =

c a
1 1
1

Example 1.2 : Find the adjoint of the matrix A = 1

2
1

Solution: The cofactors are given by

2 3
=3
1 3
1 3

= 9
2 3
1 2

=5
2 1
1 1

=4
1 3
1 1

=1
2 3
1 1

=3
2 1
1 1

=5
2 3

Cofactor of 1 = A11 =
Cofactor of 1 = A12 =
Cofactor of 1 = A13 =
Cofactor of 1 = A21 =
Cofactor of 2 = A22 =
Cofactor of 3 = A23 =
Cofactor of 2 = A31 =

1 1
=4
1 3
1 1
=
=1
1 2

Cofactor of 1 = A32 =
Cofactor of 3 = A33

3
The Cofactor matrix of A is [Aij] = 4
5
3
T
adj A = (Aij) = 9
5

9 5
1
4
4
1

1
5

4
1
3

1
2

Example 1.3 : If A =
, verify the result A (adj A) = (adj A) A = | A | I2
1 4
1 2
1 2
A=
Solution:
, | A | =
=2
1 4
1 4
4 2
adj A =

1 1
1 2 4 2 2 0
1 0
(1)
A (adj A) =

=
=2
= 2I2
0 1
1 4 1 1 0 2
4 2 1 2 2 0
1 0
(adj A) A =
(2)

=
= 2
= 2I2
0 1
1 1 1 4 0 2
From (1) and (2) we get
A (adj A) = (adj A) A = | A | I2.

Example 1.4 : If A = 1
2
1

3
1

3 , verify A (adj A) = (adj A) A = | A | I3

2
1

Solution: In example 1.2, we have found


3 4 5

adj A = 9

1
3

1
4


| A | = 1
2
1

A (adj A) = 1
2
1

= 1(6 3) 1 (3 + 6) + 1( 1 4) = 11
3
1
11 0 0
3 4 5
3 9 1
4
= 0 11 0
0 0 11
3 5 3
1
1

2
1
1
2

1
1 0 0

= 11 0 1 0 = 11 I3 = | A | I3

0 0 1
1 1
1
3 4 5

(adj A) A = 9
5

4 1
1 2

3
1 0 0

2
1

(1)

11
3 = 0
3 0

0
11
0

= 11 0 1 0 = 11 I3 = | A | I3

0 0 1

11
0

(2)

From (1) and (2) we get


A(adj A) = (adj A) A = | A | I3

1.3 Inverse :
Let A be a square matrix of order n. Then a matrix B, if it exists, such that
AB = BA = In is called inverse of the matrix A. In this case, we say that A is an
invertible matrix. If a matrix A possesses an inverse, then it must be unique. To
see this, assume that B and C are two inverses of A, then
AB = BA = In
(1)
(2)

AC = CA = In
Now AB = In

C(AB) = CIn (CA)B = C

(Q associative property)

InB = C B = C

i.e., The inverse of a matrix is unique. Next, let us find a formula for
computing the inverse of a matrix.
We have already seen that, if A is a square matrix of order n, then
A(adj A) = (adj A)A = | A | In

If we assume that A is non-singular, then | A | 0.


Dividing the above equation by | A |, we get
1

A | A | (adj A) = | A | (adj A) A = In.

From this equation it is clear that the inverse of A is nothing but


1
1
| A | (adj A). We denote this by A .
Thus we have the following formula for computing the inverse of a matrix
through its adjoint.
If A is a non-singular matrix, there exists an inverse which is given by
1
A1 = | A | (adj A).

1.3.1 Properties :
1. Reversal Law for Inverses :
If A, B are any two non-singular matrices of the same order, then AB is also
non-singular and
(AB)1 = B1 A1
i.e., the inverse of a product is the product of the inverses taken in the
reverse order.
Proof : Since A and B are non-singular, | A | 0 and | B | 0.
We know that | AB | = | A | | B |
| A | 0, | B | 0 | A | | B | 0 | AB | 0
Hence AB is also non-singular. So AB is invertible.
(AB) (B1A1) = A (BB1)A1
= AIA1 = AA1 = I
Similarly we can show that (B1A1) (AB) = I
(AB) (B1A1) = (B1A1) (AB) = I
B1 A1is the inverse of AB.
(AB)1 = B1 A1
2. Reversal Law for Transposes (without proof) :
If A and B are matrices conformable to multiplication, then (AB)T = BTAT.

i.e., the transpose of the product is the product of the transposes taken in
the reverse order.
1

3. For any non-singular matrix A, (AT)

= (A 1)

Proof : We know that AA1 = I = A1 A


T

Taking transpose on both sides of AA1 = I, we have (AA1) = IT


By reversal law for transposes we get
T

(A1) AT = I

(1)
1

Similarly, by taking transposes on both sides of A A = I, we have


T

AT(A1) = I

(2)

From (1) & (2)


T
T
(A1) AT = AT (A1) = I
T

(A1) is the inverse of AT


i.e.,

(AT)

= (A 1)

1.3.2 Computation of Inverses


The following examples illustrate the method of computing the inverses of
the given matrices.
Example 1.5 : Find the inverses of the following matrices :
3 1 1
1 2
2 1
cos sin
(i)
(ii)
(iii)
(iv) 2 2 0
1 4
4 2
sin cos
1 2 1

Solution:

1 2
1 2
(i) Let A =
, Then | A | =
=20
1 4
1 4
A is a non-singular matrix. Hence it is invertible. The matrix formed by the
cofactors is
4 1
[Aij] =

2 1
4 2
adj A = [Aij]T =

1 1

2 1
1
1 4 2
= | A | (adj A) = 2
= 1 1
1 1 2 2

2 1
2 1
(ii) Let A =
. then | A | =
=0
4 2
4 2
A is singular. Hence A1 does not exist.
cos sin
cos sin
(iii) Let A =
. Then | A | =

sin cos
sin cos
= cos2 + sin2 = 1 0
A is non singular and hence it is invertible
cos sin
Adj A =

sin cos
1
1 cos sin cos sin
A1 = | A | (Adj A) = 1
=

sin cos sin cos


3 1 1
3 1 1

(iv) Let A = 2 2
2

.
1
0

Then | A | =

2
1

2
2

=20
1
0

A is non-singular and hence A 1 exists


2 0
Cofactor of 3 = A11 =
=2
2 1
2 0
Cofactor of 1 = A12 =
= 2
1 1
2 2
Cofactor of 1 = A13 =
=6
1 2
1 1
Cofactor of 2 = A21 =
=1
2 1
3 1
Cofactor of 2 = A22 =
= 2
1 1
3 1
Cofactor of 0 = A23 =
= 5
1 2

1 1
=2
2 0
3 1
=
=2
2 0
3 1
=
=8
2 2

Cofactor of 1 = A31 =
Cofactor of 2 = A32
Cofactor of 1 = A33

[Aij] = 1
2
2

2 1 2

2 5 ; adj A = 2 2 2
6 5 8
2 8
2 1 2

1
1
A1 = | A | (adj A) = 2 2 2 2
6 5 8
2

= 1
3

1
1 2 1
1
5
2

0 1
1 2
Example 1.6 : If A =
verify that (AB)1 = B1 A1.
and B =
1 1
1 2
Solution:
| A | = 1 0 and | B | = 1 0
So A and B are invertible.

1 2 0 1 2 3
=

1 1 1 2 1 1
2 3
| AB | =
= 1 0. So AB is invertible.
1 1
1 2
adj A =

1 1
1 2
1
A1 = | A | (adj A) =

1 1
AB =

2 1

1 0

adj B =

1
2 1
B1 = | B | (adj B) =

1 0
1 3
adj AB =

1 2
1 3
1
(AB)1 = | AB | (adj AB) =

1 2

(1)

2 1 1 2 1 3
=


1 0 1 1 1 2

B1 A1 =

(2)

From (1) and (2) we have (AB)1 = B1 A1.


EXERCISE 1.1
(1) Find the adjoint of the following matrices :
1 2 3
2 5 3
3 1

(ii) 0 5 0
(i)
(iii) 3 1 2

2 4
2 4 3
1 2 1
1 2
(2) Find the adjoint of the matrix A =
and verify the result
3 5
A (adj A) = (adj A)A = | A | . I
3 3 4

(3) Find the adjoint of the matrix A = 2 3 4 and verify the result
1
A (adj A) = (adj A)A = | A | . I
(4) Find the inverse of each of the following matrices :
1 0
3
1 3 7

(i) 2 1 1
1 1 1
8 1 3
2
(iv) 5 1
10 1 4

(ii) 4 2 3

1 2 1

2 2 1
(v) 1 3 1

1 2 2

1
(iii) 1
0

2 1
5 2
verify that
and B =
7 3
1 1

(5) If A =

(i) (AB)1 = B1 A1
(6)

(7)

(ii) (AB)T = BTAT

3 3 4
Find the inverse of the matrix A = 2 3 4 and verify that A3 = A 1
0 1 1
1 2 2 T
1 2 is 3A .
Show that the adjoint of A = 2
2 2 1

4 3 3
0
1 is A itself.
(8) Show that the adjoint of A = 1

4 4 3

2 1 2, prove that A1 = AT.


1 2 2
1 2 2
4 3 4 , show that A = A1
4 4 5
2

1
(9) If A = 3

(10) For A =

1.3.3 Solution of a system of linear equations by Matrix


Inversion method :
Consider a system of n linear non-homogeneous equations in n unknowns
x1, x2, x3 xn.
a11 x1 + a12 x2 + + a1n xn = b1
a21 x1 + a22 x2 + + a2n xn = b2

an1x1 + an2 x2 + + ann xn = bn

10

a11 a12 a1n

This is of the form

an

21

a22

an2

a2n

nn

Thus we get the matrix equation AX = B

x b
=

x b
x1

b1

(1) where

a11 a12 a1n

x1

b1

a22

a2n

nn

A=

21

n1

an2

x b

; X= ;B=


x b
a

If the coefficients matrix A is non-singular, then A1 exists. Pre-multiply


both sides of (1) by A1 we get
A1 (AX) = A1B
(A1A)X = A1B
IX = A1B
X = A1B is the solution of (1)
Thus to determine the solution vector X we must compute A1. Note that
this solution is unique.
Example 1.7 : Solve by matrix inversion method x + y = 3, 2x + 3y = 8
Solution:
The given system of equations can be written in the form of
1 1 x 3

=
2 3 y 8

Here

AX = B
1 1
|A| =
=10
2 3

11

Since A is non-singular, A1 exists.


3 1
A1 =

2 1
The solution is X = A1B

x 3 1 3

=
y 2 1 8
x 1
=
y 2
x = 1, y = 2
Example 1.8 : Solve by matrix inversion method 2x y + 3z = 9, x + y + z = 6,
xy+z=2
Solution : The matrix equation is
2 1 3 x

1
1

1 1
1 1
2
A X = B, where A = 1
1

9
y = 6

z 2
1 3

9
x

1 1, X = y and B = 6


z
2
1 1
2 1 3
| A | = 1 1 1 = 2 0
1 1 1

A is a non-singular matrix and hence A1 exists.


The cofactors are A11 = 2, A12 = 0, A13 = 2
A21 = 2, A22 = 1, A23 = 1, A31 = 4, A32 = + 1, A33 = 3
The matrix formed by the cofactors is
2
0

[Aij] = 2
4

12

1
1

3
1

2
The adjoint of A = 0
2

2 4

= adj A
3

1
1
Inverse of A = | A | (adj A)
2 2 4
1
1
0 1 1
A = 2
2 1
3

The solution is given by X = A1B


2 2 4 9
x
y = 1 0 1 1 6
2


z
2 1
3 2
2
1
1
4 = 2
= 2

3
6
x = 1, y = 2, z = 3
EXERCISE 1.2
Solve by matrix inversion method each of the following system of linear
equations :
(1) 2x y = 7,
3x 2y = 11
(2) 7x + 3y = 1,
2x + y = 0
(3) x + y + z = 9,
2x + 5y + 7z = 52, 2x + y z = 0
(4) 2x y + z = 7,
3x + y 5z = 13,
x+y+z=5
(5) x 3y 8z + 10 = 0, 3x + y = 4,
2x + 5y + 6z = 13

1.4 Rank of a Matrix :


With each matrix, we can associate a non-negative integer, called its rank.
The concept of rank plays an important role in solving a system of
homogeneous and non-homogeneous equations.
To define rank, we require the notions of submatrix and minor of a matrix.
A matrix obtained by leaving some rows and columns from the matrix A is
called a submatrix of A. In particular A itself is a submatrix of A, because it is
obtained from A by leaving no rows or columns. The determinant of any square
submatrix of the given matrix A is called a minor of A. If the square submatrix
is of order r, then the minor is also said to be of order r.
13

Definition :
The matrix A is said to be of rank r, if
(i) A has atleast one minor of order r which does not vanish.
(ii) Every minor of A of order (r + 1) and higher order vanishes.
In other words, the rank of a matrix is the order of any highest order non
vanishing minor of the matrix.
The rank of A is denoted by the symbol (A). The rank of a null matrix is
defined to be zero.
The rank of the unit matrix of order n is n. The rank of an m n matrix A
cannot exceed the minimum of m and n. i.e., (A) min {m, n}.
7 1
Example 1.9 : Find the rank of the matrix

2 1
7 1
Solution : Let A =
. This is a second order matrix.
2 1
The highest order of minor of A is also 2.
7 1
The minor is given by
=90
2 1
The highest order of non-vanishing minor of A is 2. Hence (A) = 2.
2 4
Example 1.10 : Find the rank of the matrix

1 2
2 4
Solution : Let A =
.
1 2
2 4
The highest order minor of A is given by
= 0. Since the second
1 2
order minor vanishes (A) 2. We have to try for atleast one non-zero first
order minor, i.e., atleast one non-zero element of A. This is possible because A
has non-zero elements (A) = 1.
1 2 3
Example 1.11 : Find the rank of the matrix

5
1

Solution : Let A = 2

2
4
1

1
3

14

2
5

4
1

1
6

The highest order minor of A is


1 2 3

2
5

1
6 = 2 1
5
1

4
1

2
2
1

=0
1
3
3

Since the third order minor vanishes, (A) 3


2 4

= 22 0
5 1
A has atleast one non-zero minor of order 2. (A) = 2
1 1 1 3
Example 1.12 : Find the rank of the matrix

Solution : Let A = 2
5
1

1 3

2
5

1 3
1 7

11
4

11
3

1 7
This is a matrix of order 3 4
A has minors of highest order 3. They are given by
1 1 1
1 1 3

2
5

= 0 ; 2 1 4 = 0 ;
5 1 11
7
1 1 3
3

4 = 0 ; 1 3 4 = 0

1 7 11
11

1 3

1
1
1

2 3

5 7

All the third order minors vanish. (A) 3


Next, we have to try for atleast one non-zero minor of order 2. This is
1 1
possible, because A has a 2nd order minor
= 3 0 (A) = 2
2 1
Note : In the above examples, we have seen that the determination of the rank
of a matrix involves the computation of determinants. The computation of
determinants may be greatly reduced by means of certain elementary
transformations of its rows and columns. These transformations will greatly
facilitate our dealings with the problem of the determination of the rank and
other allied problems.

15

1.4.1. Elementary transformations on a Matrix:


(i) Interchange of any two rows (or columns)
(ii) Multiplication of each element of a row (or column) by any non-zero
scalar.
(iii) Addition to the elements of any row (or column) the same scalar
multiples of corresponding elements of any other row (or column)
the above elementary transformations taken inorder can be represented by
means of symbols as follows :
(ii) Ri kRi (Ci kCi)
(i) Ri Rj (Ci Cj) ;
(ii) Ri Ri + kRj (Ci Ci + kCj)
Two matrices A and B of the same order are said to be equivalent if one
can be obtained from the other by the applications of a finite sequence of
elementary transformation The matrix A is equivalent to the matrix B is
symbolically denoted by A B.
Result (without proof) :
Equivalent matrices have the same rank
Echelon form of a matrix :
A matrix A (of order m n) is said to be in echelon form (triangular form) if
(i) Every row of A which has all its entries 0 occurs below every row
which has a non-zero entry.
(ii) The first non-zero entry in each non-zero row is 1.
(iii) The number of zeros before the first non-zero element in a row is less
than the number of such zeros in the next row.
By elementary operations one can easily bring the given matrix to the
echelon form.
Result (without proof) :
The rank of a matrix in echelon form is equal to the number of non-zero
rows of the matrix.
Note :
(1) The above result will not be affected even if condition (ii) given in the
echelon form is omitted. (i.e.) the result holds even if the non-zero
entry in each non-zero row is other than 1.
(2) The main advantage of echelon form is that the rank of the given
matrix can be found easily. In this method we dont have to compute
determinants. It is enough, if we find the number of non-zero rows.

16

In the following examples we illustrate the method of finding the rank of


matrices by reducing them to the echelon form.
1 1 1
Example 1.13 : Find the rank of the matrix
1

3
2

3
4

3 2 3
1 1 1
0 5 6
0 5 6
1 1 1
0 5 6
0 0 0
1

2
3

Solution : Let A = 2 3

R2 R2 2R1
R3 R3 3R1

R3 R3 R2

The last equivalent matrix is in echelon form. The number of non-zero


rows is 2. (A) = 2
1 2 3 1
Example 1.14 : Find the rank of the matrix

1
Solution : Let A = 2
3

2
3

4 6 2
6 9

2 3 1

4 6 2

6 9
1
2 3 1 R R 2R
2
1
0 0 0 0 2

R3 R3 3R1
0 0 0 0

This equivalent matrix is in the echelon form. Since the number of


non-zero rows of the matrix in this echelon form is 1, (A) = 1.
4 2 1 3
Example 1.15 : Find the rank of the matrix 6 3 4 7

2 1 0 1

17

4 2 1 3
Solution : Let A = 6 3 4 7

2 1 0 1
1 2 4 3
4 3 6 7 C C
3

1
0 1 2 1
4
3
1 2
0 5 10 5 R R 4R
2
1

2
0 1
2
1
1 2 4 3
0 1 2 1 R 1 R
5 2

2
0 1 2 1
1 2 4 3
0 1 2 1 R R R
3
2

3
0 0 0 0
The last equivalent matrix is in the echelon form.
The number of non-zero rows in this matrix is two. (A) = 2
3 1 5 1
Example 1.16 : Find the rank of the matrix
5 1

1
1
1
3
1

5 7
2 1

2
5

1
2

1 5
1 5

1 5 7 2
5 7
1 2 1 5

R2 R2 3R1
0 7 8 14 R3 R3 R1
0 7 8 7
1 2 1 5
0 7 8 14 R3 R3 R2
0 0 0 7
3

Solution : Let A = 1 2

The last equivalent matrix is in the echelon form.


It has three non-zero rows. (A) = 3

18

R1 R2

EXERCISE 1.3
Find the rank of the following matrices :
(1)

(4)

1
3
2
0

2
1

1
2
3
1

3
4
2

3 1 2 0
(3) 1 0 1 0

2 1 3 0

6 12 6
(2) 1 2 1

4 8 4

1
0
1

1
(5) 2
3

2 1
4

2
7
3

1
(6) 2
1

2
4
2

6
4

1 3
7

1.5 Consistency of a system of linear equations :


The system of linear equations arises naturally in many areas of Science,
Engineering, Economics and Commerce. The analysis of electronic circuits,
determination of the output of a chemical plant, finding the cost of chemical
reaction are some of the problems which depend on the solutions of
simultaneous linear equations. So, finding methods of solving such equations
acquire considerable importance. In this connection methods using matrices and
determinants play an important role.
We have already seen the idea of solving a system of linear equations by
the matrix inversion method. This method is applicable provided the number of
equations is equal to the number of unknowns, and the coefficient matrix is
non-singular. Also the solution obtained under this method is unique. But this is
not so in all cases. For many of the problems the number of equations need not
be equal to the number of unknowns. In such cases, we see that any one of the
following three possibilities can occur. The system has (1) unique solution (2)
more than one solution (3) no solution at all.
Cases (1) and (3) have no significant role to play in higher studies.
Although there exist many solutions, in some cases all the points in the solution
are not attractive. Some provide greater significance than others. We have to
select the best point among them. In this section we are going to discuss the
following two methods.
(1) Cramers rule method (or Determinant method)
(2) Rank method
These methods not only decide the existence of a solution but also help us
to find the solution (if it exists) of the given system.

19

1.5.1 The Geometry of Solution sets :


The solution set of a system of linear equations is the intersection of the
solution sets of the individual equations. That is, any solution of a system must
be a solution of each of the equations in that system.
The equation ax = b (a 0) has only one solution, namely x = b/a and it
represents a point on the line. Similarly, a single linear equation in two
unknowns has a line in the plane as its solution set and a single linear equation
in three unknowns has a plane in space as its solution set.
Illustration I : (No. of unknowns No. of equations)
Consider the solution of the following three different problems.
(i) 2x = 10
(ii) 2x + y = 10
(iii) 2x + y z = 10
Solution (i) 2x = 10 x = 5
Solution (ii) 2x + y = 10
S
We have to determine the values
of two unknown from a single
equation. To find the solution we can
Fig. 1.1
assign arbitrary value to x and solve
for y, or, choose an arbitrary value to y
and solve for x.
S
Suppose we assign x an arbitrary
value k, we obtain
x = k and y = (10 2k)
Fig. 1.2
These formulae give the solution set
Y
interms of the parameter k. Particular
numerical solution can be obtained by
S
substituting values for k.
For
1
example when k = 1, 2, 5, 3, 2 , we
O
X
get
(1, 8), (2, 6), (5, 0), ( 3, 16)
Z
1
and 2 , 9 as the respective solutions.

Fig. 1.3
Solution (iii) 2x + y z = 10
In this case, we have to determine three unknowns x, y and z from a single
equation. We can assign arbitrary values to any two variables and solve for the
third variable. We assign arbitrary values s and t to x and y respectively, and
solve for z.
We get x = s, y = t and z = 2s + t 10 is the solution set.
For different values of s and t we get different solutions.

20

1.5.2 Cramers Rule Method : (Determinant Method)


Gabriel Cramer (1704 1752), a Swiss mathematician wrote on
philosophy of law and government, and history of mathematics. He served in a
public office, participated in artillery and fortifications activity for the
government instructed workers on techniques of cathedral repair and undertook
excavations of cathedral archives. Cramer, a bachelor, received numerous
honours for his achievements.
His theorem provides a useful formula for the solution of certain linear
system of n equations in n unknowns. This formula, known as Cramers Rule, is
of marginal interest for computational purposes, but it is useful for studying the
mathematical properties of a solution without actually solving the system.
Theorem 1.1 (without proof) : Cramers Rule : If AX = B is a system of
n linear equations in n unknowns such that det(A) 0, then the system has a
unique solution. This solution is
det (A1)
det (A2)
det (An)
x1 = det A , x2 = det (A) , xn = det (A)
Where Aj is the matrix obtained by replacing the entries in the jth column
b1
of A by the entries in the matrix. B =

b
2

bn

Cramers Rule for Non homogeneous equations of 2 unknowns :


Let us start with the system of two linear equations in two unknowns
x and y.
(i)
a11x + a12y = b1
(ii)

a21x + a22y = b2
a11 a12
Let =

a21 a22

a11 a12
=
a21 a22

x. = x

a11x a12

a21x a22

b1 a12y

a12

b2 a22y

a22

(by equation (i) and (ii))

21

b1 a12
a12 a12
y
(by properties of determinants)
b2 a22
a22 a22

b1 a12
y . 0 (by properties of determinants)
b2 a22

b1 a12
= x (say)
b2 a22

x. =

Similarly

a11 b1
= y (say)
a21 b2

y. =

x, y are the determinants which can also be obtained by replacing 1st and
2nd column respectively by the column of constants containing b1 and b2 i.e. by
x
b1
b Thus, we have, x = x x =
2
y = y y =

y
provided 0

Since , x, y are unique, there exists a unique solution for the above
system of equations. i.e., the system is consistent and has a unique solution.
The method stated above to solve the system of equation is known as
Cramers Rule.
Cramers rule is applicable when 0.
If = 0, then the given system may be consistent or inconsistent.
Case 1 : If = 0 and x = 0, y = 0 and atleast one of the coefficients
a11, a12, a21, a22 is non-zero, then the system is consistent and has infinitely
many solutions.
Case 2 : If = 0 and atleast one of the values x, y is non-zero, then the
system is inconsistent i.e. it has no solution.
To illustrate the possibilities that can occur in solving systems of linear
equations with two unknowns, consider the following three examples. Solve :
(1) x + 2y = 3
x+y=2

(2)

x + 2y = 3
2x + 4y = 6

22

(3)

x + 2y = 3
2x + 4y = 8

Solution (1) :
We have

x
y

Unique solution

1 2
=
= 1
1 1
3 2
=
= 1
2 1
1 3
=
= 1
1 2

S
O

X
X+2Y = 3
X+Y = 2

Fig. 1.4
Since 0, the system has unique solution. By Cramers rule
x
y
x=
=1 ; y=
= 1
(x, y) = (1, 1)

Solution (2) :
Infinitely many solution
Y
1 2
We have =
= 0
2 4
3 2
x =
= 0
o
6 4
X
S
1 3
X + 2Y = 3
y =
= 0
2X + 4Y = 6
2 6
Fig. 1.5
Since = 0 and x = 0, y = 0 and atleast one of a11, a12, a21, a22 is non zero,
it has infinitely (case 1) many solutions. The above system is reduced to a
single equation x + 2y = 3. To solve this equation, assign y = k
x = 3 2y = 3 2k
The solution is x = 3 2k, y = k ; k R
For different value of k we get different solution. In particular (1, 1), ( 1, 2),
(5 1) and (8, 2.5) are some solutions for k = 1, 2, 1 and 2.5 respectively
Solution (3) :
No Solution
Y
1
2

=
= 0 ;
2 4
3 2
1 3
x =
= 4 ; y =
=2
8 4
2 8
o
X
Since = 0 and x 0, y 0
2X + 4Y = 8
X
+
2Y
=
3
(case 2 : atleast one of the value of
x, y, non-zero), the system is
Fig. 1.6
inconsistent.
i.e. it has no solution.

23

1.5.3 Non homogeneous equations of three unknowns :


Consider the system of linear equations
a11x + a12y + a13z = b1 ; a21x + a22y + a23z = b2 ; a31x + a32y + a33z = b3
Let us define , x, y and z as already defined for two unknowns.

11
= a21
a31
a

11
y = a21
a31
a

a12
a22
a32
b1
b2
b3

a23 ,
a33
a13

a23 ,
a33

1
x = b2
b3
b

a13

a22
a32

11
a21
a31
a

z =

a12

a33
a13

a23

a12
a22
a32

b3
b1

b2

As we discussed earlier for two variables, we give the following rule for
testing the consistency of the above system.
Case 1 : If 0, then the system is consistent, and has a unique solution. Using
Cramers Rule can solve this system.
Case 2 : If = 0, we have three important possibilities.
Subcase 2(a) : If = 0 and atleast one of the values of x, y and z is
non-zero, then the system has no solution i.e. Equations are inconsistent.
Subcase 2(b) : If = 0 and x = y = z = 0 and atleast one of the 2 2
minor of is non zero, then the system is consistent and has infinitely many
solution. In this case, the system of three equations is reduced to two equations.
It can be solved by taking two suitable equations and assigning an arbitrary
value to one of the three unknowns and then solve for the other two unknowns.
Subcase 2(c) : If = 0 and x = y = z = 0 and all their (2 2) minors
are zero but atleast one of the elements of is non zero (aij 0) then the system
is consistent and it has infinitely many solution. In this case, system is reduced
to a single equation. To solve we can assign arbitrary values to any two
variables and can determine the value of third variable.
Subcase 2(d) : If = 0, x = y = z = 0, all 2 2 minors of = 0 and
atleast one 2 2 minor of x or y or z is non zero then the system is
inconsistent.

24

Theorem 1.2 (without proof) :


If a non-homogeneous system of linear equations with more number of
unknowns than the number of equations is consistent, then it has infinitely
many solutions.
To illustrate the different possibilities when we solve the above type of
system of equations, consider the following examples.
(1) 2x + y + z = 5
(2) x + 2y + 3z = 6
x+y+z=4
x+y+z=3
x y + 2z = 1
2x + 3y + 4z = 9
(3) x + 2y + 3z = 6
(4) x + 2y + 3z = 6
2x + 4y + 6z = 12
x+y+z=3
3x + 6y + 9z = 18
2x + 3y + 4z = 10
(5) x + 2y + 3z = 6
2x + 4y + 6z = 12
3x + 6y + 9z = 24
Solution (1) :
2x + y + z = 5 ; x + y + z = 4 ; x y + 2z = 1
Unique solution
We have
2 1 1
= 1 1 1 = 3

1 1 2
5 1 1
x = 4 1 1 = 3

1 1 2
Fig. 1.7

2 1 5
2 5 1

z = 1 1 4 = 3
y = 1 4 1 = 6 ;

1 1 2
1 1 1
= 3, x = 3, y = 6, z = 3
0, The system has unique solution. By Cramers rule.
x 3
y 6
z
x = = 3 = 1, y =
= 3 = 2, z =
=1

The solution is x = 1, y = 2, z = 1
(x, y, z) = (1, 2, 1)

25

Solution (2) :
x + 2y + 3z = 6 ; x + y + z = 3 ; 2x + 3y + 4z = 9
1 2 3
6 2 3

x = 3 1 1 = 0
= 1 1 1 =0 ;

2 3 4
9 3 4
1 6 3
1 2 6

y = 1 3 1 = 0 ;
z = 1 1 3 = 0

2 9 4
2 3 9
Since = 0 and x = y = z = 0 but atleast one of the 2 2 minors of is
1 2 0, the system is consistent (by case 2(b)) and has
non-zero
1 1
infinitely many solution.
The system is reduced to 2 equations. Assigning an arbitrary value to
one of unknowns, say z = k, and taking first two equations.
We get x + 2y + 3k = 6
Infinitely many solution
x+y+k = 3
i.e.,
x + 2y = 6 3k
x+y = 3k

1 2
=1
=
1 1
Fig. 1.8
6

3k
2

x =
= 6 3k 6 + 2k = k
3 k 1
1 6 3k
y =
= 3 k 6 + 3k = 2k 3
1 3 k
x
k
x=
=
= k

1
y
2k 3
=
= 3 2k
y=

1
The solution is
x = k, y = 3 2k and z = k
i.e.
(x, y, z) = (k, 3 2k, k). k R
Particularly, for k = 1, 2, 3, 4 we get
(1, 1, 1), (2, 1, 2), (3, 3, 3), (4, 5, 4) respectively as solution.

26

Solution (3) :
x + 2y + 3z = 6 ; 2x + 4y + 6z = 12 ; 3x + 6y + 9z = 18
1 2 3
6 2 3

12 4 6 = 0
2
4
6
=

= 0 ; x =

3 6 9
18 6 9
1 6 3
1 2 6
y = 2 12 6 = 0 ; z = 2 4 12 = 0

3 18 9
3 6 18
Here = 0 and x = y = z = 0.
Also all their 2 2 minors are zero, but atleast one of aij of is non- zero.
It has infinitely many solution (by
case 2(c)). The system given above is
reduced to one equation i.e. x + 2y + 3z = 6

Infinitely many solution

Assigning arbitrary values to two of the


three unknowns say y = s, z = t
We get x = 6 2y 3z = 6 2s 3t

Fig. 1.9

The solution is x = 6 2s 3t, y = s, z = t


i.e. (x, y, z) = (6 2s 3t, s, t) s, t R
For different value s, t we get different solution.
Solution (4) :
x + 2y + 3z = 6 ; x + y + z = 3 ;
1 2 3
= 1 1 1 = 0

2 3 4

2x + 3y + 4z = 10
No Solution

6 2 3
x = 3 1 1 = 1

10 3 4
Fig. 1.10
Since = 0, x 0 (atleast one of the values of x, y, z non-zero) The
system is inconsistent (by case 2(a)).
It has no solution.

27

Solution (5) :
x + 2y + 3z = 6
1
= 2

3
1
y = 2

;
2x + 4y + 6z = 12 ; 3x + 6y + 9z = 24
2 3
6 2 3
4 6 = 0 ;
x = 12 4 6 = 0

24 6 9
6 9
6 3
1 2 6

12 6 = 0 ;
z = 2 4 12 = 0

3 6 24
24 9

Here = 0 and x = y = z = 0.
All the 2 2 minors of are
zero, but we see that atleast one of
the 2 2 minors of x or y or z is
non zero. i.e.
12 4 0 minor of 3 in
x
24 6

No solution

by case 2(d), the system is


inconsistent and it has no solution.

Fig. 1.11

Example 1.17 : Solve the following system of linear equations by determinant


method.
(1)

x + y = 3,

(2) 2x + 3y = 8,

(3) x y = 2,

2x + 3y = 7

4x + 6y = 16

3y = 3x 7

Solution (1) : x + y = 3 ; 2x + 3y = 7
1 1
=
= 3 2 = 1, ; 0
2 3
3 1
1
x =
= 9 7 = 2 ; y =
7 3
2
= 1,

x = 2,

It has unique solution


3

=76=1

y = 1

By Cramers rule
x=

x 2
= 1 =2

; y =

y
1
= 1 = 1

solution is (x, y) = (2, 1)

28

Solution (2) : 2x + 3y = 8 ; 4x + 6y = 16
2 3
=
= 12 12 = 0
4 6
8 3
x =
= 48 48 = 0
16 6
2 8
y =
= 32 32 = 0
4 16
Since = 0, and x = y = 0 and atleast one of the coefficients aij of 0,
the system is consistent and has infinitely many solutions.
All 2 2 minor are zero and atleast (1 1) minor is non zero. The system
is reduced to a single equation. We assign arbitrary value to x (or y) and solve
for y (or x).
Suppose we assign x = t, from equation (1)
1
we get y = 3 (8 2t).
8 2t
The solution set is
(x, y) = t, 3 , t R

In particular

(x, y) = (1, 2)
(x, y) = ( 2, 4)
1
(x, y) = 2 , 3

for t = 1
for t = 2
1
for t = 2

Solution (3) : x y = 2 ; 3y = 3x 7
1 1
=
= 0,
3 3
2 1
x =

7 3 = 1
Since = 0 and x 0 (atleast one of the values x or y 0)
the system is inconsistent. It has no solution.
Example 1.18 : Solve the following non-homogeneous equations of three
unknowns.
(1)
x + 2y + z = 7
(2) x + y + 2z = 6
(3) 2x + 2y + z = 5
2x y + 2z = 4
3x + y z = 2
xy+z=1
x + y 2z = 1
4x + 2y + z = 8
3x + y + 2z = 4

29

(4)

x + y + 2z = 4
2x + 2y + 4z = 8
3x + 3y + 6z = 12

(5)

Solution (1) : x + 2y + z = 7,
1 2
1

x + y + 2z = 4
2x + 2y + 4z = 8
3x + 3y + 6z = 10

2x y + 2z = 4,

2 1 2 = 15
1 1 2
7
2
1

x = 4 1 2 = 15
1 1 2
1 2
7

z = 2 1 4 = 30
1 1 1

x + y 2z = 1

0 it has unique solution.

1 7 1
; y = 2 4 2 = 30

1 1 2

= 15, x = 15, y = 30, z = 30


By Cramers rule
x
y
z
x=
= 1, y = = 2, z = = 2

Solution is (x, y, z) = (1, 2, 2)


3x + y z = 2,
4x + 2y + z = 8
Solution (2) : x + y + 2z = 6,
1 1 2
6 1 2
= 3 1 1 = 0,

4
1
y = 3

2
6
2

1
2
1 = 0,

x = 2 1 1 = 0,

8
1
z = 3

2 1
1 6
1 2 = 0

2 8
8
Since = 0 and x = y = z = 0, also atleast one of the (2 2) minors of
is not zero, the system is consistent and has infinitely many solution.
Take two suitable equations and assign arbitrary value to one of the three
unknowns. We solve for the other two unknowns.
Let z = k R
equation (1) and (2) becomes
x + y =6 2k
3x + y = 2 + k

30

1 1
=13=2
3 1

6 2k 1
= 6 2k 2 k = 4 3k
2 + k 1

x =

1 6 2k
= 2 + k 18 + 6k = 7k 16
3 2 + k

y =

By Cramers rule
x=

x
4 3k
1
=
= 2 (3k 4)

y=

y
7k 16
1
=
= 2 (16 7k)

The solution set is


3k 4 16 7k
(x, y, z) = 2 ,
, k
2

kR

Particular Numerical solutions for k = 2 and 2 are


( 5, 15, 2) and (1, 1, 2) respectively
Solution (3) : 2x + 2y + z = 5,
2 2 1

= 1 1 1 = 0

x y + z = 1,

3x + y + 2z = 4
2 1

5
; x = 1 1 1 0

4 1 2

Since = 0 and x 0 (atleast one of the values of x, y, z non zero) the


system is inconsistent. i.e. it has no solution.
Solution (4) : x + y + 2z = 4,
1 1 2

= 2 2 4 = 0

3 3 6
1 4 2
y = 2 8 4 = 0,

3 12 6

2x + 2y + 4z = 8,
4 1 2

3x + 3y + 6z = 12

x = 8 2 4 = 0

12 3 6
1 1 4
z = 2 2 8 = 0

3 3 12

31

Since = 0 and x = y = z = 0 also all 2 2 minors of , x, y and z


are zero, by case 2(c), it is consistent and has infinitely many solutions. ( all
2 2 minors zero and atleast one of aij of 0, the system is reduced to single
equation).
Let us take x = s and y = t, we get from equation (1)
1
z = 2 (4 s t) the solution set is
4 s t
(x, y, z) = s, t,
2 , s, t R

Particular numerical solution for


(x, y, z) = (1, 1, 1)

when s = t = 1

3
(x, y, z) = 1, 2, 2

Solution (5) : x + y + 2z = 4,

2x + 2y + 4z = 8,

when s = 1, t = 2
3x + 3y + 6z = 10

1 1 2
= 2 2 4 = 0

3 3 6

1 4 2
y = 2 8 4 = 0

3 10 6

4 1 2
x = 8 2 4 = 0,

10 3 6

1 1 4
z = 2 2 8 = 0

3 3 10

= 0 and x = y = z = 0. Also all 2 2 minors of = 0, but not all the


minors of x, y and z are zero.
Therefore the system is inconsistent. i.e. it has no solution.
Example 1.19 : A bag contains 3 types of coins namely Re. 1, Rs. 2 and Rs. 5.
There are 30 coins amounting to Rs. 100 in total. Find the number of coins in
each category.
Solution :
Let x, y and z be the number of coins respectively in each category Re. 1,
Rs. 2 and Rs. 5. From the given information
x + y + z = 30
(i)
x + 2y + 5z = 100
(ii)
Here we have 3 unknowns but 2 equations. We assign arbitrary value k to z
and solve for x and y.

32

(i) and (ii) become


x + y = 30 k
kR

x + 2y = 100 5k

30 k 1
1 30 k
1 1
=
= 3k 40, y =
= 70 4k
= 1, x =
1 2
100 5k 2
1 100 5k
By Cramers Rule
x =

x
= 3k 40,

y=

y
= 70 4k

The solution is (x, y, z) = (3 k 40, 70 4k, k) k R.


Since the number of coins is a non-negative integer, k = 0, 1, 2
Morever 3k 40 0,

and 70 4k 0 14 k 17

The possible solutions are (2, 14, 14), (5, 10, 15), (8, 6, 16) and (11, 2, 17)

1.5.4 Homogeneous linear system :


A system of linear equations is said to be homogeneous if the constant
terms are all zero; that is, the system has the form
a11 x1 + a12 x2 + + a1n xn = 0
a21 x1 + a22 x2 + + a2n xn = 0
.
.
am1x1 + am2 x2 + .. + amn xn = 0
Every homogeneous system of linear equations is always consistent, since
all such systems have x1 = 0, x2 = 0 xn = 0 as a solution. This solution is
called trivial solution. If there are other solution they are called non trivial
solutions. Because a homogeneous linear system always has the trivial solution,
there are only two possibilities.
(i) (The system has only) the trivial solution
(ii) (The system has) infinitely many solutions in addition to the trivial
solution.

33

As an illustration, consider a
homogeneous linear system of two
equations in two unknowns.
x+y = 0
xy = 0
the graph of these equations are lines
through the origin and the trivial solution
corresponding to the point of intersection
Fig. 1. 12
at the origin.

X Y =0

S
o

X +Y =0

For the following system


X+ Y= 0
2X + 2Y = 0
xy = 0
2x 2y = 0
O
the graph shows, that the system has
X
infinitely many solutions.
There is one case in which a
Fig. 1.13
homogeneous system is assured of having
non-trivial solutions, namely, whenever
the system involves more number of unknowns than the number of equations.
Theorem 1.3 : (without proof)
A homogeneous system of linear equations with more number of
unknowns than the number of equations has infinitely many solutions.
Example 1.20 :
Solve :
x + y + 2z = 0
2x + y z = 0
2x + 2y + z = 0
Solution :
1 1 2
= 2 1 1 = 3

2 2

1
0, the system has unique solution.
The above system of homogeneous equation has only trivial solution.
i.e., (x, y, z) = (0, 0, 0).

34

Example 1.21 :
Solve :
x + y + 2z = 0
3x + 2y + z = 0
2x + y z = 0
Solution :
1 1 2
= 3 2

1 =0

2 1 1

Since = 0, it has infinitely many solutions. Also atleast one 2 2 minors


of 0, the system is reduced to 2 equations.
Assigning arbitrary value to one of the unknowns, say z = k and taking
first and last equations. (Here we can take any two equations)
we get
x + y = 2k
2x + y = k
2k 1
1 2k
1 1
=
y =
= 3k,
= 5k
= 1, x =
2 1
k 1
2 k
By Cramers Rule
x = 3k, y = 5k
Solution is (x, y, z) = (3k, 5k, k)
EXERCISE 1.4
Solve the following non-homogeneous system of linear equations by
determinant method :
(1)
3x + 2y = 5
(2)
2x + 3y = 5
x + 3y = 4
4x + 6y = 12
(3)

4x + 5y = 9
8x + 10y = 18

(4)

(5)

2x + y z = 4
x + y 2z = 0
3x + 2y 3z = 4

(6)

(7)

x + 2y + z = 6
3x + 3y z = 3
2x + y 2z = 3

(8)

35

x+y+z=4
xy+z=2
2x + y z = 1
3x + y z = 2
2x y + 2z = 6
2x + y 2z = 2
2x y + z = 2
6x 3y + 3z = 6
4x 2y + 2z = 4

1 2 1
2 4 1
(9) x + y z = 1 ; x + y + z = 5 ;

3 2 2
x yz=0

(10) A small seminar hall can hold 100 chairs. Three different colours
(red, blue and green) of chairs are available. The cost of a red chair
is Rs.240, cost of a blue chair is Rs.260 and the cost of a green chair
is Rs.300. The total cost of chair is Rs.25,000. Find atleast 3
different solution of the number of chairs in each colour to be
purchased.

1.5.5 Rank method :


Let us consider a system of m linear algebraic equation, in n unknowns
x1, x2, x3, xn as in section 1.2.
The equations can be written in the form of matrix equation as AX = B
Where the m n matrix A is called the coefficient matrix.
A set of values x1, x2, x3 xn which satisfy the above system of equations
is called a solution of the system.
The system of equations is said to be consistent, if it has atleast one
solution. A consistent system may have one or infinite number of solutions,
when the system possesses only one solution then it is called a unique solution.
The system of equations is said to be inconsistent if it has no solution.
The m (n + 1) matrix.
a11 a12 a13 a1n

a
a

21

b is called the augmented matrix of the

b
b1

a22 a23 a2n b2

a32 a33 a3n


3

a
m1
m2 am3 amn
m
system and it is denoted by [A, B]. The condition for the consistency of a
system of simultaneous linear equations can be given interms of the coefficient
and augmented matrices.
31

The system of simultaneous linear equations AX = B is consistent if and


only if the matrices A and [A, B] are of the same rank.

36

The solution of a given system of linear equations is not altered by


interchanging any two equations or by multiplying any equation by a non-zero
scalar or by adding a multiple of one equation to another equation. By applying
elementary row operations to the augmented matrix the given system of
equations can be reduced to an equivalent system and this reduced form is used
to test for consistency and to find the solutions.
Steps to be followed for testing consistency :
(i) Write down the given system of equations in the form of a matrix
equation AX = B.
(ii) Find the augmented matrix [A, B] of the system of equations.
(iii) Find the rank of A and rank of [A, B] by applying only elementary row
operations. Column operations should not be applied.
(iv) (a) If the rank of A rank of [A, B] then the system is inconsistent
and has no solution.
(b) If the rank of A = rank of [A, B] = n, where n is the number of
unknowns in the system then A is a non-singular matrix and the
system is consistent and it has a unique solution.
(c) If the rank of A = rank of [A, B] < n, then also the system is
consistent but has an infinite number of solutions.
Example 1.22 : Verify whether the given system of equations is consistent. If it
is consistent, solve them.

2x + 5y + 7z = 52,

x + y + z = 9,

2x + y z = 0

Solution : The given system of equations is equivalent to the single matrix


equation.
2 5 7 x 52

1 1 1 y = 9


2 1 1 z 0
AX = B

The augmented matrix is


2 5
[A, B] = 1 1

52

1 9

2 1 1 0

37

1
2

2
1
0

52 R1 R2

1 1 0
1
1
9
R R2 2R1
3
5
34 2
R3 R3 2R1
1 3 18
9
1 1 1

0 1 3 18 R R
3

2
0 3 5 34

0
0
1

20
9

1 3 18
0

R3 R3 + 3R2

The last equivalent matrix is in the echelon form. It has three non-zero
rows.
(A, B) = 3
1
Also A

0
0

4
1

1 3
0

Since there are three non-zero rows, (A) = 3

(A) = [A, B] = 3 = number of unknowns.


The given system is consistent and has a unique solution.
To find the solution, we see that the given system of equations is
equivalent to the matrix equation.
1 1
1
9
x

0
0

1 3
0

y =

z


18
20

x+y+z = 9

(1)

y 3z = 18

(2)

4z = 20

(3)

38

(3) z = 5 ; (2) y = 18 3z = 3 ; (1) x = 9 y z x = 935 = 1


Solution is x = 1, y = 3, z = 5
Example 1.23 :
Examine the consistency of the equations
2x 3y + 7z = 5,
3x + y 3z = 13,
2x + 19y 47z = 32
Solution :
The given system of equations can be written in the form of a matrix
equation as
2 3 7
x 5

3
2

1
19

47
3

y = 13

z 32
AX = B

The augmented matrix is

2
[A, B] = 3
2

19 47

3
2

13
32
5

7
2

7
2

19 47

R
0 112 272 112 R
0 22 54 27
3 7
5
1 2 2
2

0 112 272 112 R


0 0 0 5
3
1 2

3
2

5
2

13 R1 2

32
5
2

R2 3R1

R3 2R1

R1

R3 4R2

The last equivalent matrix is in the echelon form. It has three non-zero
rows. [A, B] = 3 and (A) = 2
(A) [A, B]
The given system is inconsistent and hence has no solution.
Note : This problem can be solved by not dividing R1 by 2 also. i.e., R2 2R2 3R1
Example 1.24 :
Show that the equations x + y + z = 6, x + 2y + 3z = 14,
x + 4y + 7z = 30 are consistent and solve them.

39

Solution : The matrix equation corresponding to the given system is


1 1 1 x 6

1 2 3 y = 14


1 4 7 z 30

AX = B
The augmented matrix is
1 1 1 6

[A, B] = 1 2 3 14

1 4 7 30
1 1 1 6 R R R
2
1
0 1 2 8 2

R3 R3 R1
0 3 6 24
1 1 1 6

0 1 2 8 R R 3R
3
2

3
0 0 0 0

In the last equivalent matrix, there are two non-zero rows.


(A, B) = 2 and (A) = 2
(A) = (A, B)
The given system is consistent. But the value of the common rank is less
than the number of unknowns. The given system has an infinite number of
solutions.
The given system is equivalent to the matrix equation
1 1 1 x 6

0 1 2 y = 8


0 0 0 z 0

x+y+z = 6
(1)
y + 2z = 8
(2)
(2) y = 8 2z ; (1) x = 6 y z = 6 (8 2z) z = z 2
Taking z = k, we get x = k 2, y = 8 2k ; k R
Putting k = 1, we have one solution as x = 1, y = 6, z = 1. Thus by giving
different values for k we get different solutions. Hence the given system has
infinite number of solutions.
Example 1.25 :
Verify whether the given system of equations is consistent. If it is
consistent, solve them :

x y + z = 5,

x + y z = 5,

40

2x 2y + 2z = 10

Solution : The matrix equation corresponding to the given system is


1 1 1
x 5

1
2

1
2

y = 5

z 10
AX = B

The augmented matrix is

1
[A, B] = 1
2

1
1

10
5

1 5

2 2
1
1 1 5 R R + R
2
1
0 0 0 0 2

R3 R3 2R1
0 0 0 0
In the last equivalent matrix, there is only one non-zero row
[A, B] = 1 and (A) = 1
Thus (A) = [A, B] = 1. the given system is consistent. Since the
common value of the rank is less than the number of unknowns, there are
infinitely many solutions. The given system is equivalent to the matrix
equation.

1 1 1 x 5
0 0 0 y = 0


0 0 0 z 0
x y + z = 5 ; Taking y = k1, z = k2, we have x = 5 + k1 k2. for various
values of k1 and k2 we have infinitely many solutions. k1, k2 R
Example 1.26 : Investigate for what values of , the simultaneous equations x
+ y + z = 6, x + 2y + 3z = 10, x + 2y + z = have (i) no solution (ii) a unique
solution and (iii) an infinite number of solutions.
Solution :
The matrix equations corresponding to the given system is
1 1 1 x 6

1 2 3 y = 10


1 2 z
AX = B

41

The augmented matrix is


1 1 1

[A, B] = 1 2 3 10

1 2
1
0


1 1

6
R R2 R1
1 2
4 2
R3 R3 R2
0 3 10
Case (i) : 3 = 0 and 10 0 i.e. = 3 and 10.
In this case (A) = 2 while [A, B] = 3 (A) [A, B]
The given system is inconsistent and has no solution.
Case (ii) : 3 0 i.e., 3 and can take any value in R.
In this case (A) = 3 and [A, B] = 3
(A) = [A, B] = 3 = number of unknowns.
The given system is consistent and has a unique solution.
Case (iii) :
3 = 0 and 10 = 0 i.e., = 3 and = 10
In this case (A) = [A, B] = 2 < number of unknowns.
The given system is consistent but has an infinite number of solutions.

1.5.6 Homogeneous linear Equations :


A system of homogeneous linear equations is given by
a11 x1 + a12 x2 + a13 x3 + ...+ a1n xn = 0
a21 x1 + a22 x2 + a23 x3 + . + a2n xn = 0

am1x1 + am2 x2 + am3 x3 + + amn xn = 0


and the corresponding augmented matrix is
a11 a12 a1n 0

[A, B] =

21

m1

a22 a2n

am2 amn

= [A, O]

0
0

42

Since rank of A = rank of [A, O] is always true, we see that the system of
homogeneous equations is always consistent.
Note that x1 = 0, x2 = 0, x3 = 0 xn = 0 is always a solution of the system.
This solution is called a trivial solution. If the rank of A = rank of
[A, B] < n then the system has non trivial solutions including trivial solution. If
(A) = n then the system has only trivial solution.
Example 1.27 : Solve the following homogeneous linear equations.
x + 2y 5z = 0, 3x + 4y + 6z = 0, x + y + z = 0
Solution : The given system of equations can be written in the form of matrix equation
1 2 5 x 0

3 4

1 1

6 y = 0

1 z 0
AX = B

The augmented matrix is


1 2
[A, B] = 3 4

1 1

1
0
0
1
0
0
1
0
0

1
2

0
5

21

1
2

6
5

2
2

21
5

0
0

0
0
0

0
0
0
0

R2 R2 3R1
R3 R3 R1

R2 R3

R3 R3 2R2

This is in the echelon form.


Clearly [A, B] = 3. and. (A) = 3
(A) = [A, B] = 3 = number of unknowns.
The given system of equations is consistent and has a unique solution.
i.e., trivial solution.
x = 0, y = 0 and z = 0

43

Note : Since (A) = 3, | A | 0 i.e. A is non-singular ;


The given system has only trivial solution x = 0, y = 0, z = 0
Example 1.28 : For what value of the equations
x + y + 3z = 0, 4x + 3y + z = 0, 2x + y + 2z = 0 have a (i) trivial
solution, (ii) non-trivial solution.
Solution : The system of equations can be written as AX = B
1 1 3 x 0

4 3

2 1 2
1
[A, B] = 4

y = 0

z 0
1 3 0
3 0

1 2 0

0 1
0 1
1 1

0 1
0 0
1

0
0

0
0
0

12 0
4
3
12

R2 R2 4R1
R3 R3 2R1
R3 R3 R2

8
Case (i) : If 8 then 8 0 and hence there are three non-zero rows.
[A] = [A, B] = 3 = the number of unknowns.
The system has the trivial solution x = 0, y = 0, z = 0
Case (ii) :
If = 8 then.
[A, B] = 2 and (A) = 2
(A) = [A, B] = 2 < number of unknowns.
The given system is equivalent to
x + y + 3z = 0 ; y + 4z = 0
y = 4z ;
x=z
Taking z = k, we get x = k, y = 4k, z = k [k R {0}]
which are non-trivial solutions.
Thus the system is consistent and has infinitely many non-trivial solutions.
Note : In case (ii) the system also has trivial solution. For only non-trivial
solutions we removed k = 0.

44

EXERCISE 1.5
(1) Examine the consistency of the following system of equations. If it is
consistent then solve the same.
(i)
4x + 3y + 6z = 25
x + 5y + 7z = 13
2x + 9y + z = 1
(ii) x 3y 8z = 10
3x + y 4z = 0
2x + 5y + 6z 13 = 0
(iii) x + y + z = 7
x + 2y + 3z = 18
y + 2z = 6
(iv) x 4y + 7z = 14
3x + 8y 2z = 13
7x 8y + 26z = 5
(v) x + y z = 1
2x + 2y 2z = 2
3x 3y + 3z = 3
(2) Discuss the solutions of the system of equations for all values of .
x + y + z = 2,
2x + y 2z = 2,
x + y + 4z = 2
(3) For what values of k, the system of equations
kx + y + z = 1,
x + ky + z = 1,
x + y + kz = 1 have
(i) unique solution (ii) more than one solution (iii) no solution

45

2. VECTOR ALGEBRA
2.1 Introduction :
We have already studied two operations addition and subtraction on
vectors in class XI. In this chapter we will study the notion of another operation,
namely product of two vectors. The product of two vectors results in two
different ways, viz., a scalar product and a vector product. Before defining these
products we shall define the angle between two vectors.

2.2 Angle between two vectors :

Let two vectors a and b be represented by OA and OB respectively. Then

the angle between a and b is the angle between their directions when these
directions both converge or both diverge from their point of intersection.
B
b

A
a

Fig. 2. 1
Fig. 2. 2
It is evident that if is the numerical measure of the angle between two
vectors, then 0 .

2.3 The Scalar product or Dot product

Let a and b be two non zero vectors inclined at an angle . Then the


scalar product of a and b is denoted by a . b and is defined as the scalar

a b cos .

| || |



Thus a . b = a b cos = ab cos
Note : Clearly the scalar product of two vectors is a scalar quantity. Therefore

the product is called scalar product. Since we are putting dot between a and

b , it is also called dot product.

| || |

46

Geometrical Interpretation of Scalar Product



Let OA = a , OB = b

Let be the angle between a and b


. From B draw BL r to OA.

OL is called the projection of b on a .


OL
From OLB, cos = OB
O

Fig. 2.3
OL = (OB) (cos )

OL = b (cos )

| |


Now by definition a . b = | a | | b | cos

= | a | (OL)

a . b = | a | [projection of b on a ]

a . b

Projection of b on a =
=

(1)

[ using (1)]

a
. b =a. b

| | | |


a . b

= a .
Projection of a on b =

| |

b

= a . b

| |

2.3.1 Properties of Scalar Product :


Property 1 :
The scalar product of two vectors is commutative

(i.e.,) a . b = b . a for any two vectors a and b


Proof :

Let a and b be two vectors and the angle between them.


a . b = a

| | |b | cos


b . a = | b | | a | cos
47

(1)

b . a = a

| | |b | cos

(2)

From (1) and (2)



a . b = b . a
Thus dot product is commutative.
Property 2 : Scalar Product of Collinear Vectors :

(i) When the vectors a and b are collinear and are in the same
direction, then = 0


Thus a . b = a b cos = a
b (1) = ab
(1)

(ii) When the vectors a and b are collinear and are in the opposite
direction, then =
Thus



a . b = a b cos = a b (cos )
(1)

= a b ( 1) = ab
Property 3 : Sign of Dot Product

The dot product a . b may be positive or negative or zero.
(i) If the angle between the two vectors is acute (i.e., 0 < < 90) then
cos is positive. In this case dot product is positive.
(ii) If the angle between the two vectors is obtuse (i.e., 90 < < 180) then
cos is negative. In this case dot product is negative.
(iii) If the angle between the two vectors is 90 (i.e., = 90) then
cos = cos 90 = 0. In this case dot product is zero.

Note : If a . b = 0, we have the following three possibilities


a . b = 0 a b cos = 0

| || |

| || |

| || |
| || |

| || |

| || |

(i)
(ii)

|a | = 0 (i.e.,) a is a zero vector and b any vector.


|b | = 0 (i.e.,) b is a zero vector and a any vector.


(iii) cos = 0 (i.e.,) = 90 (i.e.,) a b
Important Result :



Let a and b be two non-zero vectors, then a . b = 0 a b

48

Property 4 : Dot product of equal vectors :


2

a . a = a

Convention :

| | |a | cos 0 = |a | |a | = |a |
2
2
(a ) = a . a = |a | = a 2 = a2

= a2

Property 5 :

(i) i . i = j . j = k . k = 1

(ii)
i . j = j . i = j . k = k . j = k . i = i . k =0


i . i = i

| | |i | cos 0 = (1) (1) (1) = 1




i . j = | i | | j | cos 90 = (1) (1) (0) = 0
Property 6 :

If m is any scalar and a , b are any two vectors, then

(ma ) . b = m(a . b ) = a . (mb )


Property 7 :

If m, n are scalars and a , b are two vectors then


m a . n b = mn a . b = mn a . b = a . mn b

) (

Property 8 :
The scalar product is distributive over addition.



a . b + c = a . b + a . c , for any three vectors a , b , c
B
Proof :
c

C
Let
OA = a
b
c

+
b
OB = b

BC = c
O
L
M

a
Then
OC = OB + BC
Fig. 2.4

= b + c
Draw BL OA and CM OA

49

OL = Projection of b on a

LM = Projection of c on a

OM = Projection of b + c on a

We have a . b = a

| | (Projection of b on a )

a . b = | a | (OL)

a . c = | a | (Projection of c on a )

a . c = | a | (LM)


a .( b + c ) = | a | Projection of ( b + c ) on a

= | a | (OM) = | a | (OL + LM)

= | a | (OL) + | a | (LM)

Also

Now


= a . b + a . c

(1)

(2)

[by using (1) and (2)]



a . b + c = a . b + a . c

(
)


Corollary : a .( b c ) = a . b a . c
Hence

Property 9 :

(i) For any two vectors a and b ,


2

Proof :

(a + b ) = (a ) + 2a . b + (b )
2
(a + b ) = (a + b ) . (a + b )


= a2 + 2 a . b + b2


= a . a + a . b + b . a + b . b (by distribution law)
2

(a ) + a .b + a .b + (b ) ( a .b = b .a )

2
2
= ( a ) + 2 a . b + ( b ) = a2 + 2 a . b + b2
2
2
2

( a b ) = (a ) 2a . b + (b ) = a2 2a . b + b2
=

(ii)

50

(iii)
Proof :

(a + b ) . (a b ) = (a ) (b ) = a2 b2
(a + b ) . (a b ) = a . a a . b + b . a b . b
2 2
= ( a ) ( b ) = a 2 b2

Property 10 : Scalar product in terms of components :

Let a = a1 i + a2 j + a3 k
b = b1 i + b 2 j + b 3 k


a . b = a1 i + a2 j + a3 k . b1 i + b2 j + b3 k

= a1b1 i . i + a1b2 i . j + a1b3 i . k + a2b1 j . i

+ a2b2 j . j + a2b3 j . k + a3b1 k . i + a3b2 k . j + a3b3

k.k

)
(

= a1b1(1) + a1b2(0) + a1b3(0) + a2b1(0) + a2b2(1) + a2b3(0)


+ a3b1(0) + a3b2(0) + a3b3(1)
= a1b1 + a2b2 + a3b3
Thus, the scalar product of two vectors is equal to the sum of the products
of their corresponding components.
Property 11 : Angle between two vectors :

a , b be two vectors inclined at an angle .
Let


Then
a . b = a b cos

| || |

cos =


a . b

a b

| || |


a . b

a b

= cos1

| || |

If a = a1 i + a2 j + a3 k and b = b1 i + b2 j + b3 k

Then a . b = a1b1 + a2b2 + a3b3

|a | =

a12 + a22 + a32 ;

= cos1

|b | =

b12 + b22 + b32

a1b1 + a2b2 + a3b3


2

a1 + a2 + a3

51

b12

b22

b32

Property 12 : For any two vectors a and b

|a + b | |a | + |b | (Triangle inequality)
2

2 2

We have | a + b | = | a | + | b | + 2( a . b )
2

2 2

|
|
a + b = | a | + | b | + 2| a | | b | cos
2 2

| a | + | b | + 2| a | | b |
2

2

| a + b | | a | + | b |

|a + b| |a|+|b|

[ cos 1]


Example 2.1 : Find a . b when

(i) a = i 2 j + k and b = 4 i 4 j + 7 k


(ii) a = j + 2 k and b = 2 i + k

(iii) a = j 2 k and b = 2 i + 3 j 2 k
Solution :
(i)

(ii)
(iii)



a . b = i 2J + k . 4 i 4 j +7k

) (

= (1) (4) + ( 2) ( 4) + (1) (7) = 19



a . b = j + 2 k . 2 i + k = (0) (2) + (1) (0) + (2) (1) = 2

(
) (
)

a . b = ( j 2 k ) . (2 i + 3 j 2 k )

= (0) (2) + (1) (3) + ( 2) ( 2) = 7

Example 2.2 : For what value of m the vectors a and b are perpendicular to
each other

(i) a = m i + 2 j + k and b = 4 i 9 j + 2 k


(ii) a = 5 i 9 j + 2 k and b = m i + 2 j + k
Solution :

(i) Given : a b

52


a . b =0 m i +2 j + k . 4 i 9J +2k =0
4m 18 + 2 = 0 m = 4


(ii)
5 i 9J +2k . m i +2 j + k =0
16
5m 18 + 2 = 0 m = 5

Example 2.3 : If a and b are two vectors such that a = 4, b = 3 and


a . b = 6. Find the angle between a and b
Solution :

6
1

a . b
= (4) (3) = 2
=3
cos =

a b
Example 2.4 : Find the angle between the vectors

3 i 2 j 6 k and 4 i j + 8 k

Let a = 3 i 2 j 6 k ; b = 4 i j + 8 k
Solution :
Let be the angle between the vectors

a . b = 12 + 2 48 = 34

a = 7, b = 9

a . b
34
cos =
=
79

a b
34
= cos1 63

Example 2.5 : Find the angle between the vectors a and b




where a = i j and b = j k



i j . j k
a . b
=
cos =
Solution :



a b
i j
j k

) (

) (

| |

| || |

| |

| |

| || |

(
| || | |

) (
||

(1) (0) + ( 1) (1) + (0) ( 1)


2 2
2
1
cos = 2 = 3

cos =

53

)
|

| |


Example 2.6 : For any vector r

i + r . j
j+ r . k k
prove that r = r . i


Solution : Let r = x i + y j + z k be an arbitrary vector.

r. i = x i +y j +zk . i =x

(
)


r . j = (x i + y j + z k ) . j = y


r . k = (x i + y j + z k ) . k = z
(r . i ) i + (r . j ) j + (r . k ) k = xi + yj + zk = r
Example 2.7 : Find the projection of the vector

7 i + j 4 k on 2 i + 6 j + 3 k

Let a = 7 i + j 4 k ; b = 2 i + 6 j + 3 k
Solution :


7 i + j 4k . 2 i +6 j +3k

a . b
Projection of a on b =
=

2 i +6 j +3k
b

) (

| |

14 + 6 12
8
= 7
4 + 36 + 9

Example 2.8 : For any two vectors a and b


=

Solution :

| + |a b | = 2 |a | + |b |
2
2
2
2
|a + b | = (a + b ) = |a | + |b | + 2a . b
2
2
2
2
|a b | = (a b ) = |a | + |b | 2a . b


prove that a + b

(1)
(2)

Adding (1) and (2)


2

|a + b | + |a b | = |a | + |b |

2
+2a . b + a

| |

2
+|b| 2a . b
2 2
2
2
= 2| a | + 2| b | = 2| a | + | b |
54

Example 2.9 : If a and b are unit vectors inclined at an angle , then prove that

1
sin 2 = 2 a b
2




Solution : a b = a 2 + b 2 2a . b = 1 + 1 2 a b cos

= 2 2 cos = 2 (1 cos ) = 2 2 sin2 2

1

a b = 2 sin 2 sin 2 = 2 a b

Example 2.10 : If a + b + c = 0 , a = 3, b = 5 and c = 7, find the

angle between a and b



a + b + c = 0
Solution :

a + b = c

| |

| |

(a + b ) = ( c )

2
2
2
(a) + (b) +2a . b = (c)
2
2
2
|a | + |b | + 2|a | |b |cos = |c |

| |

32 + 52 + 2(3) (5) cos = 72


1

cos = 2 = 3
Example 2.11 : Show that the vectors

2 i j + k , i 3 j 5 k , 3 i + 4 j + 4 k form the sides of a right


angled triangle.

Solution : Let a = 2 i j + k ; b = i 3 j 5 k ; c = 3 i + 4 j + 4 k

We see that a + b + c = 0

a , b , c forms a triangle

Further
a . b = 2 i j + k . i 3 j 5k
= 2+35=0

a b The vectors form the sides of a right angled triangle.

) (

55

(1)
(2)

EXERCISE 2.1

Find a . b when a = 2 i + 2 j k and b = 6 i 3 j + 2 k



If a = i + j + 2 k and b = 3 i + 2 j k find

a +3b . 2a b

Find so that the vectors 2 i + j + k and i 2 j + k are


perpendicular to each other.

Find the value of m for which the vectors a = 3 i + 2 j + 9 k and


b = i + m j + 3 k are (i) perpendicular (ii) parallel

Find the angles which the vector i j + 2 k makes with the


coordinate axes.

Show that the vector i + j + k is equally inclined with the


coordinate axes.

If a and b are unit vectors inclined at an angle , then prove that



a b

1
(ii) tan 2 =
(i) cos 2 = 2 a + b

a + b
If the sum of two unit vectors is a unit vector prove that the magnitude of
their difference is 3 .

If a , b , c are three mutually perpendicular unit vectors, then prove

that a + b + c = 3

(3)
(4)

(5)

(6)
(7)

(8)
(9)

) (

(10)



If | a + b | = 60, | a b | = 40 and | b | = 46 find | a |.


(11) Let u , v and w be vector such that u + v + w = 0 .


If u = 3, v = 4 and w = 5 then find u . v + v . w + w . u

(12) Show that the vectors 3 i 2 j + k , i 3 j + 5 k and


2 i + j 4 k form a right angled triangle.


(13) Show that the points whose position vectors


4 i 3 j + k , 2 i 4 j + 5 k , i j form a right angled triangle.

| |

| |

| |

56

(14) Find the projection of


(i) i j on z-axis (ii) i + 2 j 2 k on 2 i j + 5 k



(iii) 3 i + j k on 4 i j + 2 k

2.3.2 Geometrical Applicaton of dot product


Cosine formulae :
Example 2.12 : With usual notations :
c2 + a2 b2
a2 + b2 c2
b2 + c2 a2
;
(ii)
cos
B
=
(iii)
cos
C
=
(i) cos A=
2bc
2ac
2ab
Solution (i) :
From the diagram
-A A

b
AB + BC + CA = 0 a + b + c = 0
c


a = b + c

2
2
= b + c
a

a2 = b2 + c2 + 2 b . c

( ) (

Fig. 2.5

a2 = b2 + c2 + 2bc cos( A)
a2 = b2 + c2 2bc cos A
2bc cosA = b2 + c2 a2
cos A =

b2 + c2 a2
2bc

Similarly we can prove the results (ii) & (iii)


Projection Formulae :
Example 2.13 : With usual notations
(i) a = b cos C+c cos B (ii) b = a cos C+c cos A (iii) c = a cos B+b cos A
Solution (i) :
A
From the diagram

c
b
AB + BC + CA = 0
B

C
-B
a+b+c = 0
a


a = b c
Fig. 2.6


a . a = a.b a.c

57

We have

a2 = ab cos ( C) ac cos ( B)
a2 = ab ( cos C) ac ( cos B)
a2 = ab cos C + ac cos B
a = b cos C + c cos B

Similarly (ii) and (iii) can be proved.


Example 2.14 : Angle in a semi-circle is a right angle. Prove by vector method.
Solution : Let AB be the diameter of the circle with centre O.
Let P be any point on the semi-circle.
P

To prove APB = 90

We have OA = OB = OP (radii)

Now PA = PO + OA

A
O
Also PB = PO + OB

Fig. 2.7
= PO OA

PA . PB = PO + OA . PO OA
2


= PO OA

= PO2 OA2 = 0

PA PB APB = 2
Hence angle in a semi-circle is a right angle.
Example 2.15 : Diagonals of a rhombus are at right angles. Prove by vector
methods.


Solution : Let ABCD be a rhombus. Let AB = a and AD = b
We have AB = BC = CD = DA
i.e.,

Also

a = b

| | | |

(1)

AC = AB + BC = a + b

BD = BC + CD

= AD AB = b a

b
A

Fig. 2.8

58




AC . BD = a + b . b a

(
) (
)


= (b + a).(b a)
2
2


= ( b ) ( a ) = 0 | a | = | b |



Thus AC . BD = 0 AC BD
Hence the diagonals of a rhombus are at right angles.
Example 2.16 : Altitudes of a triangle are concurrent prove by vector method.
Solution :
Let ABC be a triangle and let AD, BE be its two altitudes intersecting at O.
In order to prove that the altitudes are concurrent it is sufficient to prove
that CO is perpendicular to AB.

Taking O as the origin, let the position vectors of A, B, C be a , b , c
respectively.
A

Then OA = a ; OB = b ; OC = c
E
F
Now AD BC

OA BC
B

Fig. 2.9

OA . BC = 0


a . c b =0

a . c a . b = 0 (1)

BE CA OB CA


OB . CA = 0 b . a c = 0


b . a b . c =0
Adding (1) and (2), we get

a . c b . c =0

59

(2)

(a b ) . c = 0


BA . OC = 0 OC AB
Hence the three altitudes are concurrent.
Example 2.17 : Prove that cos (A B) = cos A cos B + sin A sin B
Solution :
Y
Take the points P and Q on the unit
P (Cos A, Sin A)
circle with centre at the origin O.
Q (Cos B, Sin B )
Assume that OP and OQ make angles
A
B
A and B with x-axis respectively.
X
O
M
L
POQ = POx QOx = A B
Clearly the coordinates of P and Q
Fig. 2.10
are (cos A, sin A) and (cos B, sin B) .

Take the unit vectors i and j along x and y axes.

OP = OM + MP = cos A i + sin A j

OQ = OL + LQ = cos B i + sin B j

By value, OP . OQ = cos A i + sin A j . cos B i + sin B j ..(1)


= cos A cos B + sin A sin B

.. (2)
By definition , OP . OQ = OP OQ cos (A B) = cos (A B)
From (1) and (2) cos (A B) = cos A cos B + sin A sin B

) (

2.3.4 Application of Scalar Product in Physics


Work done by force :
The work done by a force is a scalar quantity and its measure is equal to
the product of the magnitude of the force and the resolved part of the
A
displacement in the direction of the force.
Let a particle be placed at O and a
d

force F represented by OB be acting on


the particle at O. Due to the application

of force F , the particle is displaced in


O
L B
F

the direction of OA. Here OA is the


displacement and OL is the displacement
Fig. 2.11

in the direction of F
In right angled OLA

60

OL = OA cos

where is the angle

between F and d

= d cos

| |

The work done by a force = (Magnitude of force)


(displacement in the direction of force)


= F OL = F d cos

Work done by the force = F . d
Note : If a number of forces are acting on a particle, then the sum of the works
done by the separate forces is equal to the work done by the resultant force.
Example 2.18 : Find the work done in moving a particle from the point A,

with position vector 2 i 6 j + 7 k , to the point B, with position vector




3 i j 5 k , by a force F = i + 3 j k
Solution :



F = i + 3 j k ; OA = 2 i 6 j + 7 k ; OB = 3 i j 5 k


d = AB = OB OA = i + 5 j 12 k

Work done = F . d

| |

(i + 3j k ) . (i + 5j 12k
)

= (1) (1) + 3(5) + 12 = 28


Example 2.19 : The work done by the force F = a i + j + k in moving the
point of application from (1, 1, 1) to (2, 2, 2) along a straight line is given to be
5 units. Find the value of a.


Solution : F = a i + j + k ; OA = i + j + k ; OB = 2 i + 2 j + 2 k
Work done = 5 units

d = AB = OB OA = i + j + k

Work done = F . d


5= ai + j + k . i + j + k

) (

5 = a+1+1 a = 3

61

EXERCISE 2.2
Prove by vector method
(1) If the diagonals of a parallelogram are equal then it is a rectangle.
(2) The mid point of the hypotenuse of a right angled triangle is equidistant
from its vertices
(3) The sum of the squares of the diagonals of a parallelogram is equal to the
sum of the squares of the sides.
(4) cos (A + B) = cos A cos B sin A sin B


(5) Find the work done by the force F = 2 i + j + k acting on a particle,
if the particle is displaced from the point with position vector

2 i + 2 j + 2 k to the point with position vector 3 i + 4 j + 5 k .


(6) A force of magnitude 5 units acting parallel to 2 i 2 j + k displaces
the point of application from (1, 2, 3) to (5, 3, 7). Find the work done.

(7) The constant forces 2 i 5 j + 6 k , i + 2 j k and 2 i + 7 j act

on a particle which is displaced from position 4 i 3 j 2 k to position


6 i + j 3 k . Find the work done.


(8) Forces of magnitudes 3 and 4 units acting in the directions

6 i + 2 j + 3 k and 3 i 2 j + 6 k respectively act on a particle which


is displaced from the point (2, 2, 1) to (4, 3, 1). Find the work done by
the forces.

2.4 Vector product :


2.4.1 Right-handed and left handed systems :

Consider a set of three linearly independent vectors a , b , c through the
origin O. As they are linearly independent no two of them have parallel
directions and not all of them lie on the same plane. Let be the smaller angle

(i.e. 0 < < ) between a and b . Let an observer walk from a to b


through the angle keeping O always to his left. If the observers head is on the


same side of the plane of a and b as the vector c , we say a , b , c is a
right handed system or right handed triple (or) triad.


If c has the opposite direction, a , b , c is a left handed system.

62

z
b

b
O
y

Fig. 2. 12

Definition : The vector product of two vectors a and b is denoted as a b


and it is defined as a vector whose magnitude is a
b sin where is the

angle between a and b , 0 and whose direction is perpendicular to


both a and b in such a way that a , b and this direction constitute a right
handed system.
In other words,

| || |

a x b

<

| | |b | sin n, where

<


a b = a

b
is the angle between a and b and n
is a unit vector perpendicular to both


a and b such that a , b , n form a
a
Fig. 2.13
right handed system.
Note :

(1) a , b , n form a right handed system means that if we rotate a into

b , then n will point in the direction perpendicular to the plane

containing a and b in which a right handed screw will move if it is


turned in the same manner.

(2) a b is read as a cross b since we are putting cross between a

and b .

2.4.2 Geometrical interpretation of Vector product :



Let OA = a , OB = b

63

Let be the angle between a and b


Complete the parallelogram OACB

<

B
n

with OA and OB as adjacent sides.


Draw BN OA.
In right angled triangle ONB

BN = b sin

| |

N
A
a Fig. 2.14

a b = a

| | |b | sin n
|a b | = |a | |b | sin

Now

= (OA) (BN)
= Base height
= Area of parallelogram OACB
Area of parallelogram with

a b =

a and b as adjacent sides


1
Also, area of OAB = 2 area of a parallelogram OACB

1
1

= 2 OA OB = 2 a b

|
|
1

Vector area of OAB = 2 ( a b )

Some important results :

Result : (1) The area of a parallelogram with adjacent sides a and b is

a b


(2) The vector area of a parallelogram with adjacent sides is a b

1
(3) The area of a triangle with sides a and b is 2 a b
1
1
(4) The area of a triangle ABC is 2 AB AC (or) 2 BC BA
1
(or) 2 CA CB

64

2.4.3 Properties of Vector Product :


Property (1) : Non-Commutativity of Vector product :

Vector product is not commutative (i.e.) if a and b are any two vectors,



then a b b a however a b = b a .

a b = a

<

Let a and b be two non-zero,


non parallel vectors and let be the
angle between them. Then

b sin n where

| || |

2 -

<

n is a unit vector perpendicular to the

plane of a and b

-n

Fig. 2.15


b a = b a sin() ( n ) = a

| | |b | sin n = (a b )

| || |

Note that b , a and n form a right handed system.



Hence a b b a


But a b = b a

Property (2) :
Vector product of Collinear (Parallel) Vectors :


If the vectors a and b are collinear or parallel then a b = 0

The vectors a and b are collinear or parallel, then = 0,


sin = 0 for = 0,

Thus a b = a b sin n

| || |


= | a | | b | (O) n = 0

Result : The vector product of two non-zero vectors is zero vector if and only if
they are parallel (collinear)


i.e., a b = o a is parallel to b , where a , b are non zero vectors.
Proof (i) :

Suppose a b = O

65

then

|a | |b | sin n = O


But a 0 & b 0, n O
= 0 or

| |

| |

sin = 0

a and b are collinear (parallel)



conversely if
a || b then
= O or
sin = O



a b = a b sin n = o

a b = o

Note : If a b = o , we have the following three possibilities.

(i) a is a zero vector and b is any vector.

(ii) b is a zero vector and a is any vector

(iii) a and b are parallel (collinear)


Property (3) :
Cross Product of Equal Vectors :


a a = a a sin n

= a a (0) n

= o

a a = o for every non-zero vector a


Property (4) :

Cross product of Unit Vectors i , j , k
By the above property

i i = j j = k k = O

| || |

| || |
| || |

k
k
O

j
Y

i
i

Fig. 2. 16

66

i j = i

| | |j | sin 90k = (1) (1) (1) k = k

Also


j k = i, k i = j


j i = k, k j = i , i k = j

Similarly
and
Property (5) :


If m is any scalar and a , b are two vectors inclined at angle , then


ma b = m a b = a mb
Property (6) : Distributivity of vector product over vector addition

Let a , b , c be any three vectors. then

(i)

a b + c = a b + a c (Left distributivity)



b+c a = b a + c a
(ii)
Result :
Vector Product in the determinant form

Let
a = a1 i + a2 j + a3 k and

) (

) (Right distributivity)

b = b1 i + b2 j + b3 k be the two vectors


Then


a b = a1 i + a2 j + a3 k b1 i + b2 j + b3 k



= a1b1 i i + a1b2 i j + a1b3 i k




+ a2b1 j i + a2b2 j j + a2b3 j k

(
)
(
)
(
)



+ a3b1( k i ) + a3b2( k j ) + a3b3( k k )

= a1b2 k + a1b3( j ) + a2b1( k ) + a2b3 i

+ a3b1 j + a3b2( i )

= ( a2b3 a3b2) i (a1b3 a3b1) j + ( a1b2 a2b1) k

67

= a1
b
1


j k
a2
b2

a3

b3

Property (7) : Angle between two vectors :



Let a , b be two vectors inclined at an angle .

Then a b = a

| | |b | sin n


| a b | = | a | | b | sin n


| a b | = | a | | b |sin
|a b | = sin1 |a b |
sin =

|a | |b |
| a | | b |

Note :
In this case is always acute. Thus if we try to find the angle using vector
product, we get only the acute angle.
Hence in problems of finding the angle, the use of dot product is preferable
since it specifies the position of the angle .
Property (8) : Unit vectors perpendicular to two given vectors
(i.e.) Unit vectors normal to the plane of two given vectors.

Let a , b be two non-zero, non-parallel vectors and be the angle
between them.

a b = a

| | |b | sin n

(1)

Where n is a unit vector perpendicular to the both of a and b

|a b | = |a | |b | sin
From (1) and (2)


a b

n =

a b

68

(2)


a b

Note that
is also a unit vector perpendicular to a and b

a b

Unit vectors perpendicular to a and b are



a b

n =

a b

Vectors of magnitude normal to the plane containing a and b is given



a b
by

a b
Example 2.20 :

(
|

)
|




If a , b are any two vectors, then a b + a . b
Solution :

Let be the angle between a and b



a b = a b sin n

| (

) = |a | |b |

| || |
|a b | = |a | |b | sin
2
2
2
|a b | = |a | |b | sin2
2
2
2
(a . b ) = |a | |b | cos2
2
2
2
2
2
2
|a b | + (a . b ) = |a | |b | (sin2 + cos2) = |a | |b |

Example 2.21 : Find the vectors of magnitude 6 which are perpendicular to


both the vectors 4 i j + 3 k and 2 i + j 2 k


Solution :

Let
a = 4 i j +3k ; b =2 i + j 2k

i
j
k


Then
a b =
4 1 3 = i +2 j +2k

69

|a b | =

( 1)2 + (2)2 + (2)2 = 3


a b
Required vectors = 6

a b

= 2 i +4 j +4k

(
)


Example 2.22 : If | a | = 13, | b | = 5 and a . b = 60 then find | a b |
Solution :
2

|a b | + (a . b ) = |a | |b |
2
2
2
2
|a b | = |a | |b | (a . b )
= (13)2 (5)2 (60)2 = 625

|a b | = 25

Example 2.23 : Find the angle between the vectors 2 i + j k and


i + 2 j + k by using cross product.
Solution :




Let a = 2 i + j k ; b = i + 2 j + k

Let be the angle between a and b


a b

a b

|
|
| || |

= sin1


a b =

|a b | =
|a | =
|b | =

i
2
1


j
k
1
2

32 + ( 3)2 + 32 = 3 3
22 + 12 + ( 1)2 = 6
12 + 2 2 + 1 2 = 6

70


1 = 3 i 3 j + 3 k
1


a b
sin =

a b

|
| = 3
| | | | 6

3 3
=
6 2

=3

Example 2.24 : If p = 3 i + 4 j 7 k and q = 6 i + 2 j 3 k then find



p q . Verify that p and p q are perpendicular to each other and also


verify that q and p q are perpendicular to each other.
Solution :

i
j
k

p q = 3 4 7

= 2 i 51 j 30 k
Now


p . p q = 3 i + 4 j 7 k . 2 i 51 j 30 k

) (

) (

= 6 204 + 210 = 0


Hence p and p q are perpendicular to each other.
Now

q . p q = 6 i + 2 j 3 k . 2 i 51 j 30 k

) (

) (

= 12 102 + 90 = 0


Hence q and p q are perpendicular to each other.
Example 2.25 : If the position vectors of three points A, B and C are


respectively i + 2 j + 3 k , 4 i + j + 5 k and 7 i + k . Find AB AC .
Interpret the result geometrically.
Solution :

OA = i + 2 j + 3 k , OB = 4 i + j + 5 k ; OC = 7 i + 7 k

AB = OB OA = 4 i + j + 5 k i + 2 j + 3 k

) (

AB = 3 i j + 2 k

71

AC = OC OA = 6 i 2 j + 4 k

i
j
k

AB AC = 3 1 2 = 0

The vectors AB and AC are parallel. But they have the point A as a
common point.

AB and AC are along the same line.


A, B, C are collinear.
EXERCISE 2.3
(1) Find the magnitude of


a b if a = 2 i + k , b = i + j + k


(2) If a = 3, b = 4 and a . b = 9 then find a b
(3) Find the unit vectors perpendicular to the plane containing the vectors


2 i + j + k and i + 2 j + k
(4) Find the vectors whose length 5 and which are perpendicular to the

vectors a = 3 i + j 4 k and b = 6 i + 5 j 2 k


(5) Find the angle between two vectors a and b if a b = a . b

| |

| |

(6) If

|a | = 2, |b | = 7 and a b = 3i 2j + 6k find the angle

between a and b .


(7) If a = i + 3 j 2 k and b = i + 3 k then find a b . Verify


that a and b are perpendicular to a b separately.

(8) For any three vectors a , b , c show that




a b + c + b c + a + c a + b = 0


(9) Let a , b , c be unit vectors such that a . b = a . c = 0 and the angle


between b and c is 6 . Prove that a = 2 b c

72



(10) If a b = c d and a c = b d ,


show that a d and b c are parallel.

2.4.4 Geometrical applications of cross product :


1
Example 2.26 : Prove that the area of a quadrilateral ABCD is 2 AC BD
where AC and BD are its diagonals.
Solution :
Vector Area of

quadrilateral ABCD = Vector area of ABC + Vector area of ACD


1 1
C
= 2 AB AC + 2 AC AD
D
1 1
= 2 AC AB + 2 AC AD
1
= 2 AC AB + AD
A
B
1
= 2 AC BA + AD
Fig. 2.17
1
= 2 AC BD
1
The area of the quadrilateral ABCD = 2 AC BD
Deduction :
1


Area of a parallelogram = 2
d 1 d 2, where d 1 and d 2 are the
diagonals.
Example 2.27 :

If a , b , c are the position vectors of the vertices A, B, C of a
triangle ABC, then prove that the area of triangle ABC is
1

2 a b + b c + c a Deduce the condition for points a , b , c to be
collinear.

73

1
Solution : Area of ABC =2 AB AC

Now
AB = OB OA = b a

and
AC = OC OA = c a
1 1

Hence, area of ABC = 2 AB AC = 2 b a c a
1
=2 b c b a a c + a a
1
=2 b c + a b + c a
1
Area of ABC = 2 a b + b c + c a
If the points A, B, C are collinear, then the area of ABC = 0
1
2 a b + b c + c a =0

) (

|
|
|

)
|

|
|

|
|

|a b + b c + c a| = 0




(or) a b + b c + c a = 0



Thus a b + b c + c a = 0 is the required condition of

collinearity of the points with positions a , b , c .
b
c
a
Example 2.28 : With usual notation prove that sin A = sin B = sin C

Solution :Let BC = a , CA = b , AB = c
1 1 1
By the area property of triangles 2 a b = 2 b c = 2 c a




a b = b c = c a
ab sin (C) = bc sin(A) = ca sin (B)

| |

| |

ab sinC = bc sinA = ca sinB

-A

Divide by abc
sin B
sinC sinA
c = a = b

c
B

-B

A
b
-C
a

Fig. 2.18

74

a
b
c
sin A =sin B = sin C
Example 2.29 : Prove that sin(A + B) = sinA cosB + cosA sinB
Solution :
Y (Co
sA
Take the points P and Q on the
P , S in
A)
unit circle with centre at the origin
O. Assume that OP and OQ make
angles A and B with x-axis
A M N
O
B
respectively.
Take the reciprocals,

in B

sB

(Co Q

, -S

POQ = POx + QOx = A + B


Fig. 2.19

Clearly the coordinates of P and Q are (cosA, sinA) and (cosB, sinB).

Take the unit vectors i and j along x and y axes respectively.

OP = OM + MP = cos A i + sin A j

OQ = ON + NQ = cos B i + sin B j

( )

NQ = sinB

= cos B i sin B j

OQ OP = OQ OP sin (A + B) k = sin (A + B) k

OQ OP =

i
cosB
cosA

j
sinB
sinA

(1)


0 = k [sinA cosB + cosA sinB]
0

(2)

From (1) and (2)


sin (A + B) = sinA cosB + cosA sinB
Example 2.30 : Show that the area of a parallelogram having diagonals

3 i + j 2 k and i 3 j + 4 k is 5 3.

Solution : Let d 1= 3 i + j 2 k and d 2 = i 3 j + 4 k


1
Area of the parallelogram = 2
d 1 d 2

75


d1 d2 = 3
1


d 1 d 2 =
=


j
k
1
3


2 = 2 i 14 j 10 k
4

( 2)2 + ( 14)2 + ( 10)2


300 = 10 3

1 1
Area of the parallelogram = 2
d 1 d 2= 2 10 3 = 5 3 sq. units

2.4.5 Applications of Vector Product in Physics


The moment of a force about a point :

Let a force F be applied at a point


r x F
P of a rigid body. Then the moment of
O

force F about a point O measures the

r
tendency (amount) of F to turn the
body about point O. If this tendency of

F
P
N
rotation about O is in anti-clockwise

Fig. 2.20
direction the moment is positive,
otherwise it is negative.

Let F be the force and P be a point on the line of action of F . Let r be


the position vector of P relative to O.

The magnitude of the moment of the force F about O is the product of the

magnitude of F and the length of the perpendicular from O to the line of action
of the force.

Magnitude of the moment = F (ON)
In right angled triangle ONP
ON ON
sin = OP =

| |

|r | sin = ON
76


Magnitude of the moment = F (ON)

= r F sin

| |


= r F

Moment (or) Torque of force F about the point O is defined as the



vector M = r F
Example 2.31 :

A force given by 3 i + 2 j 4 k is applied at the point


(1, 1, 2). Find the moment of the force about the point (2, 1, 3).
Solution :
We have
A(2,-1,3)

r
F = 3 i +2 j 4k

P(1,-1,2)
OP = i j + 2 k

F
OA = 2 i j + 3 k
Fig. 2.21

r = AP = OP OA
=

(i j + 2k ) (2i j + 3k )


r =i k

The moment M of the force F about the point A is given by



i
j
k

M = r F = 1 0 1 =2 i 7 j 2k

EXERCISE 2.4
(1) Find the area of parallelogram ABCD whose vertices are
A( 5, 2, 5), B( 3, 6, 7), C(4, 1, 5) and D(2, 5, 3)
(2) Find the area of the parallelogram whose diagonals are represented by

2 i + 3 j + 6 k and 3 i 6 j + 2 k
77

(3) Find the area of the parallelogram determined by the sides


i + 2 j + 3 k and 3 i 2 j + k
(4) Find the area of the triangle whose vertices are (3, 1, 2), (1, 1, 3)
and (4, 3, 1)
(5) Prove by vector method that the parallelograms on the same base and
between the same parallels are equal in area.
(6) Prove that twice the area of a parallelogram is equal to the area of another
parallelogram formed by taking as its adjacent sides the diagonals of the
former parallelogram.
(7) Prove that sin (A B) = sin A cos B cos A sin B.


(8) Forces 2 i + 7 j , 2 i 5 j + 6 k , i + 2 j k act at a point P

whose position vector is 4 i 3 j 2 k . Find the moment of the


resultant of three forces acting at P about the point Q whose position

vector is 6 i + j 3 k .


(9) Show that torque about the point A(3, 1, 3) of a force 4 i + 2 j + k

through the point B(5, 2, 4) is i + 2 j 8 k .


(10) Find the magnitude and direction cosines of the moment about the point

(1, 2, 3) of a force 2 i + 3 j + 6 k whose line of action passes through


the origin.

2.5 Product of three vectors :



Let a , b , c be three vectors. By inserting dot and cross between

a , b , c in the same alphabetical order we introduce the following :

(a . b ) . c , (a . b ) c , (a b ) . c and (a b ) c

Consider ( a . b ) . c

Here a . b is a scalar quantity and dot product is not defined between a

scalar and vector quantity. Therefore a . b . c is not meaningful.

Similary

(a . b ) c is not meaningful.
78

But a b . c is meaningful, because a b is a vector and



a b . c is the dot product between the vectors a b and c .

(
)

Similarly

(a b ) c is meaningful.

2.5.1 Scalar Triple Product :




Let a , b , c be three vectors. Then the product a b . c is called a
scalar triple product.

Geometrical Interpretation of Scalar Triple Product :



Let a , b , c be three non-coplanar vectors. Consider a parallelopiped


having co-terminus edges OA, OB and OC such that OA = a , OB = b and

OC = c .
G
F
a x b

Then a b is a vector
perpendicular to the plane
E
C

B
c
containing a and b .
D

b
L
Let be the angle between
O
A


a
c and a b .
Fig. 2.22
Let CL be perpendicular to the base OADB. Here CL is the height of the
parallelopiped.

Here CL and a b are perpendicular to the same plane

CL is parallel to a b . OCL =

In right angled triangle OLC, CL = c cos

| |

Height of the paralleopiped CL = | c | cos

Area of the parallelogram

with a and b as adjacent sides

Base area of the parallelopiped =


Base area of the parallelopiped = a b

79

(a b ) . c = |a b | |c | cos

Now,

= [base area] [height]


Volume of the parallelopiped with
a b . c =

co-terminous edges a , b , c

Thus, the scalar triple product

(a b ) . c represents the volume of the


paralleopiped whose co-terminous edges a , b , c form a right handed system
of vectors.

2.5.2 Properties of Scalar Triple Product :


Property (1) :

(a b ) . c = (b c ) . a = (c a ) . b

[Cyclic order]

Proof :

Let a , b , c represent the co-terminous edges of a parallelopiped such
that they form a right handed system. Then the volume V of the parallelopiped

is given by V = a b . c



Clearly b , c , a as well as c , a , b form a right handed system of
vectors and represent the co-terminous edges of the same parallelopiped.


V = b c . a and V = c a . b

(
)
(
)



V = ( a b ) . c = ( b c ) . a = ( c a ) . b

(1)
Since dot product is commutative (1) gives



V= c . a b = a. b c = b . c a
(2)
From (1) and (2)

a b . c = a . b c
The dot and cross are interchangeable in a scalar triple product.
In view of this property, the scalar triple product is written in the following
notation.


a b . c = a . b c = a b c

(
)
(
) [
]



[a , b , c] = [b c a]=[c a b]
80

Property (2) :
The change of cyclic order of vectors in scalar triple product changes the
sign of the scalar triple product but not the magnitude.




(i.e.) a b c = b a c = c b a = a c b
Proof :


a b c = a b . c
We have

] [
[

] [

] (

] [

(b a ) . c Q a b = b a
[a , b , c ] = [b a c ]
(1)
=

Similarly we can prove other results.


Property (3) : The scalar triple product of three vector is zero if any two of
them are equal.

Proof : Let a , b , c be three vectors.

When a = b ,

[a b c ] = (a b ) . c = (b b ).c


= o . c =0
Q b b= o


Similarly we can prove for b = c and for c = a
Property (4) :



For any three vectors a , b , c and scalar a b c = a b c

Proof :

] [
]
[a b c ] = (a b ) . c = (a b ) . c

= [a b c]

Property (5) :
The scalar triple product of three vectors is zero if any two of them are
parallel or collinear.

Proof : Let a , b , c be three vectors such that a is parallel or collinear to

b . Then a = b for some scalar .

[a b c ] = [b b c ] = (0) = 0
81

Property (6) (without proof) :


The necessary and sufficient condition for three non-zero, non-collinear


vectors a , b , c to be coplanar is a b c = 0


i.e., a , b , c are coplanar

[a b c ] = 0

Note : Three possibilities for [ a b c ] to be zero are

(i) atleast one of the vectors a , b , c is a zero vector.

(ii) any two of the vectors a , b , c are parallel.

(iii) The vector a , b , c are co-planar.
But for cases (i) and (ii), the case (iii) is trivially true.
Result :
Scalar Triple Product in terms of components :

Let a = a1 i + a2 j + a3 k , b = b1 i + b2 j + b3 k ,

c = c1 i + c2 j + c3 k ,
a1 a2 a3

Then
a b c = b1 b2 b3

c1


Proof : We have a b = a1
b
1

c2

j
a2
b2

c3

a3

b3

= (a2b3 a3b2) i (a1b3 a3b1) j + (a1b2 a2b1) k

[a , b , c ] = (a b ) . c = (a b ) . c1i + c2j + c3k


= (a2b3 a3b2) c1 (a1b3 a3b1)c2 + (a1b2 a2b1)c3
a1 a2 a3
=

b1
c1

c3

b2 b3
c2

82

Distributivity of Cross product over Vector addition :


Result :

For any three vectors a , b , c



we have a b + c = a b + a c
This can be proved by determinant form of cross product.

Example 2.32 : If the edges a = 3 i + 7 j + 5 k , b = 5 i + 7 j 3 k

c = 7 i 5 j 3 k meet at a vertex, find the volume of the


parallelopiped.
Solution :

Volume of the parallelopiped =

[a , b , c ]

3
= 5
7

7
7

= 264
3
5

5
The volume cannot be negative
Volume of parallelopiped = 264 cu. units.
Note : Box product may be negative.

Example 2.33 : For any three vectors a b c prove that

[a + b , b + c , c + a ] = 2 [a b c ]

Solution : [ a + b , b + c , c + a ]



= ( a + b ) ( b + c ) . ( c + a )




= ( a b ) + ( a c ) + ( b b ) + ( b c ) . ( c + a )




= {a b + a c + b c}.(c + a)


= ( a b ) . c + ( a c ) . c + ( b c ). c



+ ( a b ) . a + ( a c ) . a + ( b c ). a


= [a b c]+[b c a] = 2[a b c]
83


Example 2.34 : If x . a = 0, x . b = 0, x . c = 0 and x 0 then show

that a , b , c are coplanar.
Solution :

x . a = 0 and x . b = 0 implies a and b are r to x


a b is parallel to x

x = a b


Now x . c = 0 a b . c = 0

a , b , c are coplanar

[a b c ] = 0

2.5.3 Vector Triple Product :


Definition :

Let a , b , c be any three vectors, then the product a b c and



a b c are called vector triple products of a , b , c
Result :

For any three vectors a , b , c



a b c= a . c b b . c a

This result can be proved by taking a = a1 i + a2 j + a3 k ;

b = b1 i + b2 j + b3 k ; c = c1 i + c2 j + c3 k
Property (1) :

The vector triple product a b c is a linear combination of those


two vectors which are within brackets.
Property (2) :

The vector triple product a b c is perpendicular to c and lies in

the plane which contains a and b .

Property (3) :

(b c ) = (a . c )b (a . b )c
84

Example 2.35 : If a b c = a b c then

) (
)


Prove that ( c a ) b = o

a (b c) = (a b) c
Proof : Given :
(a . c ) b (a . b ) c = (a . c ) b (b .c ) a


(a . b)c = (b.c) a


(c . b)a (a . b)c = 0
(c a ) b = 0

Example 2.36 : If a = 3 i + 2 j 4 k , b = 5 i 3 j + 6 k ,

c = 5 i j + 2 k , find (i) a b c
(ii) a b c
and show that they are not equal.
Solution :

i
j
k


b c = 5 3 6 = 20 j + 10 k
(i)

(ii)

5 1 2

i j
k

(
)
a b c = 3 2 4 = 100 i 30 j + 60 k
0 20 10

i
j
k


a b = 3 2 4 = 38 j 19 k
5 3 6

i
j
k

(a b ) c = 0 38 19 = 95i 95j + 190k


5 1 2


From (i) and (ii) a b c

) (a b ) c
85

2.5.4 Vector product of four vectors ;



For the four vectors a , b , c ,


a b and c d namely
product of four vectors.

Example 2.37 : Let a , b , c and d

(i)

d the vector product of the two vectors




a b c d is called vector

) (

be any four vectors then

(a b ) (c d ) = [a b d ] c [a b c ] d
(a b ) (c d ) = [a c d ] b [b c d ] a

(ii)
Solution :
(i)

(a b ) (c d ) = x (c d ) where x = a b


= (x . d) c (x . c) d


= ( a b ) . d c ( a b ) . c d


= [a b d] c [a b c] d


Similarly we can prove other result by taking x = c d

Note : (1) If the four vectors a , b , c , d are coplanar then

a b c d = o .


(2) Let a , b be lie on one plane and c , d lie on another plane.

These planes are perpendicular then a b . c d = o


Example 2.38 :

2
Prove that a b , b c , c a = a , b , c

) (

] [

) (

Solution :

[a b , b c , c a ] = (a b ) (b c ) . (c a )



= [ a b c ] b [ a b b ] c . ( c a )

= [ a b c ] b . ( c a ) since [ a b b ] = 0
86

[a b c ] [b c a ]
2


= [ a b c ] since [ b c a ] = [ a b c ]
=

2.5.5 Scalar product of four vectors :



For four vectors a , b , c , d the scalar product of the two vectors


namely a b and c d is called scalar product of four vectors.


i.e. a b . c d


Result : Determinant form of a b . c d

a.c
a.d


a b . c d =
i.e.

b.c b.d
Proof :




a b . c d = a b . x where x = c d

(interchange dot and cross)
= a. b x


= a .b c d


= a . b . d c b . c d




= b . d
a . c b . c
a . d

a.c a.d

=

b.c b.d

) (

)
(

) (

) (

) (

) (

)
( ) (
( )( ) (

)(

EXERCISE 2.5

(1) Show that vectors a , b , c are coplanar if and only if

a + b , b + c , c + a are coplanar.
(2) The volume of a parallelopiped whose edges are represented by

12 i + k , 3 j k , 2 i + j 15 k is 546. Find the value of .




(3) Prove that a b c = abc if and only if a , b , c are mutually
perpendicular.

87

(4) Show that the points (1, 3, 1), (1, 1, 1), ( 1, 1, 1) (2, 2, 1) are lying
on the same plane. (Hint : It is enough to prove any three vectors formed
by these four points are coplanar).

(5) If a = 2 i + 3 j k ,
b =2 i +5k, c = j 3k



Verify that a b c = a . c b a . b c

(6) Prove that a b c + b c a + c a b = o

(7) If a = 2 i + 3 j 5 k , b = i + j + 2 k and




c = 4 i 2 j + 3 k , show that a b c a b c


(8) Prove that a b c = a b c iff a and c are collinear.


(where vector triple product is non-zero).

(9) For any vector a

prove that i a i + j a j + k a k = 2 a

(
(

)(
)

)
(

(
(

)
)

( ) ( ) ( ) ( ) ( ) ( )



Find ( a b ) . ( c d ) if a = i + j + k

(10) Prove that a b . c d + b c . a d + c a . b d = 0


(11)

b =2 i + k, c =2 i + j + k, d = i + j +2k




(12) Verify a b c d = a b d c a b c

for a , b , c and d in problem 11.

) (

) [

] d

2.6 Lines :
2.6.1 Equation of a line :
Parametric and non parametric vector equations :

Let P be an any point with position vector r on the given line. A relation

satisfied by r for all points on the line is then found using certain conditions.
This relation is called the vector equation of the line.
Parametric vector equations :

If r is expressed in terms of some fixed vectors and variable scalars,


(parameters) the relation is then called a parametric vector equation.

88

Non-parametric vector equation : If no parameter is involved, the equation is


called a non-parametric vector equation.
Vector and Cartesian Equations of Straight lines :
A straight line is uniquely determined in space if
(i) a point on it and its direction are given
(ii) two points on it are given.
Note : Eventhough the syllabus does not require the derivations
(2.6.2, 2.6.3) and it needs only the results, the equations are
derived for better understanding the results.

2.6.2 Equation of a straight line passing through a given point and


parallel to a given vector :
Vector form :

Let the line pass through a given point A whose position vector is a w.r.to

O and parallel to the given vector v . Let P be any point on the line and its

position vector w.r.to O be r .



z
We have OA = a , OP = r
v
A

AP and v are parallel.


P
a

r
AP = t v for some scalar t
y
O

OP = OA + AP
x
Fig. 2.23

(1)
r = a +tv
This represents the vector equation of the given straight line.

Note : r = a + t v , where t is a variable scalar (i.e., a parameter) is the


parametric vector equation of the line.
Corollary : If the straight line is given to be passing through the origin, then

the equation (1) becomes r = t v
Cartesian form : Let the co-ordinates of the fixed point A be (x1, y1, z1) and the
direction ratios of the parallel vector be l, m, n. Then

a = x1 i + y1 j + z1 k ; v = l i + m j + n k


r = x i +y j +zk

89

r = a +t v x i + y j + z k =x1 i + y1 j + z1 k + t l i + m j + n k

Equating the coefficients of i , j , k we get
x = x1 + tl
These are the
y = y1 + tm parametric equations
of the line
z = z + tn

y y1
z z1
=
t,
m
n = t
x x1
y y1
z z1
Eliminating t, we get
=
=
l
m
n
This is the cartesian equation of the line passing through a point (x1, y1, z1)
and parallel to a vector whose drs are l, m, n.
Non-parametric vector equation :

AP = OP OA = r a


But AP | | v AP v = 0

r a v = 0

r v a v = 0

r v = a v
This is the non-parametric vector equation of the line.
x x1
= t,

2.6.3 Equation of a straight line passing through two given points:


Vector Form :
Let the line pass through two given points A and B whose position vectors

are a and b respectively.


z
Let P be any point on the line and its
A

position vector be r
B
b
P
a
We have
r


y
O
OA = a , OB = b and OP = r

x
AP and AB are parallel vectors.

Fig. 2.24
AP = t AB for some scalar t

90



= t OB OA = t b a

OP = OA + AP


r = a +t b a
(or)
(1)


r = (1 t) a + t b
This represents the vector equation of the given straight line.


Note : r = (1 t) a + t b where t is a variable scalar (i.e., a parameter) is the
parametric vector equation of the required line.
Cartesian form :
Let the co-ordinates of the fixed points A be (x1, y1, z1) and B be (x2, y2, z2)

a = x1 i + y1 j + z1 k ; b = x2 i + y2 j + z2 k ; r = x i + y j + z k
Substituting these values in equation (1) we get

x i + y j + z k = x1 i + y1 j + z1 k

+ tx2 i + y2 j + z2 k x1 i + y1 j + z1 k

Equating the coefficients of i , j , k
x = x1 + t(x2 x1)
These are the
parametric equations
y = y1 + t(y2 y1)
of the line
z = z1 + t(z2 z1)

x x1
= t,
x2 x1

y y1
= t,
y2 y1

z z1
= t
z2 z1

Eliminating t, we get
x x1
y y1
z z1
=
=
x2 x1
y2 y1
z2 z1
This is the cartesian equation of the required line.
Note : x2 x1, y2 y1, z2 z1 are the d.r.s of the line joining the points
(x1, y1, z1) and (x2, y2, z2)
Non-parametric vector equation :


AB = OB OA = b a

91

AP = OP OA = r a

Since AP and AB are collinear vectors



AP AB = 0

r a b a = 0

This is the non parametric vector equation.

) (

2.6.4 Angle between two lines :


Let r = a 1 + t u and r = a 2 + s v
be the two lines in space.
These two lines are in the direction of

u and v .
Angle between the two lines is
defined as the angle between their
directions.

a1 +
r =

t u
u

r =a
2 + s v

Fig. 2.25

u . v
If is the angle between the given lines then = cos

u v
1

| || |

Cartesian form : If the equations of the lines are in Cartesian form


x x1
y y1 z z1
x x1
y y1 z z1
=
=
and
=
a1
b1
c1
a2
b2 = c2
Where a1, b1, c1 and a2, b2, c2 are the direction ratios of two lines, then
angle between them is
a1a2 + b1b2 + c1c2

= cos1
2
2
2
2
2
2
a2 + b2 + c2
a1 + b1 + c1
Note : When two lines are perpendicular then a1a2 + b1b2 + c1c2 = 0
Example 2.39 : Find the vector and cartesian equations of the straight line

passing through the point A with position vector 3 i j + 4 k and parallel to

the vector 5 i + 7 j + 3 k
Solution : We know that vector equation of the line through the point with

position vector a and parallel to v is given by r = a + t v where t is a


scalar.

92

Here a = 3 i j + 4 k

v = 5 i +7 j +3k
Vector equation of the line is

r = 3 i j +4k +t 5 i +7 j +3k
(1)
The cartesian equation of the line passing through (x1, y1, z1) and parallel
to a vector whose d.r.s are l, m, n is
x x1
y y1 z z1
=
l
m = n
Here
(x1, y1, z1) = (3, 1, 4)

) (

(l, m, n) = ( 5, 7, 3)
x3 y+1 z4
= 7 = 3
The required equation is
5
Example 2.40 : Find the vector and cartesian equations of the straight line
passing through the points ( 5, 2, 3) and (4, 3, 6)
Solution : Vector equation of the straight line passing through two points with

position vectors a and b is given by




r = a +t b a

a = 5 i +2 j +3k
Here

b = 4 i 3 j +6k

b a = 9 i 5 j +3k
Vector equation of the line is

r = 5 i + 2 j + 3 k + t 9 i 5 j + 3 k or

r = (1 t) 5 i + 2 j + 3 k + t 4 i 3 j + 6 k
Cartesian Form :
The required equation is
x x1
y y1
z z1
=
=
x2 x1
y2 y1
z2 z1

) (

) (

(x1, y1, z1) = ( 5, 2, 3) ; (x2, y2, z2) = (4, 3, 6)


x+5
y2
z3
9 =
= 3 is the cartesian equation of the line
5

Here

93

Example 2.41 : Find the angle between the lines


r = 3 i + 2 j k + t i + 2 j + 2 k and

r =5 j +2k +s 3 i +2 j +6k

Solution : Let the given lines be in the direction of u and v


Then u = i + 2 j + 2 k , v = 3 i + 2 j + 6 k
Let be the angle between the given lines

u . v

cos =

u v

| || |

u . v = 19 ; | u | = 3 ; | v | = 7
19
cos = 21

19
= cos1 21

EXERCISE 2.6
(1) Find the d.c.s of a vector whose direction ratios are 2, 3, 6.
(2) (i) Can a vector have direction angles 30, 45, 60.
(ii) Can a vector have direction angles 45, 60, 120?
(3) What are the d.c.s of the vector equally inclined to the axes?

(4) A vector r has length 35 2 and direction ratios (3, 4, 5) , find the

direction cosines and components of r .


(5) Find direction cosines of the line joining (2, 3, 1) and (3, 1, 2).
(6) Find the vector and cartesian equation of the line through the point

(3, 4, 2) and parallel to the vector 9 i + 6 j + 2 k .


(7) Find the vector and cartesian equation of the line joining the points
(1, 2, 1) and (0, 2, 3).
(8) Find the angle between the following lines.
y+1
z4
y+2
z4
x1
2 = 3 = 6 and x + 1 = 2 = 2
(9) Find the angle between the lines

r =5 i 7 j + i +4 j +2k


r = 2 i + k + (3 i + 4 k )

94

2.6.5 Skew lines


Consider two straight lines in the space. There are three possibilities.
(i) either they are intersecting
(ii) (or) parallel
L1
(iii) (or) neither intersecting nor
A
parallel
Two lines in space which are
d
either intersecting or parallel are
L2
called coplanar lines.
B
i.e., a plane can be defined
passing through (the two lines
completely lie on the plane) two
Fig. 2.26
intersecting lines or through two
parallel lines.
Therefore, two lines lie on the same plane are called coplanar lines.
Two lines L1 and L2 in space, which are neither intersecting nor parallel
are called skew lines. (See Fig. 2.26)
i.e., two lines in space which are not coplanar are called skew lines.
Shortest distance between two lines
(i) Trivially the shortest distance between two intersecting lines is zero.
(ii) Parallel lines
Theorem : (without proof) The distance between two parallel lines


r = a 1 + t u ; r = a 2 + s u is given by
d=


u a 2 a 1

|u |

(iii) Skew lines :


Theorem : (without proof) The distance between the skew lines


r = a 1 + t u ; r = a 2 + s v is given by
d=

a 2 a 1 u v

|u v |
95

Condition for two lines to intersect :


The shortest distance between the intersecting lines

r = a 1 + t u ; r = a 2 + s v is 0

The condition for intersecting is d = 0 a 2 a 1 u v = 0 (or)
x2 x1 y2 y1 z2 z1

l1

m1

n1

l2

m2

n2

=0

if

(x1, y1, z1)and (x2, y2, z2) are the points whose position vectors are a 1 and

a 2 and l1, m1, n1 ; l2, m2, n2 are the d.rs of the vectors u and v

respectively. ( u and v are not parallel)


Example 2.42 : Find the shortest distance between the parallel lines


r = i j + t 2 i j + k and

(
) (
)



r = (2 i + j + k ) + s(2 i j + k )


Solution : Compare the given equations with r = a 1 + t u and r = a 2 + s u ,



a 1 = i j ; a 2 = 2 i + j + k and u = 2 i j + k



a2 a1 = i +2 j + k

u a 2 a 1 =

i
2
1


j
k
1
2

1=
1


u a 2 a 1 = 9 + 1 + 25 =

|u | =

3 i j +5k

35

4+1+1 = 6


The distance between u a 2 a 1
35
=
=
the parallel lines

| |

96

Note : If the equations are in the Cartesian form, write in the vector form and
find the distance between them.

Example 2.43 : Show that the two lines r = i j + t 2 i + k and

) (



r = 2 i j +s i + j k are skew lines and find the distance between
them.


Solution : Compare the given equations with r = a 1 + t u and r = a 2 + s v

) (



a 1 = i j ; a 2 = 2 i j and u = 2 i + k ; v = i + j k


a2 a1 = i
1 0
2 0

a 2 a 1 u v =

0
1 =10

1 1 1

They are skew lines.



u v =

i
2
1

|u v | =
Shortest distance between the lines =


j
k
0
1

1 =
1

i +3 j +2k

14

a 2 a 1 u v

|u v |

(1)

1
14
x4 y z+1
x1 y1 z+1
= 0 and 2 = 0 = 3
Example 2.44 : Show that the lines 3 =
1
intersect and hence find the point of intersection.
Solution : The condition for intersecting is
x2 x1 y2 y1 z2 z1
From (1) shortest distance between them is

l1

m1

n1

l2

m2

n2

=0

97

Compare with

x x1
y y1
z z1
x x2 y y2 z z2
=
=
and
l1
m1
n1
l2 = m2 = n2 , we

get
(x1, y1, z1) = (1, 1, 1) ; (x2, y2, z2) = (4, 0, 1)
(l1, m1, n1) = (3, 1, 0) ; (l2, m2, n2) = (2, 0, 3)
The determinant becomes
3 1 0

3 1 0 = 0. Note that u and v are not parallel.

0
The lines are intersecting lines.
Point of intersection :
y1 z+1
x1
= 0 =
Take 3 =
1
Any point on the line is of the form (3 + 1, + 1, 1)
x4
y z+1
Take 2 = 0 = 3 =
Any point on this line is of the form (2 + 4, 0, 3 1)
Since they are intersecting, for some ,
(3 + 1, + 1, 1) = (2 + 4, 0, 3 1) = 1 and = 0
To find the point of intersection either take = 1 or = 0
The point of intersection is (4, 0, 1).
Note : If the two lines are in the vector form convert into cartesian form and do
it.
Example 2.45 : Find the shortest distance between the skew lines



r = i j + 2 i + j + k

(
) (
) and

r = ( i + j k ) + (2 i j k )
Solution :

Compare the given equation with r = a 1 + t u and r = a 2 + s v ,




a1= i j ; a2= i + j k ; u =2 i + j + k ;

v=2 i j k

98

a 2 a 1 = 2 j k and u v = 4 j 4 k

a 2 a 1 u v =

0
2
2

|u v | = 4
distance =

a 2 a 1 u v

|u v |

2
1
1

= 12
1
1

12
3
=
4 2
2

2.6.7 Collinearity of three points :


Theorem (without proof) :

Three points A, B and C with position vectors a , b and c respectively


are collinear if and only if there exists scalars 1, 2, 3, not all zeros such that


1 a + 2 b + 3 c = o and 1 + 2 + 3 = 0
Working rule to find the collinearity :
Write the equation of the line in cartesian form using any two points and
verify the third point.
Note : If the three points are collinear then their position vectors are coplanar,
but the converse need not be true.
Example 2.46 : Show that the points (3, 1, 1), (1, 0, 1) and (5, 2, 1)
are collinear.
Solution :
The equation of the line passing through (3, 1, 1) and (1, 0, 1) is
x3 y+1 z+1
2 = 1 = 0 = (say)
Any point on the line is of the form (2 + 3, 1, 1)
The point (5, 2, 1) is obtained by putting = 1.
The third point lies on the same line. Hence the three points are
collinear.
Note : If the position vectors of the points are given then take the points and do
the problem.

99

Example 2.47 : Find the value of if the points (3, 2, 4), (9, 8, 10) and
(, 4, 6) are collinear.
Solution :
Since the three points are collinear, the position vectors of the points are
coplanar.

Let a = 3 i + 2 j 4 k ; b = 9 i + 8 j 10 k ; c = i + 4 j 6 k
3 2 4

a b c = 9 8 10 = 0

4
12 = 60 = 5

EXERCISE 2.7
(1) Find the shortest distance between the parallel lines
(i)

r = 2 i j k

(ii)

y
z+3
x3
y+1
z1
x1
= 3 = 2 and
= 3 = 2
1
1

(
) + t(i 2j + 3k )

r = ( i 2 j + k ) + s( i 2 j + 3 k )

and

(2) Show that the following two lines are skew lines :

r = 3 i + 5 j + 7 k + t i 2 j + k and

(
) (
)



r = ( i + j + k ) + s( 7 i + 6 j + 7 k )

x1 y+1 z
x2 y1 z1
1 = 1 = 3 and 1 = 2 = 1 intersect
and find their point of intersection.

(3) Show that the lines

(4) Find the shortest distance between the skew lines


and

x6 y7 z4
3 = 1 = 1

x y+9 z2
= 2 = 4
3

(5) Show that (2, 1, 3), (1, 1, 0) and (3, 1, 6) are collinear.
(6) If the points (, 0, 3), (1, 3, 1) and ( 5, 3, 7) are collinear then
find .

100

2.7 Planes :
A plane is defined as a surface such that the line joining of any two points
on it lies completely on the surface.
Vector and Cartesian Equations of the planes in parametric and nonparametric form :
A plane is determined uniquely in the following cases :
(i) Given a point on the plane and a normal to the plane.
(ii) Given a normal to the plane and distance of the plane from the origin.
(iii) Given a point and two parallel vectors to the plane.
(iv) Given two points on it and a line parallel to the plane.
(v) Given three non-collinear points.
(vi) Equation of a plane that contains two given lines.
(vii) Equation of a plane passing through the line of intersection of two
given planes and a given point.
Note : Eventhough the syllabus does not require the derivations
(2.7.1 to 2.7.5) and it needs only the results, the equations are
derived for better understanding the results.

2.7.1 Equation of a plane passing through a given point and


perpendicular to a vector.

<

Vector Form : Let the plane pass through the point A whose position vector be

a w.r.to O and perpendicular to the given vector n .


Let P be any point on the plane and its
z
n

position vector be r . Join AP


A

Here AP is perpendicular to n
P
a

r

AP. n = 0 OP OA. n = 0
O
y


Fig. 2.27
r a .n = 0 r.n = a.n
x
This is the vector equation of the required plane (non parametric form.)
Cartesian Form :

If (x1, y1, z1) are the coordinates of A and a, b, c are the direction ratios of n

then a = x1 i + y1 j + z1 k ; n = a i + b j + c k ; r = x i + y j + z k

101

Now,

(r a ) .n = 0

(x x1) i + (y y1) j + (z z1) k . a i + b j + c k = 0

a(x x1) + b(y y1) + c(z z1) = 0


This is the cartesian equation of the plane (in non-parametric form).
Corollary : The vector equation of the plane passing through the origin and

perpendicular to the vector n is r . n = 0

2.7.2 Equation of the plane when distance from the origin and unit
normal is given :
z

Let p be the length of the


perpendicular ON from the origin O

<

to the given plane. Let n be the unit


N
vector normal to the plane in the
direction O to N.
P
pn
r

Then ON = pn.
O
Let P be any point on the plane

x
and let its position vector be r
Fig. 2.28

(i.e.,) OP = r . Join NP.

NP lies on the plane and ON is perpendicular to the plane




NP . ON = 0 OP ON. ON = 0

(r pn) . pn = 0
i.e.,



r . n pn . n = 0


r .n = p

( n . n = 1)

This is the vector equation of the plane (in non-parametric form).


Cartesian form :

If l, m, n are the direction cosines of n then n = l i + m j + n k


r .n=p x i +y j +zk . l i +m j +nk =p

)(

lx + my + nz = p
This is the cartesian equation of the plane (in non-parametric form).

102


Corollary : If n is a normal vector but not a unit vector,

then n =

= p r . n = p n = q (say)
r .

| |

| |

| |


r . n =q

This is the vector equation of the plane perpendicular to the vector n .


q
The length of the perpendicular from origin to this plane is

| |

2.7.3 Equation of the plane passing through a given point and


parallel to two given vectors :

Let a be the position vector of


the given point A referred to the

origin O. Let u and v be the


given vectors, which are parallel to
the plane.
Let P be any point on the plane

and let its position vector be r



(i.e.,) OP = r .

u
C

P
B

a
O

Fig. 2.29

Through A, draw a lines AB and AC parallel to u and v lying in the




planes such that AB = u and AC = v .

Now AP is coplanar with AB and AC

AP = s AB + t AC where s and t are scalars



= su +tv

OP = OA + AP

r = a+s u +t v
(1)
This is the vector equation of the plane (in parametric form).

103

Cartesian form :

Let a = x1 i + y1 j + z1 k

u = l1 i + m1 j + n1 k ; v = l2 i + m2 j + n2 k
From (1)


r = a +su +tv

x i + y j + z k = x1 i + y1 j + z1 k + sl1 i + m1 j + n1 k

+ t l2 i + m2 j + n2 k

Equating the coefficients i , j , k
x = x1 + sl1 + tl2
y = y1 + sm1 + tm2

These are the


parametric equations
in cartesian form

z = z1 + sn1 + tn2
x x1 = sl1 + tl2
y y1 = sm1 + tm2
z z1 = sn1 + tn2

x x1
Eliminating s and t, we get l1
l2

y y1 z z1
m1

n1

m2

n2

=0

This is the cartesian equation of the required plane (in non-parametric


form).
Non-parametric vector equation


AP, AB and AC are coplanar i.e., the vectors r a , u , v are coplanar

[r a , u , v ] = 0

or

[r u v ] = [a , u , v ]

This is the vector equation of plane in non-parametric form.

104

2.7.4 Equation of the plane passing through two given points and
parallel to a given vector :
Vector Form :

Let a and b be the position


vectors of the points A and B
(respectively) referred to the origin

v
C

O. Let v be the given vector.

The required plane passes


through the points A and B and is
parallel to the vector v.
Let P be any point on the plane

and let its position vector be r



(i.e.,) OP = r .

B
y

O
x

Fig. 2.30


Through A, draw a line AC lying in the plane such that AC = v .

Now AP is coplanar with AB and AC

AP = s AB + t AC where s and t are scalars



= s OB OA + t v = s b a + t v

OP = OA + AP

r = a +s b a +t v

(1)

r = (1 s) a + s b + t v
This is the vector equation of the plane (in parametric form).
Non-parametric vector equation

AP, AB and AC are coplanar i.e., the vectors r a , b a and v are


coplanar

[r a , b a , v ] = 0

This is the required vector equation of plane in non-parametric form.

105

Cartesian form :

Let a = x1 i + y1 j + z1 k ; b = x2 i + y2 j + z2 k


v = l i +m j +nk
; r =x i +y j +zk
From (1)


x i + y j + z k = x1 i + y1 j + z1 k

+ s (x2 x1) i + (y2 y1) j + (z2 z1) k + t l i + m j + n k



Equating the coefficients of i , j , k
x = x1 + s(x2 x1) + tl
These are the
parametric equations
y = y1 + s(y2 y1) + tm
in cartesian form
z = z1 + s(z2 z1) + tn
(x x1) = s(x2 x1) + tl
(y y1) = s(y2 y1) + tm
(z z1) = s(z2 z1) + tn
x x1 y y1 z z1

Eliminating s and t we get x2 x1


l

y2 y1

z2 z1 = 0
n

m
This is the (non-parametric) equation of the plane in cartesian form.
2.7.5 Vector and cartesian equations of the plane passing through three
given non-collinear points.

z
Let a , b and c be the position
C
vectors of the points A, B and C
A
referred to the origin O.
P
The required plane passes
a
r
c
B
through the points A, B and C.
b
O
y
Let P be any point on the plane

x
and let its position vector be r

(i.e.,) OP = r .
Fig. 2.31
Now join AB, AC and AP.

AP is coplanar with AB and AC

106

AP = s AB + t AC where s and t are scalars



= s OB OA + tOC OA
=s

(b a ) + t (c a )


OP = OA + AP




r = a +s b a +t c a

) (

) (or)

(1)


r = (1 s t) a + s b + t c
This is the vector equation of the plane (in parametric form).
Non-parametric vector equation :


AP, AB and AC are coplanar.

AP, AB, AC = 0
(i.e.,)

r a, b a, c a = 0
This is the required vector equation of plane in non-parametric form.
Cartesian form :

Let a = x1 i + y1 j + z1 k ; b = x2 i + y2 j + z3 k ; c = x3 i + y3 j + z3 k


r =x i +y j +zk
From (1)


x i + y j + z k = x1 i + y1 j + z1 k

+ s (x2 x1) i + (y2 y1) j + (z2 z1) k

+ t (x3 x1) i + (y3 y1) j + (z3 z1) k


Equating the coefficients of i , j and k , we get


x = x1 + s(x2 x1) + t(x3 x1)
These are the
parametric equations
y = y1 + s(y2 y1) + t(y3 y1)
in cartesian form
z = z1 + s(z2 z1) + t(z3 z1)

107

x x1

Eliminating s and t we get x2 x1


x x
3 1

y y1

z z1

=0

z3 z1

y2 y1 z2 z1
y3 y1

This is the (non-parametric) equation of the plane in cartesian form.


Example 2.48 : Find the vector and cartesian equation of a plane which is
at a distance of 8 units from the origin and which is normal to the vector

3 i +2 j 2k

Solution : Here p = 8 and n = 3 i + 2 j 2 k

n
3 i +2 j 2k 3 i +2 j 2k

n =
=
=
9+4+4
17

| |

Hence the required vector equation of the plane is



r .n = p

3 i +2 j 2k
r .
=8
17

r . 3 i +2 j 2k

)= 8

17

Cartesian form is (xi +yj + zk). (3i + 2j 2k) = 8 17


3x + 2y 2z = 8 17
Example 2.49 :
The foot of perpendicular drawn from the origin to the plane is (4, 2,
5), find the equation of the plane.
Solution : The required plane passes through the point A(4, 2, 5) and is

perpendicular to OA.


a = 4 i 2 j 5 k and n = OA = 4 i 2 j 5 k

The required equation of the plane is r . n = a . n

r . 4 i 2 j 5k = 4 i 2 j 5k . 4 i 2 j 5k
= 16 + 4 + 25

) (

r . 4 i 2 j 5 k = 45

108

) (

(1)

Cartesian form :

(xi + yj + zk ) . (4i 2j 5k ) = 45
4x 2y 5z = 45
Example 2.50 : Find the vector and cartesian equations of the plane through the
point (2, 1, 3) and parallel to the lines
x2
y1
z3
x1
y+1
z 2
3 = 2 = 4 and 2 = 3 = 2 .
Solution : The required plane passes through A(2, 1, 3) and parallel to

u = 3 i + 2 j 4 k and v = 2 i 3 j + 2 k

The required equation is r = a + s u + t v

r = 2 i j 3 k +s 3 i + 2 j 4 k + t 2 i 3 j + 2 k
Cartesian form :
(x1, y1, z1) is (2, 1, 3) ; (l1, m1, n1) is (3, 2, 4) ; (l2, m2, n2) is (2, 3, 2)

) (

) (

The equation of the plane is

i.e.,

x 2
3
2

y+1 z+3
2

x x1
l1
l2

y y1 z z1
m1

n1

m2

n2

=0

=0

8x + 14y + 13z + 37 = 0
This is the required equation in cartesian form.
Example 2.51 : Find the vector and cartesian equations of the plane passing
through the points ( 1, 1, 1) and (1, 1, 1) and perpendicular to the plane
x + 2y + 2z = 5

Solution : The normal vector to the plane x + 2y + 2z = 5 is i + 2 j + 2 k .


This vector is parallel to the required plane.
The required plane passes through the points ( 1, 1, 1) and (1, 1, 1)

and parallel to the vector i + 2 j + 2 k .

109

Vector equation of the plane :


The vector equation of the plane passing through two given points and
parallel to a vector is


r = (1 s) a + s b + t v where s and t are scalars.

Here a = i + j + k ; b = i j + k ; v = i + 2 j + 2 k


r = (1 s) i + j + k + s i j + k + t i + 2 j + 2 k
This is the required vector equation of the plane.
Cartesian form :
(x1, y1, z1) is ( 1, 1, 1) ; (x2, y2, z2) is (1, 1, 1) ; (l1, m1, n1) is (1, 2, 2)

) (

The equation of the plane is

i.e.,

x + 1
2
1

y1 z1
2

x x1
x2 x1
l1

) (

y y1

z z1

y2 y1 z2 z1
m1

n1

=0

=0

2
2
2x + 2y 3z + 3 = 0
Example 2.52 : Find the vector and cartesian equations of the plane passing
through the points (2, 2, 1), (3, 4, 2) and (7, 0, 6)
Solution : Vector equation of the plane passing through three given noncollinear points is


r = (1 s t) a + s b + t c where s and t are scalars.

Here a = 2 i + 2 j k ; b = 3 i + 4 j + 2 k ; c = 7 i + 6 k

r = (1s t) 2 i + 2 j k + s 3 i + 4 j + 2 k + t 7 i + 6 k
Cartesian equation of the plane :
Here (x1, y1, z1) is (2, 2, 1) ; (x2, y2, z2) is (3, 4, 2) ; (x3, y3, z3) is (7, 0, 6)

) (

The equation of the plane is

x x1
x2 x1
x x
3 1
110

) (

y y1

z z1

=0

z3 z1

y2 y1 z2 z1
y3 y1

i.e.,

x 2
1
5

y2 z+1
2

=0

2
7
5x + 2y 3z = 17
This is the Cartesian equation of the plane.
EXERCISE 2.8
(1) Find the vector and cartesian equations of a plane which is at a

distance of 18 units from the origin and which is normal to the vector 2 i

+7 j +8k
(2) Find the unit normal vectors to the plane 2x y + 2z = 5.
(3) Find the length of the perpendicular from the origin to the plane

r . 3 i + 4 j + 12 k = 26.
(4) The foot of the perpendicular drawn from the origin to a plane is
(8, 4, 3). Find the equation of the plane.
(5) Find the equation of the plane through the point whose p.v. is

2 i j + k and perpendicular to the vector 4 i + 2 j 3 k .


(6) Find the vector and cartesian equations of the plane through the point

(2, 1, 4) and parallel to the plane r . 4 i 12 j 3 k = 7.


(7) Find the vector and cartesian equation of the plane containing the line
z+ 1
x2
y2
z1
x+1
y1
2 = 3 = 3 and parallel to the line
3 = 2 = 1 .
(8) Find the vector and cartesian equation of the plane through the point
(1, 3, 2) and parallel to the lines
x+1
y+2
z+3
x2
y+1
z+2
=
= 3 and 1 = 2 = 2
2
1
(9) Find the vector and cartesian equation to the plane through the point
(1, 3, 2) and perpendicular to the planes x+2y+2z = 5 and 3x+y+2z = 8.
(10) Find the vector and cartesian equation of the plane passing through the
points A(1, 2, 3) and B ( 1, 2, 1) and is parallel to the line
x2
y+1
z1
2 = 3 = 4
(11) Find the vector and cartesian equation of the plane through the points
(1, 2, 3) and (2, 3, 1) perpendicular to the plane 3x 2y + 4z 5 = 0

111

(12) Find the vector and cartesian equation of the plane containing the line
y2
z1
x2
2 = 3 = 2 and passing through the point ( 1, 1, 1).
(13) Find the vector and cartesian equation of the plane passing through the

points with position vectors 3 i + 4 j + 2 k , 2 i 2 j k and



7 i + k.
(14) Derive the equation of the plane in the intercept form.
(15) Find the cartesian form of the following planes :

(i) r = (s 2t) i + (3 t) j + (2s + t) k

(ii) r = (1 + s + t) i + (2 s + t) j + (3 2s +2 t) k

2.7.6 Equation of a plane passing through the line of intersection of


two given planes :
Vector form :
The vector equation of the plane passing through the line of intersection of


the planes r . n 1 = q1 and r . n 2 = q2 is

r . n 1 q 1 + r . n 2 q2 = 0

i.e. r . n 1 + n 2 = q1 + q2
Cartesian form :
The cartesian equation of the plane passing through the line of intersection
of the planes a1x + b1y + c1z+ d1 = 0 and a2x + b2y + c2z+ d2 = 0 is
(a1x + b1y + c1z + d1) + (a2x + b2y + c2z+ d2 ) = 0
Example 2.53 : Find the equation of the plane passing through the line of
intersection of the plane 2x 3y + 4z = 1 and x y = 4 and passing through
the point (1, 1, 1).
Solution :
Any plane through the line of intersection of the given two planes is of the
form (2x 3y + 4z 1) + (x y + 4) = 0
1
But it passes through the point (1, 1, 1). = 2
1
The equation of the required plane is (2x 3y + 4z 1) 2 (x y + 4)= 0
i.e., 3x 5y + 8z 6 = 0

112

Example 2.54 : Find the equation of the plane passing through the intersection
of the planes 2x 8y + 4z = 3 and 3x 5y + 4z + 10 = 0 and perpendicular to
the plane 3x y 2z 4 = 0
Solution :
The equation of the plane passing through the line of intersection of the
given two planes is of the form (2x 8y + 4z 3) + (3x 5y + 4z + 10) = 0
i.e., (2 + 3) x + ( 8 5)y + (4 + 4)z + ( 3 + 10) = 0 . But the required
plane is perpendicular to the plane 3x y 2z 4 = 0
Their normals are perpendicular.
i.e., (2 + 3) 3 + ( 8 5) ( 1) + (4 + 4) ( 2) = 0
6 + 6 = 0

=1

The required equation is (2x 8y + 4z 3) 1(3x 5y + 4z + 10) = 0


x 3y 13 = 0
x + 3y + 13 = 0

2.7.7 The distance between a point and a plane :


Let (x1, y1, z1) be a point and ax + by + cz + d = 0 be the equation of the
ax1 + by1 + cz1 + d
plane. The distance between the point and the plane is

a2 + b2 + c2

Corollary (1) :
The distance between the origin and the plane ax + by + cz + d = 0 is
d

2 2 2
a +b +c
Corollary (2) :
The distance between the two parallel planes ax + by + cz + d1 = 0 and
d1 d2
ax + by + cz + d2 = 0 is 2

a + b2 + c2
Note : If the given equation is in vector form, convert into cartesian form and
find the distance.
Example 2.55 : Find the distance from the point (1, 1, 2) to the plane


+s i j +t j k
r = i + j + k

) (

113

Solution :
The given plane is passing through the point (1, 1, 1) and parallel to two


vectors i j and j k .

The corresponding cartesian equation is of the form


x x1 y y1 z z1
(x1, y1, z1) = (1, 1, 1)
m1
n1
l1
(l1, m1, n1) = (1 1, 0)
=0
(l2, m2, n2) = (0, 1, 1)
m
n
l

2
x 1
i.e., 1
0

y1
1

2
z1

0 =0
1

i.e., x + y + z 3 = 0

1
Here (x1, y1, z1) = (1, 1, 2)
1
ax1 + by1 + cz1 + d 1 1 + 2 3
=
The distance =
= 3
2
2
2
1
+
1
+
1

a +b +c

Find the distance between the parallel


Example 2.56 :



r . i j + k = 3 and r . i + j k = 5
Solution :
The corresponding cartesian equations of the planes are
x y + z 3 = 0 and x + y z 5 = 0
i.e., x + y z + 3 = 0 and x + y z 5 = 0
d 1 d2 3 + 5 8
=
distance = 2
=
3
a + b2 + c2 1 + 1 + 1

planes

2.7.8 Equation of the plane which contain two given lines (i.e. passing
through two given lines)

Let r = a 1 + t u and r = a 2 + s v be the lines, lie on the plane.




Clearly r a 1, u , v are coplanar and r a 2, u , v are also coplanar


Thus r a 1, u , v = 0 and r a 2, u , v = 0
Note that the above two equations represent the same required plane. The
cartesian form is

114

x x1
l1
l2

y y1 z z1
m1

n1

m2

n2

=0

x x2
or l1
l2

y y2 z z2
m1

n1

m2

n2

=0

Where a 1 = x1 i + y1 j + z1 k ; a 2 = x2 i + y2 j + z2 k

u = l1 i + m1 j + n1 k ; v = l2 i + m2 j + n2 k
Note :
(1) If the two lines are parallel then take the two trivial points from the
lines and the parallel vector. Now find the equation of the plane
passing through two points and parallel to a vector.
(2) Through two skew lines, we cant draw a plane.
Example 2.57 : Find the equation of the plane which contains the two lines
x4 y1 z
x1 y2 z3
2 = 3 = 4 and 5 = 2 = 1
Solution :
Take the trivial point from the first line and the two parallel vectors i.e. (x1,
y1, z1) = (1, 2, 3).
(l1, m1, n1) = (2, 3, 4) and (l2, m2, n2) = (5, 2, 1)
The required equation is
x x1 y y1 z z1
x1 y2 z3

l1

m1

n1

l2

m2

n2

=0 2

3
2

4 =0

5x 18y + 11z 2 = 0
Example 2.58 : Find the point of intersection of the line passing through the
two points (1, 1, 1) ; ( 1, 0, 1) and the xy-plane.
Solution :
The equation of the line passing through (1, 1, 1) and ( 1, 0, 1) is
y1
z+1
x 1
2 = 1 = 2
It meets the xy-plane i.e. z = 0
y1
1
1
x1
2 = 1 =
x = 0, y = 2
2
1
The required point is 0, 2, 0

115

Example 2.59 : Find the co-ordinates of the point where the line

(
) + t(2i 3j + 4k ) meets the plane

r .(2 i + 4 j k ) = 3

r = i +2 j 5k

Solution :
The equation of the straight line in the cartesian form is
y2 z+5
x1
2 = 3 = 4 = (say)
Any point on this line is of the form (2 + 1, 3 + 2, 4 5).
The cartesian equation of the plane is 2x + 4y z 3 = 0
But the required point lies on this plane.
2(2 + 1) + 4( 3 + 2) (4 5) 3 = 0 = 1
The required point is (3, 1, 1)
EXERCISE 2.9
(1) Find the equation of the plane which contains the two lines
x4 y1
x+1 y2 z3
2 = 3 = 4 and 3 = 2 = z 8
(2) Can you draw a plane through the given two lines? Justify your answer.

r = i +2 j 4 k +t 2 i +3 j +6 k

(
) (
) and

r = (3 i + 3 j 5 k ) + s(2 i + 3 j + 8 k )
(3) Find the point of intersection of the line

r = j k

) + s(2i j + k ) and xz plane

(4) Find the meeting point of the line


r = 2 i + j 3k

) + t(2i j k ) and the plane

x 2y + 3z + 7 = 0
(5) Find the distance from the origin to the plane

r . 2 i j +5k

) =7

(6) Find the distance between the parallel planes


x y + 3z + 5 = 0 ; 2x 2y + 6z + 7 = 0

116

2.7.9 Angle between two given planes :


The angle between two planes is
defined as the angle between their
normals.


Let the r . n 1 = q1 and r . n 2 = q2
the equations of the given two planes

(where n 1 and n 2 are normals to the


planes.)
Now if be the angle between the two
planes (i.e., between their normals) then

n2

n1

Fig. 2.32


n 1 . n 2
= cos

n 1 n 2
1

Note :


(i) If the two planes are perpendicular then n 1 . n 2 = 0

(ii) If the two planes are parallel then n 1 = t n 2 where t is a scalar.

2.7.10 Angle between a line and a plane


The angle between a line and a plane is
the complement angle between the line
and the normal to the plane.

Let r = a + t b be the line and



r . n = q be the plane.
If is the angle between the line and the
plane then (90 ) is the angle between
the line and the normal to the plane.
i.e., (90 ) is the angle between

b . n
cos (90 ) =
sin =

b n

| || |

90-

Fig. 2.33

b and n

b . n
= sin1

b n

| || |

117


b . n

b n

| || |

Note : If the line is parallel to the plane i.e., the normal to the plane is

perpendicular to the line then b . n = 0
Example 2.60 : Find the angle between 2x y + z = 4 and x + y + 2z = 4
Solution : The normals to the given planes are

n 1 = 2 i j + k and n 2 = i + j + 2 k
Let be the angle between the planes then

n1. n2
3
1

cos =
=
=
=3

6 6 2

n 1 n 2
Example 2.61 : Find the angle between the line

(
) + (2i + j + 2k ) and the plane

r . (3 i 2 j + 6 k ) = 0

r = i +2 j k

Solution : Let be the angle between the line and the plane.

b . n
sin =

b n

| || |

b = 2 i + j +2k ; n =3 i 2 j +6k
16
16
sin =
= sin1 21

3 7
EXERCISE 2.10
(1) Find the angle between the following planes :
(i) 2x + y z = 9 and x + 2y + z = 7
(ii) 2x 3y + 4z = 1 and x + y = 4

(iii) r . 3 i + j k = 7 and r . i + 4 j 2 k = 10
(2) Show that the following planes are at right angles.

r . 2 i j + k = 15 and r . i j 3 k = 3

(3)

(
)


The planes r . (2 i + j 3 k ) = 10 and r . ( i + 3 j + k ) = 5

are perpendicular. Find .

118

(4) Find the angle between the line

y+1
z3
x2
3 = 1 = 2 and the plane

3x + 4y + z + 5 = 0


(5) Find the angle between the line r = i + j + 3 k + 2 i + j k

and the plane r . i + j = 1.

2.8 Sphere :
A sphere is the locus of a point which moves in space in such a way that its
distance from a fixed point remains constant.
The fixed point is called the centre and the constant distance is called the
radius of the sphere.
Note : Eventhough the syllabus does not require the derivations
(2.8.1, 2.8.2) and it needs only the results, the equations are
derived for better understanding the results.

2.8.1 Vector equation of the sphere whose position vector of centre is

c and radius is a.
Let O be the point of reference
(origin) and C be the centre of the

sphere having position vector c



(i.e.,) OC = c
Let P be any point on the sphere

whose position vector be r



(i.e.,) OP = r

a
P

Fig. 2.34


The radius of the sphere is given as a. (i.e.,) CP = a

From the figure (2.34)


OP = OC + CP

r = c + a

r c = a

r c = a
(1)
This is the vector equation of the sphere.
Corollary : Vector equation of a sphere whose centre is origin and radius is a.

| | |

119


When O coincides with the centre C then c = o and the vector equation

of the sphere (1) becomes r = a


Cartesian form :


Let r = x i + y j + z k

c = c1 i + c2 j + c3 k

| | | |


r c = (x c1) i + (y c2) j + (z c3) k
2

But
2

|r c |

= a2

(2)
2

(3)
From (2)
(x c1) + (y c2) + (z c3) = a
This is the cartesian equation of the sphere whose centre is (c1, c2, c3) and
raidus is a.
Corollary : If the centre is at the origin, then the equation (3) takes the form
x2 + y2 + z2 = a2.
This is known as the standard form of the equation of the sphere.
Note : General Equation of a Sphere :
The equation x2 + y2 + z2 + 2ux + 2vy + 2wz + d = 0 represents a sphere
with centre ( u, v, w) and the radius =
Note :

u2 + v2 + w2 d

(i) the coefficients of x2, y2, z2 are equal.


(ii) The equation does not contain the terms of xy, yz and zx.

2.8.2 Vector and Cartesian equations of the sphere when the


extremities of the diameter being given :
P

Let C be the centre of the sphere.


Let A and B be the end points of the
diameter AB.

Let a be the position vector of

the point A and b be the position


vector of the point B with reference
to the origin O.

r
b
O

Fig. 2.35

120



(i.e.,) OA = a and OB = b

Let P be any point on the surface of the sphere. Let r be the position

vector of P. (i.e.,) OP = r



AP = OP OA = r a



BP = OP OB = r b
We know that the diameter AB subtends a right angle at P.

AP BP

AP . BP = 0

(r a ) . (r b ) = 0

(1)

which is the required equation of the sphere.


Cartesian Form :
Let A(x1, y1, z1) and B(x2, y2, z2) be the end points of the diameter AB.
Let P (x, y, z) be any point on the surface of the sphere.


Now
a = OA = x1 i + y1 j + z1 k ; b = OB = x2 i + y2 j + z2 k



r = OP = x i + y j + z k

(r a ) . (r b ) = 0
( ) ( )
x i +y j + z k (x1 i +y1 j +z1 k ) . x i +y j +z k (x2 i +y2 j + z2 k ) = 0
From (1)

(x x )

1 i + (y y1) j + (z z1) k . (x x2) i + (y y2) j + (z z2) k =0


(x x1) (x x2) + (y y1) (y y2) + (z z1) (z z2) = 0
This is required cartesian form of the equation of the sphere in terms of the
end points of the diameter.
Example 2.62 : Find the vector and cartesian equations of the sphere whose

centre is 2 i j + 2 k and radius is 3.

121

Solution : We know that the vector equation of the sphere with centre and
radius is

r c =a

Here
c = 2 i j + 2 k and a = 3

The required vector equation is r 2 i j + 2 k = 3


Cartesian equation :


Putting r = x i + y j + z k we get



x i +y j +zk 2 i j +2k = 3

) (

|(x 2)i + (y + 1)j + (z 2)k | = 3


2
|(x 2)i + (y + 1)j + (z 2)k | = 32

(x 2)2 + (y + 1)2 + (z 2)2 = 9


x2 + y2 + z2 4x + 2y 4z = 0
Example 2.63 : Find the vector and cartesian equation of the sphere whose
centre is (1, 2, 3) and which passes through the point (5, 5, 3).
(5 1)2 + (5 2)2 + (3 3)2
16 + 9 = 25 = 5

Here a = 5 and c = i + 2 j + 3 k
Vector equation of the sphere is

r c =a

Solution :

Radius =
=

r i +2 j +3k = 5


Cartesian Equation : Let r = x i + y j + z k
From (1)



x i +y j +zk i +2 j +3k = 5

) (

|(x 1)i + (y 2)j + (z 3)k | = 5


(x 1)2 + (y 2)2 + (z 3)2 = 25
x2 + y2 + z2 2x 4y 6z 11 = 0

122

(1)

Example 2.64 : Find the equation of the sphere on the join of the points A and B

having position vectors 2 i + 6 j 7 k and 2 i 4 j + 3 k respectively as a


diameter.
Solution : Vector equation of the sphere is

(r a ) . (r b ) = 0

a = 2 i + 6 j 7 k and b = 2 i 4 j + 3 k


Let
r = x i +y j +zk
The required equation is



x i +y j + z k 2 i +6 j 7 k . x i +y j + z k 2 i 4 j +3 k = 0
Here

(
)(
) (
)(
)
[(x 2)i + (y 6)j + (z+7)k ] . [(x 2)i +(y+4)j +(z3)k ] = 0 (1)
Cartesian Equation :
From (1)
(x 2) (x 2) + (y 6) (y + 4) + (z + 7) (z 3) = 0
x2 + y2 + z2 4x 2y + 4z 41 = 0
Example 2.65 : Find the coordinates of the centre and the radius of the sphere
2

whose vector equation is r r . 8 i 6 j + 10 k 50 = 0


Solution : Let r = x i + y j + z k
2

r r . 8 i 6 j + 10 k 50 = 0
2
r = x2 + y2 + z2
x2 + y2 + z2 8x + 6y 10z 50 = 0
Here
2u = coefficient of x = 8 u = 4
2v = coefficient of y = 6 v = 3
2w = coefficient of z = 10 w = 5
Centre : ( u, v, w) = (4, 3, 5)

Radius : u2 + v2 + w2 d =
Example 2.66 :

16 + 9 + 25 + 50 = 100 = 10 uts.

Chord AB is a diameter of the sphere r 2 i + j 6 k = 18 with


coordinate of A as (3, 2, 2) Find the coordinates B.

123

Solution : The equation of the sphere is r 2 i + j 6 k = 18


Centre of the sphere is (2, 1, 6)

(i.e.,) Position vector of the centre is 2 i + j 6 k


We know that
Centre is the mid point of diameter AB
The co-ordinates of A are (3, 2, 2) and let the coordinates of B be , , )
+3 +2 2
(2, 1, 6) = 2 , 2 , 2

= 1, = 0, = 10
Coordinates of B are (1, 0, 10)

EXERCISE 2.11
(1) Find the vector equation of a sphere with centre having position vector

2 i j + 3 k and radius 4 units. Also find the equation in cartesian


form.
(2) Find the vector and cartesian equation of the sphere on the

join of the points A and B having position vectors 2 i + 6 j 7 k and

2 i + 4 j 3 k respectively as a diameter. Find also the centre and


radius of the sphere.
(3) Obtain the vector and cartesian equation of the sphere whose centre
is (1, 1, 1) and radius is the same as that of the sphere

r i + j + 2 k = 5.
(4) If A ( 1, 4, 3) is one end of a diameter AB of the sphere

x2 + y2 + z2 3x 2y + 2z 15 = 0, then find the coordinates of B.


(5) Find the centre and radius of each of the following spheres.

(i) r 2 i j + 4 k = 5

(
)



(ii) 2 r + (3 i j + 4 k ) = 4

(iii) x2 + y2 + z2 + 4x 8y + 2z = 5


(iv) r 2 r . 4 i + 2 j 6 k 11 = 0
(6) Show that diameter of a sphere subtends a right angle at a point on the
surface.

124

3. COMPLEX NUMBERS
3.1 Introduction :
The number system that we are aware of today is the gradual development
from natural numbers to integers, from integers to rational numbers and from
rational numbers to the real numbers.
If we consider the following polynomial equations (i) x 1 = 0,
(ii) x + 1 = 0, (iii) x + 1 = 1, (iv) 2x + 1 = 0 and (v) x2 3 = 0, we see that all
of them have solutions in the real number system. However this real number
system is not sufficient to solve equations of the form x2 + 9 = 0 i.e., there does
not exist any real number which satisfies x2 = 9. The mathematical need to
have solutions for equations of the above form led us to extend the real number
system to a new kind of number system that allows the square root of negative
numbers.
Let us consider solution of a simple quadratic equation x2 + 16 = 0. Its
solutions are x = 4 1 . We assume that square root of 1 is denoted by the
symbol i, called the imaginary unit. Thus for any two real numbers a and b, we
can form a new number a + ib. This number a + ib is called a complex number.
The set of all complex numbers is denoted by and the nomenclature of a
complex number was introduced by C.F. Gauss, a German mathematician.
Hence the extension of the concept of numbers from real numbers enables one
to solve any polynomial equation. The symbol i was first introduced in
mathematics by the famous Swiss mathematician, Leonhard Euler
(1707 1783) in 1748. i is the first letter of the Latin word imaginarius and
it is also referred to as iota, a Greek alphabet. Later on the subject was
enriched by the original work of A.L. Cauchy, B. Riemann, K. Weierstrass and
others.

3.2 The complex number system :


A complex number is of the form a + ib where a and b are real numbers
and i is called the imaginary unit, having the property that i2 = 1. If z = a + ib
then a is called the real part of z, denoted by Re(z) and b is called the imaginary
part of z and is denoted by Im(z) .
2
Some examples of complex numbers are 3 i2, 2 + i3, 5 + i.

125

Note that 3 is the real part and 2 is the imaginary part of 3 i2 and so on.
Two complex numbers a + ib and c + id are equal if and only if a = c and b
= d. i.e., the corresponding real parts are equal and the corresponding imaginary
parts are equal. The real numbers can be considered as a subset of the set of
complex numbers with b = 0. Hence the complex numbers 0 + i0 and 2 + i0
represents the real numbers 0 and 2 respectively. If a = 0 the complex number
0 + ib or ib is called a pure imaginary number.
Negative of a complex number :
If z = a + ib is a complex number then the negative of z is denoted by z
and it is defined as z = a + i( b)

Basic Algebraic operations :


(a + ib) + (c + id) = (a + c) + i (b + d)
Addition :
(a + ib) (c + id) = (a c)+ i(b d)
Subtraction :
To perform the operations with complex numbers we can proceed as in the
algebra of real numbers replacing i2 by 1 whenever it occurs.
Multiplication :

(a + ib) (c + id) = ac + iad + ibc + i2bd


= (ac bd) + i (ad + bc)

3.3 Conjugate of a complex number :

If z = a + ib, then the conjugate of z is denoted by z and is defined by

z = a ib
c id
a + ib a + ib
Division :
c + id = c + id c id
Multiplying the numerator and denominator by the conjugate of the
denominator, we get
a + ib ac + bd
bc ad
c + id = c2 + d2 + i c2 + d2

3.3.1 Properties :

(i) zz = (a + ib) (a ib) = a2 + b2 which is a non-negative real number.

=
(ii) Conjugate of z is z i.e., z = z

(iii) If z is real, i.e., b = 0 then z = z . Conversely, if z = z ,


i.e., if a + ib = a ib then b = b 2b = 0 b = 0
( 2 0 in the real number system). b = 0 z is real.
Thus z is real the imaginary part is 0

126

(iv) Let z = a + ib. Then z = a ib

z + z = (a + ib) + (a ib) = 2a

z+ z
a = Re(z) = 2

z z
Similarly, b = Im(z) = 2i
(v) The conjugate of the sum of two complex numbers z1, z2 is the sum of

their conjugates i.e., z1 + z2 = z1 + z2

z1 = a + ib and z2 = c + id

Proof : Let

z1 + z2 = (a + ib) + (c + id) = (a + c) + i (b + d)

Then

z1 + z2 = (a + c) i (b + d)

z1

z1

= a ib, z2 = c id

+ z2 = (a ib) + (c id) = (a + c) i(b + d)

= z1 + z2
Similarly it can be proved that the conjugate of the difference of two
complex numbers z1, z2 is the difference of their conjugates.

i.e., z1 z2 = z1 z2

(vi) The conjugate of the product of two complex numbers z1, z2 is the
product of their conjugates.
i.e.,

z1z2

= z1 z2
Proof : Let z1 = a + ib and z2 = c + id. Then
z1 z2 = (a + ib) (c + id) = (ac bd) + i(ad + bc)

z1z2

z1

z1 z2

= (ac bd) i(ad + bc)

= a ib, z2 = c id
= (a ib) (c id) = (ac bd) i(ad + bc)

= z1z2

127

(vii) The conjugate of the quotient of two complex numbers z1, z2, (z2 0)
is the quotient of their conjugates.

z1 z 1
i.e., z = (without proof)
2 z
2
n

(viii) zn = ( z )
Example 3.1 : Write the following as complex numbers

(i) 35
Solution :
(i)

35 =

(ii) 3
( 1) (35) =

7
1 .

35 = i 35

(ii) 3 7 = 3 ( 1) 7 = 3 1 7 = 3 i 7
Example 3.2 : Write the real and imaginary parts of the following numbers :
3
(ii) 2 i
(i) 4 i 3
Solution :
(i) Let z = 4 i 3 ; Re(z) = 4, Im(z) = 3
3
3
(ii) Let z = 2 i ; Re(z) = 0, Im(z) = 2
Example 3.3 :
Find the complex conjugate of (i) 2 + i 7, (ii) 4 i9 (iii) 5
Solution :
By definition, the complex conjugate is obtained by reversing the sign of
the imaginary part of the complex number. Hence the required conjugates are
(i) 2 i 7, (ii) 4 + i9 and (iii) 5 ( the conjugate of any real number is
itself).
Example 3.4 :
Express the following in the standard form of a + ib
(i) (3 + 2i) + ( 7 i)
(ii) (8 6i) (2i 7)
5 + 5i
(iii) (2 3i) (4 + 2i)
(iv)
3 4i
Solution :
(i) (3 + 2i) + ( 7 i) = 3 + 2i 7 i = 4 + i
(ii) (8 6i) (2i 7) = 8 6i 2i + 7 = 15 8i

128

(iii) (2 3i) (4 + 2i)=8 + 4i 12i 6i2 = 14 8i


(iv)

5 + 5i 5 + 5i 3 + 4i
15 + 20i + 15i + 20i2
=
3 + 4i =
3 4i 3 4i
32 + 42
=

Note :

1 7
5 + 35i
= 5 +5i
25

i4 = 1
i3 = i
i2 = 1
(i)4n = 1

(i)4n 1 = i
(i)4n 2 = 1 ; n z
Example 3.5 : Find the real and imaginary parts of the complex number
3i20 i19
2i 1
Solution :
z=

(i)

3i20 i19
z=
=
2i 1
=
=
=

10

3(i2) (i2) i
2i 1
3( 1)10 ( 1)9i
1 + 2i
3+i
1 + 2i
3+i
1 2i

1 + 2i 1 2i
3 6i i 2i2
( 1)2 + 22
1
7
1 7i
= 5 5 i
=
5
1
7
5 and Im(z) = 5

Re(z) =

3
1
Example 3.6 : If z1 = 2 + i, z2 = 3 2i and z3 = 2 + 2 i
find the conjugate of (i) z1z2

(ii) (z3)4
129

Solution :

(i) Conjugate of z1z2 is z1 z2

i.e. (2 + i) (3 2i) =
=
=

(ii)


(2 + i) (3 2i)
(2 i) (3 + 2i)
(2 i) (3 + 2i)

= 6 + 4i 3i 2i2 = 6 + 4i 3i + 2
= 8+i
4
4
3
1
z34 = z3 = 2 + 2 i

2
1
3
3 3
1
= 2 2 i = 4 + 2 i + 4 i2

3
3
1
3
1
= 2 + 2 i = 4 2 i + 4 i2

3
1
= 2 2 i
EXERCISE 3.1
(1) Express the following in the standard form a + ib
2(i 3)
(1 + i) (1 2i)
(i)
(ii)
2
1 + 3i
(1 + i)
i4 + i9 + i16
3 2i8 i10 i15
(2) Find the real and imaginary parts of the following complex numbers:
2 + 5i
1
(ii)
(iii) (2 + i) (3 2i)
(i) 1 + i
4 3i
(iii) ( 3 + i) (4 2i) (iv)

1 + i
=1
(3) Find the least positive integer n such that
1 i
(4) Find the real values of x and y for which the following equations are
satisfied.
(i) (1 i)x + (1 + i)y = 1 3i
(1 + i)x 2i (2 3i)y + i
+
=i
(ii)
3+i
3i
(iii)

x2 + 3x + 8 + (x + 4)i = y(2 + i)

130

(5) For what values of x and y, the numbers 3 + ix2y and x2 + y + 4i are
complex conjugate of each other?

3.4 Ordered pair Representation :


In view of the representation of complex numbers, it is desirable to define
a complex number a + ib as an ordered pair (a, b) of real numbers a and b
subject to certain operational definitions. These definitions are as follows:
(i) Equality : (a, b) = (c, d) if and only if a = c, b = d
(ii) Sum :
(a, b) + (c, d) = (a + c, b + d)
(iii) Product : (a, b) . (c, d) = (ac bd, ad + bc)
m(a, b) = (ma, mb)

Result :
The imaginary unit i is defined as i = (0, 1).
We have (a, b) = (a, 0) + (0, b) = a(1, 0) + b(0, 1)
and (0, 1) (0, 1) = (0 1, 0 + 0) = ( 1, 0).
By identifying (1, 0) with 1 and (0, 1) with i we see that (a, b) = a + ib.
Thus we associate the complex number a + ib with the ordered pair (a, b).
The ordered pair (0, 0) corresponds to the real number 0.
Remark :
Though the set of real numbers is ordered, the set of complex numbers is
not ordered. i.e., order relation does not exist in . Given two complex numbers
z1 and z2 we cannot say z1 < z2 or z1 > z2. We can only say that
z1 = z2 or z1 z2, since the points are represented in a plane. Thus the order
relations greater than and less than are not definied for complex numbers.
i.e., the inequalities like 1` + i > 3 2i, i > 0, (3 + i) < 2 etc. are meaningless.

3.5 Modulus of a complex number :


Let z = a + ib be a complex number.
The modulus or absolute value of z denoted by | z | is defined by
|z|=

a2 + b2

From the definition, it is obvious that | z | = | z |. Since a2 + b2 = zz ,

| z | = zz (Taking the positive square root)


Result : Let z = x + iy
Now, x < x2 + y2, then Re(z) < | z | for y 0
If y = 0 then x = | z | then Re(z) = | z |

131

(1)
(2)

Re(z) | z |

Combining (1) and (2)


Similarly Im(z) | z |

Example 3.7 : Find the modulus of the following complex numbers:


(i) 2 + 4i
(ii) 2 3i
(iii) 3 2i
(iv) 4 + 3i
Solution :
(i)

| 2 + 4i | =

( 2)2 + 42 =

20 = 2 5

(ii)

|23i| =

22 + ( 3)2 =

13

(iii)

| 3 2i | =

(iv)

| 4 + 3i | =

( 3) + ( 2) =
42 + 32 =

13

25 = 5

3.5.1 Properties :
If z1, z2, zm are complex numbers, then the following properties hold.
(i) The modulus of a product of two complex numbers is equal to the product
of their moduli.
i.e. |z1 z2 | = | z1 | | z2 |
Proof :

|z1 z2 |2 = (z1z2) z1 z2

[ zz = | z |2]

= (z1 z2) z1 z2

= z1 z1

)( z2 z2 )

= | z1 | 2 | z2 | 2
Taking the positive square root on both sides, we get
|z1 z2 | = | z1 | | z2 |
Note : This result can be extended to any finite number of complex numbers
i.e., | z1 z2 zn | = | z1 | | z2 | | zn |
(ii) The modulus of a quotient of two complex numbers is equal to the quotient
of their moduli.
z1 | z1 |
i.e.,
z = | z | where z2 0.
2
2

132

Proof : Since z2 0,

z1
z1
z1 = z . z2 and so | z1 | = z | z2 | (by the previous
2
2

result)
Therefore
Hence

| z1 |
z1
=
z
| z2 |
2
z1 | z1 |
z = | z |
2
2

(iii) Triangle inequality :


The modules of sum of two complex numbers is always less than or equal
to the sum of their moduli.
i.e., | z1 + z2 | | z1 | + | z2 |
Proof : Let z1 and z2 be two complex numbers.

We know that | z1 + z2 |2 = (z1 + z2) (z1 + z2)

[ | z |2 = zz ]

( )

= (z1 + z2) z1 + z2

= z1 z1 + z1 z2 + z2 z1 + z2 z2

= z1 z1 + z2 z2 + z1 z2 + z1 z2

( )

= | z1 |2 + | z2 |2 + 2 Re z1 z2

| z1 |2 + | z2 |2 + 2 | z1 z 2 |
= | z1 |2 + | z2 |2 + 2 | z1| | z2|

[ Re (z) | z |]

| z | = |z|

= [ | z11 + | z2 |]2
| z1 + z2 |2 [| z1 | + | z2 |]2
Thus taking positive square root we get
| z1 + z2 | | z1 | + | z2 |
Note : 1 Writing z2 for z2 in this result
We also have | z1 z2 | | z1 | + | z2 | | z1 z2 | | z1 | + | z2 |

133

Note : 2
The above inequality can be immediately extended by induction to any
finite number of complex numbers
i.e., for any n complex numbers
z1, z2, z3, , zn
|z1 + z2 + z3 + + zn | | z1 | + | z2 | + + | zn |
(iv) The modulus of the difference of two complex numbers is always greater
than or equal to the difference of their moduli.
Proof : Let z1 and z2 be two complex numbers.

We know that | z1 z2 |2 = (z1 z2) (z1 z2)

[Q| z |2 = z z ]

( )

= (z1 z2) z1 z2

= z1 z1 z1 z2 z2 z1 + z2 z2

= z1 z1 + z2 z2 2 Re z1 z2

| z1 |2 + | z2 |2 2 | z1 z 2 |

[Q Re (z) | z |
Re (z) | z |]

z2

|
= | z1 | + | z2 | 2 | z1 | |
= | z1 |2 + | z2 |2 2 | z1 | | z2 |
= [ | z1| | z2 |]2
| z1 z2 |2 [| z1 | | z2 |]2
Thus taking positive square root we get
| z1 z2 | | z1 | | z2 |
Example 3.8 : Find the modulus or the absolute value of

(1 + 3i) (1 2i)
(3 + 4i)

Solution :

(1 + 3i) (1 2i) = | 1 + 3i | | 1 2i |
| 3 + 4i |
(3 + 4i)
=

12 + 32
2

12 + (2)2

3 +4
=

10 5
=
5

134

10 5
25

3.6. Geometrical Representation


3.6.1 Geometrical meaning of a Complex Number
If real scales are chosen on two mutually perpendicular axes XOX and
YOY (called the x axis and y axis respectively), We can locate any point in the
plane determined by these lines, by the ordered pair of real numbers (a, b)
called rectangular co-ordinates of the point.
Since every complex number a + ib can
be considered as an ordered pair (a, b) of
real numbers, we can represent such
number by a point P in the xy plane,
called the complex plane. Such a
representation is also known as the
Argand diagram. The complex number
represented by P can therefore be read as
either (a, b) or a + ib.

y
P(z)

z|
x

Fig. 3.1

With this representation the modulus of the complex number z = a + ib


a2 + b2. The

complex number z = a + ib can also be represented by the vector OP


(Fig. 3.1) where P = (a, b) and pictured as an arrow from the origin to the point
(a, b). To each complex number there corresponds one and only one point in the
plane, and conversely to each point in the plane there corresponds one and only
one complex number. Because of this we often refer to the complex number z as
the point z.

represents the distance between z and the origin i.e., | z | =

Clearly, the set of real numbers (x, 0) corresponds to the x-axis called real
axis. The set of all purely imaginary number (0, y) corresponds to the y-axis
called the imaginary number axis. The origin identifies the complex number
0 = 0 + i0.

3.6.2 Polar form of a Complex Number :


Let (r, ) be the polar co-ordinates of the point
P = P(x, y) in the complex plane corresponding to the complex number
z = x + iy.

135

Then we get from the figure (Fig. 3.2),


OM x
PM y
cos = OP = r and sin = OP = r
x = r cos ; y = r sin
2

y
P(x,y)

where r = x + y = | x + iy | is called
y
the modulus or the absolute value of
)
z = x + iy denoted by mod z or | z |
x
x M
O
(i.e., the distance from the origin to the
Fig. 3.2
point z)
y
1 y
tan = x , = tan x is called the amplitude or argument of z = x + iy
denoted by amp z or arg z and is measured as the angle which the line OP
makes with the positive x-axis (in the anti clockwise sense).
Thus z = x + iy = r(cos + i sin ) is called the polar form or the modulus
amplitude form of the complex number. It is sometimes convenient to use the
abbreviation cis for cos + i sin .
y
= tan1 x is applicable only for first quadrant numbers i.e., x & y are
positive.

3.6.3 Principal Value :


The argument of z is not unique. Any two distinct arguments of z differ
from each other by an integral multiple of 2. In order to specify a unique value
of arg z, we may restrict its value to some interval of length 2. For this
purpose, we introduce the concept of principal value for arg z as follows :
For an arbitrary z 0 the principal value of arg z is defined to be the
unique value of z that satisfies < arg z .
Note : For z = 0, the argument is indeterminate.

Results :
(1) For any two complex numbers z1 and z2
(ii) arg (z1 . z2) = arg z1 + arg z2
(i) | z1 z2 | = | z1 | . | z2 |
Proof :
Let z1 = r1 (cos 1 + i sin 1) and z2 = r2 (cos 2 + i sin 2)
then

| z1 | = r1, arg z1 = 1 ; | z2 | = r2, arg z2 = 2


z1.z2 = r1r2 (cos 1 + i sin 1) . (cos 2 + i sin 2)
= r1r2 [(cos1 . cos 2 sin1 . sin 2) +
i (sin 1 . cos 2 + cos 1 . sin 2)]

136

= r1r2 [cos (1 + 2) + i sin (1 + 2)]


| z1 z2 | = r1r2 = | z1 | . | z2 | and
arg (z1 z2) = 1 + 2 = arg z1 + arg z2
Note :
This result can be extended to any finite number of complex numbers i.e.,
(i) | z1 . z2 zn | = | z1 | . | z2 | | zn |
(ii) arg (z1 z2 zn) = arg z1 + arg z2 + + arg zn
(2) For any two complex numbers z1 and z2

z1 | z1 |
z1
(i) z = | z | , (z2 0)
(ii) arg z = arg z1 arg z2
2
2
2
Proof :
Let z1 = r1 (cos 1 + i sin 1) and z2 = r2 (cos 2 + i sin 2)
Then
Now

| z1 | = r1, arg z1 = 1 and | z2 | = r2, arg z2 = 2


z1 r1 (cos 1 + i sin 1)
z2 = r2 (cos 2 + i sin 2)
r1 (cos 1 + i sin 1) (cos 2 i sin 2)
=r
2 (cos2 + i sin 2) (cos 2 i sin 2)
r1 (cos1 cos2 + sin1 sin2) + i (sin1 cos2cos1 sin 2)
=r
2
cos2 + sin2
2

r1
= r [cos (1 2) + i sin (1 2)]
2

z1 r1 | z1 |
z = r = | z | and
2
2
2
z
1
arg z = 1 2 = arg z1 arg z2
2
Exponential form of a Complex Number :
The symbol ei or exp (i) (called exponential of i) is defined by
ei = cos + i sin
This relation is known as Eulers formula.

137

If z 0 then z = r (cos + i sin ) = rei. This is called the exponential


form of the complex number z. By straight forward multiplication of
i
i
e 1 = (cos 1 + i sin 1) and e 2 = cos 2 + i sin 2
i

i( + )

we have e 1.e 2 = e 1 2
Remarks :
(1) If 1 = and 2 = in the above definition
then we have ei . ei( ) = ei( ) = ei0 = 1
1
ei( ) = i . Thus writing ei( ) as e i
e
1
we observe that ei = i
e
2

(2) If 1 = 2 = then (ei) = e2i . By mathematical induction it can be


n

shown that (ei) = ein where n = 0, 1, 2

(3) Since ei = e i, we see that, if z = rei then z = re i


(4) Two complex numbers z1 = r1ei1 and z2 = r2ei2 are equal if and only
if r1 = r2 and 1 = 2 + 2n, n Z (the set of all integers)
General rule for determining the argument .
Let z = x + iy
=
where x, y R
=+
|
y
|
Take = tan1 | x |
(i)
Both cos and sin are + e.
z lies in the first quadrant.
(ii)
Sin is + e, cos is e
z lies in the second quadrant.
(iii)
Both sin and cos are ve
z lies in the third quadrant.
(iv)
Sin is e and cos is + e,
z lies in the fourth quadrant.

138

=
=

=
=
=+
=

Example 3.9 : Find the modulus and argument of the following complex
numbers :
(ii) 1 + i 3 (iii) 1 i 3
(i) 2 + i 2
Solution :
(i) Let
2 + i 2 = r(cos + i sin )
Equating the real and imaginary parts separately
r cos = 2 r sin = 2
r2 cos2 = 2

r2 sin2 = 2

r2 (cos2 + sin2) = 4
r = 4=2
2 1
cos = 2 =
2
2
1
sin = 2 =
2

in the 2nd quadrant

3
= 4= 4
3
argument = 4

modulus r = 2,

3
3
Hence 2 + i 2 = 2 cos 4 + i sin 4

(ii) Let
1 + i 3 = r(cos + i sin )
Equating the real and imaginary parts separately
r cos = 1
2

r cos = 1

r sin = 3
r2 sin2 = 3

r2 (cos2 + sin2) = 4 r = 2
1
cos = 2
in the 1st quadrant
3
sin = 2

=3
modulus r = 2,

= tan1 y
x

argument = 3

139

Hence 1 + i 3 = 2 cos 3 + i sin 3

(iii) Let
1 i 3 = r(cos + i sin )
Equating the real and imaginary parts separately
r cos = 1 r sin = 3
r2 cos2 = 1

r2 sin2 = 3

r2 (cos2 + sin2) = 4
r=2
1
cos = 2
3
sin = 2

in the 3rd quadrant

2
= +3= 3
modulus r = 2,

2
argument = 3

2
2
2
2
Hence 1 i 3 = 2 cos 3 + i sin 3 = 2 cos 3 i sin 3

Example 3.10 : If (a1 + ib1) (a2 + ib2) (an + ibn) = A + iB,


prove that (i) (a12 + b12) (a22 + b22) (an2 + bn2) = A2 + B2
B
b1
b2
bn
(ii) tan1 a + tan1 a + + tan1 a = k + tan1 A, k Z

Solution :
Given (a1 + ib1) (a2 + ib2) (an + ibn) = A + iB
| (a1 + ib1) (a2 + ib2) (an + ibn) | = | A + iB |
| (a1 + ib1) | | (a2 + ib2) | | (an + ibn) | = | A + iB |
a12 + b12

a22 + b22

an2 + bn2 =

A2 + B2

Squaring on both sides


(a12 + b12) (a22 + b22) (an2 + bn2) = A2 + B2
Also
arg [(a1 + ib1) (a2 + ib2) (an + ibn)] = arg (A + iB)

140

arg (a1 + ib1) + arg (a2 + ib2) + arg (an + ibn) = arg (A + iB)

(1)

bi
Now arg (ai + ibi) = tan1 a
i

Hence (1) becomes


B
b1
b2
bn
tan1 a + tan1 a + + tan1 a = tan1 A

1
2
n
By taking the general value,
B
b1
b2
bn
tan1 a + tan1 a + + tan1 a = k + tan1 A

1
2
n
Example 3.11 :
P represents the variable complex number z, find the locus of P if

z+1
z1
(ii) arg z + 1 = 3
(i) Re z + i = 1

Solution :
Let z = x + iy then
z + 1 x + iy + 1
(x + 1) + iy
(i)
z + i = x + iy + i = x + i(y + 1)
[x i(y + 1)]
[(x + 1) + iy]
= x + i(y + 1)
[x i(y + 1)]
x(x + 1) + y(y + 1) + i(yx xy x y 1)
=
x2 + (y + 1)2
x(x + 1) + y(y + 1) + i( x y 1)
=
x2 + (y + 1)2
z+1
Given that
Re z + i = 1

x(x + 1) + y(y + 1)

=1
x2 + (y + 1)2
x2 + y2 + x + y = x2 + y2 + 2y + 1
x y = 1 which is a straight line.
The locus of P is a straight line.

z1
(ii) arg z + 1 = 3

141


arg (z 1) arg (z + 1) = 3

arg(x + iy 1) arg (x + iy + 1) = 3

arg [(x 1) + iy] arg [(x + 1) + iy] = 3


y
y

tan1 x + 1 = 3
tan1
x1
y
y
x+1
x1

tan1
y y = 3

1+
x 1 x + 1

2y
2 = tan 3
x 1+y
2y
= 3
2
x + y2 1

2y =

3x2 + 3y2 3

3x2 + 3y2 2y 3 = 0 is the required locus.


Result : (without proof) :
If | z z1 | = | z z2 | then the locus of z is the perpendicular bisector of the
line joining the two points z1 and z2.

3.6.4 Geometrical meaning of conjugate of a complex number :


y

Let z = x + iy be a complex number


represented by P in the Argand diagram.

Then we know that its conjugate z is

given by z = x iy.

i.e., z = (x, y) z = (x, y)

If Q represents the conjugate z , then


conjugate of z is obviously the mirror
image of the complex point z on the real
axis (Fig. 3.3). This clearly indicates that
=

z = z z is purely a real number. Also z = z.


In polar coordinates let
142

P( z )

r
O

)
)-

r
Q( z )

Fig. 3.3

z = r(cos + i sin ) then


z = r(cos ( ) + i sin ( )) so if

z = (r, ) then z = (r, )

Thus the moduli of both z and z are same i.e., r = x2 + y2 But the

amplitude of z is and that of z is . Hence | z | = | z | and

amp z = amp z.
y

Fig. 3.4 gives the simple geometric


relations among the complex number z,

its negation z and its conjugate z


z = ( x, y). It is the point
symmetrical to z about the origin.

-z

Fig. 3.4

3.6.5 Geometrical representation of sum of two complex numbers


Let A and B represent the two complex numbers z1 = x1 + iy1 and
z2 = x2 + iy2 in the Argand diagram. Complete the parallelogram OACB. Then C
represents the complex number z1 + z2.
Proof :
y
Since OACB is a parallelogram,
C (z1+z2)
diagonals OC and AB bisect at M.
From Fig. 3.5, the midpoint M of the
B( z2 )
line joining A(x1, y1) and B(x2, y2) is
x1 + x2 y1 + y2
M
2 , 2
(1)

A ( z1 )
If C is (h, k) then midpoint M of OC is
x
0+h 0+k
O
also given by 2 , 2

Fig. 3.5
k
h
i.e., M is 2 , 2
(2)

From (1) and (2)


y1 + y2
k
h x1 + x2
=
;
=
2
2
2
2
h = x1 + x2 ; k = y1 + y2

143

C is (x1 + x2, y1 + y2)


Hence C represents the complex number, z1 + z2
Note : OA = | z1 |, OB = | z2 | and OC = | z1 + z2 |
In any triangle the sum of two sides is greater than the third side.
from OAC we have OA + AC > OC or OC < OA + AC
| z1 + z 2 | < | z 1 | + | z 2 |
(1)
Further, if the points are collinear
| z1 + z2 | = | z1 | + | z2 |
(2)
Combining (1) and (2) we get
| z1 + z2 | | z1 | + | z2 |
This is the reason why this inequality is called the triangle inequality.

3.6.6 Vector interpretation of complex numbers :


Let A and B represent the two complex numbers z1 = x1 + iy1 and
z2 = x2 + iy2 in the Argand diagram. Complete the parallelogram OACB. Then C
represents the complex number z1 + z2.
A complex number z = x + iy can
y
B
be considered as a vector OP whose
initial point is the origin O and
whose
terminal
point
is
A
P = P(x, y). We sometimes call
OP = x + iy the position vector of
P (x,y)
P. Two vectors having the same
length or magnitude and the same
x
direction but different initial points
O
such as OP and AB are considered
Fig. 3.6
equal. (Fig. 3.6) OP=AB = x + iy
Based on the above interpretation
y
B
of complex numbers as vectors,
z2
addition of complex numbers
A
corresponds to the parallelogram
z1
z2
law for addition of vectors. Thus
z1
+
z1
to add the complex numbers z1 and
C
z2, we complete the parallelogram
z2
O
x
OABC whose sides OA and OC
correspond to z1 and z2. The
Fig. 3.7
diagonal OB of this parallelogram
corresponds to z1 + z2. (Fig. 3.7)

144

3.6.7. Geometrical representation of difference of two complex


numbers
Let A and B represent the two
complex numbers z1 = x1 + iy1 and
z2 = x2 + iy2 respectively in the
Argand diagram. Produce BO to B
such that OB = OB. Then B
represents the complex number
z2. Complete the parallelogram
OADB. Then D represents the sum
of the complex numbers z1 and
z2 or z1 z2 i.e., D represents
the difference of complex numbers
z1 and z2. (Fig. 3.8)

C(z1+z2)
B(z2)
A(z1)

x
D(z1 - z2)

B(-z2)

Fig. 3.8

Result :
From the diagram OD = AB. But OD = | z1 z2 |. AB is the distance
between z1 and z2. Thus, distance between two complex numbers z1 and z2 is
| z1 z2 | .
Note :
Complete the parallelogram OACB. Then C represents the complex
number z1 + z2.

3.6.8 Geometrical representation of product of two complex


numbers:
Let A and B represent the two
complex numbers z1 and z2
respectively in the Argand diagram.
Let z1 = r1 (cos 1 + i sin 1) and
z2 = r2 (cos 2 + i sin 2)

y
C (z1, z2)

Then OA = r1, XOA = 1


)

) 1

r1

A (z1)

145

r2

OB = r2, XOB = 2 .
Take a point L on OX such that OL
= 1 unit. Draw the triangle OBC
directly similar to OLA. (Fig. 3.9)

B (z2)

L (1,0)

Fig. 3.9

then

OB OC
OL = OA

r2
OC
i.e., 1 = r
1

OC = r1r2
Also

XOC = XOB + BOC


= XOB + XOA
= 2 + 1 or 1 + 2

XOA = BOC

The point C represents the complex number z1z2 with polar coordinates
(r1 r2, 1 + 2)
y

Note :
If P represents the complex
number

Q (-y, x)
iz
x

z = r (cos + i sin ) = rei

-y

then the effect of multiplication by


(cos + i sin ) = ei is the rotation

P (x,y)

Fig. 3.10

of P(z) counter clockwise about O through an angle .


i

In particular, since i = cos 2 + i sin 2= e 2 , the effect of multiplication of


any complex numbers P(z) by i is the rotation of P counter clockwise about the
origin through an angle 90. (Fig. 3.10)

3.6.9 Geometrical representation of the quotient of two complex


numbers
Let A and B represent two complex
numbers z1 and z2 in the Argand
diagram.
Let z1 = r1(cos 1 + i sin 1) and
z2 = r2 (cos 2 + i sin 2) ; (z2 0)

A (z1)

r1

C (z1/z2)

146

OB = r2, XOB = 2 .
Take a point L on OX such that OL
= 1 unit. Draw the triangle OAC
directly similar to OBL. (Fig. 3.11)

Then OA = r1, XOA = 1


O

) 2
1

r2

B (z2)

L (1,0)

Fig. 3.11

OA OC
OB = OL

then

r1
OC
i.e., r = 1
2

r1
OC = r
2
XOC = XOA COA = 1 2

r1

C is the point whose polar coordinates are r , 1 2 . Hence C


2

z1
represents the complex number z
2
Example 3.12 : Graphically prove that | z1 + z2 + z3 | | z1 | + | z2 | + | z3 |
Solution :
y
By triangle inequality in OAB,
B
| z1 + z2 | | z 1 | + | z 2 |
(1)
|z 2|
A

|z |
1

By triangle inequality in OBP,


| z1 + z2 + z3 | | z1+ z2 | + | z3|
| z1 | + | z2 | + | z3 | from (1)

|
+z 2
|z 1
+z |
|z 1+z 2 3

| z1 + z2 + z3 | | z1 | + | z2 | + | z3 |

|z3|

P
x

Fig. 3.12
Example 3.13 : Prove that the complex numbers 3 + 3i, 3 3i, 3 3 + 3 3 i
are the vertices of an equilateral triangle in the complex plane.
Solution :
y
Let A, B and C represent the
C(-3 3, 3 3 )
complex numbers
(3 + 3i),
A (3,3)
( 3 3i) and ( 3 3 + i3 3) in
the Argand diagram.
AB = | (3 + 3i) ( 3 3i) |
x

= | 6 + 6i | = 72
B (-3,-3 )

Fig. 3.13
BC = |( 3 3i) ( 3 3 + 3 3i)|
=

|( 3 + 3 3) + i ( 3 3 3)| =

147

72

|( 3
= |( 3

CA =

3 + 3 3i) (3 + 3i)|
3 3) + i (3 3 3)| = 72

A (0, 2)

AB = BC = CA
ABC is an equilateral triangle.
Example 3.14 : Prove that the points representing the complex numbers
2i, 1 + i, 4 + 4i and 3 + 5i on the Argand plane are the vertices of a rectangle.
Solution :
y
Let A, B, C and D represent the
D (3,5)
complex numbers 2i, (1 + i), (4 + 4i) and
(3 + 5i) in the Argand diagram
C (4,4)
respectively.
AB = | 2i (1 + i) |
= | 1 + i | = ( 1)2 + (1)2 = 2
BC = | (1 + i) (4 + 4i) |

B (1,1 )

= | 3 3i |
Fig. 3.14
= ( 3)2 + ( 3)2 = 9 + 9 = 18
CD = | (4 + 4i) (3 + 5i) |
= |1i| =

12 + ( 1)2 = 2

DA = | (3 + 5i) 2i | = | 3 + 3i | =
AB = CD and BC = DA
AC = | (0 + 2i) (4 4i) |
= | 4 2i |
=
2

( 4)2 + ( 2)2 =

32 + 32 = 18

16 + 4 = 20

AB + BC = 2 + 18 = 20
AC2 = 20
Hence

AB2 + BC2 = AC2

As pairs of opposite sides are equal and B = 90, ABCD is a rectangle.


Example 3.15 : Show that the points representing the complex numbers
7 + 9i, 3 + 7i, 3 + 3i form a right angled triangle on the Argand diagram.

148

Solution :
Let A, B and C represent the complex
numbers
7 + 9i, 3 + 7i and 3 + 3i in the
Argand diagram respectively.
AB = | (7 + 9i) ( 3 + 7i) |
= | 10 + 2i | =

A(7,9)

)
B (-3, 7

C(3,3)

102 + 22 = 104

BC = | ( 3 + 7i) (3 + 3i) |

Fig. 3.15

= | 6 + 4i |
= ( 6)2 + 42 = 36 + 16 = 52
CA = | (3 + 3i) (7 + 9i) |
= | 4 6i |
=

( 4)2 + ( 6)2 = 16 + 36 = 52

AB2 = BC2 + CA2


BCA = 90
Hence ABC is a right angled isosceles triangle.
Example 3.16 : Find the square root of ( 7 + 24i)
Solution :
Let 7 + 24i = x + iy
On squaring,
7 + 24i = (x2 y2) + 2ixy
Equating the real and imaginary parts
x2 y2 = 7 and 2xy = 24
x2 + y2 =

(x2 y2) + 4x2y2

= ( 7)2 + (24)2 = 25
Solving,
x2 y2 = 7 and x2 + y2 = 25
we get
x2 = 9 and y2 = 16
x = 3 and y = 4
Since xy is positive, x and y have the same sign.
(x = 3, y = 4) or (x = 3, y = 4)

7 + 24i = (3 + 4i) or ( 3 4i)

149

EXERCISE 3.2
(1) If (1 + i) (1 + 2i) (1 + 3i) (1 + ni) = x + iy
(2)
(3)
(4)

(5)

(6)
(7)
(8)

show that 2.5.10 (1 + n2) = x2 + y2


Find the square root of ( 8 6i)
If z2 = (0, 1) find z.
Prove that the triangle formed by the points representing the complex
numbers (10 + 8i), ( 2 + 4i) and ( 11 + 31i) on the Argand plane is
right angled.
Prove that the points representing the complex numbers (7 + 5i), (5 + 2i),
(4 + 7i) and (2 + 4i) form a parallelogram. (Plot the points and use
midpoint formula).
Express the following complex numbers in polar form.
(i) 2 + 2 3 i
(ii) 1 + i 3
(iii) 1 i
(iv) 1 i

If arg (z 1) = 6 and arg (z + 1) = 2 3 then prove that | z | = 1


P represents the variable complex number z. Find the locus of P, if
2z + 1
(ii) | z 5i | = | z + 5i |
(i) Im iz + 1 = 2

z1
z1

(iv) | 2z 3 | = 2
(v) arg z + 3 = 2
(iii) Re z + i = 1

3.7 Solutions of polynomial equations :


Consider the quadratic equation x2 4x + 7 = 0
b2 4ac = ( 4)2 (4) (7) (1)
= 16 28 = 12 which is negative.
The roots of this quadratic equation are not real. The roots are given by
Its discriminant is

( 4)
2

12

12
2

=2i 3

Thus we see that the roots 2 + i 3 and 2 i 3 are conjugate to each other.
We shall now consider the cube roots of unity.
Let x be the cube root of unity then
1

x = (1)3
x3 = 1
(x 1) (x2 + x + 1) = 0

150

x 1 = 0 ; x2 + x + 1 = 0
Hence x = 1 and

x=

Cube roots of unity are 1,

1 (4) (1) (11)


2

1 + 3 i 1 3i
,
2
2

Here again the two complex roots

1+ 3i
1 3i
and
are conjugate
2
2

to each other.
From the above two examples one can infer that in an equation with real
coefficients, imaginary roots occur in pairs (i.e., one root is the conjugate of the
other). This paved the way for the following theorem.
Theorem :
For any polynomial equation P(x) = 0 with real coefficients, imaginary
(complex) roots occur in conjugate pairs.
Proof :
Let P(x) = anxn + an1xn 1 + + a1x + a0 = 0 be a polynomial equation
of degree n with real coefficients.

Let z be a root of P(x) = 0. We show that z is also a root of P(x) = 0.


Since z is a root of P(x) = 0
P(z) = anzn + an 1zn 1 + + a1z + a0 = 0
Taking the conjugate on both sides

(1)

P(z) = anzn + an 1zn 1 + + a1z + a0 = 0

Using the idea that the conjugate of the sum of the complex numbers is
equal to the sum of their conjugates,

anzn

+ an 1zn 1 + + a1z + a0 = 0


i.e. an zn + an1 zn1 + + a1 z + a0 = 0

Since zn = ( z )

and
a0, a1, a2 an are real numbers, each of them is its own conjugate and
hence we get

an zn + an 1zn1 + + a1 z + a0 = 0

151

which is same as P( z ) = 0

This means z is also a root of P(x) = 0.


Hence the result.
Example 3.17 : Solve the equation x4 4x2 + 8x + 35 = 0, if one of its roots is
2+ 3i
Solution :
Since 2 + i 3 is a root, 2 i 3 is also a root.
Sum of the roots = 4
Product of the roots (2 + i 3) (2 i 3) = 4 + 3 = 7
The corresponding factor is x2 4x + 7
x4 4x2+ 8x + 35 = (x2 4x + 7) (x2 + px + 5)
Equating x term, we get 8 = 7p 20 p = 4
Other factor is (x2 + 4x + 5)
x2 + 4x + 5 = 0 x = 2 i
Thus the roots are 2 i 3 and 2 i
EXERCISE 3.3
4

(1) Solve the equation x 8x + 24x2 32x + 20 = 0 if 3 + i is a root.


(2) Solve the equation x4 4x3 + 11x2 14x + 10 = 0 if one root is 1 + 2i
(3) Solve : 6x4 25x3 + 32x2 + 3x 10 = 0 given that one of the roots is 2 i

3.8 De Moivres Theorem and its applications :


Theorem :
For any rational number n, cos n + i sin n is the value or one of the
values of (cos + i sin )n
Proof :
Case I : Let n be a positive integer.
By simple multiplication we have
(cos1 + isin1) (cos2 + isin2) = cos(1 + 2) + isin(1 + 2)
Similarly (cos1 + isin1) (cos2 + isin2) (cos3 + i sin 3)
= cos(1 + 2 + 3) + i sin(1 + 2 + 3)
By extending it to the product of n complex number we have
(cos1 + i sin1) (cos2 + i sin2) (cosn + i sinn)
= cos(1 + 2 + + n) + i sin(1 + 2 + + n)

152

In this expression put 1 = 2 = n = , then


we have
(cos + isin )n = cosn + isin n
Case II : Let n be a negative integer and equal to m ; (m is a +ve integer)
(cos + isin)n = (cos + isin) m
1
=
(cos + i sin)m
1
by case I
=
cosm + i sin m
cos m i sin m
=
(cos m + i sinm) (cosm i sin m)
cosm i sinm
=
cos2m + sin2m
= cosm i sinm
= cos( m) + i sin( m)
= cosn + i sinn
p
Case III : Let n be a fraction and equal to q , where q is a positive integer and
p is any integer.

q
Consider cos q + i sin q = cos + i sin

Therefore cos q + i sin q is such that its qth power is cos + i sin.
1

Hence cos q + i sin q is one of the values of (cos + i sin )q.


Raise each of these quantities to the pth power.
1

P
cos q + i sin q is one of the values of (cos + i sin)q

p
p
i.e., cos q + i sin q is one of the values of (cos + i sin)q
ie., cos n + i sin n is one of the values of (cos + i sin)n.
Note : De Moivres theorem holds good for irrational values also but the proof
is beyond the scope of this book.

153

Properties :
(i)

(cos + i sin) n = cos( n) + i sin ( n)


= cos n i sin n

(ii)

(cos i sin)n = {cos() + i sin( )}n


= cos ( n) + i sin( n)
= cosn i sin n
n

(iii)

(sin + i cos) = cos 2 + i sin 2


= cos n 2 + i sin n2

Example 3.18 : Simplify :

(cos 2 + i sin 2)3 (cos 3 i sin 3) 3


(cos 4 + i sin 4 ) 6 (cos + i sin )8

Solution :
The given expression =

(cos 2 + i sin 2)3 (cos 3 i sin 3) 3


(cos 4 + i sin 4 ) 6 (cos + i sin )8
3

(ei2) . (ei3)
i4 6

(e

i 8

(e )

ei6 ei9
= i24 i8
.e
e

= ei15 . ei16
= ei31 = cos31 + i sin 31
Alternative method :
The given expression =

(cos + i sin )6 (cos + i sin )9


(cos + i sin ) 24 (cos + i sin )8

= (cos + i sin )6 + 9 + 24 8
= (cos + i sin )31
= cos 31 + i sin 31
Example 3.19 : Simplify :

(cos + i sin)4
(sin + i cos)5

154

Solution :
(cos + i sin)4
(cos + i sin)4
=
5
(sin + i cos)5
cos + i sin
2
2

= cos 4 5 2 + i sin 4 5 2

5
5
= cos 9 2 + i sin 9 2

5
5
= cos 2 9 i sin 2 9

= cos 2 9 i sin 2 9

= sin 9 i cos 9
Alternative method :
(cos + i sin)4 1 (cos + i sin)4
=

(sin + i cos)5 i5 (cos i sin)5


= i (cos 4 + i sin 4 ) (cos 5 + i sin 5)
= i [cos 9 + i sin 9]
= sin 9 i cos 9
1

Result : | z | = 1 z = z
Example 3.20 : If n is a positive integer, prove that
n

1 + sin + i cos = cos n + i sin n


1 + sin i cos
2
2

Solution :
Let z = sin + i cos
1
z = sin i cos
n

1 + sin + i cos
1 + z
n
n
=

1 = z = (sin + i cos )
1 + sin i cos
1 + z

= cos2 + i sin 2

155

= cos n2 + i sin n2

Example 3.21 : If n is a positive integer, prove that


n
( 3 + i)n + ( 3 i)n = 2n + 1 cos 6
Solution :
Let ( 3 + i)= r(cos + i sin )
Equating real and imaginary parts separately, we have
r cos = 3 and r sin = 1
r=

( 3 )2 + 1 2

4=2

3
1

cos = 2 , sin = 2 = 6
Hence

3 + i) = 2 cos 6 + i sin 6

3 + i)

= 2 cos 6 + i sin 6 = 2ncos 6 + i sin 6


n
n
= 2ncos 6 + i sin 6
(1)

To determine 3 i, we replace i in the above result by i we get

( 3 i) = 2cos 6 i sin 6
n
n
n
( 3 i) = 2n cos 6 i sin 6

Adding (1) and (2) we have


n
n
( 3 + i)n + ( 3 i) = 2n 2 cos 6
n
= 2n + 1 . cos 6

(2)

Example 3.22 : If and are the roots of x2 2x + 2 = 0 and cot = y + 1,


sin n
(y + )n (y + )n
=
show that

sinn
Solution :
The roots of the equation x2 2x + 2 = 0 are 1 i.

156

Let = 1 + i and = 1 i
(y + )n = [(cot 1) + (1 + i)]n

Then

= (cot + i)n
1
=
[cos + i sin]n
n
sin
1
[cosn + i sin n]
=
sinn
1
[cosn i sin n]
Similarly
(y + )n =
sinn
2i sin n
(y + )n (y + )n=
sinn
Further
= (1 + i) (1 i) = 2i
(y + )n (y + )n 2i sin n
sin n
=
=
n

2i sin
sinn
EXERCISE 3.4
(1) Simplify :

(cos 2 i sin 2)7 (cos 3 + i sin 3) 5


(cos 4 + i sin 4)12 (cos 5 i sin 5) 6

(cos + i sin )3
(sin + i cos )4
(3) If cos + cos + cos = 0 = sin + sin + sin , prove that
(i) cos 3 + cos 3 + cos 3 = 3 cos ( + + )
(ii) sin 3 + sin 3 + sin 3 = 3 sin ( + + )
(iii) cos 2 + cos 2 + cos 2 = 0
(iv) sin 2 + sin 2 + sin 2 = 0
(Hints : Take a = cis , b = cis , c = cis
a + b + c = 0 a3+ b3 + b3 = 3abc

(2) Simplify :

1/a + 1/b + 1/c = 0 a2 + b2 + c2 = 0)


3
(v) cos2 +cos 2 + cos2 = sin2 + sin2 + sin2 = 2
For problems 4 to 9, m, n N
(4) Prove that
n+2

(i) (1 + i)n + (1 i)n = 2 2

n
cos 4

157

n
n
n
(ii) (1 + i 3) + (1 i 3) = 2n + 1 cos 3
n
n
(iii) (1 + cos + i sin )n + (1+cos i sin )n = 2n + 1 cos ( / 2) cos 2

(iv) (1 + i)4n and (1 + i)4n + 2 are real and purely imaginary respectively
(5) If and are the roots of the equation x2 2px + (p2 + q2) = 0 and
(y + )n (y +)n
sin n
q
= qn 1
tan = y + p show that

sinn
(6) If and are the roots of x2 2x + 4 = 0
n
Prove that n n = i2n + 1 sin 3 and deduct 9 9
1
(7) If x + x = 2 cos prove that
1
1
(i) xn + n = 2 cos n
(ii) xn n = 2 i sin n
x
x
1
1
(8) If x + x = 2 cos and y + y = 2 cos show that
yn
xm
(i) n + m = 2 cos (m n)
y
x

xm
yn
(ii) n m = 2 i sin (m n)
y
x

(9) If x = cos + i sin ; y = cos + i sin


1
prove that xmyn + m n = 2 cos (m + n)
x y
(10) If a = cos2 + i sin 2, b = cos2 + i sin 2 and c = cos 2 + i sin 2
prove that
1
(i) abc +
= 2 cos ( + + )
abc
(ii)

a2 b2 + c2
= 2 cos 2( + )
abc

3.9. Roots of a complex number


Definition :
A number is called an nth root of a complex number z, if n = z and
1

we write = zn

158

Working rule to find the nth roots of a complex number :


Step 1
Step 2
Step 3
Step 4
Illustration :
Let

:
:
:
:

Write the given number in polar form.


Add 2k to the argument
apply De Moivres theorem (bring the power to inside)
Put k = 0, 1 upto n 1

z = r(cos + i sin)
= r{cos (2k + ) + i sin (2k + )}, k is an integer.
1

zn = [r{cos (2k + ) + i sin (2k + )]n


1

2k +
2k +
= rn cos n + i sin n

where k = 0, 1, 2 (n 1)
1

Only these values of k will give n different values of zn provided z 0


Note :
(1) The number of nth roots of a non-zero complex number is n.
(2) The moduli of these roots is the same non negative real number.
(3) The argument of these n roots are equally spaced. That is if is the
principal value of arg z i.e., then the arguments of other

2 4
roots of z are obtained by adding respectively n , n , to n
(4) If k be given integral values greater than or equal to n, these n values
are repeated and no fresh root is obtained.

3.9.1 The nth roots of unity


1= (cos 0 + i sin 0) = cos 2k + i sin 2k
1

nth roots of unity = 1n = (cos 2k + i sin 2k)n


2k
2k
= cos n + i sin n where k = 0, 1, 2, n 1

2
2
The nth roots of unity are cos0 + i sin 0 , cos n + i sin n ,
4
4
6
6
2
2
cos n + i sin n , cos n + i sin n , , cos (n 1) n + i sin (n 1) n
2

2
2
i
Let = cos n + i sin n = e n

159

Then the nth roots of unity are


2

2(n 1)

i
i
i
i n
e0, e n , e n , e n , e
become 1, , 2 n 1 .
It is clear that the roots are in geometric progression.
Results :
(1) n = 1
n

2
2
= cos n + i sin n = cos 2 + i sin 2 = 1
n

(2) Sum of the roots is 0


i.e., 1 + + 2 + + n 1 = 0
LHS = 1 + + 2 + + n 1 is a G.P. with n terms.
1 rn
1.(1 n)

1 + r + r2 + + rn 1 =
1r
1

= 0 = R.H.S.
(3) The roots are in G.P with common ratio
2
(4) The arguments are in A.P with common difference n
=

(5) Product of the roots = ( 1)n + 1


1

3.9.2. Cube roots of unity : (1)3


1

Let x = (1)3
1

x = (cos 0 + i sin 0)3


1

= (cos 2k + i sin 2k)3 , where k is an integer.


2k
2k
x = cos 3 + i sin 3 , where k = 0, 1, 2

The three roots are


2
4
4
2
cos 0 + i sin0, cos 3 + i sin 3 , cos 3 + i sin 3
1
1
3
3
i.e., 1, 2 + i 2 , 2 i 2
1
3
The roots are 1, 2 i 2

160

Result :
1

12
0

120

The modulus of each root of (1)3 is 1


B
All these roots lie on the
circumference of the unit circle. Let A, B
and C be points represented by the three
3
1
x
roots 1, 2 i 2 in ordered pair form.
A
The angles between OA and OB, OB and
2
OC, OC and OA are each 3 radians or
C
120. Hence when these points are joined
by straight lines they will form the
Fig. 3.16
vertices of an equilateral triangle.
2
2
If we denote the second root cos 3 + i sin 3 by then the other root
0
12

4
2
2
4
cos 3 + i sin 3 = cos 3 + i sin 3 becomes 2.

Hence we observe that the cube roots of unity, namely 1, , 2 are in G.P.
Note :
1i 3
1+i 3
(i) Even if
is taken as it can be proved that
= 2
2
2
(ii) 1 + + 2 = 0 (by actual addition) i.e., the sum of the cube roots of
unity is zero.
(iii) Since is a root of the equation x3 = 1, we see that 3 = 1.

Fourth roots of unity :


Let x be a fourth root of unity. Then x =

1
(1)4

x4 = 1 = (cos 2k + i sin 2k) where k is an integer.


1

x = (cos 2k + i sin 2k)4


2k
2k
= cos 4 + i sin 4

k
k
= cos 2 + i sin 2 where k = 0, 1, 2, 3

161

The four roots are

3
3
cos 0 + i sin 0, cos 2 + i sin 2, cos + i sin , cos 2 + i sin 2

i.e., 1, i, 1(= i2), i (= i3) . Let us denote cos 2 + i sin 2 by . Then the
four roots of unity are 1, , 2, 3.
y

The fourth roots of unity form


the vertices of a square all lying
on the unit circle.
We observe that the sum of
the fourth roots of unity is zero.
i.e., 1 + + 2 + 3 = 0
and 4 = 1

/2

2
O

Fig. 3.17
Note : The values of used in cube roots of unity and in fourth roots of unity
are different.
Sixth roots of unity : Let x be a sixth root of unity. Then x = (1)1/6
1 = cos 0 + i sin 0
(1)1/6 = (cos 2k + i sin 2k)1/6
where k is an integer.
By De Moivres theorem
1
2k
2k
x = (1)6 = cos 6 + i sin 6 , where k = 0, 1, 2, 3, 4, 5

The six roots are


y
cos 0 + i sin 0 = 1

cos 3 + i sin 3
2
2
cos 3 + i sin 3
1
3
3
3
x
cos 3 + i sin 3
4
4
cos 3 + i sin 3
5
5
5
4
cos 3 + i sin 3
Fig. 3.18

162

Then the six, sixth roots of unity are 1, , 2, 3, 4, 5


y

From the above figure it can


be noted that the six roots of
unity form the vertices of a
hexagon all lying on the unit
circle (Fig. 3.18). Thus it can be
seen that the n, nth roots of unity
form the vertices of n sided
regular polygon all lying on the
unit circle (Fig. 3.19).

3
2

1
n-1
n-2
n-3

Fig. 3.19
9

Example 3.23 : Solve the equation x + x x 1 = 0


Solution :
x9 + x5 x4 1 = 0 x5 (x4 + 1) 1 (x4 + 1) = 0
(x5 1) (x4 + 1) = 0
x5 1 = 0 ; x4 + 1 = 0
1

x = (1)5 ; ( 1)4
1

(i)

x = (1)5 = (cos 0 + i sin 0)5


1

= (cos 2k + i sin 2k)5


2k
2k
= cos 5 + i sin 5 , k = 0, 1, 2, 3, 4
1

(ii)

( 1)4 = (cos + i sin )4


1

= {cos (2k + 1) + i sin (2k + 1)}4


= cos

(2k + 1)
(2k + 1)
+ i sin
k = 0, 1, 2, 3
4
4

Thus we have 9 roots.


Example 3.24 : Solve the equation x7 + x4 + x3 + 1 = 0
Solution :
x7 + x4 + x3 + 1 = 0 x4 (x3 + 1) + 1 (x3 + 1) = 0
(x4 + 1) (x3 + 1) = 0

163

x4 = 1 ; x3 = 1
1

(i)

x = ( 1)4
1

= (cos + i sin )4
1
)]4

= [cos (2k + ) + i sin (2k +


(2k + 1)
(2k + 1)
= cos
+ i sin
4
4

; k = 0, 1, 2, 3

i.e.,

(ii)

x3 = 1

x = ( 1)3
1

= (cos + i sin )3
1

= [cos (2k + ) + i sin (2k + )] 3

= cos (2k + 1) 3 + i sin (2k + 1) 3 , k = 0, 1, 2


Note :

3
3
5
5
The roots are cos 4 + i sin 4 ; cos 4 + i sin 4 ; cos 4 + i sin 4

7
7

and cos 4 + i sin 4 , cos 3 + i sin 3 , (cos + i sin ) and

cos 5 + i sin 5
3
3

1
1
1
1
1
1 1
1
i.e.,
+i
,
+i ,
i ,
i
2
2
2
2
2
2 2
2
3
3
1
1
2 + i 2 , 1, 2 i 2
2

Example 3.25 : Find all the values of ( 3 + i)3


Solution :
Let 3 + i = r (cos + i sin )
r cos = 3 , r sin = 1
2

r =
( 3) + 1 = 2
3
1

Cos = 2 , sin = 2 = 6
2

( 3 + i)3 = 23 cos 6 + i sin 6 3

164

2
23

2 3

cos + i sin
6
6


= 23 cos 3 + i sin 3 3

2
23

cos 2k + + i sin 2k + 3
3
3

= 23 cos (6k + 1) 9 + i sin (6k + 1) 9 where k = 0, 1, 2

Note: The values are


2

7
7
13
13
23 cos 9 + i sin 9 , 23 cos 9 + i sin 9 , 23 cos 9 + i sin 9

Aliter :
2

3 + i)3 = 23 cos 6 + i sin 63

2
23

cos 2k + + i sin 2k + 3
6
6

= 23 cos (12k + 1) 6 + i sin (12k + 1) 6 3

2
23

cos (12k + 1) + i sin (12k + 1) where k = 0, 1, 2


9
9

13
13
The different values are 23 cos 9 + i sin 9 , 23 cos 9 + i sin 9 ,

2
25
25
23 cos 9 + i sin 9

2
23

cos + i sin , 23
cos 7 + i sin 7, and
i.e.,
9
9
9
9

13
13
25
25
7
7
cos

9 + i sin 9 since cos 9 + i sin 9 = cos 9 + i sin 9

2
23

Thus we have obtained the same values in this case also.


Note : If we add 2k before taking the power 2 inside, we will get the same
answer.

165

EXERCISE 3.5
(1) Find all the values of the following :
1

(i) (i)3

(ii) (8i))3

(iii) ( 3 i)3

(2) If x = a + b, y = a + b2, z = aw2 + b show that


(i) xyz = a3 + b3
(ii) x3 + y3 + z3 = 3 (a3 + b3) where is the complex cube root of unity.
(3) Prove that if 3 = 1, then
(i) (a + b + c) (a + b + c2) (a + b2 + c) = a3 + b3 + c3 3abc
5

(ii)

1 + i 3 + 1 i 3 = 1
2
2

1
1
1

+
= 0
1 + 2
1+
2+
(4) Solve :
(i) x4 + 4 = 0 (ii) x4 x3 + x2 x + 1 = 0
(iii)

34
1
(5) Find all the values of 2 i 2

and hence prove that the product of the


values is 1.

166

4. ANALYTICAL GEOMETRY
4.1 Introduction :
Tracing the history of Mathematics, around 430 B.C., study of conic
sections or conics, i.e., study of plane sections of a right circular cone began.
The study included degenerate or singular conics (comprising single point, pair
of distinct lines, two coincident lines etc., which were already dealt with in
detail in lower classes) and non-degenerate or non-singular conics (comprising
of circles, parabolas, ellipses and hyperbolas).
The study of conic sections from Greek Geometry, developed by
Apollonius, is described today as graphs of quadratic equations in the
co-ordinate plane. The Greek mathematicians of Platos time (429 347 B.C.)
described these curves as the curves formed by slicing a double cone (right
circular cone of two nappes) with a plane and hence the name conic sections.
Analytic Geometry grew out of need for establishing uniform techniques
for solving geometrical problems, the aim being to apply them to the study of
curves, which are of particular importance in practical problems.
The aim was achieved in the co-ordinate method viz., cartesian, polar,
bi-polar (where calculations are fundamental and constructions play a
subordinate role). Thus solving problems by the method of Analytical
Geometry requires less inventiveness. This method of the ancient Greek origin
( 1 2 B.C.) was systematically developed in the first half of the 17th century
by great mathematicians Fermat, Descartes, Kepler, Newton, Euler, Leibnitz,
LHpital, Clairaut, Cramer and the Jacobis.
A major breakthrough in the study occurred with the development of the
hypothesis of Planetary Phenomena by the German mathematician cum
physicist Johannes Kepler. He stated that all the planets in the solar system
including the earth are moving in elliptical orbits with sun at one of a foci,
governed by inverse square law. This led to the development of Newtons
gravitation theory.
Euler applied the co-ordinate method in a systematic study of space curves
and surfaces, which was further developed by Albert Einstein in his theory of
relativity.
Needless to say that today the development in this area has conquered
industry, medicine and scientific research. And we shall cite a few of them
before getting into the depth of actual study of conics.

167

4.1.1 Geometry and Practical applications of a parabola :


A parabola is a conic section
obtained on slicing a right circular cone
by a plane parallel to the line joining
vertex and any other point of the cone
(Fig.4.1)
Fig. 4. 1
If P is any point on the
parabola with focus F and vertex
V, the angles subtended by FP
and PX with the tangent at P are
equal where PX is parallel to the
axis VFA of the parabola. (Fig.
4.2)
This property is made use of
in parabolic reflectors (surface
obtained by revolving the
parabola about its axis and coated
with silver paint) of sound, light
and radio waves when the
respective source is placed at the
focus S as given in (Fig. 4.3).
Light (or sound or radio waves)

X
V F

Fig. 4. 2

Fig. 4. 3
from S falls on the reflecting surface gets reflected parallel to the axis of
parabola. For example, Flash light, head light of motor vehicles, parabolic
mirrors, spot light reflectors, selective microphone sounding boards etc.
The same reflectors can be
employed in intensifying signals.
Electromagnetic waves arriving
x
parallel to the axis of the
R
V
parabolic reflector will be
focussed at the focus where a
suitable receiver R could be
Fig. 4. 4
placed. (Fig. 4.4)
For example, Radio telescope, television satellite dishes, solar heaters,
radar antennas etc.
The strongest simple arch is parabolic in shape.

168

The supporting cable of a uniformly loaded bridge is parabolic in shape


(weight of cable neglected in comparison with weight of the bridge).
The path of an object thrown or projected obliquely upwards is a parabola.
Also bombs dropped from a moving war plane or food packets dropped from
helicopters during cyclone time to people in need (not moving vertically
upwards or downwards) traces a parabola.
Some comets have parabolic path with sun at the focus.

4.1.2 Geometry and Practical Applications of an ellipse :


An ellipse is a conic obtained on
slicing across obliquely one nappe of a
cone. (Fig. 4.5)
If P is any point on the ellipse and
F1and F2 its foci, the angle subtended by
F1P and F2P with the tangent at P are
Fig. 4. 5
equal and if a source of light or sound is
placed at one focus of an ellipsoidal
P
reflector (surface generated by revolving
an ellipse about its major axis) all the
F2
F1
waves will be reflected so as to pass
Fig. 4. 6
through the other focus (Fig. 4.6)
This property is also used in Wispering Gallery, the roof or walls of
which are shaped like an ellipsoidal reflector.
The ellipsoidal reflectors are designed for Nd : YAG (ND3+ Neodymium
ions; YAG Yttrium Aluminium Garnetz) laser that is widely used in medicine,
industry and scientific research.
A light reflector in the form of a
tube whose cross section is an ellipse
has Nd : YAG rod and a linear flash
lamp placed at the foci of the ellipse.
(Fig. 4.7). Here light emitted from
the lamp is effectively coupled to the
Nd : YAG rod to produce laser beam.
In Bohr-Sommerfeld theory of
the atom electron orbit can be
circular or elliptical.

M1

Laser rod

Output

Flashtube
Capacitor bank

Power Supply

Ballast resistor

Fig. 4. 7

169

M2

Ellipsoidal
reflector
Trigger
pulse

The orbits of our planet earth and all other planets and planetoids in our
solar system are elliptical with sun situated at one of the foci. Also all the
satellites, either natural or artificial to all the planets in the solar system have
elliptic orbits (with the force binding them following inverse square law).
[Fig.4.8(a)]
Uranus Mercury

Jupiter Venus

Pluto
Saturn

Halleys Comet

Saturn
JupiterSun
Earth

Sun
Neptune

Mars

Earth

Pluto

Asteroids

Fig. 4.8(b)
Fig. 4.8(a)
Path of Halleys Comet (which returns after every 75 years) is an ellipse
with e 0.97 and the sun at a focus [Fig. 4.8 (b)], e being the eccentricity.
Elliptical arches are often used for their beauty.
Steam boilers are believed
to have greatest strength when
heads are made elliptical with
major and minor axes in the
ratio 2 : 1.

Fig. 4. 9
Gears are sometimes (for particular need) made elliptical in shape (Fig. 4.9)

Fig. 4. 10
The orbit of Comet Kohoutek is an ellipse with e 0.9999 (Fig. 4.10).
The shape of our mother earth is an oblate spheroid i.e., the solid of
revolution of an ellipse about its minor axis, bulged along equatorial region and
flat along the polar region.
The area of action of an airplane which leaves a moving carrier and returns
in a given time (with no wind) is an ellipse with the take off and landing
positions of the carrier as foci.
The track of a plane making an
On-pylon turn in a wind of constant
velocity is an ellipse with one focus
directly over the pylon (Fig. 4.11).
Fig. 4. 11

170

4.1.3 Geometry and Practical Applications of a Hyperbola :


A hyperbola is a conic obtained on slicing a
double napped cone by a plane parallel to the axis
of the cone (Fig.4.12)

Fig. 4. 12
The lines from the foci to any point on
a hyperbola make equal angles with the
tangent at that point. Hence if the surface of
a reflector is generated by revolving a
hyperbola about its transverse axis, all rays
of light converging on one focus are
reflected to the other (Fig.4.13)

P
F2

F1

Fig. 4. 13

This property is made use of in some


telescopes in conjunction with a parabolic
reflector. American space research
foundation NASAs Hubble space
telescope uses hyperbolic reflectors in
conjunction with parabolic reflectors
(Fig. 4.14).
Hyperbolas are useful in rangefinding. (The difference in the times at
which a sound is heard at two listening
posts is proportional to the difference of
the distances from the posts to the point
of emanation of sound. A third listening
post serves to give another hyperbola and
the point of emanation is at the point of
intersection of the two curves).

Fig. 4. 14

Boyles law pv = constant is hyperbolic in relationship. The same is true of


relationship of any two quantities, which are inversely proportional.
Hyperbolic paths arise in Einsteins theory of relativity and form a basis
for LORAN (Long Range Navigation) radio navigation system.
171

4.2 Definition of a Conic :

Fixed Line (directrix)

Consider a circle C. Let A be the


A
line through the centre of C and
perpendicular to the plane of C and

let V be a point on A not in the plane


V
of C. Let P be a point on C and draw
the infinite straight lines through P
that also passes through V. As P
C
moves around C, what sweeps out is
P
called a right circular cone with the
Fig. 4. 15
axis A and vertex V. Each of the lines
PV is called a generator of the cone, and the angle between the axis and the
generator is called a vertex angle (semi-vertical angle). The upper and lower
portions of the cone that meet at the vertex are called nappes of the cone
(Fig. 4.15).
The curves obtained by slicing the cone with a plane not passing through
the vertex are called conic sections or simply conics.
A conic is the locus of a point which
l
moves in a plane, so that its distance from
P (Moving point)
M
a fixed point bears a constant ratio to its
distance from a fixed straight line.
(Fig.4.16)
. )
F( Fixed point
The fixed point is called focus, the
fixed straight line is called directrix, and
Fig. 4. 16
the constant ratio is called eccentricity, which is denoted by e.
FP
From the figure we have PM = constant = e

4.2.1 General equation of a Conic :


Let F(x1, y1) be the focus, lx + my + n = 0, the equation of the directrix l
and e the eccentricity of the conic.
Let P(x, y) be any point on the conic.
l
Drop a perpendicular from P to l.
P(x,y)
FP =
(x x1)2 + (y y1)2
PM = Perpendicular distance from
P(x, y) to the line lx + my + n = 0
lx + my + n
=
l2 + m 2

172

F(x1,y1)

Fig. 4. 17

FP
By the definition of a conic, PM = e
FP2 = e2 PM2
2

(x x1) + (y y1) = e

lx + my + n

l2 + m2

Simplifying this we get an equation of second degree in x and y of the form


Ax2 + Bxy + Cy2 + Dx + Ey + F = 0.

4.2.2 Classification with respect to the general equation of a conic :


The equation Ax2 + Bxy + Cy2 + Dx + Ey + F = 0 represents either a
(nondegenerate) conic or a degenerate conic. If it is a conic, then it is
(i) a parabola if B2 4AC = 0
(ii) an ellipse if B2 4AC < 0
(iii) a hyperbola if B2 4AC > 0

4.2.3. Classification of conics with respect to eccentricity :


1. If e < 1, then the conic is an ellipse.
From the figure 4.18 we observe
that F2Pi is always less than PiMi.
F2Pi
i.e., P M = e < 1, (i = 1, 2, 3)
i i

y
M3
M2
M1

P3
P2
P1

F2

F1

Fig. 4. 18

2. If e = 1, then the conic is a


parabola.
From the figure 4.19 we observe
that FPi is always equal to PiMi.
FPi
i.e., P M = e = 1, (i = 1, 2, 3)
i i

M3

P2

M2
M1

P3

P1

Fig. 4. 19
y

3. If e > 1, then the conic is a


hyperbola.
From the figure 4.20 we
observe that F1Pi is always
greater than PiMi.
F1Pi
i.e., P M = e > 1, (i = 1, 2, 3)
i i

M3
M2
M1
F2

P3
P2
P1

Fig. 4. 20

173

F1

4.3 Parabola :
The locus of a point whose distance from a fixed point is equal to its
distance from a fixed line is called a parabola. That is a parabola is a conic
whose eccentricity is 1.
Note : Eventhough the syllabus does not require the derivation of
standard equation and the tracing of parabola (4.3.1, 4.3.2) and it
needs only the standard equation and the diagram, the equation is
derived and the curve is traced for better understanding.
Now we derive and trace the standard equation of a parabola.

4.3.1. Standard equation of a parabola :


y

Given :
Fixed point (F)
Fixed line (l)
Eccentricity (e = 1)
Moving point P(x, y)
Construction :
Plot the fixed point F and
draw the fixed line l.

M
(-a, y)

Z
(-a,0)

P (x,y)

F (a,0)

Fig. 4. 21

Drop a perpendicular (FZ) from F to l.


Take FZ = 2a and treat it as x-axis.
Draw a perpendicular bisector to FZ and treat it as y-axis.
Let V(0, 0) be the origin.
Drop a perpendicular (PM) from P to l.
The known points are F(a, 0), Z( a, 0) and hence M is ( a, y).
FP
2
2
By the definition of a conic,
PM = e = 1 FP = PM
(x a)2 + (y 0)2 = (x + a)2 + (y y)2
x2 2ax + a2 + y2 = x2 + 2ax + a2 which simplifies to y2 = 4ax.
This is the standard equation of the parabola.
To trace a curve, we shall use the tools dealt in detail in chapter 6.

4.3.2.Tracing of the parabola y2 = 4ax :


(i) Symmetry property :
It is symmetrical about x-axis.
i.e., x-axis divides the curve into two symmetrical parts.

174

(ii) Special points :


The parabola passes through the origin since (0, 0) satisfies the equation
y2 = 4ax.
To find the points on x-axis, put y = 0. We get x = 0 only.
the parabola cuts the x-axis only at the origin (0, 0).
To find the points on y-axis, put x = 0. We get y = 0 only.
the parabola cuts the y-axis only at the origin (0, 0).
(iii) Existence of the curve :
For x < 0, y2 becomes negative. i.e., y is imaginary. Therefore the curve
does not exist for negative values of x. i.e., the curve exists only for
non-negative values of x.
y
(iv) The curve at infinity :
As x increases, y2 also
increases.

P (x, y)

M
(-a, y)

i.e., as x , y2
i.e., as x , y

the curve is open


rightward. [Fig. 4.22]

Z
(-a, o)

F(a, o)

Fig. 4. 22

4.3.3. Important definitions regarding a parabola :


Focus : The fixed point used to draw the parabola is called the focus (F).
Here, the focus is F(a, 0).
Directrix : The fixed line used to draw a parabola is called the directrix of
the parabola. Here, the equation of the directrix is x = a.
Axis : The axis of the parabola is the axis of symmetry. The curve
2
y = 4ax is symmetrical about x-axis and hence x-axis or y = 0 is the axis of the
parabola y2 = 4ax. Note that the axis of the parabola passes through the focus
and perpendicular to the directrix.
Vertex : The point of intersection of the parabola and its axis is called its
vertex. Here, the vertex is V(0, 0).
Focal distance : The focal distance is the distance between a point on the
parabola and its focus.
Focal chord : A chord which passes through the focus of the parabola is
called the focal chord of the parabola.
Latus Rectum : It is a focal chord perpendicular to the axis of the
parabola. Here, the equation of the latus rectum is x = a.

175

End points of latus rectum and length of latus rectum :


y

To find the end points,


solve the equation of latus
rectum x = a and y2 = 4ax.
x = a in y2 = 4ax
Using
we get y2 = (4a)a = 4a2
y = 2a

F
L

Fig. 4. 23

If L and L are the end points of latus rectum then L is (a, 2a) and L is
(a, 2a). The length of latus rectum = LL = 4a. Length of semi-latus rectum
= FL = FL = 2a. So far we have discussed standard equation of a parabola
which is open rightward. But we have parabolas which are open leftward, open
upward and open downward.

4.3.4. Other standard parabolas :


1. Open leftward :
y2 = 4ax [a > 0]
If x > 0, then y becomes
imaginary. i.e., the curve does
not exist for x > 0 i.e., the
curve exist for x 0.
2. Open upward :
x2 = 4ay [a > 0]
If y < 0, then y becomes
imaginary. i.e., the curve does
not exist for y < 0 i.e., the
curve exist for y 0.

y2 = -4ax

x =a

F(-a,0)

Fig. 4. 24
x 2=

4ay

F(0, a)
x

y =- a

Fig. 4. 25
3. Open downward :
x2 = 4ay [a > 0]
If y > 0, then y becomes
imaginary. i.e., the curve does
not exist for y > 0 i.e., the
curve exist for y 0.

y
y =a

x 2=

- 4a

F(0,- a)

Fig. 4. 26

176

177

Remark : So far we have discussed four standard types of parabolas.


There are plenty of parabolas which cannot be classified under these
standard types. For example, consider the following parabolas.
y

y
x

x
x

Fig. 4. 27
For the above parabolas, the axes are neither parallel to x-axis nor parallel
to y-axis. In such cases the equation of the parabolas include xy term, which is
beyond the scope of this book, eventhough we will find the equation of the
parabolas which are not in standard form. Note that for the standard types the
axis is either parallel to x-axis or parallel to y-axis. We will study only these
four types.
All the parabolas discussed so far have vertex at the origin. In general the
vertex need not be at the origin for any parabola. Hence we need the concept of
shifting the origin or translation of the axes.

4.3.5 The process of shifting the origin or translation of axes :


Consider the xoy system. Draw a line parallel to x-axis (say X-axis) and
draw a line parallel to y-axis (say Y-axis). Let P(x, y) be a point with respect to
xoy system and P(X, Y) be the same point with respect to XOY system.
Let the co-ordinates of O with respect to xoy system be (h, k)
The co-ordinate of P with
Y
y
(X, Y)
respect to xoy system :
P(x, y)
OL = OM + ML = h + X
Y
O
X
i.e., x = X + h
k X
Similarly y = Y + k
h
x
O (0,0) M L
The new co-ordinates of P
with respect to XOY system
X=xh
Fig. 4. 28
Y=yk

178

4.3.6 General form of the standard equation of a parabola, which is


open rightward (i.e., the vertex other than origin) :
Consider a parabola with vertex V whose co-ordinates with respect to XOY
system is (0, 0) and with respect to xoy system is (h, k).
Since it is open rightward, the equation of the parabola w.r.t. XOY system
is Y2 = 4aX.
By shifting the origin X = x h and Y = y k, the equation of the parabola
with respect to old xoy system is (y k)2 = 4a(x h).
This is the general form of the standard equation of the parabola, which is
open rightward. Similarly the other general forms are
(y k)2 = 4a (x h) (open leftwards)
(x h)2 = 4a (y k) (open upwards)
(x h)2 = 4a (y k) (open downwards)
Note : To find the general form, replace x by x h and y by y k if the
vertex is (h, k)
Remark : The above form of equations do not have xy term.
Example 4.1: Find the equation of the following parabola with indicated focus
and directrix.
(i) (a, 0)

x=a

(ii) ( 1, 2)

x 2y + 3 = 0

(iii) (2, 3)

y2=0

a>0

Solution: (i) Let P(x, y) be any point on the parabola. If PM is drawn


perpendicular to the directrix,
x =- a
FP
PM = e = 1
P(x,y)
2
2
M

FP = PM
x + a
(x a) + (y 0) =

12

F(a,0)

(x a)2 + y2 = (x + a)2

y2 = 4ax
This is the required equation.

Fig. 4. 29

179

Alternative method :
From the given data, the parabola is
open rightward.
The equation of the parabola is of
the form (y k)2 = 4a (x h)
We know that the vertex is the
midpoint of Z( a, 0) and focus F(a, 0),
where Z is the point of intersection of the
directrix and the x-axis.

y
x =- a
F(a,0)
Z
(-a,0)

V (0,0)

Fig. 4. 30

a+a
Vertex is 2 ,

0 + 0
2 = (0, 0) = (h, k)
Again the distance between the vertex and the focus VF = a
The required equation is (y 0)2 = 4a (x 0) i.e., y2 = 4ax
(ii) Let P(x, y) be any point on the parabola. If PM is drawn perpendicular to
the directrix,
FP
P(x, y)
M
PM = e = 1
FP2 = PM2

(x + 1)2 + (y + 2)2 =

x 2y + 3

12+22

x - 2y + 3 = 0

F( -1, -2 )

Fig. 4. 31
4x2 + 4xy + y2 + 4x + 32y + 16 = 0
Note : Here the directrix is parallel to neither x-axis nor y-axis. This type
is not standard type. Therefore we cant do this problem as in the alternative
method of previous problem.
(iii) Let P(x, y) be any point on the parabola. If PM is drawn perpendicular to
the directrix
FP
M
P(x,y)
PM = e = 1
2

y 2
i.e., (x 2) + (y + 3) =

2
2

(x 2) + (y + 3) = (y 2)

y - 2 =0

FP2 = PM2

F(2,-3)

Fig. 4. 32

x 4x + 10y + 9 = 0

180

Note : Since the directrix y = 2 is parallel to x-axis, the type is standard


and hence this problem can be solved by alternative method of 4.1(i).
Example 4.2 : Find the equation of the parabola if
(i) the vertex is (0, 0) and the focus is ( a, 0), a > 0
(ii) the vertex is (4, 1) and the focus is (4, 3)
Solution: (i) From the given data the parabola is open leftward
y
The equation of the parabola is of the form
(y k)2 = 4a(x h)
Here, the vertex (h, k) is (0, 0) and VF = a
V (o,o)
x
The required equation is
F
2
(y 0) = 4a (x 0)
(-a,o)
2
y = 4ax
Fig. 4. 33
(ii) From the given data the parabola is open downward.
The equation is of the form
y
2
(x h) = 4a (y k)
x =4
Here, the vertex (h, k) is (4, 1) and
the distance between the vertex and
V(4,1)
the focus
x
VF = a
(4 4)2 + (1 + 3)2 = 4 = a
the required equation is
(x 4)2 = 4(4) (y 1)

F(4,-3)

Fig. 4. 34
(x 4)2 = 16(y 1)
Example 4.3: Find the equation of the parabola whose vertex is (1, 2) and the
equation of the latus rectum is x = 3.
Solution: From the given data the parabola is open rightward.
y
The equation is of the form
(y k)2 = 4a(x h)
Here, the vertex V (h, k) is (1, 2)
Draw a perpendicular from V to the latus
rectum.
It passes through the focus.
F is (3, 2)
Again VF = a = 2

181

x =3
V

F(3,2)

(1,2)

Fig. 4. 35

y =2
x

The required equation is


(y 2)2 = 4(2) (x 1)
(y 2)2 = 8(x 1)
Example 4.4: Find the equation of the parabola if the curve is open rightward,
vertex is (2, 1) and passing through point (6, 5).
Solution: Since it is open rightward, the equation of the parabola is of the form
(y k)2 = 4a(x h)
The vertex V(h, k) is (2, 1)
(y 1)2 = 4a (x 2)
But it passes through (6, 5)
42 = 4a (6 2) a = 1
The required equation is (y 1)2 = 4(x 2)
Example 4.5 : Find the equation of the parabola if the curve is open upward,
vertex is ( 1, 2) and the length of the latus rectum is 4.
Solution: Since it is open upward, the equation is of the form
(x h)2 = 4a(y k)
Length of the latus rectum = 4a = 4 and this gives a = 1
The vertex V (h, k) is ( 1, 2)
Thus the required equation becomes
(x + 1)2 = 4 (y + 2)
Example 4.6 : Find the equation of the parabola if the curve is open leftward,
vertex is (2, 0) and the distance between the latus rectum and directrix is 2.
Solution: Since it is open leftward, the equation is of the form
(y k)2 = 4a(x h)
The vertex V(h, k) is (2, 0)
The distance between latus rectum and directrix = 2a = 2 giving a = 1
and the equation of the parabola is
(y 0)2 = 4(1) (x 2)
or y2 = 4(x 2)
Example 4.7 : Find the axis, vertex, focus, directrix, equation of the latus
rectum, length of the latus rectum for the following parabolas and hence draw
their graphs.
(ii) x2 = 4y
(iii) (y + 2)2 = 8(x + 1)
(i) y2 = 4x
(v) x2 2x + 8y + 17 = 0
(iv) y2 8x + 6y + 9 = 0

182

Solution:
(i)

y2 = 4x

(y 0)2 = 4(1) (x 0)
Here (h, k) is (0, 0) and a = 1
y
Axis
: The axis of symmetry is
x-axis.
x = -1
Vertex : The vertex V (h, k) is (0, 0)
x =1
Focus : The focus F (a, 0) is (1, 0)
F(1,0)
x
Directrix : The equation of the directrix is
V (0,0)
x = a i.e. x = 1
Latus Rectum : The equation of the latus
Fig. 4. 36
rectum is x = a i.e. x = 1
and its length is 4a = 4(1) = 4
the graph of the parabola looks as in
Fig. 4.36.
(ii)
x2 = 4y
(x 0)2 = 4(1) (y 0)
y
Here (h, k) is (0, 0) and a = 1
y =1
Axis
: y-axis or x = 0
V (0,0)
x
Vertex
: V (0, 0)
y = -1
Focus
: F (0, a) i.e. F (0, 1)
F
Directrix
: y = a i.e. y = 1
(0,-1)
Latus rectum : y = a i.e. y = 1
: length = 4
Fig. 4. 37
the graph looks as in Fig. 4.37
(iii)

(y + 2)2 = 8 (x + 1)
Y2 = 8X

where X = x + 1

Y=y+2
a=2
Y = 4(2) X
The type is open leftward.
Referred to X, Y
Referred to x, y
X = x + 1, Y = y + 2
Axis
Y=0
Y=0 y+2=0
Vertex
(0, 0)
X=0 ; Y=0
x+1=0 ; y+2=0
x = 1, y = 2
V ( 1, 2)

183

Focus

( a, 0) i.e. ( 2, 0)

Directrix

X=a

Latus rectum

X=a

Length of
Latus rectum

4a = 8

i.e. X = 2
i.e. X = 2

X=2 ; Y=0
x + 1 = 2, y + 2 = 0
x = 3, y = 2
F ( 3, 2)
X=2 x+1=2
x=1
X=2 x+1=2
x=3
8

y
x =1

(0,0)
F
(-3,-2 )

V (-1,-2)

x
X

Fig. 4. 38
(iv)

y 8x + 6y + 9 = 0
y2 + 6y = + 8x 9
(y + 3)2 9 = + 8x 9
(y + 3)2 = 8x
Y2 = 8X
2

Y = 4(2)X
a=2
The type is open rightward
Referred to X, Y
Axis
Vertex

Y=0
(0, 0)

184

where X = x
Y = y+3

Referred to x, y
X=x,Y=y+3
Y=0 y+3=0
X=0 ; Y=0
x=0 ; y+3=0
V (0, 3)

Focus

(a, 0) i.e. (2, 0)

Directrix
Latus rectum
Length

X = a i.e. X = 2
X = a i.e. X = 2
4a = 8
x = -2

X=+2 ; Y=0
x = 2, y + 3 = 0
F (2, 3)
X=2 x=2
X=2 x=2
8

y
x
x =2

V
( 0,-3)

x2 2x + 8y + 17
x2 2x
(x 1)2 1
(x 1)2
(x 1)2
X2
X2
a
The type is open downward
(v)

F(2,-3)

y = -3
X

Fig. 4. 39
=0
= 8y 17
= 8y 17
= 8y 16
= 8(y + 2)
= 8Y
where X = x 1
= 4(2)Y
Y = y+2
=2
Y
y
x
V (1,-2)
y = -2

y = -4

F (1,-4)

Fig. 4. 40

185

Referred to X, Y

Referred to x, y
X=x1,Y=y+2
X=0 x1=0
x=1
X=0 ; Y=0
x 1 = 0, y + 2 = 0
V (1, 2)

Axis

X=0

Vertex

(0, 0)

Focus

(0, a) i.e. (0, 2)

X=0 ; Y=2
x 1 = 0, y + 2 = 2
F (1, 4)

Directrix

Y=a

Y=2 y+2=2
y=0

Latus rectum

Y=a

Length

4a = 8

i.e. Y = 2
i.e. Y = 2

Y=2 y+2= 2
y=4
8

4.3.7 Some practical problems :


Example 4.8 :
The girder of a railway bridge is in the parabolic form with span
100 ft. and the highest point on the arch is 10 ft. above the bridge. Find the
height of the bridge at 10 ft. to the left or right from the midpoint of the bridge.
y

Solution:
Girder

C
10
(-50,-10)

Fig. 4. 41
Consider the parabolic girder as open downwards
i.e.,

x2 = 4ay

It passes through (50, 10)


50 50 = 4a ( 10)

186

B
10
A

(50,-10)

250
a= 4
250
x2 = 4 4 y

x2 = 250y
Let B(10, y1) be a point on the parabola.
100 = 250y1
100
2
y1 = 250 = 5
Let AB be the height of the bridge at 10 ft to the right from the mid point
2
AC = 10 and BC = 5
2
3
AB = 10 5 = 9 5 ft
3
i.e. the height of the bridge at the required place = 9 5 ft.
Example 4.9 :
The headlight of a motor vehicle is a parabolic reflector of diameter 12cm
and depth 4cm. Find the position of bulb on the axis of the reflector for
effective functioning of the headlight.
Solution:
By the property of parabolic
reflector the position of the bulb
should be placed at the focus.
By taking the vertex at the
origin, the equation of the
reflector is y2 = 4ax
Let PQ be the diameter of the
reflector

12 cm

-- - 4 - -V

Fig. 4. 42
P is (4, 6) and since P(4, 6) lies on the parabola, 36 = 4a 4 a = 2.25
The focus is at a distance of 2.25cm from the vertex on the x-axis.

187

The bulb has to be placed at a distance of 2.25 cms from the centre of
the mirror.
Example 4.10 :
On lighting a rocket cracker it gets projected in a parabolic path and
reaches a maximum height of 4mts when it is 6 mts away from the point of
projection. Finally it reaches the ground 12 mts away from the starting point.
Find the angle of projection.
Solution:

V (0,0)
4

6 mts

12

(- 6, - 4)

Fig. 4. 43

6 mts
(6, - 4)

Fig. 4. 44
2

The equation of the parabola is of the form x = 4ay (by taking the vertex at
the origin). It passes through (6, 4)
9
36 = 16a a = 4
The equation is
x2 = 9y
(1)
Find the slope at ( 6, 4)
Differentiating (1) with respect to x, we get
dy
dy
2
2x = 9 dx dx = 9 x
dy
2
4
4
At ( 6, 4), dx = 9 6 = 3 i.e. tan = 3
4
= tan1 3

4
The angle of projection is tan1 3

Example 4.11 :
A reflecting telescope has a parabolic mirror for which the distance from
the vertex to the focus is 9mts. If the distance across (diameter) the top of the
mirror is 160cm, how deep is the mirror at the middle?

188

Solution:
Let the vertex be at the origin.
VF = a = 900
The equation of the parabola is
y2 = 4 900 x
Let x1 be the depth of the mirror at the
middle
Since (x1, 80) lies on the parabola
16
802 = 4 900 x1 x1 = 9
16
depth of the mirror = 9 cm
Example 4.12 :

(x1, 80)

x1

V(0,0)

----------900------

(x1, - 80)

Fig. 4. 45

Assume that water issuing from the end of a horizontal pipe, 7.5m above
the ground, describes a parabolic path. The vertex of the parabolic path is at the
end of the pipe. At a position 2.5m below the line of the pipe, the flow of water
has curved outward 3m beyond the vertical line through the end of the pipe.
How far beyond this vertical line will the water strike the ground?
Solution:
y

Pipe line
2.5 mts
7.5 mts

V (0,0)

3 mts

2.5

x
P(3, -2.5)

7.5

Ground
A

Fig. 4. 46

x1

Q (x1, -7.5)

Fig. 4. 47
As per the given information, we can take the parabola as open downwards
i.e. x2 = 4ay
Let P be the point on the flow path, 2.5m below the line of the pipe and 3m
beyond the vertical line through the end of the pipe.
P is (3, 2.5)
Thus 9 = 4a ( 2.5)
9
a = 10

189

9
The equation of the parabola is x2 = 4 10 y
Let x1 be the distance between the bottom of the vertical line on the ground
from the pipe end and the point on which the water touches the ground. But the
height of the pipe from the ground is 7.5 m
The point (x1, 7.5) lies on the parabola
9
x12 = 4 10 ( 7.5) = 27
x1 = 3 3
The water strikes the ground 3 3 m beyond the vertical line.
Example 4.13 :
A comet is moving in a parabolic orbit around the sun which is at the focus
of a parabola. When the comet is 80 million kms from the sun, the line segment

from the sun to the comet makes an angle of 3 radians with the axis of the orbit.
find (i) the equation of the comets orbit (ii) how close does the comet come
nearer to the sun? (Take the orbit as open rightward).
Solution:
Take the parabolic orbit as open
rightward and the vertex at the origin.
Let P be the position of the comet in
which FP = 80 million kms.
Draw a perpendicular PQ from P to the
axis of the parabola.
Let
FQ = x1
From the triangle FQP,

PQ = FP . sin 3
3
= 80 2 = 40 3

y
Comet
P
80
V

/3
F )
Sun x1 Q

Fig. 4. 48

1
Thus FQ = x1 = FP . cos 3 = 80 2 = 40
VQ = a + 40 if VF = a ; P is (VQ, PQ) = (a + 40, 40 3)
Since P lies on the parabola y2 = 4ax

190

(40 3)

= 4a(a + 40)

a = 60 or 20
a = 60 is not acceptable.
The equation of the orbit is
y2 = 4 20 x
y2 = 80x
The shortest distance between the Sun and the Comet is VF i.e. a
The shortest distance is 20 million kms.
Example 4.14 :
A cable of a suspension bridge hangs in the form of a parabola when the
load is uniformly distributed horizontally. The distance between two towers is
1500 ft, the points of support of the cable on the towers are 200ft above the road
way and the lowest point on the cable is 70ft above the roadway. Find the
vertical distance to the cable (parallel to the roadway) from a pole whose height
is 122 ft.
Solution :
B

D
Q

P
C
C

750

70

130

52

70

70

A
A

Fig. 4. 49
Take the lowest point on the cable as the vertex and take it as origin. Let
AB and CD be the towers. Since the distance between the two towers is 1500 ft.
VA = 750 ft ; AB = 200 ft
AB = 200 70 = 130 ft
Thus the point B is (750, 130)
The equation of the parabola is x2 = 4ay
Since B is a point on x2 = 4ay
(750)2 = 4a(130)
75 750
4a =
13

191

The equation is x2 =

75 750
y
13

Let PQ be the vertical distance to the cable from the pole RQ.
RQ = 122, RR = 70 RQ = 52
Let VR be x1 Q is (x1, 52)
Q is a point on parabola
x12 =

75 750
52
13

x1 = 150 10
PQ = 2x1 = 300 10 ft.
EXERCISE 4.1
(1) Find the equation of the parabola if
(i)
Focus : (2, 3) ; directrix : 2y 3 = 0
(ii) Focus : ( 1, 3) ; directrix : 2x + 3y = 3
(iii) Vertex : (0, 0) ; focus : (0, 4)
(iv) Vertex : (1, 4) ; focus : ( 2, 4)
(v) Vertex : (1, 2) ; latus rectum : y = 5
(vi) Vertex : (1, 4) ; open leftward and passing through the point
( 2, 10)
(vii) Vertex : (3, 2) ; open downward and the length of the latus
rectum is 8.
(viii) Vertex : (3, 1) ; open rightward ; the distance between the
latus rectum and the directrix is 4.
(ix) Vertex : (2, 3) ; open upward ; and passing through the point
(6, 4).
(2) Find the axis, vertex, focus, equation of directrix, latus rectum, length of
the latus rectum for the following parabolas and hence sketch their
graphs.
(i) y2 = 8x

(ii) x2 = 20y

(iii) (x 4)2 = 4(y + 2)

(iv) y2 + 8x 6y + 1 = 0

(v) x2 6x 12y 3 = 0
(3) If a parabolic reflector is 20cm in diameter and 5cm deep, find the
distance of the focus from the centre of the reflector.

192

(4) The focus of a parabolic mirror is at a distance of 8cm from its centre
(vertex). If the mirror is 25cm deep, find the diameter of the mirror.
(5) A cable of a suspension bridge is in the form of a parabola whose span is
40 mts. The road way is 5 mts below the lowest point of the cable. If an
extra support is provided across the cable 30 mts above the ground level,
find the length of the support if the height of the pillars are 55 mts.

4.4 Ellipse :
Definition : The locus of a point in a plane whose distance from a fixed point bears
a constant ratio, less than one to its distance from a fixed line is called ellipse.
Note : Eventhough the syllabus does not require the derivation of
standard equation and the tracing of ellipse (4.4.1, 4.4.2) and it
needs only the standard equation and the diagram, the equation is
derived and the curve is traced for better understanding.
Now, we will derive the standard equation of an ellipse.

4.4.1 Standard equation of the ellipse :


Given :
y
x =a / e
Fixed point F
Fixed line l
P(x,y)
M
Eccentricity e (e < 1)
Moving point P (x, y)
(a,0)
C
x
Construction :
Z
F(ae,0) A
A (-a,0)
(a/e, 0)
Plot the fixed point F
and draw the fixed
Fig. 4. 50
line l
Drop a perpendicular (FZ) from F to l
Drop a perpendicular (PM)
Plot the points A, A which divides FZ internally and externally in the ratio e : 1
Take AA = 2a and treat it as x-axis.
Draw a perpendicular bisector to AA and treat it as y-axis.
Let C be the origin.
The known points are the origin C(0, 0), A(a, 0), A( a, 0)
To find the co-ordinates of F and M, do the following :
Since A, A divides FZ internally and externally in the ratio e : 1 respectively,
193

FA
e
=1
AZ

FA
e
AZ = 1
FA = e AZ
i.e., CA CF = e (CZ CA)
a CF = e (CZ a) (1)
(2) + (1)
(2) (1)

a + CF = e(a + CZ)
a
2a = e [2CZ] CZ = e

FA = e AZ
i.e. AC + CF = e (AC + CZ)

2CF = e(2a)

(2)

CF = ae

a
M is e, y and F is (ae, 0)

To obtain the equation of the ellipse, do the following :


Since P is any point on the ellipse
FP
2
2
2
PM = e FP = e PM

a
i.e. (x ae)2 + (y 0)2 = e2x e + (y y)2
2

x2 e2x2 + y2 = a2 a2e2
(1 e2) x2 + y2 = a2 (1 e2)
Dividing by a2 (1 e2), we get
x2
y2
+
=1
a2
a2(1 e2)
x2 y2
+
= 1, where b2 = a2 (1 e2)
a2 b2
This is known as the standard equation of an ellipse.
i.e.

x2 y2
4.4.2 Tracing of the ellipse 2 + 2 = 1, a > b
a
b
(i) Symmetry property :
It is symmetrical about x-axis and y-axis simultaneously and hence about
the origin.
(ii) Special points :
It does not pass through the origin.

194

To find the points on x-axis, put y = 0, we get x = a. Therefore the curve


meets the x-axis at A(a, 0) and A( a, 0).
To find the points on y-axis, put x = 0, we get y = b. Therefore the curve
meets the y-axis at B(0, b) and B(0, b)
(iii) Existence of the curve :
b
Write the equation of the ellipse as y = a
a2 x2 . y is real only if
a2 x2 0. i.e., the curve does not exist for a2 x2 < 0 or x2 a2 > 0
Equivalently the curve does not exist for x > a and x < a. Thus the curve
exists only when a x a.
a
Write the equation of the ellipse as x = b
b2 y2 . x is real only if
b2 y2 0. The curve does not exist for b2 y2 < 0 i.e., y2 b2 > 0 i.e., the
curve does not exist for y > b and y < b. The curve exist only when
b y b. Ellipse is a closed curve bounded by the lines x = a and
y = b. Thus the curve is
y
x = - a/e
M

Z
(- a/e, 0)

A
(- a,0)

x = a/e

C
(0,0)

P(x,y)

F1
(ae,0)

A
Z
(a,0) (a/e, 0)

Fig. 4. 51
4.4.3 Important definitions regarding ellipse :
Focus : The fixed point is called focus, denoted as F1 (ae, 0).
Directrix : The fixed line is called directrix l of the ellipse and its equation
a
is x = e .
Major axis : The line segment AA is called the major axis and the length
of the major axis is 2a. The equation of the major axis is y = 0.

195

Minor axis : The line segment BB is called the minor axis and the length
of minor axis is 2b. Equation of the minor axis is x = 0. Note that the length of
major axis is always greater than minor axis.
Centre : The point of intersection of the major axis and minor axis of the
ellipse is called the centre of the ellipse. Here C(0, 0) is the centre of the ellipse.
Note that the centre need not be the origin of the ellipse always.
End points of latus rectum and length of latus rectum :
To find the end points, solve x = ae (1)

and

x2 y2
+
= 1 (2)
a2 b2

Using (2) in (1) we get


y2
a2 e2
+
=1
a2
b2

y2
= 1 e2
b2

y2 = b2 (1 e2)

b2 = a2 (1 e2)
b
= b 2 2 b2
2
a or 2 = 1 e
a
2

b2
y= a

b2
If L1 and L1 are the end points of the latus rectum then L1 is ae, a and

b2
L1 is ae, a .

b2
The end points of the other latus rectum are ae, a .

2b2
The length of the latus rectum is a .
For the above discussed ellipse, the major axis is along x-axis. There is
another standard ellipse in which the major axis is along the y-axis.
Vertices : The points of intersection of the ellipse and its major axis are
called its vertices. Here the vertices of the ellipse are A(a, 0) and A( a, 0).

196

Focal distance : The focal distance with respect to any point P on the
ellipse is the distance of P from the referred focus.
Focal chord : A chord which passes through the focus of the ellipse is
called the focal chord of the ellipse.
A special property : Thanks to the symmetry about the origin, it permits
(i) the second focus F2 ( ae, 0)
a
(ii) the second directrix x = e
Latus rectum : It is a focal chord perpendicular to the major axis of the
ellipse. The equations of latus rectum are x = ae, x = ae.
Eccentricity
e=

b2
1 2
a

Remark :
b
In the case of an ellipse 0 < e < 1. As e 0, a 1 i.e., b a or the
length of the minor and major axes are close in size. i.e., the ellipse is close to
being a circle.
b
As e 1, a 0 and the ellipse degenerates into a line segment
(degenerate conic) i.e., the ellipse is flat.

4.4.4 The other standard form of the ellipse :


If the major axis of the ellipse is along the y-axis, then the equation of the
x2 y2
ellipse takes the form 2 + 2 = 1, for a > b.
b
a
For this type of ellipse, we have the following as explained in the earlier
ellipse.
Centre
: C (0, 0)
Vertices
: A (0, a), A (0, a)
Foci

F1 (0, ae), F2 (0, ae)

Equation of major axis is x = 0


Equation of minor axis is y = 0
End points of minor axis : B (b, 0), B( b, 0)
Equation of directrices : y = a/e

197

b2
b2

End points of latus rectums : a , ae , a , ae

y
y =a/e

A(0,a)

(-b2/a ,ae) L11

L1 (-b2/a,ae)

B1

(-b2/a -,ae)L21

L2 (b2/a,-ae)

A(0,-a)

y =- a/e

Fig. 4. 52
4.4.5 General forms of standard ellipses :
To obtain the general forms of standard ellipses, replace x by x h and y
by (y k) if the centre is C(h, k).
Thus the general forms of standard ellipses are

(y k)2
(x h)2
+
= 1,
a2
b2

(x h)2 (y k)2
+
= 1, a > b
b2
a2
Focal property of an ellipse :
The sum of the focal distances of any point on an ellipse is constant and is
equal to the length of the major axis.
y
x = - a/e

x = a/e
P

M
Z

F2 C

F1

Fig. 4. 53

198

M
Z

Proof :
To prove : F1P + F2P = 2a
Let P be a point on the ellipse. Drop the perpendiculars PM and PM to the
a
a
directrices x = e and x = e respectively .
F1P
F2P
We know that PM = e,
=e
PM
F1P = ePM, F2P = e PM
F1P + F2P = e(PM + PM)
= e(MM)
2a
= e. e
= 2a
= length of the major axis
Example 4.15 : Find the equation of the ellipse whose foci are (1, 0) and
1
( 1, 0) and eccentricity is 2 .
Solution:
The centre of the ellipse is the midpoint of FF where F is (1, 0) and F is
( 1, 0).
y
11 0+0
Centre C is 2 , 2 = (0, 0)

1
C
x
But F1F2 = 2ae = 2 and e = 2
F1 (1,0)
(-1,0) F2
1
2a 2 = 2
Fig. 4. 54
a=2
1
b2 = a2 (1 e2) = 4 1 4 = 3

From the given data the major axis is along x-axis.


the equation of the ellipse is of the form
(y k)2
x2
y2
(x h)2
+
= 1 4 + 3 =1
2
2
a
b

199

Example 4.16 : Find the equation of the ellipse whose one of the foci is (2, 0)
1
and the corresponding directrix is x = 8 and eccentricity is 2
Solution:
Let P(x, y) be a moving point. By definition
FP
PM = e

x =8

y
P(x,y)

FP2 = e2 PM2

M (8,y)

1
x 8
(x 2)2 + (y 0)2 = 4

1
(x 2)2 + y2 = 4 (x 8)2
2

4 [(x 2) + y ] = (x 8)

F(2,0)

Fig. 4. 55

3x2 + 4y2 = 48
x2 y2
16 + 12 = 1
Aliter :
From the given data, the major
axis is along the x-axis and the
equation of the ellipse may be
taken as

y
x =8

x2 y2
+
=1
a2 b2
a
FZ = e ae = 6
1
1
But e = 2 2a 2 a = 6
3
2a = 6 a=4

F(2,0)

Z(8,0)

Fig. 4. 56

1
3
b2 = a2 (1 e2) = 16 1 4 = 16 4 = 12

x
y
The required equation is 16 + 12 = 1

200

Example 4.17 : Find the equation of the ellipse with focus ( 1, 3), directrix
4
x 2y = 0 and eccentricity 5

FP2 = e2 PM2
16
x 2y
(x + 1) + (y + 3) = 25

1 + 4
2

x - 2y =0

Solution:
Let P(x, y) be a moving point. By definition
FP
PM = e

F(-1,-3)

125 [(x + 1)2 + (y + 3)2 ] = 16 (x 2y)2


2

P(x,y)

Fig. 4. 57

109x + 64xy + 61y + 250x + 750y + 1250 = 0


Example 4.18 : Find the equation of the ellipse with foci ( 4, 0) and vertices
( 5, 0)
Solution:
y
Let the foci be F1(4, 0) and
F2 ( 4, 0), vertices be A(5, 0)
and A( 5, 0). The centre is
C (0,0) F1 A
A F2
the midpoint of AA
x
5+5 0+0
i.e., C is 2 , 2

= (0, 0)

(-5,0) (-4,0)

(4,0) (5,0)

Fig. 4. 58
From the given data, the major axis is along the x-axis and the equation of
the ellipse is of the form
x2 y2
+
=1
a2 b2
Here CA = a = 5
4
since e = 5

CF = ae = 4

b2 = a2 (1 e2) = 25 16 = 9 and the


x2 y2
required equation of the ellipse is 25 + 9 = 1

201

Example 4.19 : The centre of the ellipse is (2, 3). One of the foci is (3, 3). Find
the other focus.
Solution:
From the given data the major axis is
parallel to the x axis. Let F1 be (3, 3)
F
C
F
2

Let F2 be the point (x, y). Since


C (2, 3) is the midpoint of F1 and F2 on
the major axis y = 3
x+3
y+3
2 = 2 and 2 = 3

y =3

Fig. 4. 59

This gives x = 1 and y = 3. Thus the other focus is (1, 3).


Example 4.20 : Find the equation of the ellipse whose centre is (1, 2), one of
1
the foci is (1, 3) and eccentricity is 2
Solution:
The major axis is parallel to y-axis.
The equation is of the form

y
x =1
F1(1,3)
C(1,2)

(x h)2 (y k)2
+
=1
b2
a2
CF1 = ae = 1

1
But e = 2 a = 2, a2 = 4

Fig. 4. 60

1
b2 = a2(1 e2) = 41 4 = 3 ; C(h, k) = (1, 2)

The required equation is

(x 1)2 (y 2)2
+
=1
3
4

Example 4.21 : Find the equation of the ellipse whose major axis is along
1
x-axis, centre at the origin, passes through the point (2, 1) and eccentricity 2
Solution:
Since the major axis is along the x-axis and the centre is at the origin, the
x2 y2
equation of the ellipse is of the form 2 + 2 = 1
a
b

202

(1)

It passes through the point (2, 1). 2 + 2 = 1


a b
1
e=2

1
b2 = a2 (1 e2) b2 = a2 1 4

(2)
4b = 3a
16
2
2
Solving (1) and (2) we get
a = 3 , b =4
2
x
y2
The required equation is 16/3 + 4 = 1
Example 4.22 : Find the equation of the ellipse if the major axis is parallel to
y-axis, semi-major axis is 12, length of the latus rectum is 6 and the centre is (1,
12)
Solution:
Since the major axis is parallel to y-axis the equation of the ellipse is of the
form
(x h)2 (y k)2
+
=1
b2
a2
The centre C (h, k) is (1, 12)
a = 12 a2 = 144
Semi major axis
2b2
2b2
Length of the latus rectum a = 6 12 = 6
(x 1)2 (y 12)2
+ 144
=1
36
Example 4.23 : Find the equation of the ellipse given that the centre is (4, 1),
focus is (1, 1) and passing through (8, 0).
Solution :
y
From the given data since the
major axis is parallel to the x axis,
x
C(4,-1)
the equation is of the form
y =-1
b2 = 36 and the required equation is

F2(1,-1)

(x h)2 (y k)2
+
=1
a2
b2
The centre C(h, k) is (4, 1)
2

Fig. 4. 61

(x 4)
(y + 1)
+
=1
a2
b2

203

16 1
It passes through (8, 0) 2 + 2 = 1
a
b
But CF1 = ae = 3

(1)

b2 = a2 (1 e2) = a2 a2 e2 = a2 9
16
1
=1
2+ 2
a
a 9

16a2 144 + a2 = a4 9a2

a4 26a2 + 144 = 0
a2 = 18 or 8

Case (i)

(1)

a2 = 18
b2 = a2 9 = 18 9 = 9

Case (ii)

a2 = 8
b2 = 8 9 = 1 which is not possible

Thus the equation is

a2 = 18,

b2 = 9

(x 4)2 (y + 1)2
+
=1
18
9

Example 4.24 : Find the equation of the ellipse whose foci are (2, 1), ( 2, 1)
and length of the latus rectum is 6.
Solution :
From the given data the major axis is
parallel to the x axis.
The equation is of the form
(x h)2 (y k)2
+
=1
a2
b2
Since the centre is the midpoint of
F1F2
2 + 2 1 + 1
, 2
C is 2

= (0, 1)
and the equation becomes
x2 (y 1)2
+
=1
a2
b2

204

F2(-2,1)

F1(2,1)

y =1
x

Fig. 4. 62

F1 F2 = 2ae = 4 a2 e2 = 4
a2 e2 = a2 b2
a2 b 2 = 4
2b
a

The length of the latus rectum

a2 3a 4
a
a
a

(1)

=6
=
=
=
=

(1)
b2 = 3a

0
4 or 1
1 is absurd
4

(2)
(by (2))

b2 = 3a = 12
x2 (y 1)2
=1
16 + 12

Thus the equation is

Example 4.25 : Find the equation of the ellipse whose vertices are ( 1, 4) and
1
( 7, 4) and eccentricity is 3 .
y
Solution :
y
From the given data the major axis is
parallel to x axis.
A
A
y =4
The equation is of the form
(-7,4)
(-1,4)
(x h)2 (y k)2
+
=1
a2
b2

The centre is the midpoint of AA


1 7 4 + 4
, 2
C is 2

= ( 4, 4)
Thus the equation becomes

Fig. 4. 63

(y 4)2
(x + 4)2
+
=1
a2
b2
AA = 2a = 6

a=3
1
b2 = a2 (1 e2) = 9 1 9 = 8

We know that

The required equation is

(x + 4)2
(y 4)2
+
=1
9
8

205

Example 4.26 : Find the equation of the ellipse whose foci are (1, 3) and (1, 9)
1
and eccentricity is 2
y
Solution :
x =1
From the given data the major axis is
parallel to y axis.
F2(1,9)
The equation is of the form
(x h)2 (y k)2
+
=1
b2
a2
The centre of the ellipse is the
midpoint of F1 F2
1+1 3+9
C is 2 , 2 = (1, 6)

F1(1,3)
x

Fig. 4. 64

F1 F2 = 2ae = 6
ae = 3
1
But e = 2

a=6

1
b2 = a2 (1 e2) = 36 1 4 = 27

Thus the required equation is


(x 1)2
(y 6)2
+
27
36 = 1
Property (without proof) :
A point moves such that the sum of its distances from two fixed points in a
plane is a constant. The locus of this point is an ellipse.
Example 4.27 : Find the equation of a point which moves so that the sum of its
distances from ( 4, 0) and (4, 0) is 10.
y
P(x,y)

F1(4,0)

F2(-4,0)

Fig. 4. 65

206

Solution :
Let F1 and F2 be the fixed points (4, 0) and ( 4, 0) respectively and
P(x1, y1) be the moving point.
It is given that
i.e.,

F1P + F2P = 10

(x1 4)2 + (y1 0)2 +

(x1 + 4)2 + (y1 0)2 = 10

Simplifying we get
9x12 + 25y12 = 225. The locus of (x1, y1) is
x2
y2
+
25
9 =1
Example 4. 28 : Find the equations and lengths of major and minor axes of
(x 1)2 (y + 1)2
x2 y2
(iii)
+ 16
=1
(i) 9 + 4 = 1 (ii) 4x2 + 3y2 = 12
9
Solution :
(i) The major axis is along x-axis and the minor axis is along y-axis. This
gives the equation of major axis as y = 0 and the equation of minor axis as
x = 0. We have a2 = 9 ; b2 = 4 a = 3, b = 2
The length of major axis is 2a = 6 and the length of minor axis is 2b = 4
(ii)

x2 y2
3 + 4 =1

The major axis is along y-axis and the minor axis is along x-axis.
The equation of major axis is x = 0 and the equation of minor axis is
y = 0. Here a2 = 4 ; b2 = 3 a = 2, b = 3
The length of major axis (2a) = 4
The length of minor axis (2b) = 2 3
(iii) Let x 1 = X and y + 1 = Y
X2 Y2
The given equation becomes 9 + 16 = 1 Clearly the major axis is along
Y-axis and the minor axis is along X-axis.
The equation of major axis is X = 0 and the equation of minor axis is
Y=0
i.e., the equation of major axis is x 1 = 0 and the equation of minor axis
is y + 1 = 0

207

Here a2 = 16, b2 = 9
a = 4, b = 3
Length of major axis (2a) = 8
Length of minor axis (2b) = 6
Example 4. 29 : Find the equations of axes and length of axes of the ellipse
6x2 + 9y2 + 12x 36y 12 = 0
Solution :
6x2 + 9y2 + 12x 36y 12 = 0
(6x2 + 12x) + (9y2 36y) = 12
6(x2 + 2x) + 9(y2 4y) = 12
6{(x + 1)2 1} + 9 {(y 2)2 4} = 12
6(x + 1)2 + 9 (y 2)2 = 12 + 6 + 36

Let X = x + 1 ;

6(x + 1)2 + 9(y 2)2 = 54


(x + 1)2
(y 2)2
+
=1
9
6
Y=y2

X 2 Y2
9 + 6 =1
Clearly the major axis is along X-axis and the minor axis is along Y-axis.
The equation of the major axis is Y = 0 and the equation of the minor
axis is X = 0.
The equation of the major axis is y 2 = 0 and of minor axis is x + 1 = 0
i.e., the equation of the major axis is y 2 = 0
The equation becomes

Here a2 = 9, b2 = 6 a = 3, b = 6
The length of major axis (2a) = 6
The length of minor axis (2b) = 2 6
Example 4.30 : Find the equations of directrices, latus rectum and length of
latus rectums of the following ellipses.
x2 y2
(i) 16 + 9 = 1 (ii) 25x2 + 9y2 = 225
Solution :
(i) The major axis is along x-axis
Here a2 = 16, b2 = 9

208

(iii) 4x2 + 3y2 + 8x + 12y + 4 = 0

b2
1 2 =
a
a
x = e

e=
Equations of directrices are

x=
Equations of the latus rectums are

7
9
1 16 = 4

16
7

x = ae
x= 7

Length of the latus rectum


(ii) 25x2 + 9y2 = 225
Here a2 = 25,

2b
a

29
9
4 = 2

x2 y2
9 + 25 = 1

b2 = 9
e=

b2
1 2 =
a

9
4
1 25 = 5

a
y =e

The equations of the directrices are

y=

25
4

Equations of the latus rectum are y = ae


y = 4
Length of the latus rectum is
(iii)

2b2
29
18
a = 5 = 5

4x2 + 3y2 + 8x + 12y + 4 = 0


(4x2 + 8x) + (3y2 + 12y) + 4 = 0
4(x2 + 2x) + 3(y2 + 4y) = 4
4{(x + 1)2 1} + 3 {(y + 2)2 4} = 4
4(x + 1)2 + 3(y + 2)2 = 12
(y + 2)2
(x + 1)2
+
=1
3
4

209

X 2 Y2
3 + 4 = 1 where X = x + 1 , Y = y + 2
The major axis is along Y axis. Here a2 = 4,

1
b2 = 3 and e = 2

a
Y =e

Equations of the directrices are

2
i.e. Y = (1/2)

Y = 4
(i) Y = 4
y+2=4
y=2
(ii) Y = 4 y + 2 = 4 y = 6
The directrices are y = 2 and y = 6
Y = ae

Equations of the latus rectum are

1
i.e. Y = 2 2

Y = 1
y+2=1
y=1
(ii) Y = 1 y + 2 = 1
y=3
Equation of the latus rectum are y = 1 and y = 3

(i) Y = 1

23
2b2
Length of the latus rectum is a = 2 = 3
Example 4.31 : Find the eccentricity, centre, foci, vertices of the following
x2 y2
x2 y2
(ii) 4 + 9 = 1
ellipses : (i) 25 + 9 = 1
(iii)

(x + 3)2 (y 5)2
+
=1
6
4

(iv) 36x2 + 4y2 72x + 32y 44 = 0

x2 y2
Solution : (i)25 + 9 = 1
The major axis is along x-axis a2 = 25, b2 = 9
4
e = 5 and ae = 4
Clearly centre C is (0, 0),
A F2
Foci are ( ae, 0) = ( 4, 0)
(-5,0) (-4,0)
Vertices are ( a, 0) = ( 5, 0)

C (0,0)

(4,0) (5,0)

Fig. 4. 66

210

F1

(ii) The major axis is along y-axis a2 = 9, b2 = 4


5
e = 3 and ae = 5
Clearly centre C is (0, 0)
Foci are (0, ae) = (0, 5 )
Vertices are (0, a) = (0, 3)

y
A(0,3)
F1 (0,5)

C (0,0)

F2 (0,-5)

A(0,-3)

Fig. 4. 67
(iii) Let x + 3 = X, y 5 = Y
Y2
X2
The equation becomes 6 + 4 = 1
The major axis is along X-axis
a2 = 6, b2 = 4
1
and ae = 2
e=
3
Referred to X, Y
Centre

(0, 0)

( a, 0) i.e. ( 6, 0)
(i) ( 6, 0)
Vertices

(ii) ( 6, 0)

Referred to x, y
X=x+3; Y=y5
X = 0 ; Y=0

x + 3 = 0, y 5 = 0
x = 3, y = 5
Centre C( 3, 5)
(i)
X = 6, Y=0
x+3 = 6 , y5=0
x = 6 3, y = 5
A ( 3 + 6, 5)
(ii)

211

X = 6,Y=0
x+3 = 6,y5=0
x = 3 6 ,y=5
A ( 3 6, 5)

( ae, 0)
i.e. ( 2 , 0)

(i)

X = 2, Y=0
x+3 = 2, y5=0
x = 3+ 2,y=5
F1 ( 3 + 2 , 5)

(ii)

X = 2 , Y=0
x+3 = 2,y5=0
x = 3 2,y=5
F2 ( 3 2 , 5)

(i) ( 2 , 0)
foci

(ii) ( 2, 0)

Y
A1
(-3 - 6, 5)

F2

C (-3,5) F1

AA
(-3 +6, 5)
(-3+2,5)

(-3- 2, 5)

y =5
X

Fig. 4. 68
(iv) 36x2 + 4y2 72x + 32y 44 = 0
36 (x2 2x) + 4 (y2 + 8y) = 44
36 {(x 1)2 1} + 4 {(y + 4)2 16} = 44
36(x 1)2 + 4 (y + 4)2 = 144
(x 1)2
(y + 4)2
+
=1
4
36
i.e.,

X2
Y2
+
4
36 = 1 where X = x 1 , Y = y + 4

The major axis is along Y-axis.


a2 = 36, b2= 4
2 2
e = 3 and ae = 4 2

212

Referred to x, y
X=x1; Y=y+4
X = 0 ; Y=0

x 1 = 0, y + 4 = 0
x = 1, y = 4
Centre C(1, 4)
(i)
X = 0, Y = 6
x1 = 0, y+4=6
x = 1, y = 2
A (1, 2)
X = 0, Y = 6
(ii)
x 1 = 0, y + 4 = 6
x1=0,y+4=6
x = 1, y = 10
A (1, 10)
(i)
X = 0; Y=4 2
x 1 = 0, y + 4 = 4 2
x = 1, y = 4 2 4
F1 (1, 4 2 4)

Referred to X, Y
Centre

(0, 0)

(0, a) i.e. (0, 6)


(i) (0, 6)
Vertices
(ii) (0, 6)

(0, ae)
i.e. (0, 4 2)
(i) (0, 4 2)
Foci
(ii) (0, 4 2)

(ii)

X = 0, Y=4 2
x 1= 0 ; y + 4 = 4 2
x = 1, y = 4 4 2
F2 (1, 4 4 2)

y
Y

x =1
A(1,2)
F1

C (1,-4)
F2

A (1,-10)

Fig. 4. 69

213

x
X
Where
F1(1, 4 2 4)
F2(1, - 4 4 2)

4.4.6 Some practical problems :


Example 4.32 : An arch is in the form of a semi-ellipse whose span is 48 feet
wide. The height of the arch is 20 feet. How wide is the arch at a height of 10
feet above the base?
Solution :
Take the mid point of the
(x1,10)
base as the centre C (0, 0)
20
Since the base wide is 48 feet,
10
x1
Q
P
the vertices A and A are (24, 0)
A
C (0,0)
A
and ( 24, 0) respectively.
(24,0)
(-24,0)
Fig. 4. 70
Clearly 2a = 48 and b = 20.
x2
y2
+
=1
(1)
242 202
Let x1 be the distance between the pole whose height is 10m and the centre.

The corresponding equation is

Then (x1, 10) satisfies the equation (1)

x12
24

102
=1
202

x1 = 12 3

Clearly the width of the arch at a height of 10 feet is 2x1 = 24 3


Thus the required width of arch is 24 3 feet.
Example 4.33 : The ceiling in a hallway 20ft wide is in the shape of a semi
ellipse and 18 ft high at the centre. Find the height of the ceiling 4 feet from
either wall if the height of the side walls is 12ft.
Solution :
R (6, y1)
Let PQR be the height of the
ceiling which is 4 feet from the
6
10
6
4
wall.
A
A
C (0,0)
Q
(10,0)
(-10,0)
From the diagram PQ = 12 ft
To find the height QR
12
Since the width is 20ft, take A,
A as vertices with A as (10, 0) and
P
A as ( 10, 0). Take the midpoint
Fig. 4. 71
of AA as the centre which is (0, 0)

214

From the diagram


and

AA = 2a = 20 a = 10
b = 18 12 = 6

x2
y2
+
100
36 = 1
Let QR be y1 then R is (6, y1)

Since R lies on the ellipse,


2

y1
36
100 + 36 = 1 y1 = 4.8
PQ + QR = 12 + 4.8
The required height of the ceiling is 16.8 feet.
Example 4.34 : The orbit of the earth around the sun is elliptical in shape with
sun at a focus. The semi major axis is of length 92.9 million miles and
eccentricity is 0.017. Find how close the earth gets to sun and the greatest
possible distance between the earth and the sun.
Solution :
y
Semi-major axis CA is
H
a = 92.9 million miles
RT
EA
Given e = 0.017
SUN A
A
x
The closest distance of the
C (0,0)
F
earth from the sun = FA
and farthest distance of the
earth from the sun = FA
Fig. 4. 72
CF = ae = 92.9 0.017
FA = CA CF = 92.9 92.9 0.017
= 92.9 [1 0.017]
= 92.9 0.983 = 91.3207 million miles
FA = CA + CF = 92.9 + 92.9 0.017
= 92.9 (1 + 0.017)
= 92.9 1.017 = 94.4793 million miles
Example 4.35 : A ladder of length 15m moves with its ends always touching
the vertical wall and the horizontal floor. Determine the equation of the locus of
a point P on the ladder, which is 6m from the end of the ladder in contact with
the floor.

215

Solution :
Let AB be the ladder and P(x1, y1) be
a point on the ladder such that AP = 6m.
Draw PD perpendicular to x-axis and
PC perpendicular to y-axis.
Clearly the triangles ADP and PCB
are similar.
PC
PB
BC
DA = AP = PD
i.e.,

B
9
C

x1

y1
O

x1
9
BC
DA = 6 = y1

P(x1y1)
6

y1
x1

Fig. 4. 73

6x1
2x1
DA = 9 = 3

9y1
3
; BC = 6 = 2 y1

2x1
3y1
5
5
OA = OD + DA = x1 + 3 = 3 x1 ; OB = OC + BC = y1 + 2 = 2 y1
But

OA2 + OB2 = AB2

25 2 25 2
9 x1 + 4 y1 = 225

x12 y12
9 + 4 =9
x2 y2
The locus of (x1, y1) is 81 + 36 = 1, which is an ellipse.
EXERCISE 4.2
(1) Find the equation of the ellipse if
(i)

one of the foci is (0, 1), the corresponding directrix is


3
3x + 16 = 0 and e = 5

(ii)

1
the foci are (2, 1), (0, 1) and e = 2

(iii)

the foci are ( 3, 0) and the vertices are ( 5, 0)

(iv)

3
the centre is (3, 4), one of the foci is (3 + 3, 4) and e = 2

216

2
the centre at the origin, the major axis is along x-axis, e = 3 and
5
passes through the point 2, 3

80
(vi) the length of the semi major axis, and the latus rectum are 7 and 7
respectively, the centre is (2, 5) and the major axis is parallel to
y-axis.
(vii) the centre is (3, 1), one of the foci is (6, 1) and passing through
the point (8, 1).
32
(viii) the foci are ( 3, 0), and the length of the latus rectum is 5 .
(v)

3
the vertices are ( 4, 0) and e = 2
(2) If the centre of the ellipse is (4, 2) and one of the focus is (4, 2),
find the other focus?
(3) Find the locus of a point which moves so that the sum of its distances
from (3, 0) and ( 3, 0) is 9
(4) Find the equations and length of major and minor axes of
(ix)

(i) 9x2 + 25y2 = 225

(iii) 9x2 + 4y2 = 20

(ii) 5x2 + 9y2 + 10x 36y 4 = 0 (iv) 16x2 + 9y2 + 32x 36y 92 = 0
(5) Find the equations of directrices, latus rectum and lengths of latus
rectums of the following ellipses :
(i) 25x2 + 169y2 = 4225

(ii) 9x2 + 16y2 = 144

(iii) x2 + 4y2 8x 16y 68 = 0


(iv) 3x2 + 2y2 30x 4y + 23 = 0
(6) Find the eccentricity, centre, foci, vertices of the following ellipses and
draw the diagram :
(ii) x2 + 4y2 8x 16y 68 = 0
(i) 16x2 + 25y2 = 400
(iii) 9x2 + 4y2 = 36
(iv) 16x2 + 9y2 + 32x 36y = 92
(7) A kho-kho player in a practice session while running realises that the sum
of the distances from the two kho-kho poles from him is always 8m. Find
the equation of the path traced by him if the distance between the poles is
6m.
(8) A satellite is travelling around the earth in an elliptical orbit having the
earth at a focus and of eccentricity 1/2 . The shortest distance that the
satellite gets to the earth is 400 kms. Find the longest distance that the
satellite gets from the earth.

217

(9) The orbit of the planet mercury around the sun is in elliptical shape with
sun at a focus. The semi-major axis is of length 36 million miles and the
eccentricity of the orbit is 0.206. Find (i) how close the mercury gets to
sun? (ii) the greatest possible distance between mercury and sun.
(10) The arch of a bridge is in the shape of a semi-ellipse having a horizontal
span of 40ft and 16ft high at the centre. How high is the arch, 9ft from
the right or left of the centre.

4.5 Hyperbola :
Definition: The locus of a point whose distance from a fixed point bears a constant
ratio, greater than one to its distance from a fixed line is called a hyperbola.
Note : Eventhough the syllabus does not require the derivation of
standard equation and the tracing of hyperbola (4.5.1, 4.5.2) and it
needs only the standard equation and the diagram, the equation is
derived and the curve is traced for better understanding.
We shall now derive the standard equation of the hyperbola.

4.5.1. Standard equation of the hyperbola :


y
Given :
l
Fixed point (F)
Fixed line (l)
P(x,y)
M
Eccentricity e, (e > 1)
Moving point P(x, y)
x
Construction
A C Z A F(ae,0)
Plot the fixed point F and
draw the fixed line l.
Drop a perpendicular (FZ)
Fig. 4. 74
from F to l.
Drop a perpendicular (PM) from P to l.
Plot the points A, A which divides FZ internally and externally in the ratio
e : 1 respectively.
Take AA = 2a and treat it as x-axis.
Draw a perpendicular bisector of AA and treat it as y-axis.
Let C be the origin. The known points are C(0, 0), A(a, 0), A( a, 0).
To find the co-ordinates of F and M do the following :
Since A, A divides FZ internally and externally in the ratio e : 1 respectively,

218

FA
e
=1
AZ

FA
e
AZ = 1
FA = e AZ
i.e. CF CA = e (CA CZ)
CF a = e (a CZ) (1)

FA = e AZ
i.e. AC + CF = e (AC + CZ)

a + CF = e(a + CZ)
a
(2) (1) 2a = e [2CZ] CZ = e
(2) + (1) 2CF = e(2a) CF = ae

(2)

a
M is e, y and F is (ae, 0)

To obtain the equation of the hyperbola we do the following:


Since P is a point on the hyperbola,
FP
2
2
2
We have
PM = e FP = e PM
a 2

(x ae)2 + (y 0)2 = e2 x e + (y y)2

2 2

[e x 2aex + a ]
x2 2aex + a2e2 + y2 = e2
e2
x2 e2x2 + y2 = a2 a2e2
(e2 1)x2 y2 = a2(e2 1)
2
y2
x

=1
a2
a2(e2 1)
y2
x2
2
2 2

2 2 = 1 where b = a (e 1) is a positive quantity.


a
b
This is the required standard equation of the hyperbola.

4.5.2 Tracing of the hyperbola

x2 y2

=1
a2 b2

(i) Symmetry :
The hyperbola is symmetric about x-axis, y-axis and hence the hyperbola is
symmetric about the origin.
(ii) Special points :
The hyperbola does not pass through the origin.
To find the points on x-axis, put y = 0, we get x = a. Therefore the curve
meets the x-axis at A(a, 0) and A( a, 0).

219

To find the points on y-axis, put x = 0, we get y2 = b2. i.e., y is imaginary.


Therefore the curve does not meet the y-axis.
(iii) Existence of the curve :
b
Write the equation of the hyperbola as y = a
x2 a2 . If x2 a2 < 0
i.e., a < x < a, y is imaginary. i.e., the curve does not exist for a < x < a.
Therefore the curve exists for x a and x a. Note that for all y, the curve
exists.
y
(iv) The curve at infinity :
As x increases y2 also
increases i.e., as x ,
y2 . as x , y .
Thus the curve branches
out to infinity on either side.

x = a/e

x = -a/e

F2 A
(-ae,0)

B
A

Z
B

F1
(ae,0)

Fig. 4. 75

4.5.3 Important definitions regarding hyperbola :


Focus : The fixed point is called a focus F1 (ae, 0) of the hyperbola.
Directrix : The fixed line is called the directrix of the hyperbola and its
a
equation is x = e .
Transverse axis : The line segment AA joining the vertices is called the
transverse axis and the length of the transverse axis is 2a. The equation of
transverse axis is y = 0. Note that the transverse axes cut both the branches of
the curve.
Conjugate axis : The line segment joining the points B(0, b) and

B (0, b) is called the conjugate axis. The length of the conjugate axis is 2b.
The equation of the conjugate axis is x = 0
Centre : The point of intersection of the transverse and conjugate axes of
the hyperbola is called the centre of the hyperbola. Here C(0, 0) is called the
centre of the hyperbola.
Vertices : The points of intersection of the hyperbola and its transverse
axis is called its vertices. The vertices of the hyperbola are A(a, 0) and
A( a, 0).
As in the case of ellipse, hyperbola also has the special property of the
a
second focus F2( ae, 0) and the second directrix x = e .

220

Eccentricity : e =

b2
1+ 2
a

Remark :
b
In the case of a hyperbola e > 1. As e 1, a 0 i.e., as e 1, b is very
small related to a and the hyperbola becomes a pointed nose. As e , b is
very large related to a and the hyperbola becomes flat.
Latus rectum : It is a focal chord perpendicular to the transverse axis of
the hyperbola. The equations of the latus rectum are x = ae.
End points of latus rectum and length of latus rectum :
To find the end points, solve x = ae (1)

and

x2 y2

= 1 (2)
a2 b2

Using (1) in (2) we get


y2
a2 e2

=1
a2
b2

y2
= e2 1
b2

y2 = b2 (e2 1)

b
= b 2 . 2
a
2

( b2 = a2 (e2 1))

b2
y= a

b2
If L1 and L1 are the end points of one latus rectum then L1 is ae, a and

b2
L1 is ae, a .

b2
Similarly the end points of the other latus rectum are ae, a and the

2b2
length of the latus rectum is a .
For the above discussed hyperbola, the transverse axis is along x-axis.
There is another standard hyperbola in which the transverse axis is along
y-axis.

221

4.5.4 The other form of the hyperbola:


If the transverse axis is along y-axis and the conjugate axis is along x-axis,
x2
y2
then the equation of the hyperbola is of the form 2 2 = 1
a
b
For this type of hyperbola, we have the following as explained in the
earlier hyperbola
Centre
: C (0, 0)
Vertices

A (0, a), A (0, a)

Foci

F1 (0, ae), F2 (0, ae)

Equation of transverse axis is :

x=0

Equation of conjugate axis is

y=0

End points of conjugate axis

(b, 0), ( b, 0)

Equations of latus rectum

Equations of directrices

y = ae
a
y=e

End points of latus rectum

b2
b2

a , ae , a , ae

y
F1 (0, ae)
A(0,a)

y = a/e

Z
C
B

B
y = -a/e

Z
A(0, -a)
F2 (0, - ae)

Fig. 4. 76

222

Example 4. 36 : Find the equation of hyperbola whose directrix is 2x + y = 1,


focus (1, 2) and eccentricity 3 .
Solution:
Let P(x, y) be any point on the hyperbola. Draw PM perpendicular to the
directrix.
FP
By definition, PM = e FP2 = e2 . PM2
2x + y 1

4+1

i.e., (x 1)2 + (y 2)2= 3

P(x,y)

M
2x +y - 1=0

3
(x 1)2 + (y 2)2 = 5 (2x + y 1)2
i.e., 7x2 + 12xy 2y2 2x + 14y 22 = 0
This is the required equation of the
hyperbola.

F(1,2)

Fig. 4. 77
Example 4.37 : Find the equation of the hyperbola whose transverse axis is
along x-axis. The centre is (0, 0) length of semi-transverse axis is 6 and
eccentricity is 3.
Solution:
Since the transverse axis is along x-axis and the centre is (0, 0), the
x2 y2
equation of the hyperbola is of the form 2 2 = 1
a b
y
Given that semi-transverse axis
a = 6, eccentricity e = 3
We know that b2 = a2 (e2 1)
b2 = 36(8)
= 288
The equation of the hyperbola is
x2
y2

36
288 = 1

A C
F2

A
6

F1

Fig. 4. 78

Example 4.38 : Find the equation of the hyperbola whose transverse axis is
parallel to x-axis, centre is (1, 2), length of the conjugate axis is 4 and
eccentricity e = 2.

223

Solution:
Since the transverse axis is parallel to x-axis, the equation is of the form
(x h)2
(y k)2

=1
a2
b2
Here centre C(h, k) is (1, 2).
The length of conjugate axis 2b = 4 and e = 2
b2 = a2 (e2 1)
4 = a2 (4 1)
4
a2 = 3
(y 2)2
(x 1)2

=1
The required equation is 4/3
4
Example 4.39 : Find the equation of the hyperbola whose centre is (1, 2). The
20
distance between the directrices is 3 , the distance between the foci is 30 and
the transverse axis is parallel to y-axis.
Solution:
Since the transverse axis is parallel to y-axis, the equation is of the form
(y k)2
(x h)2

=1
a2
b2
Here centre C(h, k) is (1, 2)
a
10
2a 20
The distance between the directrices e = 3 e = 3
The distance between the foci, 2ae = 30 ae = 15
a
10
2
e (ae) = 3 15 a = 50
9
ae
Also a/e e2 = 2
9
b2 = a2 (e2 1) b2 = 50 2 1 = 175

(y 2)2
(x 1)2

50
175 = 1
Example 4.40 : Find the equation of the hyperbola whose transverse axis is
parallel to y-axis, centre (0, 0), length of semi-conjugate axis is 4 and
eccentricity is 2.
The required equation is

224

Solution:
From the given data the hyperbola is of the form

y2 x2
= 1

a2 b2

Given that semi-conjugate axis b = 4 and e = 2,


b2 = a2 (e2 1)
42 = a2 (22 1)
16
a2 = 3
y2
x2
Hence the equation of the hyperbola is 16/3 16 = 1
or
3y2 x2 = 16
Example 4.41 : Find the equation of the hyperbola whose foci are ( 6, 0) and
length of the transverse axis is 8.
Solution:
y
From the given data the
transverse axis is along x-axis.
The equation is of the form
C (0,0)
(x h)2
(y k)2
x

=1
2
2
F2(-6,0)
F1(6,0)
a
b
The centre is the midpoint of
Fig. 4. 79
F1 and F2
6 + 6 0 + 0
, 2
i.e., C is 2

= (0, 0)
The length of the transverse axis 2a = 8, a = 4
F1F2 = 2ae = 12 ae = 6
4e = 6
6
3
e=4 = 2
9
16 5
b2 = a2 (e2 1) = 16 4 1 = 4 = 20

y
x
The required equation is 16 20 = 1

225

Example 4.42 : Find the equation of the hyperbola whose foci are (5, 4) and
3
eccentricity is 2 .
Solution:
From the given data the transverse
axis is parallel to y-axis and hence
the equation of the hyperbola is of
the form
(y k)2
(x h)2

=1
a2
b2
The centre C (h, k) is the midpoint
of F1 and F2

y
F1(5,4)
x

C (0,0)
F2(5,-4)

Fig. 4. 80

5+5 44
i.e., C is 2 , 2 = (5, 0)

F1F2 = 2ae = (5 5)2 + (4 + 4)2 = 8


ae = 4
3
8
But e = 2 a = 3
64 9
b2 = a2 (e2 1) = 9 4 1

80
= 9
The required equation is
9y2
9(x 5)2
(y 0)2 (x 5)2

=
1
or

=1
64/9
80/9
64
80
Example 4.43 : Find the equation of the hyperbola whose centre is (2, 1), one
of the foci is (8, 1) and the corresponding directrix is x = 4.
Solution:
y
From the given data the equation
x =4
is of the form
C(2,1)

(x h)2 (y k)2

=1
a2
b2
Centre C (h, k) is (2, 1)
CF1 = ae = 6

F1(8,1)

Fig. 4. 81

(Draw CZ perpendicular to x = 4)

226

y =1
x

The distance between the centre and directrix


a
CZ = e = 2
a
ae . e = 6 2 a2 = 12
6
ae
2
a/e = 2 e = 3
b2 = a2 (e2 1) b2 = 12(3 1) = 24
The required equation is
(y 1)2
(x 2)2

12
24 = 1
Example 4.44 : Find the equation of the hyperbola whose foci are (0, 5) and
the length of the transverse axis is 6.
Solution:
y
From the given data the transverse axis is
along y-axis and hence the equation is of
F1(0,5)
the form
(y k)2 (x h)2
=1

a2
b2
The centre C (h, k) is the midpoint of
F1 and F2

(0,0)
C
F2(0,-5)

Fig. 4. 82
0+0 55
i.e. C is 2 , 2 = (0, 0)

F1F2 = 2ae = 10
The length of the transverse axis = 2a = 6
5
a = 3 and e = 3
b2 = a2 (e2 1)
25
= 9 9 1

= 16
x2
y2
The required equation is 9 16=1

227

Example 4.45 : Find the equation of the hyperbola whose foci are (0, 10)
and passing through (2, 3).
Solution:
From the data, the transverse axis is along
the y-axis. it is of the form
y2 x2
=1
a 2 b2
Given that the foci are (0, ae) =
(0, 10)
ae =
2

F1(0, 10)
(0,0)
C

F2(0, -10)

10
2

2 2

Also b = a (e 1) = a e a

b2 = 10 a2
Equation of the hyperbola is

Fig. 4. 83

x2
y2

= 1
10 a2
a2

It passes through (2, 3),


9
4
=1
2
a
10 a2
9(10 a2) 4a2
=1
a2 (10 a2)
90 9a2 4a2 = 10a2 a4
or a4 23a2 + 90 = 0
(a2 18) (a2 5) = 0
a2 = 18 or 5
If a2 = 18, b2 = 10 18 = 8 which is impossible.
If a2 = 5, b2 = 10 5 = 5
x2
y2
Equation of the hyperbola is 5 5 = 1 or y2 x2 = 5
Example 4.46 : Find the equations and length of transverse and conjugate axes
y2
x2
of the hyperbola 9 4 = 1

228

Solution:
The centre is at the origin, the transverse axis is along x-axis and the
conjugate axis is along the y-axis. i.e., transverse axis is x-axis i.e., y = 0 and the
conjugate axis y-axis i.e., x = 0.
Hence a2 = 9, b2 = 4 a = 3, b = 2
Length of transverse axis = 2a = 6
Length of conjugate axis = 2b = 4
Example 4.47 : Find the equations and length of transverse and conjugate axes
of the hyperbola 16y2 9x2 = 144
y2 x2
Solution: 9 16 = 1
The centre is at the origin, the transverse axis is along y-axis, and the
conjugate axis is along x-axis.
The transverse axis is y-axis, i.e. x = 0
The conjugate axis is x-axis i.e. y = 0.
Here a2 = 9, b2 = 16 a = 3, b = 4
The length of transverse axis = 2a = 6
The length of conjugate axis = 2b = 8
Example 4.48 : Find the equations and length of transverse and conjugate axes
of the hyperbola 9x2 36x 4y2 16y + 56 = 0
Solution:
9(x2 4x) 4(y2 + 4y) = 56
9{(x 2)2 4} 4 {(y + 2)2 4} = 56
9(x 2)2 4(y + 2)2 = 36 16 56
9(x 2)2 4(y + 2)2 = 36
4(y + 2)2 9(x 2)2 = 36
(y + 2)2
(x 2)2
=1

9
4
X = x 2
Y2
X2

9
4 = 1 where Y = y + 2
Clearly the transverse axis is along y-axis and the conjugate axis is along xaxis. i.e. transverse axis is y-axis or X = 0 i.e., x 2 = 0
The conjugate axis is X-axis or Y = 0 i.e., y + 2 = 0

Here a2 = 9,

b2 = 4

a = 3,
229

b =2

The length of transverse axis = 2a = 6


The length of conjugate axis = 2b = 4
Example 4.49 : Find the equations of directrices, latus rectum and length of
x2 y2
latus rectum of the hyperbola 9 4 = 1
Solution:
The centre is at the origin and the transverse axis is along x-axis.
a
The equations of the directrices are x = e
The equations of the latus rectum are x = ae
2b2
Length of the latus rectum = a
Here a2 = 9,

b2 = 4
b2
1+ 2 =
a

e=

4
1+9 =

13
3

The equations of the directrices are


x=

3
9
i.e. x =
13/3
13

The equation of the latus rectum are x = 13


2b2 8
Length of the latus rectum is a = 3
Example 4. 50 : Find the equations of directrices, latus rectum and length of
latus rectum of the hyperbola 16y2 9x2 = 144
x2
y2
Solution: 9 16 = 1
5
e = 3
The transverse axis is along the y-axis.
Here a2 = 9,

b2 = 16

a
The equations of the directrices are y = e

9
i.e., y = 5

The equation of the latus rectum are y = ae i.e., y = 5


2b2
32
Length of the latus rectum is a = 3

230

Example 4.51 : Find the equations of directrices, latus rectum and length of
latus rectum of the hyperbola 9x2 36x 4y2 16y + 56 = 0
Y = y + 2
Y2
X2
Solution: By simplifying we get 9 4 = 1 where X = x 2

Here a2 = 9,

b2 = 4
13
b2
1+ 2 = 3
a
a
9
13 , e =
13

e=
ae =
The transverse axis is along Y-axis.

a
The equations of the directrices are Y = e

i.e. Y =

9
13

9
9
9
y+2 =
y=
2
13
13
13
9
9
9
y+2 =
y=
2
(ii) Y =
13
13
13
The equations of the latus rectum are Y = ae i.e. Y = 13
(i) Y = 13
y+2 =
13 y = 13 2
(ii) Y = 13 y + 2 = 13 y = 13 2
(i) Y =

2b2
8
Length of the latus rectum is a = 3
Example 4.52 : The foci of a hyperbola coincide with the foci of the ellipse
x2 y2
25 + 9 = 1. Determine the equation of the hyperbola if its eccentricity is 2.
Solution :
x2 y2
The equation of the ellipse is 25 + 9 = 1
a2 = 25, b2 = 9,

e =

b2
1 2 =
a

9
4
1 25 = 5

ae = 4
The foci of the ellipse are ( ae, 0) = ( 4, 0)
Given that the foci of the hyperbola coincide with the foci of the ellipse,
foci of the hyperbola are ( ae, 0) = ( 4, 0)

231

ae = 4
Given that the eccentricity of the hyperbola is 2
a(2) = 4 a = 2
For a hyperbola

b2 = a2 (e2 1)
= a2 e2 a2
= 16 4 = 12

x2 y2
The equation of the hyperbola is 4 12 = 1
Property (without proof) :
A point moves such that the difference of its distances from two fixed
points in a plane is a constant. The locus of this point is a hyperbola and this
difference is equal to the length of the transverse axis.
Example 4.53 : Find the equation of the locus of all points such that the
differences of their distances from (4, 0) and ( 4, 0) is always equal to 2.
Solution :
By the property, the locus is a hyperbola. Take the fixed points as foci.
F1 is (4, 0) and F2 is ( 4, 0)
Let P(x, y) be a point on the hyperbola.
F2P = length of transverse axis = 2a = 2
F1P
a=1
Centre is the midpoint of F1F2 = (0, 0)

Hence from

the given data the


x2 y2
hyperbola is of the form 2 2 = 1
a b
F1F2 = 2ae = 8
ae = 4 e = 4

P(x,y)

F2

b2 = a2 (e2 1)
= 1(16 1) = 15

C (0,0)

F1

Fig. 4. 84

y2
x2
The equation is 1 15 = 1
Alternate method:
Let P(x, y) be a point on the hyperbola and let F1and F2 be the fixed points
(4, 0) and ( 4, 0).

232

It is given that F1P


(x 4)2 + (y 0)2

F2P = 2
(x + 4)2 + (y 0)2 = 2

x2
y2
Simplifying, we get 1 15 = 1
Example 4.54 : Find the eccentricity, centre, foci and vertices of the hyperbola
y2
x2

4
5 = 1 and also trace the curve
Solution :
y
a2 = 4, b2 = 5
b2
3
1+ 2 = 2
F2 A
C (0,0) A F1
x
a
(2,0) (3,0)
(-3,0) (-2,0)
3
ae = 2 2 = 3.
The transverse axis is along the
Fig. 4. 85
x-axis
Centre : (0, 0)
Foci
: ( ae, 0) = ( 3, 0)
vertices : ( a, 0) = ( 2, 0)
Example 4.55 : Find the eccentricity, centre, foci and vertices of the hyperbola
x2
y2

6
18 = 1 and also trace the curve.
y
Solution :
e=

F1 (0,26)

a2 = 6 b2 = 18

e=

b2
1+ 2=
a

24
6 =2

ae = 2 6
The transverse axis is along the y-axis

A(0, 6)
C (0,0)
A(0, -6)
F2 (0,-26)

Fig. 4. 86
Centre
: (0, 0)
Foci are
: (0, ae) = (0, 2 6)
vertices are : (0, a) = (0, 6)

233

Example 4.56 : Find the eccentricity, centre, foci and vertices of the hyperbola
9x2 16y2 18x 64y 199 = 0 and also trace the curve.
9(x2 2x) 16 (y2 + 4y) = 199

Solution:

9{(x 1)2 1} 16 {(y + 2)2 4} = 199


9(x 1)2 16(y + 2)2 = 199 + 9 64
9(x 1)2 16(y + 2)2 = 144
(y + 2)2
(x 1)2
=1

16
9
i.e.,
a2 = 16,

b2 = 9

X = x 1
X2
Y2

16
9 = 1 where Y = y + 2

e=

b2 5
1+ 2= 4
a
5
ae = 4 4 = 5

The transverse axis is parallel to X-axis.


Referred to X, Y
Centre

Referred to x, y
X = x 1, Y = y + 2
X=0
; Y=0
x1=0 ; y+2=0
x=1
; y=2
C (1, 2)
(i) X = 5
; Y=0
x1=5 ; y+2=0
x=6
; y=2
F1 (6, 2)

(0, 0)

( ae, 0) is ( 5, 0)
(i) (5, 0)
Foci

(ii) X = 5
; Y=0
x 1 = 5; y + 2 = 0
F2 ( 4, 2)

(ii) ( 5, 0)

234

Vertices

( a, 0) i.e. ( 4, 0)
(i) (4, 0)

(i) X = 4
x1=4
A (5, 2)

(ii) ( 4, 0)

(ii) X = 4
;Y=0
x1=4
; y+2=0
A ( 3, 2)

;Y=0
; y+2=0

x
F2
(-4, -2)

A
(-3,-2)

C (1,-2) A
(5,-2)

F1
(6,-2)

Fig. 4. 87
Example 4.57 : Find the eccentricity, centre, foci and vertices of the following
hyperbola and draw the diagram : 9x2 16y2 + 36x + 32y + 164 = 0
Solution:
9(x2 + 4x) 16(y2 2y) = 164
9{(x + 2)2 4} 16 {(y 1)2 1} = 164
9(x + 2)2 16(y 1)2 = 164 + 36 16
16(y 1)2 9(x + 2)2 = 144
(x + 2)2
(y 1)2

=1
9
16
X = x + 2
X2
Y2

9
16 = 1 where Y = y 1

a2 = 9,

b2 = 16

e=

b2 5
1+ 2= 3
a
ae = 5

235

The transverse axis is parallel to Y-axis.


Referred to X, Y
Centre

Referred to x, y
X = x + 2, Y = y 1
X=0
; Y=0
x+2=0 ; y1=0
x=2
; y=1
C ( 2, 1)
(i) X = 0
; Y=5
x+2=0 ; y1=5
x=2
; y=6
F1 ( 2, 6)

(0, 0)

(0, ae) i.e., (0, 5)


(i) (0, 5)

(ii) (0, 5)

(ii) X = 0
; Y= 5
x+2=0 ; y1=5
x=2
; y=4
F2 ( 2, 4)
(i) X = 0
; Y=3
x+2=0 ; y1=3
A ( 2, 4)
(ii) X = 0
; Y=3
x+2=0 ; y1=3
x=2
; y=2

Foci
(0, a)
(i) (0, 3)
Vertices
(ii) (0, 3)

A ( 2, 2)

x =-2
y

F1 (-2,6)
A (-2,4)
y =1
C (-2,1)
A (-2,-2)
F2 (-2,-4)

Fig. 4. 88
236

X
x

Example 4.58 :
Points A and B are 10 km apart and it is determined from the sound of an
explosion heard at those points at different times that the location of the
explosion is 6 km closer to A than B. Show that the location of the explosion is
restricted to a particular curve and find an equation of it.
y

P(x,y)

A(-5, 0)

B(5, 0)

Fig. 4. 89
Given :
i.e.,

PB PA = 6
(x 5)2 + y2

Simplifying we get

(x + 5)2 + y2 = 6
9y2 + 16x2 = 144

y2 x2
x2
y2
+
=
1
i.e.,

16
9
9
16 = 1 which is a hyperbola.
EXERCISE 4.3
(1) Find the equation of the hyperbola if
(i)
(ii)
(iii)

(iv)
(v)

focus : (2, 3) ; corresponding directrix : x + 2y = 5, e = 2


7
centre : (0, 0) ; length of the semi-transverse axis is 5 ; e = 5 and
the conjugate axis is along x-axis.
centre : (0, 0) ; length of semi-transverse axis is 6 ; e = 3, and the
transverse axis is parallel to y-axis.
5
centre : (1, 2) ; length of the transverse axis is 8 ; e = 4 and the
transverse axis is parallel to x-axis.
centre : (2, 5) ; the distance between the directrices is 15, the
distance between the foci is 20 and the transverse axis is parallel to
y-axis.

237

(2)

(3)

(4)
(5)

(vi) foci : (0, 8) ; length of transverse axis is 12


(vii) foci : ( 3, 5) ; e = 3
(viii) centre : (1, 4) ; one of the foci (6, 4) and the corresponding
9
directrix is x = 4 .
(ix) foci : (6, 1) and ( 4, 1) and passing through the point (4, 1)
Find the equations and length of transverse and conjugate axes of the
following hyperbolas :
(i) 144x2 25y2 = 3600
(ii) 8y2 2x2 = 16
(iii) 16x2 9y2 +96x + 36y 36 = 0
Find the equations of directrices, latus rectums and length of latus rectum
of the following hyperbolas :
(ii) 9x2 4y2 36x + 32y + 8 = 0
(i) 4x2 9y2 = 576
Show that the locus of a point which moves so that the difference of its
distances from the points (5, 0) and ( 5, 0) is 8 is 9x2 16y2 = 144.
Find the eccentricity, centre, foci and vertices of the following
hyperbolas and draw their diagrams.
y2 x2
(i) 25x2 16y2 = 400
(ii) 9 25 = 1
(iv) x2 3y2 + 6x + 6y + 18 = 0
(iii) x2 4y2 + 6x + 16y 11 = 0

4.6 Parametric form of Conics:


Conic
Parabola
Ellipse
Hyperbola

Parametric
equations
x = at2
y = 2at
x = a cos
y = b sin
x = a sec
y = b tan

Parameter
t

Range of
parameter
<t<

0 2

0 2

Any point on
the conic
t or
(at2, 2at)
or
(a cos, b sin)
or
(a sec , b tan )

Note: For ellipse, we have another parametric form of equations x =


y=

a(1 t2)
,
1 + t2

b.2t
2 , < t < . This result will be obtained by putting tan 2 = t in the
1+t

parametric equations x = a cos and y = b sin .

238

Thus we have two forms of representations of conics i.e., cartesian form


and parametric form. Now we will derive the equations of chord, tangent and
normal to the conics.

4.7 Chords, tangents and normals


We derive these equations using both forms of conics.

4.7.1 Cartesian form


(i) Parabola
Equation of the chord joining A(x1, y1) and B(x2, y2) on the parabola
y2 = 4ax
y
A(x1,y1)
Since (x1, y1) and (x2, y2) lie on
the parabola,
y12 = 4ax1, y22 = 4ax2
x

y12 y22 = 4a (x1 x2)

y1 y2
4a
=
x1 x2 y1 + y2

B(x2,y2)
y2 =4ax

Fig. 4. 90
4a
i.e., the slope (m) of the chord AB = y + y
1
2
The equation of the chord, using slope (m) and point (x1, y1) is
4a
(y y1) = y + y (x x1)
1
2

Ta

4a
(y y1) = y + y (x x1)
1
1
yy1 = 2a(x + x1)
(use

y12

ng
en
t

If the point (x2, y2) coincides with (x1, y1) then the chord becomes the
tangent at (x1, y1). Therefore, to obtain tangent at (x1, y1), put x2 = x1 and
y2 = y1 in the equation of the chord. the equation of the tangent is

(x1,y1)
x

= 4ax1)

Thus the equation of the tangent


at (x1, y1) to the parabola y2 = 4ax
is yy1 = 2a(x + x1)

y2 =4ax

Fig. 4. 91

239

No
rm
al

2ax y1y + 2ax1 = 0

Ta

Equation of the tangent is

ng
en
t

To find the equation of the normal using perpendicularity.

(x1,y1)

the normal is of the form

y1x + 2ay = k

y2 =4ax

But it passes through (x1, y1)


k = x1y1 + 2ay1

Fig. 4. 92

Thus the equation of the normal at (x1, y1) to the parabola is


y1x + 2ay = x1y1 + 2ay1
(ii) Ellipse
Equation of the chord joining A(x1, y1) and B(x2, y2) on the ellipse
y2
x
+
=1
a2 b2
2

Since (x1, y1) and (x2, y2) lie on


the ellipse,
x1

x22
2

y1
+ 2 = 1,
a
b
a
2

A(x1,y1)

y2
+ 2 = 1
b

B(x2,y2)

By simplification, the slope


m=

y1 y2 b2 (x1 + x2)
= 2
x1 x2
a (y1 + y2)

Fig. 4. 93

the equation of the chord is


(y y1) =

b2 (x1 + x2)
a2(y1 + y2)

(x x1)

To get the equation of the tangent at (x1, y1) put x2 = x1 and y2 = y1 in the
equation of the chord.

240

The equation of the tangent at


(x1, y1) is

y
(x1,y1)

Tan
g

(y y1) =

b (x1 + x1)
2

a (y1 + y1)

(x x1)

en t

yy1
+ 2 =1
a
b

xx1
2

No
rm
a

Fig. 4. 94
To get the equation of the normal, use the perpendicularity property to a
straight line.
The equation of the tangent is
y
x1b2x + y1a2y a2b2 = 0
The equation of the normal is of
the form y1a2x x1b2y = k

(x1,y1)
x

But it passes through (x1, y1)


k = (a2 b2) x1 y1
Fig. 4. 95
The required equation is
a2x b2y
y1a2x x1b2y = (a2 b2) x1y1 or x y = a2 b2
1
1
(iii) Hyperbola
Following the same procedure as in the case of ellipse we get the equation
of the chord as
b2(x1 + x2)
(x x1)
y y1 = 2
a (y1 + y2)
The equation of the tangent at (x1, y1) as

xx1
a

yy1
b2

= 1

a2 x b 2 y
and the normal at (x1, y1) as x + y = a2 + b2
1
1
Note : To get the results for the hyperbola replace b2 as b2 in the results
of ellipse.
241

4.7.2 Parametric form :


To get the parametric forms of equations of chord, tangent and normal to
conics, replace (x1, y1), by the corresponding any point in the parametric form.
(i) Parabola :
The equation of the chord joining (x1, y1) and (x2, y2) on the parabola is
4a
y y1 = y + y (x x1)
1
2
The equation of the chord joining (at12, 2at1) and (at22, 2at2) or t1 and
t2 on the parabola is
4a
y 2at1 = 2at + 2at (x at12)
1
2
i.e.

y(t1 + t2) = 2x + 2a t1t2

To find the equation of the tangent at t put t1 = t2 = t in the equation of


the chord. We get
y(2t) = 2x + 2at2
i.e. yt = x + at2
Another method:
The tangent at (x1, y1) to y2 = 4ax is yy1 = 2a(x + x1)
The tangent at (at2, 2at) is
y(2at) = 2a (x + at2)
i.e.,

yt = x + at2

Applying the perpendicularity, we get the equation of the normal at t as


y + tx = 2at + at3
Similarly we can derive the equation of chord, tangent and normal for
ellipse and hyperbola.

242

243

Note : The equation of tangent at (x1, y1) is obtained from the equation of the
1
1
curve by replacing x2 by xx1, y2 by yy1, xy by 2 (xy1 + x1y), x by 2 (x + x1) and
1
y by 2 (y + y1)
To find the condition that y = mx + c may be a tangent to the conics
(1) Parabola :
Let y = mx + c be a tangent to the parabola y2 = 4ax at (x1, y1).
We know that at (x1, y1), the equation of the tangent is yy1 = 2a(x + x1)
The above two equations represent the same tangent and hence their
corresponding coefficients are proportional
2ax y1y + 2ax1 = 0
mx y + c = 0

2ax1
2a y1
=
=
c
m
1

c
2a
x1 = m , y1 = m

Since (x1, y1) lies on the parabola, y12 = 4ax1 ,

c
4a2
= 4a . m
m2

a
i.e. , c = m
Thus we have three results to the parabola y2 = 4ax.
a
(1) The condition for the tangency is c = m
c 2a
a 2a
(2) The point of contact is m , m i.e., 2 , m .

a
(3) The equation of any tangent is of the form y = mx + m
Note : Instead of taking the equation of the tangent in the cartesian form, we
can prove the same result by taking the tangent in the parametric form.
Similarly, we can derive the results for other conics also.

244

Results connected with ellipse :


x2 y2
(i) The condition that y = mx + c may be a tangent to the ellipse 2 + 2 = 1
a
b
is c2 = a2m2 + b2
a2m b2
(ii) The point of contact is c , c where c2 = a2m2 + b2

a2m2 + b2

(iii) The equation of any tangent is of the form y = mx


Note : In y = mx

a2m2 + b2 , either y = mx +

a2m2 + b2 holds

or y = mx a2m2 + b2 holds
Results connected with hyperbola :
(i) The condition that y = mx + c may be a tangent to the hyperbola is
c2 = a2m2 b2
a2m b2
(ii) The point of contact is c , c where c2 = a2m2 b2

a2m2 b2

(iii) The equation of any tangent is of the form y = mx


Note : In y = mx
or y = mx

a2m2 b2 , either y = mx +

a2m2 b2

a2m2 b2 is correct but not both.

4.7.3 Equation of chord of contact of tangents from a point (x1, y1)

(x

,y

2)

x2 y2
x2 y2
to the (i) Parabola y2 = 4ax (ii) ellipse 2 + 2 = 1 (iii) hyperbola 2 2 = 1
a b
a b
Solution :
y
The equation of tangent at
Q(x2, y2) is yy2 = 2a(x + x2)
It passes through the point
P(x1, y1)
y1y2 = 2a (x1 + x2)
(1)
(x1,y1)
P V
The equation of tangent at
x
R(x3, y3) is yy3 = 2a(x + x3)
It passes through the point
P(x1, y1)
y1y3 = 2a(x1 + x3)
(2)
The result (1) and (2) show that
Q(x2, y2) and R(x3, y3) lie on the
straight line yy1 = 2a(x + x1).
Fig. 4. 96
)
,y 3
x3
R(

245

Equation of QR, the chord of contact of tangents is yy1 = 2a(x + x1)


Similarly we can find the required equations of the chord of contact for
xx1
yy1
xx1 yy1
ellipse as 2 + 2 = 1 and for the hyperbola as 2 2 = 1
a
b
a
b
Example 4.59 : Find the equations of the tangents to the parabola y2 = 5x from
the point (5, 13). Also find the points of contact.
Solution:
5
Here 4a = 5 a = 4
The equation of the parabola is y2 = 5x
5
a
Let the equation of the tangent be y = mx + m i.e., y = mx + 4m
(1)
Since it passes through (5, 13) we have
5
13 = 5m + 4m
20m2 52m + 5 = 0
(10m 1) (2m 5) = 0
1
5
m = 10 or m = 2
Using the values of m, we get the equations of tangents are 2y = 5x + 1,
10y = x + 125.
5
5
1
a 2a
The points of contact are given by 2 , m , where a = 4 m = 2 , 10
m

1
the points of contact are 5 , 1 , (125, 25)

Example 4.60 : Find the equation of the tangent at t = 1 to the parabola y2 = 12x
Solution: Equation of the parabola is y2 = 12x.
Here 4a = 12, a = 3
t represents the point (at2, 2at). t = 1 represents the point = (3, 6)
(x + x1)
Equation of tangent at (x1, y1) to the parabola y2 = 12x is yy1 = 12
2
12 (x + 3)
i.e., x y + 3 = 0
Equation of tangent at (3, 6) is y(6) =
2
Alternative form :
The equation of the tangent at t is yt = x + at2
Here
4a = 12 a = 3
Also
t=1

246

The equation of the tangent is y = x + 3


xy+3 = 0
Example 4.61 : Find the equation of the tangent and normal to the parabola
x2 + x 2y + 2 = 0 at (1, 2)
Solution:
The equation of the parabola is x2 + x 2y + 2 = 0
Equation of the tangent at (x1, y1) to the given parabola is
xx1 +

(y + y1)
x + x1
x+1
(y + 2)

2
+ 2 = 0 i.e., x(1) + 2 2 2 + 2 = 0
2
2

On simplification we get 3x 2y + 1 = 0
Equation of the normal is of the form 2x + 3y + k = 0
This normal passes through (1, 2)
2+6+k = 0 k=8
Equation of the normal is 2x + 3y 8
=
0
Example 4.62 : Find the equations of the two tangents that can be drawn from
the point (5, 2) to the ellipse 2x2 + 7y2 = 14
Solution:
Equation of the ellipse is
2x2 + 7y2 = 14
i.e.,

x2 y2
7 + 2 =1

Here a2 = 7, b2 = 2
Let the equation of the tangent be y = mx +

a2m2 + b2

y = mx + 7m2 + 2
Since this line passes through the point (5, 2) we get
2 = 5m +
i.e.

2 5m =

7m2 + 2

7m2 + 2

(2 5m)2 = 7m2 + 2
4 + 25m2 20m = 7m2 + 2
18m2 20m + 2 = 0
9m2 10m + 1 = 0

247

(9m 1) (m 1) = 0
m=1

1
or m = 9

To find the equations of the tangents, use slope-point form


(i) m = 1,
The equation is y 2 = 1(x 5) i.e., x y 3 = 0
(ii) m = 1/9
1
The equation is y 2 = 9 (x 5), i.e., x 9y + 13 = 0.
Thus the equations of the tangents are x y 3 = 0, x 9y + 13 = 0
Example 4.63 : Find the equation of chord of contact of tangents from the point
(2, 4) to the ellipse 2x2 + 5y2 = 20
Solution:
The equation of chord of contact of tangents
from (x1, y1) to 2x2 + 5y2 20 = 0 is 2xx1 + 5yy1 20 = 0
the required equation from (2, 4) is 2x(2) + 5y(4) 20 = 0
i.e. x + 5y 5 = 0
EXERCISE 4.4
(1) Find the equations of the tangent and normal
(i) to the parabola y2 = 12x at (3, 6)
(ii) to the parabola x2 = 9y at ( 3, 1)
(iii) to the parabola x2 + 2x 4y + 4 = 0 at (0, 1)
(iv) to the ellipse 2x2 + 3y2 = 6 at ( 3 , 0)
(v) to the hyperbola 9x2 5y2 = 31 at (2, 1)
(2) Find the equations of the tangent and normal
1
(i) to the parabola y2 = 8x at t = 2

(ii) to the ellipse x2 + 4y2 = 32 at = 4


1
(iii) to the ellipse 16x2 + 25y2 = 400 at t =
3
x2 y2

(iv) to the hyperbola 9 12 = 1 at = 6

248

(3) Find the equations of the tangents


(i) to the parabola y2 = 6x, parallel to 3x 2y + 5 = 0
(ii) to the parabola y2 = 16x, perpendicular to the line 3x y + 8 = 0
x2 y2
(iii) to the ellipse 20 + 5 = 1, which are perpendicular to x + y + 2 = 0
(iv) to the hyperbola 4x2 y2 = 64, which are parallel to
10x 3y + 9 = 0
(4) Find the equation of the two tangents that can be drawn
(i) from the point (2, 3) to the parabola y2 = 4x
(ii) from the point (1, 3) to the ellipse 4x2 + 9y2 = 36
(iii) from the point (1, 2) to the hyperbola 2x2 3y2 = 6.
(5) Prove that the line 5x + 12y = 9 touches the hyperbola x2 9y2 = 9 and
find its point of contact.
(6) Show that the line x y + 4 = 0 is a tangent to the ellipse x2 + 3y2 = 12.
Find the co-ordinates of the point of contact.
(7) Find the equation to the chord of contact of tangents from the point
(i) ( 3, 1) to the parabola y2 = 8x
(ii) (2, 4) to the ellipse 2x2 + 5y2 = 20
(iii) (5, 3) to the hyperbola 4x2 6y2 = 24
Results without Proof :
(1) Two tangents can be drawn to (i) a parabola (ii) an ellipse and
(iii) a hyperbola, from any point on the plane.
(2) (a) Three normals can be drawn to a parabola
(b) Four normals can be drawn to (i) an ellipse and (ii) a hyperbola from
any point on the plane.
(3) The equation of chord of contact of tangents from a point (x1, y1)
(i) a parabola y2 = 4ax is yy1 = 2a(x + x1)
xx1 yy1
y2
x2
(ii) an ellipse 2 + 2 = 1 is 2 + 2 = 1
a
b
a
b
2
2
xx1 yy1
x
y
(iii) a hyperbola 2 2 = 1 is 2 2 = 1
a
b
a
b
(4) The chord of contact of tangents from any point on the directrix (i) of a
parabola passes through its focus (ii) passes through the corresponding
focus for ellipse and hyperbola
249

(5) The condition that lx + my + n = 0 may be a tangent to


(i) the parabola y2 = 4ax is am2 = ln
x2 y2
(ii) the ellipse 2 + 2 = 1 is a2l2 + b2m2 = n2
a b
x2 y2
(iii) the hyperbola 2 2 = 1 is a2l2 b2m2 = n2
a b
(6) The condition that lx + my + n = 0 may be a normal to
(i) the parabola y2 = 4ax is al3 + 2alm2 + m2n = 0
a2 b2 (a2 b2)
x2 y2
(ii) the ellipse 2 + 2 = 1 is 2 + 2 =
a b
l
m
n2

x2 y2
a2 b2 (a2 + b2)
(iii) the hyperbola 2 2 = 1 is 2 2 =
a b
l
m
n2
(7) The locus of the foot of the perpendicular from a focus to a tangent to
(i) the parabola y2 = 4ax is x = 0
x2 y2
(ii) the ellipse 2 + 2 = 1 is the circle x2 + y2 = a2
b
a
x2 y2
(iii) the hyperbola 2 2 = 1 is the circle x2 + y2 = a2
b
a
(This circle is also called auxiliary circle)
(8) The locus of the point of intersection of perpendicular tangents to
(i) the parabola y2 = 4ax is x = a (the directrx)
x2
(ii) the ellipse 2 +
a
director circle)

y2
= 1 is x2 + y2 = a2 + b2 (This circle is called
b2

x2 y2
(iii) an hyperbola 2 2 = 1 is x2 + y2 = a2 b2 (This circle is also
a
b
called director circle)
(9) The point of intersection of the tangents at t1 and t2 to the

parabola y2 = 4ax is [at1t2, a(t1 + t2)]


(10) The normal at the point t1 on the parabola y2 = 4ax meets the parabola
2
again at the point t2, then t2 = t1 + t
1

250

(11) If t1 and t2 are the extremities of any focal chord of the parabola
y2 = 4ax, then t1t2 = 1
Note : For the proof of above results one may refer the Solution Book.

4.8. Asymptotes
Consider the graph of a function y = f(x). As a point P on the curve moves
farther and farther away from the origin, it may happen that the distance
between P and some fixed line tends to zero. This fixed line is called an
asymptote.
Note that it is possible only when the curve is open. Since hyperbola is
open and y as x + and x hyperbola have asymptotes.
Definition :
An asymptote to a curve is the tangent to the curve such that the point of
contact is at infinity. In particular the asymptote touches the curve at
+ and .
y

xe
Fi

A
F2
P

in
dL
A

e(

as

pto
ym

te)

F1

Fig. 4. 97
x 2 y2
The equations of the asymptotes to the hyperbola 2 2 = 1
a b
Assume that the equation of an asymptote is of the form y = mx + c.
To find the points of intersection of the hyperbola and the asymptote, solve
2
x
y2

= 1 and y = mx + c.
a2
b2

x2
(mx + c)2

=1
a2
b2

251

1 m2 2 2mc
c2
2 2 x 2 x 2 + 1 = 0
b
a b
b

The points of contact are at infinity. i.e., the roots of the equations are
infinite. Since the roots are infinite, the coefficients of x2 and x must be zero.
1
m2
2mc
2 2 = 0 and
=0
a
b2
b
b
i.e., m = a and c = 0
b
Then y = a x
there are two asymptotes to the hyperbola whose equations are
b
b
y = a x and y = a x
x y
x
y
i.e. a b = 0 and a + b = 0
The combined equation of asymptotes is

x y x + y = 0 i.e.
a b a b
x/a

x2
y2
2 2=0
a
b
y

y/b
=0

F2
(-ae,0)

l1

A'
Z'

-y
x/a

A
Z

0
/b=

F1
(ae,0)

Fig. 4. 98
Results regarding asymptotes :
(1) The asymptotes pass through the centre C(0, 0) of the hyperbola.

252

b
b
(2) The slopes of asymptotes are a and a i.e., the transverse axis and
conjugate axis bisect angles between the asymptotes.
x
y
(3) If 2 is the angle between the asymptotes then the slope of a b = 0 is
b
tan = a .
b
angle between the asymptotes is 2 = 2 tan1 a
(4) We know that sec2 = 1 + tan2
a2 + b2
b2
sec2 = 1 + 2 =
= e2
a
a2
sec = e = sec1e
angle between the asymptotes 2 = 2 sec1e
Important Note :
Eventhough the asymptotes are straight lines, if the angle between the
asymptotes is obtuse, take obtuse angle as the angle between them and
not the corresponding acute angle.
(5) The standard equation of hyperbola and combined equation of
asymptotes differs only by a constant.
(6) If l1 = 0 and l2 = 0 are the separate equations of asymptotes, then the
combined equation of the asymptotes is l1 l2 = 0.
the equation of the corresponding hyperbola is of the form l1l2 = k,
where k is a constant. To find this k, we need a point on the hyperbola.
Example 4.64 : Find the separate equations of the asymptotes of the hyperbola
3x2 5xy 2y2 + 17x + y + 14 = 0
Solution: The combined equation of the asymptotes differs from the hyperbola
by a constant only.
the combined equation of the asymptotes is
3x2 5xy 2y2 + 17x + y + k = 0
Consider 3x2 5xy 2y2 = 3x2 6xy + xy 2y2
= 3x (x 2y) + y(x 2y)
= (3x + y) (x 2y)
The separate equations are 3x + y + l = 0, x 2y + m = 0

253

(3x + y + l) (x 2y + m) = 3x2 5xy 2y2 + 17x + y + k


Equating the coefficients of x, y terms and constant term, we get
l + 3m = 17
(1)
2l + m = 1
lm = k
Solving (1) and (2) we get l = 2, m = 5 and k = 10

(2)

Hence separate equations of asymptotes are 3x + y + 2 = 0, x 2y + 5 = 0


The combined equation of asymptotes is
3x2 5xy 2y2 + 17x + y + 10 = 0
Note : The hyperbola, discussed above is not a standard hyperbola.
Example 4.65 : Find the equation of the hyperbola which passes through the
point (2, 3) and has the asymptotes 4x + 3y 7 = 0 and x 2y = 1.
Solution:
The separate equations of the asymptotes are 4x + 3y 7 = 0, x 2y 1 = 0
combined equation of asymptotes is (4x + 3y 7) (x 2y 1) = 0
The equation of the hyperbola differs from this combined equation of
asymptotes by a constant only.
the equation of the hyperbola is of the form
(4x + 3y 7) (x 2y 1) + k = 0
But this passes through (2, 3)
(8 + 9 7) (2 6 1) + k = 0 k = 50
The equation of the corresponding hyperbola is
(4x + 3y 7) (x 2y 1) + 50 = 0
i.e.,
4x2 5xy 6y2 11x + 11y + 57 = 0
Example 4.66 : Find the angle between the asymptotes of the hyperbola
3x2 y2 12x 6y 9 = 0
3x2 y2 12x 6y 9 = 0
Solution:
3(x2 4x) (y2 + 6y) = 9
3 {(x 2)2 4} {(y + 3)2 9} = 9
3(x 2)2 (y + 3)2 = 12
(x 2)2
(y + 3)2

4
12 = 1
Here a = 2, b = 12 = 2 3

254

The angle between the asymptotes is


b
2 3
2
2 = 2 tan1 a = 2tan1 2 = 2 tan1 3 = 2 3 = 3
Another method : a2 = 4, b2 = 12
b2
1+ 2 =
a
The angle between the asymptotes is

12
1+ 4 = 2

e=

2
2 = 2 sec12 = 2 3 = 3
Example 4.67 : Find the angle between the asymptotes to the hyperbola
3x2 5xy 2y2 + 17x + y + 14 = 0
Solution: Combined equation of the asymptotes differs from that of the
hyperbola by a constant only.
Combined equation of asymptotes is 3x2 5xy 2y2 + 17x + y + k = 0
3x2 5xy 2y2 = 3x2 6xy + xy 2y2
= 3x(x 2y) + y(x 2y)
= (x 2y) (3x + y)
Separate equations are x 2y + l = 0, 3x + y + m = 0
1
Let m1 and m2 be the slopes of these lines, then m1 = 2 , m2 = 3
m1 m2
1/2 ( 3)
angle between the lines is
tan = 1 + m m =
1 + 1/2 ( 3) = 7

1 2

= tan1 (7)
Alternative method : Combined equation of asymptotes is nothing but pair of
straight lines. Hence the angle between the asymptotes is

2 h2 ab

a+b

tan =

Comparing with ax2 + 2hxy + by2 + 2gx + 2fy + c = 0


We have a = 3, b = 2, 2h = 5
25

2 4 + 6

tan =
32
27
= 2 =7

= tan1 (7)

255

Note : Since the above hyperbola is not in the standard form, it is difficult to
identify whether the angle between the asymptotes is obtuse or acute.
According to the above method we will get only the acute angle as the angle
between the asymptotes.
b
Therefore if the hyperbola in the standard form, use either 2 tan1 a or
2 sec1e to find the angle between the asymptotes and take the angle as it is.
Example 4.68 : Prove that the product of perpendiculars from any point on the
a2b2
x2 y2
hyperbola 2 2 = 1 to its asymptotes is constant and the value is 2
a b
a + b2
Solution:
x12 y12
x2 y2
Let P (x1, y1) be any point on the hyperbola 2 2 = 1 2 2 = 1 (1)
a
b
a
b
The perpendicular distance from (x1, y1) to the asymptote
x y
a b = 0 is

x1 y1
a b
x y
and to a + b = 0 is
1 1
+
a 2 b2

x1 y1
a + b
1 1
+
a2 b 2

y
x/a

y/b

-y
x/a

=0

0
/b=

P(x1,y1)
F2

F1

Fig. 4. 99
Product of perpendicular distances =

256

x1 y1
a + b
.
1 1
+
a2 b 2

x1 y1
a b
1 1
+
a2 b2

x12

y12

a2 b2
1
= 1 1 = 2
b + a2
2+ 2
a b
a2 b2

(by (1))

a2 b2
= 2
, which is a constant.
a + b2
EXERCISE 4.5
(1) Find the equation of the asymptotes to the hyperbola
(i) 36x2 25y2 = 900
(ii) 8x2 + 10xy 3y2 2x + 4y 2 = 0
(2) Find the equation of the hyperbola if
(i) the asymptotes are 2x + 3y 8 = 0 and 3x 2y + 1 = 0 and (5, 3) is a
point on the hyperbola
(ii) its asymptotes are parallel to x + 2y 12 = 0 and x 2y + 8 = 0,
(2, 4) is the centre of the hyperbola and it passes through (2, 0).
(3) Find the angle between the asymptotes of the hyperbola
(i) 24x2 8y2 = 27
(ii) 9(x 2)2 4(y + 3)2 = 36
2
2
(iii) 4x 5y 16x + 10y + 31 = 0

4.9 Rectangular hyperbola


Definition:
A hyperbola is said to be a rectangular hyperbola if its asymptotes are at
right angles.
b
The angle between the asymptotes is given by 2tan1 a . But angle between
the asymptotes of the rectangular hyperbola is 90 .
b
b
2tan1 a = 90 a = tan45 a = b.

y2
x2
2 2 = 1, we get the equation of
a
b
2
2
2
rectangular hyperbola as x y = a . Hence the combined equation of the
asymptotes is x2 y2 = 0. The separate equations are x y = 0 and x + y = 0.
i.e., x = y and x = y. The transverse axis is y = 0, conjugate axis is x = 0.
Using a = b in the hyperbola

All the results corresponding to the rectangular hyperbola of the form


x2 y2 = a2 are obtained simply by putting a = b in the corresponding results of
y2
x2
the hyperbola 2 2 = 1
a
b

257

This type of rectangular hyperbola is not a standard one. For standard type,
the asymptotes are the co-ordinate axes.
The standard rectangular hyperbola xy = c2 is obtained by rotating the
rectangular hyperbola x2 y2 = a2 through an angle 45 about the origin in the
anticlockwise direction.
y

y
x y = c2
x2 -

y 2 = a2
A

90

45

Fig. 4. 100
4.9.1 Standard equation of a rectangular hyperbola :
For a standard rectangular hyperbola the asymptotes are co-ordinate axes.
Since the axes are the asymptotes, the equations of the asymptotes are x = 0 and
y = 0. The combined equation of the asymptotes is xy = 0. Therefore the
equation of the standard rectangular hyperbola is of the form xy = k. To find k,
we need a point on the rectangular hyperbola.
y
D1
D2

F1(a,a)
A (a/2, a/2)
C

45

M
A
D1

F2(-a,-a)
D2

Fig. 4. 101
258

Let the asymptotes meet at C. Let AA = 2a be the length of the transverse


axis. Draw AM perpendicular to x-axis. Since the asymptotes bisect the angle
a
a
between the axes, ACM = 45. CM = a cos 45 =
, AM = a sin 45 =
2
2
a a
co-ordinates of A are , . This point lies on the rectangular
2 2
hyperbola xy = k. k =

a
a
a2
.
or k = 2 and
2
2

a2
the equation of the rectangular hyperbola is xy = 2 or
xy = c2

a2
where c2 = 2 .

Eccentricity of the hyperbola is given by b2 = a2(e2 1). Since a = b in a


rectangular hyperbola, a2 = a2 (e2 1)
Eccentricity of the rectangular hyperbola is e = 2 .
a a
a
a
Also the vertices of the rectangular hyperbola are , , ,
2
2 2 2
and foci are (a, a), ( a, a).
The equation of transverse axis is y = x and the conjugate axis is y = x.
If the centre of the rectangular hyperbola is at (h, k) and the asymptotes are
parallel to x and y-axis, the general form of standard rectangular hyperbola is (x
h) (y k) = c2.
The parametric equation of the rectangular hyperbola xy = c2 are
c
x = ct, y = t where t is the parameter and t is any non-zero real number.
c
Any point on the rectangular hyperbola is ct , t . This point is often

referred to as the point t.


Results :
Equation of the tangent at (x1, y1) to the rectangular hyperbola xy = c2
is xy1 + yx1 = 2c2
Equation of the tangent at t is x + yt2 = 2ct.
Equation of normal at (x1, y1) is xx1 yy1 = x12 y12 .

259

c
Equation of normal at t is y xt2 = t ct3
Two tangents and four normals can be drawn from a point to a
rectangular hyperbola.
Example 4.69 : Find the equation of the standard rectangular hyperbola whose
2
3
centre is 2 , 2 and which passes through the point 1, 3

Solution:
The equation of the standard rectangular hyperbola with centre at (h, k) is
(x h) (y k) = c2
3
The centre is 2 , 2 .

3
the equation of the standard rectangular hyperbola is (x+2) y + 2 = c2

2
2 3

2
2 5
It passes through 1 , 3

(1 + 2) 3 + 2 = c c = 2
3 5
Hence the required equation is (x + 2) y + 2 = 2 or

2xy + 3x + 4y + 1 = 0
Example 4.70 : The tangent at any point of the rectangular hyperbola xy = c2
makes intercepts a, b and the normal at the point makes intercepts p, q on the
axes. Prove that ap + bq = 0
y
Ta
nt

N
or
m
al

e
ng

b
}q
}
p

Fig. 4. 102

260

Solution: Equation of tangent at any point t on xy = c2 is x + yt2 = 2ct


y
x
or 2ct + 2c/t = 1
2c
Intercept on the axes are a = 2ct, b = t .
c
Equation of normal at t on xy = c2 is y xt2 = t ct3
y
x
+
=1
c ct3 c ct3
t
t

2
t

c
1 c
p = 2 t ct3 , q = t ct3

t
2c c
1 c
ap + bq = 2ct 2 t ct3 + t t ct3

t
2c c
2c c
= t t ct3 + t t ct3

=0
Example 4.71 : Show that the tangent to a rectangular hyperbola terminated by
its asymptotes is bisected at the point of contact.
Solution:
y
The equation of tangent at
c
P ct, t is x + yt2 = 2ct

Putting y = 0 in this equation


we get the co-ordinates of A as
B
(2ct, 0). Putting x = 0 we get
P(ct, c/t)
2c

the co-ordinates of B as 0, t
O

x
intercept on axes are

Fig. 4. 103
2c

2ct + 0 0 + t
c
The mid-point of AB is 2 , 2 = ct , t


which is the point P. This shows that the tangent is bisected at the point of
contact.

261

EXERCISE 4.6
(1) Find the equation of the standard rectangular hyperbola whose centre is
1 , 1 and which passes through the point 1 , 1 .
2 2
4
(2) Find the equation of the tangent and normal (i) at (3, 4) to the rectangular
1
hyperbolas xy = 12
(ii) at 2 , 4 to the rectangular hyperbola

2xy 2x 8y 1 = 0
(3) Find the equation of the rectangular hyperbola which has for one of its
asymptotes the line x + 2y 5 = 0 and passes through the points (6, 0)
and ( 3, 0).
(4) A standard rectangular hyperbola has its vertices at (5, 7) and ( 3, 1).
Find its equation and asymptotes.
(5) Find the equation of the rectangular hyperbola which has its centre at
(2, 1), one of its asymptotes 3x y 5 = 0 and which passes through the
point (1, 1).
(6) Find the equations of the asymptotes of the following rectangular
hyperbolas.
(i) xy kx hy = 0
(ii) 2xy + 3x + 4y +1 = 0
(iii) 6x2 + 5xy 6y2 + 12x + 5y + 3 = 0
(7) Prove that the tangent at any point to the rectangular hyperbola forms
with the asymptotes a triangle of constant area.
Results without proof :
(1) The foot of the perpendicular from a focus of a hyperbola on an
asymptote lies on the corresponding directrix.
(2) (i) Two tangents (ii) four normals can be drawn from a point to the
rectangular hyperbola xy = c2.
(3) The condition that the line lx + my + n = 0 may be a tangent to the
rectangular hyperbola xy = c2 is 4c2lm = n2
(4) If the normal to the rectangular hyperbola xy = c2 at t1 meets the
curve again at t2 prove that t13 t2 = 1.
Note : For the proof of above results one may refer the Solution Book.

262

Now, we summarise the results of the four standard types of parabolas.


Type

Equation

Diagram

Focus

Equation of
Directrix

Axis

x =a

y=0

(0, 0)

x=a

4a

x=a

y=0

(0, 0)

x=a

4a

y =a

x=0

(0, 0)

y=a

4a

y=a

x=0

(0, 0)

y=a

4a

Vertex

Equation of
Latus Rectum

Length of
Latus Rectum

Open rightwards

y = 4ax

(a, 0)

y
2

Open leftwards

y = 4ax

Open upwards

x2 = 4ay

( a, 0)

(0, a)
x
y

Open downwards

x = 4ay

(0, a)

177

Thus we get the following :


Cartesian form :
Equation of chord joining
(x1, y1) and (x2, y2)

Parabola
4a
y y1 = y + y (x x1)
1
2

y y1 =

Equation of tangent at (x1, y1)

yy1 = 2a(x + x1)

xx1 / a2 + yy1/b2 = 1

Equation of normal at (x1, y1)

xy1 + 2ay = x1y1 + 2ay1

Parametric form :
Equation of chord

Parabola
Chord joining the
points t1 and t2 is
y(t1 + t2) = 2x + 2at1t2

Equation of tangent

at t is yt = x + at2

Equation of normal

at t is
tx + y = 2at + at3

Ellipse

Hyperbola

b (x1+x2)
2

a (y1+y2)

(x x1)

b (x1+x2)
(x x1)
y y1 = 2
a (y1+y2)
xx1 / a2 yy1/b2 = 1

a2x b2y
2
2
x1 y1 = a b
Ellipse
Chord joning the points 1 and 2 is

a2x b2y
2
2
x1 + y1 = a + b
Hyperbola
Chord joning the points 1 and 2 is

(1 2)
x (1 + 2) y (1 + 2)
cos
+
sin
=
cos
a
b
2
2
2
x
y
at is a cos + b sin = 1
by
ax

= a2 b2
cos sin

(1 + 2)
x (1 2) y (1 + 2)

cos
sin
=
cos
a
b
2
2
2
x
y
at is a sec b tan = 1
by
ax
+
= a2 + b2
sec tan

243

5. DIFFERENTIAL CALCULUS
APPLICATIONS - I
5.1 Introduction :
In higher secondary first year we discussed the theoretical aspects of
differential calculus, assimilated the process of various techniques involved and
created many tools of differentiation. Geometrical and kinematical significances
for first and second order derivatives were also interpreted. Now let us learn
some practical aspects of differential calculus.
At this level we shall consider problems concerned with the applications to
(i) plane geometry, (ii) theory of real functions, (iii) optimisation problems and
approximation problems.

5.2 Derivative as a rate measure :


If a quantity y depends on and varies with a quantity x then the rate of
dy
change of y with respect to x is dx .
Thus for example, the rate of change of pressure p with respect to height
dp
h is dh . A rate of change with respect to time is usually called as the rate of
change, the with respect to time being assumed. Thus for example, a rate of
di
d
change of current i is dt and a rate of change of temperature is dt and so
on.
Example 5.1 : The length l metres of a certain metal rod at temperature C is
given by l = 1 + 0.00005 + 0.00000042. Determine the rate of change of
length in mm/C when the temperature is (i) 100C and (ii) 400C.
dl
.
Solution : The rate of change of length means
d
Since length l = 1 + 0.00005 + 0.00000042,
dl
= 0.00005 + 0.0000008 .
d
(i) when = 100C
dl
= 0.00005 + (0.0000008) (100)
d
= 0.00013 m/C = 0.13 mm/C

(ii) when = 400C


dl
= 0.00005 + (0.0000008) (400)
d
= 0.00037 m/C = 0.37 mm/C
Example 5.2 : The luminous intensity I candelas of a lamp at varying voltage
V is given by : I = 4 104V2. Determine the voltage at which the light is
increasing at a rate of 0.6 candelas per volt.
dI
Solution : The rate of change of light with respect to voltage is given by dV .
Since I = 4 104V2
dI
4
dV = 8 10 V.
dI
When the light is increasing at 0.6 candelas per volt then dV = + 0.6. Therefore
we must have + 0.6 = 8 10-4 V, from which,
0.6
= 0.075 104 = 750 Volts.
Voltage V =
8 104
Velocity and Acceleration :
y
Distance

A car describes a distance x


metres in time t seconds along a
straight road. If the velocity v is
x
constant, then v = t m/s i.e., the
slope (gradient) of the distance/time
graph shown in Fig.5.1 is constant.

x
t
x

Time

Fig. 5.1
If, however, the velocity of the
car is not constant then the distance /
time graph will not be a straight line.
It may be as shown in Fig.5.2
The average velocity over a
small time t and distance x is
given by the gradient of the chord AB
i.e., the average velocity over time t
x
is .
t

y
Distance

B
x
A

t
Time

Fig. 5.2

Velocity

As t 0, the chord AB becomes a tangent, such that at point A the


dx
velocity is given by v = dt . Hence the velocity of the car at any instant is
given by gradient of the distance / time graph. If an expression for the distance x
is known in terms of time, then the velocity is obtained by differentiating the
expression.
The acceleration a of the car is
y
defined as the rate of change of
D
velocity. A velocity / time graph is
shown in Fig.5.3. If v is the change
y
in v and t is the corresponding
v
C
. As
change in time, then a =
t
t
x
Time
t 0 the chord CD becomes a
tangent such that at the point C,
Fig. 5.3
dv
the acceleration is given by a = dt
Hence the acceleration of the car at any instant is given by the gradient of
the velocity / time graph. If an expression for velocity is known in terms of time
t, then the acceleration is obtained by differentiating the expression.
dx
dv
Acceleration a = dt , where v = dt
d2x
d dx
Hence a = dt dt = 2
dt
The acceleration is given by the second differential coefficient of distance
x with respect to time t. The above discussion can be summarised as follows. If
a body moves a distance x meters in time t seconds then
(i) distance x = f(t).
dx
(ii) velocity v = f (t) or dt , which is the gradient of the
distance / time graph.
dv
d2x
(iii) Acceleration a = dt = f (t) or 2 , which is the gradient of the
dt
velocity / time graph.
Note : (i) Initial velocity means velocity at t = 0
(ii) Initial acceleration means acceleration at t = 0.
(iii) If the motion is upward, at the maximum height, the velocity is zero.
(iv) If the motion is horizontal, v = 0 when the particle comes to rest.

Example 5.3 : The distance x metres described by a car in time t seconds is


given by: x = 3t3 2 t2 + 4t 1. Determine the velocity and acceleration when
(i) t = 0 and (ii) t = 1.5 s
Solution :

distance x = 3t3 2 t2 + 4t 1
dx
velocity v = dt = 9t2 4 t + 4 m/s
d2x
= 18t 4 m/s2
dt2
When time t = 0

acceleration a =
(i)

velocity v = 9(0)2 4(0) + 4 = 4 m/s


and acceleration a = 18(0) 4 = 4 m/s2
(ii)
when time t = 1.5 sec
velocity v = 9(1.5)2 4(1.5) + 4 = 18.25 m/sec
and acceleration a = 18(1.5) 4 = 23 m/sec2
Example 5.4 : Supplies are dropped from an helicopter and distance fallen in
1
time t seconds is given by x = 2 gt2 where g = 9.8 m/sec2. Determine the
velocity and acceleration of the supplies after it has fallen for 2 seconds.
1
1
distance x = 2 gt2 = 2 (9.8) t2 = 4.9 t2 m
Solution :
dx
velocity v = dt = 9.8t m/sec
d2x
acceleration a = 2 = 9.8 m/sec2
dt
When time t = 2 seconds
velocity v = (9.8)(2) = 19.6 m/sec
and acceleration a = 9.8 m/sec2 which is the acceleration due to
gravity.
Example 5.5 : The angular displacement radians of a fly wheel varies with
time t seconds and follows the equation = 9t2 2t3. Determine
(i) the angular velocity and acceleration of the fly wheel when time
t = 1 second and
(ii) the time when the angular acceleration is zero.
Solution : (i)

angular displacement = 9t2 2t3 radians.


d
angular velocity = dt = 18t 6t2 rad/s

When time t = 1 second,


= 18(1) 6(1)2 = 12 rad/s
angular acceleration =

d 2
2
2 = 18 12t rad/s
dt

when t = 1, angular acceleration = 6 rad/ s2


(ii) Angular acceleration is zero 18 12t = 0, from which t = 1.5 s
Example 5.6 : A boy, who is standing on a pole of height 14.7 m throws a stone
vertically upwards. It moves in a vertical line slightly away from the pole and
falls on the ground. Its equation of motion in meters and seconds is
x = 9.8 t 4.9t2 (i) Find the time taken for upward and downward motions.
(ii) Also find the maximum height reached by the stone from the ground.
Solution :
(i) x = 9.8 t 4.9 t2
At the maximum height v = 0
Max. Ht.
dx
v = dt = 9.8 9.8 t
s =0
v = 0 t = 1 sec
The time taken for upward
motion is 1 sec. For each position x,
s =-147
Ground
there corresponds a time t. The
ground position is x = 14.7, since
Fig. 5.4
the top of the pole is taken as x = 0.
To get the total time, put x = 14.7 in the given equation.
i.e., 14.7 = 9.8 t 4.9t2 t = 1, 3
t = 1 is not admissible and hence t = 3
The time taken for downward motion is 3 1 = 2 secs
(ii) When t = 1, the position x = 9.8(1) 4.9(1) = 4.9 m
The maximum height reached by the stone = pole height + 4.9 = 19.6 m

5.3 Related Rates :


In the related rates problem the idea is to compute the rate of change of one
quantity in terms of the rate of change of another quantity. The procedure is to
find an equation that relates the two quantities and then use the chain rule to
differentiate both sides with respect to time.
We suggest the following problem solving principles that may be followed
as a strategy to solve problems considered in this section.

(1) Read the problem carefully.


(2) Draw a diagram if possible.
(3) Introduce notation. Assign symbols to all quantities that are functions
of time.
(4) Express the given information and the required rate in terms of
derivatives.
(5) Write an equation that relates the various quantities of the problem. If
necessary, use the geometry of the situation to eliminate one of the
variables by substitution.
(6) Use the chain rule to differentiate both sides of the equation with
respect to t.
(7) Substitute the given information into the resulting equation and solve
for the unknown rate.
Illustration : Air is being pumped into a spherical balloon so that its volume
increases at a rate of 100 cm3/s. How fast is the radius of the balloon increasing
when the diameter is 50 cm.
Solution :
We start by identifying two things.
(i) The given information : The rate of increase of the volume of air is
100 cm3/s. and
(ii) The unknown : The rate of increase of the radius when the diameter is
50 cm.
In order to express these quantities mathematically we introduce some
suggestive notation.
Let V be the volume of the balloon and let r be its radius.
The key thing to remember is that the rates of change are derivatives. In
this problem, the volume and the radius are both functions of time t. The rate of
dV
increase of the volume with respect to time is the derivative dt and the rate of
dr
increase of the radius is dt . We can therefore restate the given and the unknown
as follows :
dV
dr
Given : dt = 100 cm3/s and unknown : dt when r = 25 cm.
dV
dr
In order to connect dt and dt we first relate V and r by the formula for
4
the volume of a sphere V = 3 r3.

dy/dt =?

Wall

In order to use the given information, we differentiate both sides of this


equation with respect to t. To differentiate the right side, we need to use chain
rule as V is a function of r and r is a function of t.
dV dV dr
dr
dr
4
i.e., dt = dr . dt = 3 3r2 dt = 4r2 dt
dr
1 dV
Now we solve for the unknown quantity dt =
.
4r2 dt
dV
If we put r = 25 and dt = 100 in this equation,
dr
1 100
1
we obtain dt =
2 = 25
4(25)
1
cm/s.
i.e., the radius of the balloon is increasing at the rate of
25
Example 5.7 : A ladder 10 m long rests against a vertical wall. If the bottom of
the ladder slides away from the wall at a rate of 1 m/sec how fast is the top of
the ladder sliding down the wall when the bottom of the ladder is 6 m from the
wall ?
Solution : We first draw a diagram
y
and lable it as in Fig. 5.5
Let x metres be the distance
from the bottom of the ladder to the
wall and y metres be the vertical
10
y
distance from the top of the ladder to
the ground. Note that x and y are both
dx/dt =1
x
functions of timet. We are given
x
Ground
dx
that dt = 1 m/sec and we are asked
dy
Fig. 5.5
to find dt when x = 6 m.
In this question, the relationship between x and y is given by the
Pythagoras theorem : x2 + y2 = 100
Differentiating each side with respect to t, using chain rule, we have
dy
dx
2x dt + 2y dt = 0
and solving this equation for the derived rate we obtain,
dy
x dx
dt = y dt

When x = 6, the Pythagoras theorem gives, y = 8 and so substituting these


dx
dy
6
-3
values and dt = 1, we get dt = 8 (1) = 4 m/sec.
3
The ladder is moving downward at the rate of 4 m/sec.
Example 5.8 : A car A is travelling from west at 50 km/hr. and car B is
travelling towards north at 60 km/hr. Both are headed for the intersection of the
two roads. At what rate are the cars approaching each other when car A is 0.3
kilometers and car B is 0.4 kilometers from the intersection?
Solution :
x
A
C
We draw Fig. 5.6 where C is the
intersection of the two roads. At a given
time t, let x be the distance from car A to C,
y
z
let y be the distance from car B to C and let
z be the distance between the cars A and B
B
where x, y and z are measured in kilometers.
Fig. 5.6
dy
dx
We are given that dt = 50 km/hr and dt = 60 km/hr.
Note that x and y are decreasing and hence the negative sign. We are asked
dz
to find dt . The equation that relate x, y and z is given by the Pythagoras
theorem z2 = x2 + y2
Differentiating each side with respect to t,
dx
dy
dz 1
dy
dx
dz
we have 2z dt = 2x dt + 2y dt dt = z x dt + y dt

When x = 0.3 and y = 0.4 km, we get z = 0.5 km and we get


1
dz
dt = 0.5 [0.3 ( 50) + 0.4 (60)] = 78 km/hr.
i.e., the cars are approaching each other at a rate of 78 km/hr.
Example 5.9 : A water tank has the shape of an inverted circular cone with base
radius 2 metres and height 4 metres. If water is being pumped into the tank at a
rate of 2m3/min, find the rate at which the water level is rising when the water
is 3m deep.

Solution :
2m
We first sketch the cone
rm
and label it as in Fig. 5.7. Let V, r
4m
and h be respectively the volume of
h
the water, the radius of the cone
and the height at time t, where
Fig. 5.7
t is measured in minutes.
dV
dh
We are given that dt = 2m3/min. and we are asked to find dt when h is 3m.
1
The quantities V and h are related by the equation V = 3 r2h. But it is very
useful to express V as function of h alone.
r
2
In order to eliminate r we use similar triangles in Fig. 5.7 to write h = 4
1

h 2
h
r = 2 and the expression for V becomes V = 3 2 h = 12 h3.

Now we can differentiate each side with respect to t and we have
2 dh
dh
4 dV
dV
dt = 4 h dt dt = h2 dt
dV
Substituting h = 3m and dt = 2m3/min.
4
8
dh
we get, dt =
2 . 2= 9 m/min
(3)
EXERCISE 5.1
(1) A missile fired from ground level rises x metres vertically upwards in
25
t seconds and x = 100t - 2 t2. Find (i) the initial velocity of the missile,
(ii) the time when the height of the missile is a maximum (iii) the
maximum height reached and (iv) the velocity with which the missile
strikes the ground.
(2) A particle of unit mass moves so that displacement after t secs is given by
x = 3 cos (2t 4). Find the acceleration and kinetic energy at the end of 2
K.E. = 1 mv2, m is mass
secs.
2

(3) The distance x metres traveled by a vehicle in time t seconds after the
brakes are applied is given by : x = 20 t 5/3t2. Determine (i) the speed
of the vehicle (in km/hr) at the instant the brakes are applied and (ii) the
distance the car travelled before it stops.
9

(4) Newtons law of cooling is given by = 0 ekt, where the excess of


temperature at zero time is 0C and at time t seconds is C. Determine
the rate of change of temperature after 40 s, given that 0 = 16 C and

[e1.2 = 3.3201)

k = 0.03.

(5) The altitude of a triangle is increasing at a rate of 1 cm/min while the area
of the triangle is increasing at a rate of 2 cm2/min. At what rate is the
base of the triangle changing when the altitude is 10 cm and the area is
100 cm2.
(6) At noon, ship A is 100 km west of ship B. Ship A is sailing east at 35
km/hr and ship B is sailing north at 25 km/hr. How fast is the distance
between the ships changing at 4.00 p.m.
(7) Two sides of a triangle are 4m and 5m in length and the angle between
them is increasing at a rate of 0.06 rad/sec. Find the rate at which the
area of the triangle is increasing when the angle between the sides of
fixed length is /3.
(8) Two sides of a triangle have length 12 m and 15 m. The angle between
them is increasing at a rate of 2 /min. How fast is the length of third side
increasing when the angle between the sides of fixed length is 60?
(9) Gravel is being dumped from a conveyor belt at a rate of 30 ft3/min and
its coarsened such that it forms a pile in the shape of a cone whose base
diameter and height are always equal. How fast is the height of the pile
increasing when the pile is 10 ft high ?

5.4 Tangents and Normals (Derivative as a measure of slope)


y

In this section the applications


of derivatives to plane geometry is
discussed. For this, let us consider a
curve whose equation is y = f(x).
On this curve take a point
P(x1,y1). Assuming that the tangent

y =f (x)
P (x1,y1)

No
rm
al

at this point is not parallel to the coordinate axes, we can write the
equation of the tangent line at P.

Time

Fig. 5.8

10

The equation of a straight line with slope (gradient) m passing through


(x1,y1) is of the form y y1 = m(x x1). For the tangent line we know the slope
dy
m = f (x1) = dx at (x1,y1) and so the equation of the tangent is of the form

y y1= f (x1) (x x1). If m=0, the curve has a horizontal tangent with equation
lim
y = y1 at P(x1,y1). If f(x) is continuous at x = x1, but x x f (x) = the
1
curve has a vertical tangent with equation x = x1.
In addition to the tangent to a curve at a given point, one often has to
consider the normal which is defined as follows :
Definition : The normal to a curve at a given point is a straight line passing
through the given point, perpendicular to the tangent at this point.
From the definition of a normal it is clear that the slope of the normal m
1
and that of the tangent m are connected by the equation m = m .
1
1
= dy
i.e., m =
f (x1) (x ,y )
dx 1 1
Hence the equation of a normal to a curve y = f(x) at a point P(x1,y1) is
1
of the form y y1=
( x x1).
f (x1)
The equation of the normal at (x1,y1) is
(i) x = x1 if the tangent is horizontal (ii) y = y1 if the tangent is vertical and
1
(iii) y y1 = m (x x1) otherwise.
Example 5.10: Find the equations of the tangent and normal to the curve y = x3
at the point (1,1).
Solution :

We have y = x3 ; slope y= 3x2.

At the point (1,1), x = 1 and m = 3(1)2 = 3.


Therefore equation of the tangent is y y1 = m(x x1)
y 1 = 3(x 1) or y = 3x 2
1
The equation of the normal is y y1 = m (x x1)
1
4
1
y 1 = 3 (x 1) or y = 3 x + 3

11

Example 5.11 : Find the equations of the tangent and normal to the curve
y = x2 x 2 at the point (1, 2).
dy
We have y = x2 x 2 ; slope, m = dx = 2x 1.
Solution :
At the point (1,2), m = 1
Hence the equation of the tangent is y y1 = m(x x1) i.e., y (2) = x 1
i.e., y = x 3
1
Equation of the normal is y y1 = m (x x1)
1
i.e., y (2) = 1 (x 1)
or y = x 1
Example 5.12 : Find the equation of the tangent at the point (a,b) to the
curve xy = c2.
Solution : The equation of the curve is xy = c2.
Differentiating w.r.to x we get,
dy
y +x dx = 0
y
b
dy
dy
or dx = x and m = dx
(a, b)= a .
Hence the required equation of the tangent is
b
y b = a (x a)
i.e., ay ab = bx + ab
x y
bx + ay = 2ab or a + b = 2

Example 5.13 : Find the equations of the tangent and normal at = 2 to the
curve x = a ( + sin ), y = a (1 + cos ).

dx
= a (1 + cos) = 2a cos2 2
Solution : We have
d
dy

= a sin = 2a sin 2 cos 2


d
dy
d

dy
Then dx = dx = tan 2
d

12


dy
Slope m = dx
= / = tan 4 = 1
2

Also for = 2 , the point on the curve is a 2 + a, a.

Hence the equation of the tangent at = 2 is

y a = (1) x a 2 + 1

1
1
i.e., x + y = 2 a + 2a or x + y 2 a 2a = 0
Equation of the normal at this point is

y a = (1) x a 2 + 1

1
or x y 2 a = 0
Example 5.14 : Find the equations of tangent and normal to the curve
16x2 + 9y2 = 144 at (x1,y1) where x1 = 2 and y1 > 0.
Solution : We have 16x2 + 9y2 = 144
(x1,y1) lies on this curve, where x1 = 2 and y1 > 0
(16 4) + 9 y12 = 144 or 9 y12 = 144 64 = 80
80
80
3 . But y1 > 0 y1 = 3
80

The point of tangency is (x1,y1) = 2 , 3

80
y12 = 9

y1 =

We have 16x2 + 9y2 = 144


32 x
16 x
dy
Differentiating w.r.to x we get
dx = 18 y = 9 y
The slope at

2 , 80 = dy
3
dx 2 ,

16
= 9

13

80
3

2
8
=
3 5
80
3

80
8
The equation of the tangent is y 3 =
(x 2)
3 5
On simplification we get 8x + 3 5 y = 36
Similarly the equation of the normal can be found as 9 5 x 24 y + 14 5 = 0
Example 5.15 : Find the equations of the tangent and normal to the ellipse

x = a cos, y = b sin at the point = 4 .

Solution : At = 4 , (x1,y1) = a cos 4 , b sin 4 =

2 , 2
dx
dy
= a sin ,
= b cos .
y
d
d
dy
T
d
b
dy
P
=
=
cot
=/4
dx
a
dx
x
d
O
N

b
b
m = = a cot 4 = a
Fig. 5.9
b
b
a
,
and the slope is m = a .
Thus the point of tangency is
2
2
b
b
a
The equation of the tangent is y
= a x or bx + ay ab 2 = 0
2
2

b
a
a
The equation of the normal is y
= b x
2
2

or (ax by) 2 (a2 b2) = 0.


Example 5.16 : Find the equation of the tangent to the parabola, y2 = 20 x
which forms an angle 45 with the x axis.
Solution : We have y2 = 20x . Let (x1,y1) be the tangential point
10
10
Now 2yy = 20 y = y ie., at (x1, y1) m = y
1
But the tangent makes an angle 45 with the x axis.
slope of the tangent m=tan 45 = 1
10
From (1) and (2) y = 1 y1 = 10
1
But (x1,y1) lies on y2 = 20x y12 = 20 x1

14

(1)
(2)

100 = 20 x1 or x1 = 5
i.e., (x1,y1) = (5,10)
and hence the equation of the tangent at (5, 10) is
y 10 = 1(x 5)
or y = x + 5.
Note : This problem is suitable for equation of any tangent to a parabola
a
i.e., y = mx + m

5.5 Angle between two curves :


The angle between the curves C1 and C2 at a point of intersection P is
defined to be the angle between the tangent lines to C1 and C2 at P (if these
tangent lines exist) Let us represent the two curves C1 and C2 by the Cartesian
equation y = f(x) and y = g(x) respectively. Let them intersect at P (x1,y1) .
If 1 and 2 are the angles made by the tangents PT1 and PT2 to
C1 and C2 at P, with the positive direction of the x axis, then m1 = tan 1 and
m2 = tan 2 are the slopes of PT1 and PT2 respectively.
y

Let be the angle between PT1


and PT2. Then = 2 1 and

y =g (x)
C2

y =f (x)
C1

tan = tan (2 1)
tan 2 tan 1
=
1 + tan 1 tan 2

T1

m2 m1
= 1+m m
1 2

180

where 0 <

T2

Time

Fig. 5.10

We observe that if their slopes are equal namely m1 = m2 then the two
curves touch each other. If the product m1 m2 = 1 then these curves are said to
cut at right angles or orthogonally. We caution that if they cut at right angles
then m1 m2 need not be 1.
Note that in this case 1 is acute and 2 is obtuse and = 2 1. If 1 is
obtuse and 2 is acute, then = 12.

15

Combining together the angle between tangents can be given as 12 or


tan 1 tan2
m1 m2
= 1 + m m
tan = tan(12) =
1 + tan 1 tan 2
1 2
Example 5.17 : Find the angle between the curves y = x2 and y = (x 2)2 at the
point of intersection.
Solution : To get the point of intersection of the curves solve the equation
y =x2

y =(x-2)2

we get x2 = (x 2)2
This gives x = 1. When x = 1, y = 1
The point of intersection is (1, 1)
dy
Now y = x2 dx = 2x
dy
m1 = dx
(1,1) = 2

2
1

(1,1)
2

(0,0)

Tan-1(4/3)

Fig. 5.11
dy
dy
y = (x 2)2 dx = 2(x 2) m2 = dx
(1,1) = 2.
If is the angle between them, then
22
4
1 4
tan = 1 4 =

3 = tan 3
Example 5.18 : Find the condition for the curves
ax2 + by2 = 1, a1x2 + b1y2= 1 to intersect orthogonally.
Solution :
If (x1,y1) is the point of intersection, then ax12 + by12 = 1 ; a1x12 + b1y12 = 1
b1 b
a a1
then, x12 = ab a b , y12 = ab a b (By Cramers rule)
1
1
1
1

ax
dy
1
For ax2 + by2 = 1, m1 = dx
(x ,y ) = by1
1 1

a1x1
= b y
(x ,y )
1 1
1 1
For orthogonal intersection, we have m1m2 = 1. This gives

and for a1x2 + b1y2 = 1,

dy
m2 = dx
2

a a1x1
ax1 a1x1
= 1.
by b y = 1 or
1 1 1
bb1y12

16

b1 b
a a1
aa1x12 + bb1y12 = 0 aa1 ab a b + bb1 ab a b = 0

b1 b
a a1
+ aa = 0
aa1 (b1 b) + bb1 (a a1) = 0 bb
1
1
1 1
1
1
1 1 1 1
or b b + a a = 0 or a a = b b which is the required condition.
1
1
1
1
Example 5.19 : Show that x2 y2 = a2 and xy = c2 cut orthogonally.
Solution : Let (x1,y1) be the point of intersection of the given curves
x12 y12 = a2 and x1 y1 = c2
dy
dy x
2x 2y dx = 0 dx = y
x1
x1
dy
m1 = dx
= y ie., m1 = y
(x ,y ) 1
1

x2 y2 = a2

1 1

xy = c2

c2
y = x

dy
dx =

c2
x2

c2
c2
dy
m2 = dx
= 2 i.e., m2 = 2
(x ,y ) x1
x1
1 1
c2
x1 c2 c2
m1m2 = y 2 = x y = 2 = 1
c
1 x1 1 1
the curves cut orthogonally.
Example 5.20 : Prove that the sum of the intercepts on the co-ordinate axes of

any tangent to the curve x = a cos4, y = a sin4, 0 2 is equal to a.


Solution : Take any point as (a cos4, a sin4, )
dx
y
= 4a cos3 sin ;
Now
d
= /2
dy
= 4a sin3 cos
and
d
(0,a)
sin2
dy
dx =
cos2
O

=0
x
(a,0)

Fig. 5.12

17

i.e., slope of the tangent at is =

sin2
cos2

Equation of the tangent at is (y a sin4) =

sin2
4
2 (x a cos )
cos

or x sin2 + y cos2 = a sin2 cos2


y
x
=1

2 +
a cos a sin2
i.e., sum of the intercepts = a cos2 + a sin2 = a
EXERCISE 5.2
(1) Find the equation of the tangent and normal to the curves
(i)

(ii) y = x sin x cos x, at x = 2

y = x2 4x 5 at x = 2

(iii) y = 2 sin2 3x at x = 6

1 + sinx

(iv) y = cos x at x = 4

(2) Find the points on curve x2 y2=2 at which the slope of the tangent is 2.
(3) Find at what points on the circle x2 + y2 = 13, the tangent is parallel to
the line 2x + 3y = 7
(4) At what points on the curve x2 + y2 2x 4y + 1 = 0 the tangent is
parallel to (i) x axis (ii) y axis.
(5) Find the equations of those tangents to the circle x2 + y2 = 52, which
are parallel to the straight line 2x + 3y = 6.
(6) Find the equations of normal to y = x3 3x that is parallel to
2x + 18y 9 = 0.
(7) Let P be a point on the curve y = x3 and suppose that the tangent line at
P intersects the curve again at Q. Prove that the slope at Q is four times
the slope at P.
(8) Prove that the curve 2x2 + 4y2 = 1 and 6x2 12y2= 1 cut each other at
right angles.
(9) At what angle do the curves y = ax and y = bx intersect (a b) ?
(10) Show that the equation of the normal to the curve
x = a cos3 ; y = a sin3 at is x cos y sin = a cos 2.
(11) If the curve y2 = x and xy = k are orthogonal then prove that 8k2 = 1.

18

5.6 Mean value theorems and their applications :


In this section our main objective is to prove that between any two points
of a smooth curve there is a point at which the tangent is parallel to the chord
joining two points. To do this we need the following theorem due to Michael
Rolle.
5.6.1 Rolles Theorem : Let f be a real valued function that satisfies the
following three conditions :
(i) f is defined and continuous on the closed interval [a, b]
(ii) f is differentiable on the open interval (a, b)
(iii) f (a) = f (b)
Then there exists atleast one point c (a,b) such that f (c) = 0
Some observations :
Rolles theorem is applied to the position function s = f(t) of a moving
object.
If the object is in the same place at two different instants t = a and
t = b then f(a) = f(b) satisfying hypothesis of Rolles theorem.
Therefore the theorem says that there is some instant of time t = c
between a and b where f (c) = 0 i.e., the velocity is 0 at t = c.
Note that this is also true for an object thrown vertically upward
(neglecting air resistance).
Rolles Theorem applied to theory of equations : If a and b are two
roots of a polynomial equation f(x) = 0, then Rolles Theorem says
that there is atleast one root c between a and b for f (x) = 0.
Rolles theorem implies that a smooth curve cannot intersect a
horizontal line twice without having a horizontal tangent in between.
Rolles theorem holds trivially for the function f(x) = c, where c is a
constant on [a,b].
The converse of Rolles Theorem is not true ie., if a function
f satisfies f (c) = 0 for c (a,b) then the conditions of hypothesis
need not hold.
Example 5.21 : Using Rolles theorem find the value(s) of c.
(i) f(x) = 1 x2 , 1 x 1
(ii) f(x) = (x a) (b x),
a x b, a b.
1
(iii) f(x) = 2x3 5x2 4x + 3,
2 x 3

19

Solution :
(i) The function is continuous in [1,1] and differentiable in (1,1).
f(1) = f (1) = 0 all the three conditions are satisfied.
1 2x
x
=
f (x) = 2
2
1 x
1 x2
f (x) = 0 x = 0.
(Note that for x = 0, denominator = 1 0) Thus the suitable point for which
Rolles theorem holds is c = 0.
(ii) f(x) = (x a) (b x), a x b, a b.
f (x) is continuous on [a,b] and f (x) exists at every point of (a,b).
f(a) = f(b) = 0 All the conditions are satisfied.
f (x) = (b x) (x a)
a+b
f (x) = 0 2x = b a x = 2
a+b
The suitable point c of Rolles theorem is c = 2
1
(iii) f(x) = 2x3 5x2 4x + 3, 2 x 3
1
1
f is continuous on 2 , 3and differentiable in 2 , 3

f() = 0 = f(3). All the conditions are satisfied.


f (x) = 6x2 10x 4
1
f (x) = 0 3x2 5x 2 = 0 (3x + 1) (x 2) = 0 x = 3 or x = 2.
1
1
x = 3 does not lie in 2, 3 x = 2 is the suitable c of Rolles theorem

Remark : Rolles theorem cannot be applied if any one of the conditions does not hold.
Example 5.22 : Verify Rolles theorem for the following :
(i) f(x) = x3 3x + 3 0 x 1
(ii) f(x) = tan x, 0 x
(iii) f(x) = | x |, 1 x 1
(iv) f(x) = sin2 x, 0 x
(v) f(x) = ex sin x,

0x
(vi) f(x) = x (x 1) (x 2), 0 x 2

20

Solution :
(i) f(x) = x3 3x + 3 0 x 1
f is continuous on [0,1] and differentiable in (0,1)
f(0) = 3 and f(1) = 1 f (a) f (b)
Rolles theorem, does not hold, since f (a) = f (b) is not satisfied.
Also note that f (x) = 3x2 3 = 0 x2 = 1 x = 1
There exists no point c (0,1) satisfying f (c) = 0.
(ii) f(x) = tan x, 0 x

f (x) is not continuous in [0,] as tan x tends to + at x = 2,


Rolles theorem is not applicable.
(iii) f(x) = | x |, 1 x 1
f is continuous in [1,1] but not differentiable in (1,1) since f (0) does
not exist.
Rolles theorem is not applicable.
(iv) f(x) = sin2 x, 0 x
f is continuous in [0,] and differentiable in (0,). f(0) = f () = 0
(ie.,) f satisfies hypothesis of Rolles theorem.
f (x) = 2 sin x cos x = sin 2x

3
f (c) = 0 sin 2c = 0 2c = 0, , 2, 3, ... c = 0, 2, , 2 , ...

since c = 2 (0,), the suitable c of Rolles theorem is c = 2.


(v) f(x) = ex sin x, 0 x
ex and sin x are continuous for all x, therefore the product ex sin x is
continuous in 0 x .
f (x) = ex sin x + ex cos x = ex (sin x + cos x) exist in 0 < x <
f (x) is differentiable in (0,).
f(0) = e0 sin 0 = 0
f() = e sin = 0
f satisfies hypothesis of Rolles theorem
Thus there exists c (0, ) satisfying f (c) = 0 ec(sin c + cos c) = 0
ec = 0 or sin c + cos c = 0

21

ec = 0 c = which is not meaningful here.


sin c
3
sin c = cos c cos c =1 tan c = 1 = tan 4
3
c = 4 is the required point.
(vi) f(x) = x (x 1) (x 2), 0 x 2,
f is continuous in [0,2] and differentiable in (0,2)
f(0) = 0 = f(2), satisfying hypothesis of Rolles theorem
Now f (x) = (x 1) (x 2) + x (x 2) + x (x 1) = 0
1
3x2 6x + 2 = 0 x = 1
3
1
The required c in Rolles theorem is 1
(0,2)
3
Note : There could exist more than one such c appearing in the statement of
Rolles theorem.
Example 5.23 : Apply Rolles theorem to find points on curve y = 1 + cos x,
where the tangent is parallel to x-axis in [0, 2].
Solution :
y
f(x) is continuous in [0,2] and
2

x
differentiable in (0,2)
(0,0)
f(0) = 0 = f(2) satisfying hypothesis
-1
of Rolles theorem.
-2
Now f (x) = sin x = 0 sin x = 0
(,-2)
x = 0, , 2, . . .
Fig. 5.13
x = , is the required c in (0,2). At x = , y = 1 + cos = 2.
the point (,2) is such that at this point the tangent to the curve is parallel
to x-axis.
EXERCISE 5.3
(1) Verify Rolles theorem for the following functions :
(i) f(x) = sin x,
0x
(ii) f(x) = x2,
(iii) f(x) = | x 1|,

0 x 1
0x2
3
3
(iv) f(x) = 4x3 9x, 2 x 2

22

(2) Using Rolles theorem find the points on the curve y = x2+1, 2 x 2
where the tangent is parallel to x axis.

5.6.2 Mean Value Theorem (Law of the mean due to Lagrange) :


Many results in this section depend on one central fact called law of the
mean or mean value theorem due to Joseph Louis Lagrange.
Theorem :Let f(x) be a real valued function that satisfies the following
conditions :
(i) f(x) is continuous on the closed interval [a,b]
(ii) f(x) is differentiable on the open interval (a,b)
Then there exists at least one point c (a,b) such that
f(b) f(a)
f (c) =
(1)
ba
Some Observations :
Note that if f(a) = f(b) then the law of the mean reduces to the Rolles
theorem.
Interpretation of law of the mean when applied to an equation of motion
s = f(t) :
The quantity s = f(b) f(a) is the change in s corresponding to
t = b a and R.H.S. of (1) is
s
f(b) f(a)
=
= average velocity from t = a to t = b.
ba
t
The equation then tells us that there is an instant c between a and b at
which the instantaneous velocity f (c) is equal to the average velocity. For
example, if a car has traveled 180 kms in 2 hours then the speedometer must
have read 90 kms/hr at least once.

23

y
C

( x)

y =f

The slope f (c)of the curve at C (c, f(c))


f(b) f(a)
is the same as the slope
of the
ba
chord joining the points A (a, f(a))
and
B (b, f(b)). Geometrically means that if the
function f is continuous on [a,b] and
differentiable on (a,b) then there is atleast one
number c in (a,b) where the tangent to the
curve is parallel to the chord through A and B.

A
x

(0,0)

Fig. 5.14

Remarks (1) : Since the value of c satisfies the condition a < c < b, it follows
ca
that (c a) < (b a) or
(< 1) = , (say).
ba
i.e.,

ca
= c a = (b a), 0 < < 1.
ba

But then c = a + (b a)
the law of the mean can be put in the form
f(b) f(a) = (b a) f (c)
= (b a) f [a + (b a)], 0 < < 1
and this is used in calculating approximate values of functions.
(2) Letting b a = h, the above result can be written as
f(a + h) = f(a) + hf (a + h), 0 < < 1
(3) If we let a = x, h = x, law of the mean becomes
f(x + x) = f(x) + x f (x + x) for some such that 0 < < 1.
Example 5.24 : Verify Lagranges law of the mean for f(x) = x3 on [2,2]
Solution : f is a polynomial, hence continuous and differentiable on [ 2, 2].
f(2) = 23 = 8 ; f (2) = (2)3 = 8
f (x) = 3x2 f (c) = 3c2
By law of the mean there exists an element c ( 2, 2) such that
f(b) f(a)
8 (8)
=4
f (c) =
3c2 =
4
ba
i.e.,

4
2
c2 = 3 c =
3

The required c in the law of mean are

2
2
and
as both lie in [2,2].
3
3

Example 5.25 :
A cylindrical hole 4 mm in diameter and 12 mm deep in a metal block is
rebored to increase the diameter to 4.12 mm. Estimate the amount of metal
removed.
Solution : The volume of cylindrical hole of radius x mm and depth 12 mm is
given by

24

12mm

V = f(x) = 12 x2
f (c) = 24c.
To estimate f(2.06) f(2) :
4mm
By law of mean,
f(2.06) f(2) = 0.06 f (c)
= 0.06 (24 c), 2 < c < 2.06
Take c = 2.01
f(2.06) f(2) = 0.06 24 2.01
= 2.89 cubic mm.
Fig. 5.15
Note : Any suitable c between 2 and 2.06 other than 2.01 also will give other
estimates.
Example 5.26 : Suppose that f(0) = 3 and f (x) 5 for all values of x, how
large can f(2) possibly be?
Solution : Since by hypothesis f is differentiable, f is continuous everywhere.
We can apply Lagranges Law of the mean on the interval [0,2]. There exist
atleast one c(0, 2) such that
f(2) f(0) = f (c) ( 2 0)
f(2) = f(0) + 2 f (c)
= 3 + 2 f (c)
Given that f (x) 5 for all x. In particular we know that f (c) 5.
Multiplying both sides of the inequality by 2, we have
2f (c) 10
f(2) = 3 + 2 f (c) 3 + 10 = 7
i.e., the largest possible value of f(2) is 7.
Example 5.27 : It took 14 sec for a thermometer to rise from 19C to 100C
when it was taken from a freezer and placed in boiling water. Show that
somewhere along the way the mercury was rising at exactly 8.5C/sec.
Solution : Let T be the temperature reading shown in the thermometer at any
time t. Then T is a function of time t. Since the temperature rise is continuous
and since there is a continuous change in the temperature the function is
differentiable too. By law of the mean there exists a t0 in (0, 14)
such that
T(t2) T(t1)
= T (t0)
t2 t1
Here T (t0) is the rate of rise of temperature at C.
25

Here t2 t1 = 14, T(t2) = 100 ; T(t1) = 19


100 + 19 119
= 14 = 8.5C/sec
T (t0) =
14
EXERCISE 5.4
(1) Verify Lagranges law of mean for the following functions :
(i) f(x) = 1 x2, [0,3]

1
(ii) f(x) = x , [1,2]

(iii) f(x) = 2x3 + x2 x 1, [0,2]

(iv) f(x) = x2/3, [2,2]

(v) f(x) = x3 5x2 3x , [1,3]


(2) If f(1) = 10 and f (x) 2 for 1 x 4 how small can f(4) possibly be?
(3) At 2.00 p.m a cars speedometer reads 30 miles/hr., at 2.10 pm it reads
50 miles / hr. Show that sometime between 2.00 and 2.10 the
acceleration is exactly 120 miles /hr2.

Generalised Law of the Mean :


If f(x) and g(x) are continuous real valued functions on [a,b] and f and g
are differentiable on (a,b) with g (x) 0 everywhere on (a,b) then there exist
f (c)
f(b) f(a)
=
atleast one value of x, say x = c, between a and b such that
g(b) g(a) g(c)
Remarks :
(1) This theorem is also known as Cauchys generalised law of the mean.
(2) Lagranges law of the mean is a particular case of Cauchy law of the
mean for the case g(x) = x for all x [a,b]
(3) Note that g(b) g(a), for, suppose g(b) = g(a), then by Rolles
theorem, g(x) = 0 for some x in (a,b) contradicting hypothesis of the
generalized law of the mean.
Extended Law of the mean :
If f(x) and its first (n 1) derivatives are continuous on [a,b] and if f(n)(x)
exists in (a,b), then there exist atleast one value of x, x = c say, in (a,b) such
that
f (a)
f (a)
f (n1)( a)
f (n)(c)
(ba)n1+ n! (ba)n...(1)
f(b)=f(a)+ 1! (ba)+ 2! (ba)2+...+
(n 1)!

26

Remarks : (1) If in the extended law of the mean b a = h then b = a + h


and (1) becomes
f (a)
f (a)
f (n1)( a) n 1 f (n)(c) n
f(a + h) = f(a) + 1! h + 2! h2 + ...
h
+ n! h ...(2)
(n 1)!
for some c (a, a + h) and this is known as Taylors theorem.
(2) When b is replaced by the variable x then (1) becomes
f (a)
f (n1)( a)
f (n)(c)
(x a)n 1 + n! (x a)n
f(x) = f(a) + 1! (x a) +...
(n 1)!
for some c (a, x)
(3) If n becomes sufficiently large (i.e., ; as n ) in Taylors theorem, then
(2) becomes
f (a)
f (a)
f (n)(a)
...(3)
f(a + h) = f(a) + 1! h + 2! h2 + . . . + n! hn + ...
provided f is differentiable any number of times. This series of expansion
of f(a + h) about the point a is usually known as Taylors Series.
(4) If in the extended law of the mean a is replaced by 0 and b is replaced with
the variable x, (1) becomes,
f (0)
f (0)
f (n1)(0) n 1 f (n) (c) n
f(x) = f(0) + 1! x + 2! x2 +...+
x
+ n!
x ___(4)
(n1)!
for some c (0,x) and is known as Maclaurins theorem.
(5) If n is sufficiently large (i.e., n ) in Maclaurins theorem, then it
f (0)
f (0)
becomes f(x) = f(0) + 1! x + 2! x2 + . . .
provided f is differentiable any number of times, This series expansion of f(x)
about the point 0 is usually known as Maclaurins Series.

Illustration : The Taylors series expansion of f(x) = sin x about x = 2 is


obtained by the following way.

f(x) = sin x
; f 2 = sin 2 = 1

f (x) = cos x
; f 2 = cos 2 = 0

f (x) = sin x ; f 2 = 1

f (x) = cos x ; f 2 = 0

27


f(x) = sin x = f 2 +

f 2


1! x 2 +

f 2

x
2!
2 + ...
2

(1)

= 1 + 0 x 2 + 2! x 2 + ...

1
1
2

sin x = 1 2! x 2 + 4! x 2 ...

Example 5.28 :
Obtain the Maclaurins Series for
1) ex

3) arc tan x or tan1x

2) loge(1 + x)

Solution :
(1)

f(x) = ex

f(0) = e0 = 1

f (x) = ex

; f (0) = 1

f (x) = e

f (0) = 1

!
1.x 1
1
f(x) = ex = 1 + 1! + 2! x2 + 3! x3
x
x2
x3
= 1 + 1! + 2! + 3! + ... holds for all x
f(x) = loge(1 + x) :
1
f (x) = 1 + x
;
1
;
f (x) =
(1 + x)2
+1.2
f (x) =
;
(1 + x)3
1.2.3
;
f (x) =
(1 + x)4

(2)

f(0) = loge1 = 0
f (0) = 1
f (0) = 1
f (0) = 2!
f (0) = (3!)

1
2!
1
3!
f(x) = loge(1 + x) = 0 + 1! x 2! x2 + 3! x3 4! x4 ... + .
x2 x3 x4
x 2 + 3 4 + .... 1 < x 1.

28

(3)

f(x) = tan1x
f (x) =

; f(0) = 0

1
= 1 x2 + x4 x6. ; f (0) = 1 = 1!
1 + x2

f (x) = 2x + 4x3 6x5 . ; f (0) = 0


f (x) = 2 + 12x2 30x4 . ; f (0) = 2 = (2!)
f iv(x) = 24x 120x3 . ; f iv(0) = 0
f v(x) = 24 360x2 . ; f v(0) = 24 = 4!
0
2
0
4!
1
tan1 x = 0 + 1! x + 2! x2 3! x3 + 4! x4 + 5! x5 +
1
1
= x 3 x3 + 5 x5
holds in | x | 1.
EXERCISE 5.5
Obtain the Maclaurins Series expansion for :
1

(2) cos2x
(4) tan x, 2 < x < 2
(3) 1 + x
(1) e2x

5.7 Evaluating Indeterminate forms :


Suppose f(x) and g(x) are defined on some interval [a,b], satisfying
Cauchys generalized law of the mean and vanish at a point x = a of this interval
f(x)
such that f(a) = 0 and g(a) = 0, then the ratio g(x) is not defined for x = a
0
and gives a meaningless expression 0 but has a very definite meaning for
values of x a. Evaluating the limit x a of this ratio is known as evaluating
0
indeterminate forms of the type 0.
3x 2
If f(x) = 3x 2 and g(x) = 9x + 7, then 9x + 7 is an indeterminate form

of the type
as the numerator and denominator becomes in the limiting

case, x tends to .

29

We also have other limits

lim
lim ex lim
lim
,
(x ex), x 0 xx,
x1/x
x x x
x

1
/(x1)
lim
which lead to other indeterminate forms of the types
and x 1 x
0 . , , 00, 0 and 1 respectively. These symbols must not be taken
literally. They are only convenient labels for distinguishing types of behaviour
at certain limits. To deal with such indeterminate forms we need a tool that
facilitates the evaluation. This tool was devised by John Bernoulli for
calculating the limit of a fraction whose numerator and denominator approach
zero. This tool today is known as lHpitals rule after Guillaume Francois
Antoinede lHpital.
lHpitals rule :
Let f and g be continuous real valued functions defined on the closed
interval [a,b], f, g be differentiable on (a,b) and g(c) 0.

f (c)
= L it follows that
g(c)
x c

Then if lim f(x) = 0, lim g(x) = 0 and if lim


x c

x c

f(x)
lim g(x) = L.

x c

Remarks :
(1) Using lHpitals method, evaluation of the limits of indeterminate
forms works faster than conventional methods. For instance, consider
sin x
lim
x . This limit we know is 1, which we obtained through
x 0
geometrical constructions, a laborious method.
sin x
cos x
But lim x = lim 1 = cos 0 = 1
x 0
x 0
(2) Note that lHpitals rule can be applied only to differentiable
functions for which the limits are in the indeterminate form. For,
x+1
1
x+1 1
lim
is 3 while if lHpitals rule is applied lim x + 3 = 1 = 1.
x
+
3
x 0
x 0
Here f(x) = x + 1 g(x) = x + 3 are both differentiable but not in the
indeterminate form
(3) The conclusion of lHpitals rule is unchanged if lim f(x) = 0 and
x a

lim g(x) = 0 and replaced by lim f(x) = and lim f(x) = .


x a

x a

30

x a

(4) All other indeterminate forms mentioned above can also be reduced to
0

0 or by a suitable transformation.
We need the following result in some problems
Composite Function Theorem :
Result : If lim g(x) = b and f is continuous at b,
x a

then lim f(g(x) = f lim g(x)


x a

xa

x
Example 5.29 : Evaluate : lim tan x
x 0
x
0
Solution : lim tan x is of the type 0 .
x 0
x
1
1
lim tan x = lim
2 =1=1
x 0
x 0 sec x
1
sin x
if exists
Example 5.30 : Find
lim
11
x + tan x
1
Let y = x As x , y 0
Solution :
1
sin x
sin y 0
= lim
lim
1 = 0
1
1
y 0 tan y
x + tan x
cos y
1
= lim 1 = 1 = 1
y0
2
1 + y
log(sin x)
Example 5.31 : lim
2
( 2x)
x /2

0
Solution : It is of the form 0
log(sin x)
lim
lim
2 =
( 2x)

x /2

1
sin x cos x
2( 2x) (2)

x /2

31

= lim

cotx
0
=
4( 2x) 0

= lim

cosec2x 1
=
42 8

x /2

x /2

Note that here lHpitals rule, applied twice yields the result.
x2
Example 5.32 : Evaluate : lim
ex
x
x2

Solution : lim
x is the type
e
x
x2
2x
2
2
lim
x = lim
x = lim
x = =0
e
e
e
x
x
x
1
Example 5.33 : Evaluate : lim cosec x x

x 0
1
Solution : lim cosec x x is of the type .

x 0
1
1
1
x sin x 0
lim cosec x
= lim sin x x = lim x sinx = 0
x

x 0
x 0
x 0
1 cos x 0
sinx
= 0 type = lim
sin
x
+
x
cos
x

x
x x sin x
cos
+
cos
x 0
x 0
0
=2 = 0
lim

sin x

Example 5.34 : Evaluate : lim (cot x)


x 0
sin x

Solution : lim (cot x)

is of the type 0.

x 0
sin x

log y = sin x log (cot x)


Let y = (cot x)
lim (log y) = lim sin x log (cot x)
x 0

x 0

log (cot x)

is of the type
cosec
x

x 0

= lim

32

Applying lHpitals rule,


1
( cosec2 x)
cotx
log (cot x)
lim
lim
cosec x = x 0 cosec x cot x
x 0
sin x
1
0
= lim cos x cos x = 1 = 0
x 0
i.e., lim log y = 0
x 0

By Composite Function Theorem, we have


0 = lim log y = log lim y lim y = e0 = 1

x 0

x 0

x 0

Caution : When the existence of lim f(x) is not known, log lim f(x) is
x a

x a

meaningless.
sinx

Example 5.35 : Evaluate lim x


x 0 +

Solution :

lim x

sinx

is of the form 00.

x 0 +
sinx

Let y = x

log y = sin x log x.

Note that x approaches 0 from the right so that log x is meaningful


log x
i.e., log y = cosec x
log x

lim log y = lim


which is of the type
.
cosec
x

x 0 +
x 0 +
Applying lHpitals rule,
1
x
log x
lim
= lim
cosec
x
x 0 +
x 0 +cosec x cot x
sin2x
0
= lim x cos x of the type 0

x 0 +

33

2 sin x cos x
= 0
x
x 0 + sin x cos x

= lim
ie.,

lim logy = 0
x 0 +

By Composite Function Theorem, we have


0 = lim log y = log lim y lim y = e0 = 1
x 0 +
x 0 +
x 0 +
Example 5.36 :
The current at time t in a coil with resistance R, inductance L and subjected
Rt
E
to a constant electromotive force E is given by i = R 1 e L . Obtain a
suitable formula to be used when R is very small.
Solution :

Rt

E 1 e L
i
lim
=
R0
R
R0
lim

0
(is of the type 0.)

Rt

lim
R0

t
E L e L
1

Et
Et
= L lim i = L is the suitable formula.
R0

EXERCISE 5.6
Evaluate the limit for the following if exists,
sin x
tan x x
(1) lim
(2) lim
2
x
x 2
x 0 x sinx
sin 1x
x
x 0

x n 2n
x 2 x 2
1
1
2 tan1 x

x2
(6) lim
1
x
x

(3) lim

(5)

lim
x

(4) lim

2
sin x
1/x
logex
x

(7)

lim
x

(9)

lim x2 logex.
x0+

cotx
lim cot 2x
x0
1
x1
(10) lim x
x1
(8)

34

(11)

lim


x /2

lim

cos x
(tanx)

lim
(12) x0+ xx

/x

(13) x 0 (cos x)

5.8 Monotonic Functions :


Increasing, Decreasing Functions
Differential calculus has varied applications. We have already seen some
applications to geometrical, physical and practical problems in sections 5.2, 5.3
and 5.4 In this section, we shall study some applications to the theory of real
functions.
In sketching the graph of a
y
y =f(x)
Positive
function it is very useful to
B
Negative
Gradient
know where it raises and where
Gradient
(slope)
Positive
(slope) D
it falls. The graph shown in
Gradient
(slope)
Fig. 5.16 raises from A to B,
C
falls from B to C, and raises
again from C to D.
A
The function f is said to be

increasing on the interval [a,b],

x
decreasing on [b,c], and
c d
O a x1 x2 b
increasing again on [c,d]. We
Fig. 5.16
use this as the defining property
of an increasing function.
Definition : A function f is called increasing on an interval I if
f(x1) f(x2) whenever x1 < x2 in I. It is called decreasing on I if f(x1) f(x2)
whenever x1 < x2 in I.
A function that is completely increasing or completely decreasing on I is
called monotonic on I.
In the first case the function f preserves the order.
i.e., x1 < x2 f(x1) f(x2) and in the later case the function f reverses the
order i.e., x1 < x2 f(x1) f(x2). Thanks to the order preserving property,
increasing functions are also known as order preserving functions. Similarly,
the decreasing functions are also known as order reversing functions.
Illustrations :
(i) Every constant function is an increasing function.
(ii) Every identity function is an increasing function.
35

(iii) The function f(x) = sin x is not an increasing function on R; but

f(x) = sin x is increasing on 0, 2 .

(iv) The function f(x) = 4 2x is decreasing

(v) The function f(x) = sin x is decreasing in the interval 2,



Note that f is increasing is equivalent to ( f) is decreasing.
Do you agree that each constant function is both increasing and
decreasing?
Caution : It is incorrect to say that if a function is not increasing, then it is
decreasing. It may happen that a function is neither increasing nor decreasing.
For instance, if we consider the interval [0,], the function sin x is neither

increasing nor decreasing. It is increasing on 0, 2 and decreasing on 2, .


There are other functions that are even worse. They are not monotonic on any
subinterval also. But most of the functions that we consider are not so bad.
Usually, by looking at the graph of the function one can say whether the
function is increasing or decreasing or neither. The graph of an increasing
function does not fall as we go from left to right while the graph of a decreasing
function does not rise as we go from left to right. But if we are not given the
graph, how do we decide whether a given function is monotonic or not ?
Theorem 1 gives us a criterion to do just that.
Theorem 1 : Let I be an open interval. Let f : I R be differentiable. Then
(i) f is increasing if and only if f (x) 0 for all x in I.
(ii) f is decreasing if and only if f (x) 0 for all x in I.
Proof : (i) Let f be increasing and x I. Since f is differentiable f (x) exists and
lim f(x + h) f(x)
is given by f (x) = h0
. If h > 0, then x + h > x and since f is
h
increasing, f(x + h) f(x). Hence f(x + h) f(x) 0.
If h < 0, then x + h < x and f (x + h) f(x). Hence f(x + h) f(x) 0
So either f(x + h) f(x) and h are both non-negative or they are both
non positive.
f(x + h) f(x)
Therefore
is non-negative for all non-zero values of h and
h
f(x + h) f(x)
must also be non-negative. Thus, f (x) 0
lim
h
h0

36

Conversely, let f (x) 0, for all x in I. Let x1 < x2 in I. We shall prove


that f(x1) f(x2).
f(x2) f(x1)
By the Law of mean, x x
= f (c) , for x1 < c < x2
2
1
f(x2) f(x1)
Since, f (c) 0, we have x x
0. Also x2 x1> 0 ( x1 < x2)
2
1
Thus f(x2) f(x1) 0 or f(x1) f(x2). Hence f is increasing
(ii) can be proved in a similar way. It can also be deduced by applying result
(i) to the function ( f).
Geometrical interpretation : The above theorem expresses the following
geometric fact. If on an interval I = [a,b] a function f(x) increases, then the
tangent to the curve y = f(x) at each point on this interval forms an acute angle
with the x-axis or (at certain points) is horizontal (See Fig.5.16), the tangent of
this angle is not negative. Therefore f (x) = tan 0. If the function f(x)
decreases on the interval [b,c] then the angle of inclination of the tangents form
an obtuse angle (or, at some points, the tangent is horizontal) ; the tangent of
this angle is not positive f (x) = tan 0.
From the class of increasing functions we can separate out functions which
are strictly increasing. The following definition gives the precise meaning of the
term strictly increasing function.
Definition : f : I R is said to be strictly increasing if x1 < x2 implies that f(x1)
< f(x2). We can similarly say that a function defined on I is strictly decreasing
if x1 < x2 implies f(x1) > f(x2)
For example, a constant function is not strictly increasing, nor is it strictly
decreasing (Fig. 5.17). The greatest integer function f(x)= x too, is increasing
(Fig. 5.18), but not strictly increasing, where as the function f(x) = x is strictly
increasing (Fig. 5.19).
y

y
=
x

1 f(x) =1
0

Fig. 5.17

f(

x)

3
-2 1
1
-2

Fig. 5.18

37

Fig. 5.19

Theorem 2 :
(i) Let f be positive on I. Then f is strictly increasing on I.
(ii) Let f be negative on I. Then f is strictly decreasing on I.
The proof of the theorem is easy and is left as an exercise.
Corollary : f is strictly monotonic on the interval I, if f is of the same sign
through out I.
You may have noticed that there is a difference between the statement of
Theorem 1 and Theorem 2.
f is increasing if and only if f is non negative
If f > 0, then f is strictly increasing.
Can we have if and only if in Theorem 2 also ?
The answer is no as shown in the following example.
Illustration : Define f : R R by f(x) = x3.
Suppose x1 < x2, Then x2 x1 > 0 and x12 + x22 > 0
This implies x23 x13 = (x2 x1) (x22 + x12 + x1 x2)
1
= (x2 x1) 2 [(x12 + x22)+ (x1 + x2)2] > 0
x13 < x23
Thus whenever x1 < x2, f(x1) < f(x2).
Hence f(x) = x3 is strictly increasing.
But its derivate f (x) = 3x2 and f (0) = 0.
Hence its derivate f is not strictly positive.
Note: If a function changes its signs at different points of a region (interval)
then the function is not monotonic in that region. So to prove the
non- monotonicity of a function, it is enough to prove that f has different signs
at different points.
Example 5.37 : Prove that the function f (x) = sin x + cos2x is not monotonic on

the interval 0, 4 .

Let f(x) = sin x + cos 2x


Solution :
Then f (x) = cos x 2sin 2x
Now f (0) = cos 0 2 sin 0 = 1 0 = 1 > 0

and f 4 = cos 4 2 sin 2 4





1
=
21<0
2

38

Thus f is of different signs at 0 and 4 Therefore f is not monotonic on 0, 4

Example 5.38 : Find the intervals in which f(x) = 2x3 + x2 20x is increasing
and decreasing.
Solution : f (x) = 6x2 + 2x 20 = 2(3x2 + x 10) = 2 (x + 2) ( 3x 5)
Now f (x) = 0 x = 2, and x = 5/3. The values 2 and 5/3 divide the real
line (the domain of f(x)) into intervals (, 2), ( 2, 5/3)and (5/3, ) .

0 5/3

-2

Fig. 5.20
Interval
< x < 2
2 < x < 5/3

x +2

3x 5

f (x)
+

Interval of inc / dec


Increasing on ( , 2]

decreasing on [ 2, 5/3]

5/3 < x <

increasing on [5/3, )

Note (i) : If the critical numbers are not included in the intervals, then the
intervals of increasing (decreasing) becomes strictly increasing (strictly
decreasing)
Note : (ii) The intervals of inc / dec can be obtained by taking and checking a
sample point in the sub-interval.
Example 5.39 : Prove that the function f(x) = x2 x + 1 is neither increasing nor
decreasing in [0,1]
Solution :

f (x) = x2 x + 1

f (x) = 2x 1
1
1
1
f (x) 0 for x 2 i.e., x 2 , 1 f(x) is increasing on 2 , 1

1
1
1
Also f (x) 0 for x 2 x 0, 2. Also f (x) is decreasing on 0, 2

Therefore in the entire interval [0,1] the function f(x) is neither increasing
nor decreasing.
Example 5.40 : Discuss monotonicity of the function
f(x) = sin x, x [0, 2]

39

3
f (x) = sin x and f (x) = cos x = 0 for x = 2 , 2 in [0,2] Now

3
f (x) 0 for 0 x 2 and 2 x 2. Therefore f(x) = sin x is increasing on
0, and 3, 2 i.e., sin x is increasing on 0, 3 , 2
2
2

2 2

Solution :

3
Also, f (x) 0 for 2 x 2 . Therefore f(x) = sin x is decreasing on
, 3
2 2
x 2
Example 5.41 : Determine for which values of x, the function y = x + 1 ,
x 1 is strictly increasing or strictly decreasing.
Solution :
dy (x + 1) 1 (x 2) 1
3
=
> 0 for all x 1.
2
dx =
(x + 1)
(x +1)2

x 2
y = x + 1 , x 1

y is strictly increasing on R {1}.


Example 5.42 : Determine for which values of x, the function
f(x) = 2x3 15x2 + 36x + 1 is increasing and for which it is decreasing. Also
determine the points where the tangents to the graph of the function are parallel
to the x axis.
Solution :

f (x) = 6x2 30x + 36 = 6(x 2) (x 3)

f (x) = 0 x = 2, 3. Therefore the points 2 and 3 divide the real line


into ( , 2), (2, 3) (3, ).
Interval

x 2

x3

f (x)

Intervals of inc / dec

<x<2

increasing on ( , 2]

2<x<3

decreasing on [2, 3]

+
+
+
3<x<
increasing on [3, )
The points where the tangent to the graph of the function are parallel to the
x axis are given by f (x)= 0, ie., when x = 2, 3 Now f(2) = 29 and f(3) = 28.
Therefore the required points are (2, 29) and (3, 28)

40

Example 5.43 :
Show that f(x) = tan1 (sin x + cos x), x > 0 is a strictly increasing function

in the interval 0, 4 .

Solution :

f(x) = tan1(sin x + cos x).


1
cos x sin x
f (x) =
(cos x sin x) = 2 + sin 2x > 0
2
1 + (sin x + cos x)

since cos xsin x > 0 in the interval 0, 4

and 2 + sin 2x > 0)

f(x) is strictly increasing function of x in the interval 0 , 4

EXERCISE 5.7
(1) Prove that ex is strictly increasing function on R.
(2) Prove that log x is strictly increasing function on (0, )
(3) Which of the following functions are increasing or decreasing on the
interval given ?
1 1
(i) x2 1 on [0,2]
(ii) 2x2 + 3x on 2 , 2

(iii) ex on [0,1]

(iv) x(x 1) (x + 1) on [2, 1]

(v) x sin x on 0, 4

(4) Prove that the following functions are not monotonic in the intervals
given.
(i) 2x2 + x 5 on [1,0]

(ii) x (x 1) (x + 1) on [0,2]

(iii) x sin x on [0,]

(iv) tan x + cot x on 0, 2

(5) Find the intervals on which f is increasing or decreasing.


(i) f(x) = 20 x x2

(ii)

f(x) = x3 3x + 1

(iii) f(x) = x3 + x + 1

(iv)

(v) f(x) = x + cos x in [0, ]

(vi)

f(x) = x 2sin x, [0, 2]


f(x) = sin4 x + cos4 x in [0, /2]

41

Inequalities :
Example 5.44 :
Prove that ex > 1 + x for all x > 0.
f (x) = ex 1 > 0 for x > 0

Let f(x) = ex x 1

Solution :

i.e., f is strictly increasing function. for x > 0, f(x) > f(0)


i.e., (ex x 1) > (e0 0 1) ; ex > x + 1
Example 5.45 :
Prove that the inequality (1 + x)n > 1+nx is true whenever x > 0 and n > 1.
Solution : Consider the difference f(x) = (1 + x)n (1 + nx)
Then f (x) =

n(1 + x)n1 n = n[(1 + x)n1 1]

Since x > 0 and n 1 > 0, we have (1 + x)n1 > 1, so f (x) > 0.


Therefore f is strictly increasing on [0, ).
For x > 0 f(x) > f(0) i.e., (1 + x)n (1 + nx) > (1 + 0) (1 + 0)
i.e., (1 + x)n (1 + nx) > 0

i.e., (1 + x)n > (1 + nx)

Example 5.46 : Prove that sin x < x < tan x, x0, 2

f (x) = 1 cos x > 0 for 0 < x < 2

x =/2
y=
x

Let f(x) = x sin x

y =tan x

Solution :

f is strictly increasing.
y =sin x

For x > 0, f(x) > f(0)


x sin x > 0 x > sin x

(1)

/2

Let g(x) = tan x x

g(x) = sec2x 1 = tan2x > 0 in 0, 2

Fig. 5.21

g is strictly increasing
For x > 0, f(x) > f(0) tan x x > 0 tan x > x (2)
From (1) and (2)

sin x < x < tan x

42

EXERCISE 5.8
(1) Prove the following inequalities :
x3
x2
(ii) sin x > x 6 , x > 0
(i) cos x > 1 2 , x > 0
(iii) tan1 x < x for all x > 0

(iv) log (1 + x) < x for all x > 0.

5.9 Maximum and Minimum values and their applications :

(slope)

Positiv
e

Gradie
nt

ve
Negati
nt
Gradie
)
e
p
lo
(s

Positiv
e Grad
ient

Some of the most important


applications of differential calculus are
optimization problems, in which we are
required to find the optimal (best) way of
doing something. In many cases these
problems can be reduced to finding the
maximum or minimum values of a
function. Many practical problems
require us to minimize a cost or maximize
an area or somehow find the best possible
outcome of a situation.

(slope
)

For since the fabric of the Universe is most perfect and the work of a
most wise creator, nothing at all takes place in the Universe in which some rule
of maximum or minimum does not appear
Leonard Euler

Fig. 5.22

Let us first explain exactly what we mean by maximum and minimum values.
In fig 5.22 the gradient (rate
of change) of the curve changes
from positive between O and P
to negative between P and Q and
positive again between Q and R.
At point P, the gradient is zero
and as x increases, the gradient
(slope) of the curve changes
from positive just before P to
negative just after. Such a point
is called a maximum point and
appears as the crest of a wave.

y
Maximum
Point

Maximum
Point
Point of
Inflexion

x
Minimum
Point

Fig. 5.23

43

At point Q, the gradient is


also zero and as x increases the
gradient of the curve changes
from negative just before Q to
positive just after. Such a point is
called a minimum point and
appears as the bottom of a
valley. Points such as P and Q
are given the general name,
turning points.

f (d)
f (a)
aO b

Fig. 5.24

It is possible to have a turning point, the gradient on either side of which is


the same. Such a point is given the special name of a point of inflection as
shown in Fig 5.23.

Definition : A function f has an absolute maximum at c if


f(c) f(x) for all x in D, where D is the domain of f. The number f(c) is called
maximum value of f on D. Similarly f has an absolute minimum at c if
f(c) f(x) for all x in D and the number f(c) is called the minimum value of f on
D. The maximum and minimum values of f are called extreme values of f.:
Fig.5.24 shows the graph of a function f with absolute maximum at d and
absolute minimum at a. Note that (d, f(d)) is the highest point on the graph and
(a, f(a)) is the lowest point.
In Fig. 5.24 if we consider only values of x near b, for instance, if we
restrict our attention to the interval (a,c) then f(b) is the largest of those values
of f(x) and is called a local maximum value of f. Likewise f(c) is called a local
minimum value of f because f(c) f(x) for x near c, in the interval (b,d). The
function f also has a local minimum at e. In general we have the following
definition.
Definition : A function f has a local maximum (or relative maximum) at c
if there is an open interval I containing c such that f(c) f(x) for all x in I.
Similarly, f has a local minimum at c if there is an open interval I containing c
such that f(c) f(x) for all x in I.
Illustrations : (1) The function f(x)=cos x takes on its (local and absolute)
maximum value of 1 infinitely many times since cos 2n = 1 for any integer and
1 cos x 1 for all x. Like wise cos (2n + 1) = 1 is its (local and absolute)
minimum value, n is any integer.

44

(2) If f(x) = x2, then f(x) f(0)


because x2 0 for all x. Therefore
f(0) = 0 is the absolute (and local)
minimum value of f. This
corresponds to the fact that the
origin is the lowest on the parabola
y = x2 See Fig.5.25 However, there
is no highest point on the parabola
and so this function has no
maximum value.

y = x2

x
O
Min. value =0; No Max.

Fig. 5.25
y
8
6 y =x3

(3) If f(x) = x3 then from the graph

of f(x) shown in Fig 5.26, we see

2
-4

that this function has neither an

-2

absolute maximum value nor an

-2

absolute minimum value. In fact it

-4

has no local extreme values either.

-6

No Minimum
No Maximum

-8

Fig. 5.26
y

(4) Consider the function


4

f(x) = 3x 16x + 18x ; 1 x 4.


The graph is shown in Fig. 5.27.
We can see that f(1) = 5 is a local
maximum, whereas the absolute maximum
is f(1)=37. Also f(0) = 0 is a local minimum
and f(3)= 27 is both local and absolute
minimum.
We have seen that some functions have
extreme values, while others do not. The
following theorem gives conditions under
which a function is guaranteed to possess
extreme values.

45

(-1,37)

40

(4,32)

30
20
10 (1,5)
-1 0
-10

1 2
(2,-8)

-20
-30

(3,-27)

Fig. 5.27

The Extreme value theorem : If f is continuous on a closed interval [a,b]


then f attains an absolute maximum value f(c) and an absolute minimum value
f(d) at some number c and d in [a,b]
The next two examples show that a function need not possess extreme
values if either of the hypotheses (continuity or closed interval) is omitted from
the extreme value theorem.
y

(5) Consider the function


x2

, 0x<1
0 , 1 x 2
The function is defined on the
closed interval [0,2] but has no
maximum value. Notice that the
range of f is the interval [0,1). The
function takes on value close to 1
but never attains the value 1.

f(x) =

Fig. 5.28

This is because the hypotheses of f to be continuous fails. Note that x = 1 is a


point of discontinuity, for,
Lim
x 1

Lim
Lim
f(x) = x 1 (x2) = 1 ; x 1 + f(x) = 0

(6) The function f(x) =x2, 0 < x < 2


is continuous on the interval (0,2)
but has neither a maximum nor a
minimum value. The range of f is
the interval (0,4). The values 0 and
4 are never taken on by f. This is
because the interval (0,2) is not
closed.

y
4
f (x) =x2, 0<x<2
No Maximum
No Minimum

Fig. 5.29
If we alter the function by including either end point of the interval (0,2)
then we get one of the situations shown in Fig. 5.30, Fig. 5.31, Fig. 5.32 In
particular the function f(x) = x2, 0 x 2 is continuous on the closed interval
[0,2]. So the extreme value theorem says that the function has an absolute
maximum and an absolute minimum.

46

f2 (x) =x2, 0x<2

f1 (x) =x2, 0<x2

Fig. 5.30

f3 (x) =x2, 0x2

No Maximum
Minimum f2 (0) =0

Maximum f1 (2) =4
No Minimum

Maximum f3 (2) =4
Minimum f3 (0) =0

Fig. 5.31

Fig. 5.32
Inspite of the above examples we point out that there are functions which
are neither continuous nor differentiable but still attains minimum and
maximum values. For instance, consider
1 , x is irrational
0 , x is rational
(This function is known as characteristic function on the set of irrational
numbers)
This function is nowhere differentiable and everywhere discontinuous. But
the maximum value is 1 and the minimum value is 0.
The extreme value theorem says that a continuous function on a closed
interval has a maximum value and minimum value, but it does not tell us how to
find their extreme values.
y
Fig. 5.33 shows the graph of a
(c, f (c))
function f with a local maximum at
c and a local minimum at d. It
appears that at the maximum and
(d, f (d))
minimum points the tangent line is
horizontal and therefore has slope
zero. We know that the derivative
x
is the slope of the tangent line, so it
O
c
d
Fig. 5.33
appear that f (c) = 0 and f (d) = 0.

f(x) =

The following theorem shows that this is always true for differentiable
functions.
Fermats Theorem : If f has a local extremum (maximum or minimum) at c
and if f (c) exists then f (c) = 0.
The following examples caution us that we cannot locate extreme values
simply by setting f (x) = 0 and solving for x.

47

(7) The function f(x) = | x | has its


(local and absolute) minimum value at
0, but that value cannot be found by
setting f (x) = 0 because f (x) does
not exist.

y =- x

y =x

y =|x|

Fig. 5.34
(8) The function f(x) = 3x 1, 0 x 1
has its maximum value when x = 1 but
f (1) = 3 0. This does not contradict
Fermats Theorem. Since f(1) = 2 is not a
local maximum.
Note that the number 1 is not
contained in an open interval in the
domain of f.

y
(1,2)

y =3x 1
0x1

1
-1

-1 (0,-1)

Fig. 5.35

Remark : The above examples demonstrate that even when f (c) = 0 there
need not be a maximum or minimum at c. Further more, there may be an
extreme value even when f (c) 0 or when f (c) does not exist.
y

(9) If f(x) = x3. Then f (x) = 3x2,


so f (0) = 0.
But f has no maximum or minimum
at 0 as you can see from its graph.
(observe that x3 > 0 for x > 0 and x3 < 0
for x < 0).
The fact that f (0) = 0 simply means
that the curve y = x3 has a horizontal
tangent at (0,0). Instead of having a
maximum or minimum at (0, 0) the curve
crosses its horizontal tangent there.

8
6

y =x3

4
2
-2 0
2
-2

-4
-6
-8

Fig. 5.36
Fermats theorem does suggest that we should atleast start looking for
extreme values of f at the numbers c where f (c) = 0 or f (c) does not exist.
Definition : A critical number of a function f is a number c in the domain of f
such that either f (c) = 0 or f (c) does not exist.

48

Stationary points are critical numbers c in the domain of f, for which f (c)= 0.
3

/
Example 5.47 : Find the critical numbers of x 5 (4 x)

Solution :

3
8
/
/
f(x) = 4 x 5 x 5

f (x) =
=

12 2/5
8 3/5
x

5
5 x
4 2/5
(3 2x)
5 x

3
Therefore f (x) = 0 if 3 2x = 0 i.e., if x = 2 . f (x) does not exist when
3
x = 0. Thus the critical numbers are 0 and 2 .
Note that if f has a local extremum at c, then c is a critical number of f, but
not vice versa.
To find the absolute maximum and absolute minimum values of a
continuous function f on a closed interval [a,b] :
(1) Find the values of f at the critical numbers, of f in (a,b).
(2) Find the values of f(a) and f(b)
(3) The largest of the values from steps 1 and 2 is the absolute maximum
value, the smallest of these values is the absolute minimum value.
Example 5.48 : Find the absolute maximum and minimum values of the
1

function. f(x) = x3 3x2 + 1 , 2 x 4


1

Solution : Note that f is continuous on ; 2 , 4


f(x) = x3 3x2 + 1

f (x) = 3x2 6x = 3x (x 2)
-1/2 0

Fig. 5.37
Since f (x) exists for all x, the only critical numbers of f are x = 0, x = 2.
1
Both of these critical numbers lie in the interval 2 , 4 . Value of f at

these critical numbers are f(0) = 1 and f(2) = 3.

49

The values of f at the end points of the interval are


1
1
1 3
1 2
f 2 = 2
+1=8
3 2

( ) ( )

( )

and f(4) = 43 3 42 + 1 = 17
Comparing these four numbers, we see that the absolute maximum value is
f(4) = 17 and the absolute minimum value is f(2) = 3.
Note that in this example the absolute maximum occurs at an end point, whereas
the absolute minimum occurs at a critical number.
Example 5.48(a): Find the absolute maximum and absolute minimum values of
f(x) =x 2sin x, 0 x 2.
f(x) =x 2 sin x, is continuous in [0, 2]
Solution :
f (x) = 1 2 cos x
1
5

f (x) = 0 cos x = 2 x = 3 or 3
The value of f at these critical points are

f 3 = 3 2 sin 3 = 3 3

5
5
5
f 3 = 3 2 sin 3

5
= 3 + 3
6.968039
The values of f at the end points are f(0) = 0 and f(2) = 2 6.28

Comparing these four numbers, the absolute minimum is f 3 = 3 3 and



5
5
the absolute maximum is f 3 = 3 + 3 . In this example both absolute

minimum and absolute maximum occurs at the critical numbers.
Let us now see how the second derivatives of functions help determining
the turning nature (of graphs of functions) and in optimization problems.
The second derivative test : Suppose f is continuous on an open interval
that contains c.
(a) If f (c) = 0 and f (c) > 0, then f has a local minimum at c.
(b) If f (c) = 0 and f (c) < 0, then f has a local maximum at c.

50

Example 5.49 : Discuss the curve y = x4 4x3 with respect to local extrema.
Solution :

f(x) = x4 4x3
f (x) = 4x3 12x2 , f (x) = 12x2 24x

To find the critical numbers we set f (x) = 0 and obtain x = 0 and x = 3. To


use the second derivative test we evaluate the sign of f at these critical
numbers.
f (0) = 0, f (3) = 36 > 0. Since f (3) = 0 and f (3) > 0, f(3) = 27 is a local
minimum value and the point (3, 27) is a minimum point. Since f (0) = 0 the
second derivative test gives no information about the critical number 0. But
since f (x) < 0 for x < 0 and also for 0 < x < 3, the first derivative test tells us
that f does not have a local extremum at 0.
We summarise the above discussion as follows :
Procedure for finding and distinguishing stationary points.
dy
(i) Given y = f(x) determine dx (i.e., f (x))
dy
(ii) Let dx = 0 and solve for the critical numbers x.
(iii) Substitute the values of x into the original function y = f(x) to find the
corresponding y-coordinate values. This establish the co-ordinates of
the stationary points. To determine the nature of the stationary points,
d2y
and substitute into it the values of x found in (ii).
dx2
If the result is :
(a) positive the point is a minimum one
(b) negative the point is a maximum one
(c) zero the point cannot be an extremum (minimum or maximum)
OR
(iv) Find

(v) Determine the sign of the gradient (slope f (x) of the curve just before
and just after the stationary points. If the sign change for the gradient
of the curve is
(a) positive to negative this point is a maximum one
(b) negative to positive this point is a minimum one
Example 5.50 : Locate the extreme point on the curve y = 3x2 6x and
determine its nature by examining the sign of the gradient on either side.

51

Solution : Following the above procedure


dy
(i) Since y = 3x2 6x, dx = 6x 6
dy
(ii) At a stationary point, dx = 0, hence x = 1
(iii) When x = 1, y = 3(1)2 6(1) = 3. Hence the coordinates of the
stationary point is (1, 3).
dy
If x is slightly less than 1, say 0.9, then dx = 6(0.9) 6 = 0.6 < 0.
dy
If x is slightly greater than 1, say 1.1 then dx = 6(1.1) 6 = 0.6 > 0.
Since the gradient (slope of the curve) changes its sign from negative to
positive (1, 3) is a minimum point.
Example 5.51 :
Find the local minimum and maximum values of f(x) = x4 3x3 + 3x2 x
f(x) = x4 3x3 + 3x2 x

Solution :

f (x) = 4x3 9x2 + 6x 1


At a turning point, f (x) = 0 gives 4x3 9x2 + 6x 1 = 0
1
(x 1)2 (4x 1) = 0 x = 1, 1, 4
1 1 27
When x = 1, f(1) = 0 and when x = 4, f 4 = 256

1 27
Hence the coordinates of the stationary points are (1, 0) and 4 , 256

f (x) = 12x 18x + 6 = 6(2x 3x + 1) = 6(x 1) (2x 1)


2

When x = 1, f (1) = 0. Thus the second derivative test gives no


information about the extremum nature of f at x = 1.
1
1 9
1 27
When x = 4 , f 4 = 4 > 0, hence 4 , 256 is a minimum point.

Caution :
No function will attain local maximum / minimum at the end points of its
domain.

52

EXERCISE 5.9
(1) Find the critical numbers and stationary points of each of the following
functions.
(i) f(x) = 2x 3x2

(ii) f(x)

= x3 3x + 1

(iii) f(x) = x4/5 (x 4)2

(iv) f(x)

x+1
x +x+1
2

(v) f() = sin2 2 in [0, ]


(vi) f() = + sin in [0, 2]
(2) Find the absolute maximum and absolute minimum values of f on the
given interval :
(i)

f(x) = x2 2x + 2,

[0,3]

(ii)

f(x) = 1 2x x2,

[4,1]

(iii) f(x) = x3 12x + 1,


(iv) f(x) =
(v)

9 x2 ,

x
f(x) = x + 1,

[1,2]
[1,2]

(vi) f(x) = sin x + cos x,


(vii) f(x) =

[3,5]

0,
3

x 2 cos x,

[, ]

(3) Find the local maximum and minimum values of the following :
(i) x3 x

(ii) 2x3 + 5x2 4x

(iii) x4 6x2

(iv) (x2 1)3

(v) sin2 ,

(vi) t + cos t

[0, ]

5.10 Practical problems involving maximum and minimum values :


The methods we have learnt in this section for finding extreme values have
practical applications in many areas of life. A business person wants to
minimise costs and maximise profits. We also solve such problems as
maximising areas, volumes and profits and minimising distances, times and
costs. In solving such practical problems, the greatest challenge is often to
convert the word problem into maximum minimum problem by setting up the
function that is to be maximised or minimised.

53

As a problem solving technique we suggest the following principles.


(1) Understand the problem : The first step is to read the problem
carefully until it is clearly understood. Ask yourself what is the
unknown? What are the given quantities? What are the given conditions?
(2) Draw diagram : In most problems it is useful to draw a diagram and
identify the given and required quantities on the diagram.
(3) Introduce notation : Assign a symbol to the quantity to be maximised or
minimised, say Q. Also select symbols (a,b,c ,x, y, z) for the other
unknown quantities and lable the diagram with these symbols.
(4) Express Q in terms of some other symbols from step 3.
(5) If Q has been expressed as a function of more than one variable in
step 4, use the given information to find relationship (in the form of
equation) among these variables. Then use these equations to
eliminate all but one of these variables in the expression for Q.Thus Q
will be given as a function of one variable x, say, Q = f(x). Write the
domain of this function.
(6) Use the methods discussed to find the absolute maximum or minimum
value of f.
Remarks :
(1) If the domain is a closed interval then we apply the absolute max/min
property to maximize / minimize the given function (see 5.52, 5.58).
(2) If the domain is an open interval then we apply either first derivative
test (5.53) or second test for finding local max / min. Instead of first
derivative one can also apply second derivative test if the second test
exist. Similarly instead of second derivative test one can also apply
first derivative test.
(3) All these cases ultimately lead us to the absolute max / min only.
Example 5.52 : A farmer has 2400 feet of fencing and want to fence of a
rectangular field that borders a straight river. He needs no fence along the river.
What are the dimensions of the field that has the largest area ?
y
Solution :
We wish to maximize the area A
x
x
of the rectangle. Let x and y be the
width and length of the rectangle (in
feet). Then we express A in terms of
x and y as A = xy
Fig. 5.38

54

We want to express A as a function of just one variable, so we eliminate y


by expressing it in terms of x. To do this we use the given information that the
total length of the fencing is 2400 ft. Therefore 2x + y = 2400
Hence

y = 2400 2x and the area is A= x (2400 2x) = 2400 x 2x2

Note that x 0 and x 1200 (otherwise A < 0). So the function that we
wish to maximize is
A (x) = 2400 x 2x2, 0 x 1200.
A(x) = 2400 4x, so to find the critical numbers we solve the equation
2400 4x = 0 which gives x = 600. The maximum of A must occur either at this
critical number or at an end point of the interval.
Since A(0) = 0, A(600) = 7,20,000 and A(1200) = 0, thus the maximum
value is A (600) = 720,000. When x = 600, y = 2400 1200 = 1200
Hence the rectangular field should be 600 ft wide and 1200 ft long.
Note : This problem also be done by using second derivative test (local). In this
case x > 0 and y > 0.
Example 5.53 :
to the point (1,4)
Find a point on the parabola y2 = 2x that is closest
y
Solution : Let (x,y) be the point
on the parabola y2 = 2x. The
distance between the points (1,4)
and
(x,y) is d =

(1,4)
y2 = 2x
(x,y)

(x 1)2 + (y 4)2 .
0

y2
(x,y) lies on y = 2x x = 2 ,
y2
so d2= f(y) = ( 2 1)2 + (y 4)2
2

Fig. 5.39

(Note that the minimum of d occurs at the same point as the minimum of d2)

y2
Now f (y) = 2 2 1 (y) + 2 (y 4)

= y3 8 = 0 at a critical point.
y3 8 = 0 y = 2 (since y2 + 2y + 4 = 0 is not possible)

55

Observe that f (y) < 0 when y < 2 and f (y) > 0 when y > 2, so by the
first derivate test, for absolute extrema, the absolute minimum occurs when
y2
y = 2. The corresponding value of x is x = 2 = 2. Thus the point on y2 = 2x
closest to (1,4) is (2,2).
Note : This problem also be done by using second derivative test
Example 5.54 :
Find the area of the largest rectangle that can be inscribed in a semi circle
of radius r.
Solution :
x2 +y2 =r2
Let be the angle made by OP
P(r cos , r sin )
with the positive direction of xaxis.
r
Then the area of the rectangle A is

x
A() = (2 r cos) (r sin)
Fig. 5.40
= r2 2 sin cos = r2 sin 2
Now A() is maximum when sin 2 is maximum. The maximum value of

sin 2 = 1 2 = 2 or = 4 . (Note that A () = 0 when = 4 )

Therefore the critical number is 4. The area A4 = r2.



Note : The dimensions of the largest rectangle that can be inscribed in a
r
semicircle are 2r ,
2

A () = 2r2 cos 2 = 0 2 = 2 ; = 4
Aliter :

A () = 4r2 sin 2 < 0, for = 4 = 4 gives the

maximum point and the maximum point is 4 , r2

From the above problem, we understand that the method of calculus gives
the solution faster than the algebraic method.
Example 5.55 : The top and bottom margins of a poster are each 6 cms and the
side margins are each 4 cms. If the area of the printed material on the poster is
fixed at 384 cms2, find the dimension of the poster with the smallest area.
Solution : Let x and y be the length and breadth of printed area, then the area
xy = 384

56

Dimensions of the poster area are


(x + 8) and (y + 12) respectively.
Poster area
A = (x + 8) (y + 12)
= xy +12x + 8y + 96
= 12x + 8y + 480
384
= 12x + 8 x + 480

1
A = 12 8 384 2
x

x +8
6 cms
4

x
y

y +12

6 cms

Fig. 5.41

1
A = 16 384 3
x
A = 0 x = 16
But x > 0
x = 16
when x = 16, A > 0
when x = 16, the area is minimum
y = 24
x + 8 = 24, y + 12 = 36
Hence the dimensions are 24cm and 36 cm.
Example 5.56 : Show that the volume of the largest right circular cone that can
8
be inscribed in a sphere of radius a is 27 (volume of the sphere).
Solution : Given that a is the radius of
the sphere and let x be the base radius of
a
the cone. If h is the height of the cone,
c
then its volume is

y
1
2
V = 3 x h
O x
1
= 3 x2 (a + y)
(1)
Fig. 5.42
where OC = y so that height h = a + y.
From the diagram x2 + y2 = a2

(2)

Using (2) in (1) we have


1
V = 3 (a2 y2) (a + y)

57

For the volume to be maximum :


1
V =0 3 [a2 2ay 3y2] = 0
3y = +a or y = a
a
y = 3 and y = a is not possible
2
a
Now V = 3 (a + 3y) < 0 at y = 3
a
the volume is maximum when y = 3 and the maximum volume is
1
8a2
1
8 4 3
8

3
9 (a + 3 a) = 27 (3 a ) = 27 (volume of the sphere)
Example 5.57 : A closed (cuboid) box with a square base is to have a volume
of 2000 c.c. The material for the top and bottom of the box is to cost Rs. 3 per
square cm. and the material for the sides is to cost Rs. 1.50 per square cm. If the
cost of the materials is to be the least, find the dimensions of the box.
Solution : Let x, y respectively denote the length of the side of the square base
and depth of the box. Let C be the cost of the material
Area of the bottom = x2
Area of the top = x2
Combined area of the top and bottom = 2x2
Area of the four sides = 4xy
Cost of the material for the top and bottom = 3(2x2)
Cost of the material for the sides = (1.5) (4xy) = 6xy
Total cost C = 6x2 + 6xy

(1)
2

Volume of the box V = (area) (depth) = x y=2000 (2)


12000
Eliminating y from (1) & (2) we get C(x) = 6x2 + x
(3)
where x > 0, ie., x (0,+ ) and C(x) is continuous on (0, + ).
12000
C (x) = 12x
x2
C (x) = 0 12x3 12000 = 0 12(x3 103) = 0
x = 10 or x2 + 10x + 100 = 0

58

x2 + 10x + 100 = 0 is not possible


The critical numbers is x = 10.
24000
24000
; C (10) = 12 + 1000 = 36 > 0
Now C (x) = 12 +
3
x

C is minimum at (10,C(10)) = (10, 1800) the base length is 10cm and


2000
depth is y = 100 = 20 cm.
Example 5.58 :

8-x

A man is at a point P on a bank of a straight river, 3 km wide, and wants to


reach point Q, 8 km downstream on the opposite bank, as quickly as possible.
He could row his boat directly across the river to point R and then run to Q, or
he could row directly to Q, or he could row to some point S between Q and R
and then run to Q. If he can row at 6 km/h and run at 8 km/h where should he
land to reach Q as soon as possible ?
Solution :
3km
P
R
Let x be the distance from R to S. Then the
(
x2
+9 x
running distance is 8 x and the distance
)
distance
PS = x2 + 9 . We know that time = rate .
S
Then the rowing time
Rt =

x2 + 9
6

and the running time rt =

(8 x)
8

Fig. 5.43
Therefore the total time T = Rt + rt =

(8 x)
x2 + 9
+ 8 , 0 x 8.
6

Notice that if x = 0, he rows to R and if x = 8 he rows directly to Q.


x
1
T (x) = 0 T (x) =
8 = 0 for critical points.
2
6 x +9
4x = 3

x2 + 9

16x2 = 9 (x2 + 9)
7x2 = 81

59

9
9
since x =
is not admissible.
7
7
9
The only critical number is x =
. We calculate T at the end point of the
7
9
.
domain 0 and 8 and at x =
7
x=

T(0) = 1.5, T

9
7
73
= 1 + 8 1.33, and T(8) = 6 1.42
7

Since the smallest of these values of T occurs when x =


should land the boat at a point

9
, the man
7

9
km ( 3.4 km) down stream from his starting
7

point.
(1)
(2)
(3)
(4)
(5)
(6)

EXERCISE 5.10
Find two numbers whose sum is 100 and whose product is a maximum.
Find two positive numbers whose product is 100 and whose sum is
minimum.
Show that of all the rectangles with a given area the one with smallest
perimeter is a square.
Show that of all the rectangles with a given perimeter the one with the
greatest area is a square.
Find the dimensions of the rectangle of largest area that can be
inscribed in a circle of radius r.
5
Resistance to motion, F, of a moving vehicle is given by, F = x + 100x.
Determine the minimum value of resistance.

5.11 Concavity (convexity) and points of inflection :


Figure 5.44 (a), (b) shows the graphs of two increasing functions on [a, b].
Both graphs join point A to point B but they look different because they bend in
different directions. How can we distinguish between these two types of
behaviour? In fig. 5.44 (c), (d) tangents to these curves have been drawn at
several points. In (a) the curve lies above the tangents and f is called concave
upward (convex downward) on [a, b]. In (b) the curve lies below the tangents
and g is called concave downward (convex upward) on [a, b]

60

A
0

Fig. 5.44 (a)

Fig. 5.44 (b)

Fig. 5.44(c)

Fig. 5.44 (d)

Definition :
If the graph of f lies above all of its tangents on an interval I, then it is
called concave upward (convex downward) on I. If the graph of f lies below all
of its tangents on I, it is called concave downward (convex upward) on I.
Let us see how the second derivative helps to determine the intervals of
concavity (convexity). Looking at Fig.5.44(c), you can see that, going from left
to right, the slope of the tangent increases. This means that the derivative f (x)
is an increasing function and therefore its derivative f (x) is positive. Likewise
in Fig.5.44 (d) the slope of the tangent decreases from left to right, so f (x)
decreases and therefore f (x) is negative. This reasoning can be reversed and
suggests that the following theorem is true.
The test for concavity (convexity) :
Suppose f is twice differentiable on an interval I.
(i) If f (x) > 0 for all x in I, then the graph of f is concave upward
(convex downward) on I.
(ii) If f(x) < 0 for all x in I, then the graph of f is concave downward
(convex upward) on I.

61

Definition : A point P on a curve is called a point of inflection if the curve


changes from concave upward (convex downward) to concave downward
(convex upward) or from concave downward (convex upward) to concave
upward (convex downward) at P.
That is the point that separates the convex part of a continuous curve from
the concave part is called the point of inflection of the curve.
It is obvious that at the point of inflection the tangent line, if it exists, cuts
the curve, because on one side the curve lies under the tangent and on the other
side, above it. The following theorem says under what situation a critical point
of f becomes a point of inflection.
Theorem :
Let a curve be defined by an equation y = f(x). If f (x0) = 0 or
f (x0) does not exist and if the derivative f (x) changes sign when passing
through x = x0, then the point of the curve with abcissa x = x0 is the point of
inflection. Equivalently the point (x0, f(x0)) is a point of inflection of the graph
of f if there exists a neighbourhood (a, b) of x0 such that
f (x) > 0 for every x in (a, x0) and f (x) < 0 for every x in (x0, b) or vice versa.
y of x0, f (a) andyf (b) differ in sign.y
That yis in the neighbourhood
A

B
x

Fig. 5.45
Remark :
We caution the reader that points of inflections need not be critical points
and critical points need not be points of inflections. However x = x0 is a critical
point such that f (x) does not change its sign as f(x) passes through x0, then
x0 is a point of inflection and for points of inflections x0, it is necessary that
f (x0) = 0. If f (x) does not change its sign even if f (x0) = 0 then x0 cannot be
a point of inflection. Thus the conjoint of the above discussion is that for points
of inflections x0, f (x0) = 0 and in the immediate neighbourhood (a, b) of x0, f

(a) and f (b) must differ in sign.

62

If x = x0 is a root of odd order simple, triple, etc. of the function


f (x) = 0, then x = x0 yields a maximum or minimum. If x = x0 is a root of even
order, x = x0 yield a point of inflection with a horizontal tangent. These
concepts are made clear in the following illustrative example y = x3.
y = 3x2 and y = 6x.
Here y(0) = 0 and y(0) = 0 and x = 0 happens to be a critical point of both
y and y. Clearly y (x) > 0 for x < 0 and x > 0. Thus y does not change its sign
as f(x) passes through x = 0.
That is y ( 0.1) > 0 and y(0.1) > 0 i.e., in the neighbourhood ( 0.1, 0.1)
of 0, y does not change its sign. Thus the first derivative test confirms that
(0, 0) is a point of inflection.
y
Again y(0) = 0, y( 0.1) < 0
y =x3
and y(0.1) > 0. Here y changes its
Concave
sign as y(x) passes through x = 0. In
upward
Convex
this case the second derivative
downward
(concavity) test also confirms that (0,
0
0) is a point of inflection. Note that (0,
x
Convex
3
0) separates the convex part of y = x
upward
from the concave part.
Note also that y(x) = 3x2 and
x = 0 is a double root of y(x) = 0. The
root order test also confirms that (0, 0)
is a point of inflection with x-axis as
the horizontal tangent at (0, 0)
Example 5.59 :
Determine the domain of concavity
y = 2 x2.

Concave
downward

Fig. 5.46
(convexity)

of

the

y = 2 x2
y = 2x and y = 2 < 0 for x R
Here the curve is everywhere concave downwards (convex upwards).
Example 5.60 :
Determine the domain of convexity of the function y = ex.
y = ex ; y = ex > 0 for x
Solution :
Hence the curve is everywhere convex downward.

Solution :

63

curve

Example 5.61 : Test the curve y = x4


for points of inflection.
Solution :

y=x

y
16

y =x4

12

y = 12x2 = 0 for x = 0

and y > 0 for x < 0 and x > 0


8
Therefore the curve is concave
4
upward and y does not change sign
0
as y(x) passes through x = 0. Thus the
-2 -1
1
2
curve does not admit any point of
Fig. 5.47
inflection.
Note : The curve is concave upward in ( , 0) and (0, ).

Example 5.62 : Determine where the curve y = x3 3x + 1 is cancave upward,


and where it is concave downward. Also find the inflection points.
Solution :
f(x) = x3 3x + 1

x =0

f (x) = 3x 3 = 3(x 1)
2

Fig. 5.48

Now f (x) = 6x
Thus f (x) > 0 when x > 0 and f (x) < 0 when x < 0.
The test for concavity then tells us that the curve is concave downward on
( , 0) and concave upward on (0, ). Since the curve changes from concave
downward to concave upward when x = 0, the point (0, f(0)) i.e., (0, 1) is a
point of inflection. Note that f (0) = 0
Example 5.63 :
Discuss the curve y = x4 4x3 with respect to concavity and points of
inflection.
Solution :
f(x) = x4 4x3 f (x) = 4x3 12x2
f (x) = 12x2 24x = 12x (x 2)
Since f (x) = 0 when x = 0
or 2, we divide the real line into
three intervals.

Fig. 5.49
( , 0), (0, 2), (2, ) and complete the following chart.

64

f (x) = 12x (x 2)
+

Inerval
( , 0)
(0, 2)
(2, )

concavity
upward
downward
upward

The point (0, f(0)) i.e., (0, 0) is an inflection point since the curve changes
from concave upward to concave downward there. Also (2, f(2)) i.e., (2, 16) is
an inflection point since the curve changes from concave downward to concave
upward there.
Note : The intervals of concavity can be obtained by taking and checking a
sample point in the sub-interval.
Example 5.64 : Find the points of inflection and determine the intervals of
convexity and concavity of the Gaussion curve y = ex
2

Solution : y = 2xex ; y = 2ex (2x2 1)


(The first and second derivatives exist everywhere). Find the values of x
for which y = 0
2

2ex (2x2 1) = 0
1
1
x=
, or x =
2
2

-1/2

1/2

Fig. 5.50
1
1
we have y > 0 and when x >
we have y < 0
when x <
2
2
The second derivative changes sign from positive to negative when passing
1
1
. Hence, for x =
, there is a point of inflection
through the point x =
2
2

on the curve; its co-ordinates are

1

, e 2
2

1
1
we have y < 0 and when x >
we have y > 0 . Thus
2
2
1
; its co-ordinates are
there is also a point of inflection on the curve for x =
2
When x <

1
1

, e 2. (Incidentally, the existence of the second point of inflection follows

directly from the symmetry of the curve about the y-axis). Also from the signs
of the second derivatives, it follows that

65

for < x <


for
for

1
the curve is concave upward ;
2

1
1
<x<
the curve is convex upward ;
2
2

1
< x < the curve is concave upward.
2

Example 5.65 :
Determine
y = x3 3x + 2

the

points

of

inflection

if

any,

of

the

function

y = x3 3x + 2
dy
2
dx = 3x 3 = 3(x + 1) (x 1)

Solution :

d2y
= 6x = 0 x = 0
dx2
Now

d2y
( 0.1) = 6( 0.1) < 0 and
dx2

d2y
(0.1) = 6(0.1) > 0. In the neighbourhood ( 0.1, 0.1)
dx2
of 0, y ( 0.1) and y(0.1) are of opposite signs. Therefore (0, y (0)) i.e.,
(0, 2) is a point of inflection.
Note : Note that x = 0 is not a critical point since y (0) = 3 0.
Example 5.66 :
Test for points of inflection of the curve y = sinx, x (0, 2)
Solution :

y = cosx
y = sinx = 0 x = n, n = 0, 1, 2, ...

since x (0, 2), x = corresponding to n = 1.


Now y (.9) = sin (.9) < 0 and
y(1.1) = sin (1.1 ) > 0 since sin (1.1) is negative
The second derivative test confirms that (, f()) = (, 0) is a point of
inflection.

66

Note : Note that x = is not a stationary point since y() = cos = 1 0.


In fact y = sin x admits countable number of points of inflections in the range
( , ), each of which is given by (n, 0), n = 0, 1, 2, and in none of the
cases, y(n) = ( 1)n vanishes. This shows that points of inflections need not be
stationary points.
EXERCISE 5.11
Find the intervals of concavity and the points of inflection of the following
functions :
(1)

f(x) = (x 1)1/3

(2)

f(x) = x2 x

(3)

f(x) = 2x3 + 5x2 4x

(4)

f(x) = x4 6x2

(5)

f() = sin 2 in (0, )

(6)

= 12x2 2x3 x4

67

Testing a differentiable function for maximum and minimum with a first


derivative
This gives us the following diagram of possible cases.
Signs of derivative f (x) when passing through Character of critical
point
critical point x0
x = x0
x > x0
x < x0
+

f (x0) = 0 or is

Maximum point

discontinuous
f (x0) = 0 or is

Minimum point

discontinuous
f (x0) = 0 or is

Neither maximum nor


minimum
(function
increases). But is a point
of inflection.
Neither maximum nor
minimum
(function
decreases) But is a point
of inflection.

discontinuous

f (x0) = 0 or is

discontinuous

Second derivative test


This gives us the following diagram of possible cases.
Signs of derivative f (x) at the critical point of f(x) or f (x)

Character of
the point

x = x0
f (x0)

f (x0)

x < x0

Maximum
point
Minimum
point

f (x0)

Critical point
of f
Critical point
of f
x > x0

Point of
Inflection
Point of
inflection
Unknown
Unknown

0 or 0

0 or 0

0 or 0
0 or 0

0
0

68

6. DIFFERENTIAL CALCULUS
APPLICATIONS-II
6.1 Differentials : Errors and Approximation
dy
dx to denote the derivative of
y with respect to x but we have regarded it as a single entity and not as a ratio.
In this section we give the quantities dy and dx separate meanings in such a way
that their ratio is equal to the derivative. We also see that these quantities, called
differentials, are useful in finding the approximate values of functions.
We have used the Liebnitz notation

Definition 1 : Let y = f(x) be a differentiable function. Then the quantities


dx and dy are called differentials. The differential dx is an independent variable
that is dx can be given any real number as the value. The differential dy is then
defined in terms of dx by the relation
dy = f (x) dx (dx x)
Note :
(1) The differentials dx and dy are both variables, but dx is an independent
variable, where as dy is a dependent variable it depends on the
values of x and dx. If dx is given a specific value and x is taken to be
some specific number in the domain of f, then the numerical value of
dy is determined.
(2) If dx 0 we can divide both sides of dy = f (x) dx by dx to obtain
dy
dy
dx = f (x). Thus dx now is the ratio of differentials.
Example 6.1 : If y = x3 + 2x2 (i)find dy
(ii) find the value of dy when x = 2 and dx = 0.1
Solution :
(i) If f(x) = x3 + 2x2, then f (x) = 3x2 + 4x, so dy = (3x2 + 4x) dx
(ii) Substituting x = 2 and dx = 0.1, we get dy = (3 22 + 4 2)0.1 = 2.

69

6.1.1 Geometric meaning of differentials :


Let P(x,f(x)) and Q(x + x, f(x +x)) be
points on the graph of f and set
dx = x. The corresponding change in y is
y = f(x + x) f(x)
The slope of the tangent line PR is the
derivate f (x). Thus the directed distance
from S to R is f (x) dx = dy.

R
Q

y =f (x) P

dy

dx = x
0

x + x

Fig. 6.1
Therefore dy represents the amount that the tangent line rises or falls
whereas y represents the amount that the curve y = f(x) rises or falls when x
changes by an amount dx.
dy
y
y dy
lim
Since dx = x 0
, we have

.(1) when x is small.


x
x dx
Geometrially, this says that the slope of the secant line PQ is very close to
the slope of the tangent line at P when x is small. If we take dx = x, then (1)
becomes y dy .(2) which says that if x is small, then the actual change
in y is approximately equal to the differential dy. Again this is geometrically
evident in the case illustrated by Fig. 6.1. The actual change in y is referred as
absolute error.
The actual error in y is y dy.
y Actual change in y
The quantity y = Actual value of y is called relative error and the
y
quantity y 100 is called percentage error.

The approximation given by (2) can be used in computing approximate
values of functions. Suppose that f(a) is a known number and an approximate
value is calculated for f(a + x) where dx is small, since f(a + x) = f(a) + y,
(2) gives, f(a + x) f(a) + dy.(3)
Example 6.2 : Compute the values of y and dy if y = f(x) = x3 + x2 2x + 1
where x changes (i) from 2 to 2.05 and (ii) from 2 to 2.01
Solution :
(i) We have f(2) = 23 + 22 2(2) + 1 = 9
f(2.05) = (2.05)3 + (2.05)2 2(2.05) + 1 = 9.717625.
and y = f(2.05) f(2) = 0.717625.
In general
dy = f (x) dx = (3x2 + 2x 2)dx
When x = 2, dx = x = 0.05 and dy = [(3(2)2+2(2)2] 0.05 = 0.7

70

(ii)

f(2.01) = (2.01)3 (2.01)2 2(2.01) + 1 = 9.140701


y = f(2.01) f(2) = 0.140701

When dx = x = 0.01, dy = [3(2)2 + 2(2) 2]0.01 = 0.14


Remark : The approximation y dy becomes better as x becomes smaller
in Example 6.2. Also dy was easier than to compute y. For more complicated
functions it may be impossible to compute y exactly. In such cases the
approximation by differentials is especially useful.
Example 6.3 : Use differentials to find an approximate value for
Solution : Let y = f(x) =

1
3

1
x = x. Then dy = 3 x.

2
3

65.

dx

Since f(64) = 4. We take x = 64 and dx = x = 1


2

1
1
1
This gives dy = 3 (64) 3 (1) = 3(16) = 48

1
65 = f(64 + 1) f(64) + dy = 4 + 48 4.021

3
Note : The actual value of 65 is 4.0207257... Thus the approximation by
differentials is accurate to three decimal places even when x = 1.
Example 6.4 : The radius of a sphere was measured and found to be 21 cm with
a possible error in measurement of atmost 0.05 cm. What is the maximum error
in using this value of the radius to compute the volume of the sphere ?
4

Solution : If the radius of the sphere is r, then its volume is V = 3 r3. If the
error in the measured value of r is denoted by dr = r, then, the corresponding
error in the calculated value of V is V. which can be approximated by the
differential dV = 4r2 dr.
When r = 21 and dr=0.05, this becomes dV = 4(21)2 0.05 277.
The maximum error in the calculated volume is about 277 cm3.
Note : Although the possible error in the above example may appear to be
rather large, a better picture of the error is given by the relative error, which is
computed by dividing the error by the total volume.
V dV
277
V V 38,808 0.00714

71

dr

0.05

Thus a relative error of r = 21 0.0024 in the radius produces a


relative error of about 0.007 in the volume. The errors could also be expressed
as percentage errors of 0.24% in the radius and 0.7% in the volume.
Example 6.5 : The time of swing T of a pendulum is given by T = k l where k
is a constant. Determine the percentage error in the time of swing if the length
of the pendulum l changes from 32.1 cm to32.0 cm.
1

Solution :

If T = k l

= k l2

1
dT
k
Then dl = k 2 l 2 =
2 l and dl = 32.0 32.1 = 0.1 cm

Error in T = Approximate change in T.


dT
k
T dT = dl dl =
(0.1)

2 l
k
(0.1)
2 l
T

Percentage error = T 100 % =


100 %

k l
0.1
0.1
= 2l 100 % = 2(32.1) 100%

= 0.156%
Hence the percentage error in the time of swing is a decrease of 0.156%.
Aliter :
T = k l
1
Taking log on both sides,
log T = log k + 2 log l
1 1
1
Taking differential on both sides, T dT = 0 + 2 l dl
T
1
1 1
i.e, T T dT = 0 + 2 l dl
T
1 dl
T 100 = 2 l 100
1 (0.1)
= 2 32.1 100
= 0.156%
ie., the percentage error in the time of swing is a decrease of 0.156.
1

Caution : Differentiation is carried out with the common understanding that the
function involved admit logarithmic differentiation.

72

Example 6.6 : A circular template has a radius of 10 cm ( 0.02). Determine the


possible error in calculating the area of the templates. Find also the percentage
error.
dA
Solution : Area of circular template A = r2, hence dr = 2r, Approximate
change in area A (2r)dr. When r = 10 cm and dr = 0.02
A = (2 10) (0.02) 0.4 cm2 i.e, the possible error in calculating the
template area is approximately 1.257 cm2
0.4
Percentage error
100 = 0.4%
(10)2
Example 6.7 : Show that the percentage error in the nth root of a number is
1
approximately n times the percentage error in the number .
1
n

Solution : Let x be the number. Let y = f(x) = (x)


1
Then log y = n log x
1 1
1
Taking differential on both sides, we have y dy = n x dx
1
1
1
y
i.e., y y dy = n . x dx
y
1 dx
y 100 n x 100

1
= n times the percentage error in the number.
Example 6.8 : Find the approximate change in the volume V of a cube of side x
meters caused by increasing the side by 1%
Solution : The volume of the cube of side x is,
V = x3 ; dV = 3x2 dx
When dx = 0.01x,

dV = 3x2 (0.01x) = 0.03 x3 m3.

EXERCISE 6.1
(1) Find the differential of the functions
4

(i) y = x5

(ii) y =

x2
(iv) y = 2x + 3

(v) y = sin 2x

73

(iii) y =

x4 + x2 + 1

(vi) y = x tan x

(2) Find the differential dy and evaluate dy for the given values of x and dx.
(i)

1
y = 1 x2 , x = 5, dx = 2

(ii)

y = x4 3x3+ x 1, x = 2, dx = 0.1.

(iii)

y = (x2 + 5)3, x = 1, dx = 0.05

(iv)

y =

(v)

y = cos x, x = 6 dx = 0.05

1 x , x = 0, dx = 0.02

(3) Use differentials to find an approximate value for the given number
(i)

1
(ii) 10.1

36.1

(iii) y =

1.02 +

(iv) (1.97)6

1.02

(4) The edge of a cube was found to be 30 cm with a possible error in


measurement of 0.1 cm. Use differentials to estimate the maximum
possible error in computing (i) the volume of the cube and (ii) the
surface area of cube.
(5) The radius of a circular disc is given as 24 cm with a maximum error in
measurement of 0.02 cm.
(i) Use differentials to estimate the maximum error in the calculated
area of the disc.
(ii) Compute the relative error ?

6.2 Curve Tracing :


The study of calculus and its applications is best understood when it is
studied through the geometrical representation of the functions involved. In
order to investigate the nature of a function (graph) it is not possible to locate
each and every point of the graph. But we can sketch the graph of the function
and know its nature by certain specific properties and some special points. To
do this we adopt the following strategies.
(1) Domain, Extent, Intercepts and origin :
(i) Domain of a function y = f(x) is determined by the values of x for
which the function is defined.

74

(ii) Horizontal (vertical) extent of the curve is determined by the intervals


of x (y) for which the curve exists.
(iii) x = 0 yields the y intercept and y = 0 yields the x intercept
(iv) If (0,0) satisfies the given equation then the curve will pass through
the origin.
(2) Symmetry : Find out whether the curve is symmetrical about any line with
the help of the following rules :
The curve is symmetrical about
(i) the x-axis if its equation is unaltered when y is replaced by y
(ii) the y-axis if its equation is unaltered when x is replaced by x.
(iii) the origin if it is unaltered when x is replaced by x and y is replaced
by y simultaneously.
(iv) the line y = x if its equation is unchanged when x and y are replaced by
y and x.
(v) the line y = x if its equation is unchanged when x and y are replaced
by y and x.
(3) Asymptotes (parallel to the co-ordinate axes only) :
If y c, c finite [x k, k finite] whenever x [y ] then the
line y = c [x = k] is an asymptote parallel to x axis [y axis].
(4) Monotonicity : Determine the intervals of x for which the curve is
decreasing or increasing using the first derivates test.
(5) Special points (Nature of bending) :
Determine the intervals of concavity and inflection points using the first
and second derivatives test.
Illustrative Example :
Example 6.9 : Trace the curve y = x3 + 1
Solution :
(1) Domain, Extent, intercepts and origin :
The function is defined for all real values of x and hence the domain is the
entire interval (, ). Horizontal extent is < x < and vertical extent is
< y < . Clearly x = 0 yields the y intercept as + 1 and y = 0 yields the
x intercepts as 1. It is obvious that the curve does not pass through (0,0).

75

(2) Symmetry Test : The symmetry test shows that the curve does not possess
any of the symmetry properties.
(3) Asymptotes : As x c (for c finite) y does not tend to and vice versa.
Therefore the curve doest not admit any asymptote.
(4) Monotonicity : The first derivative test shows that the curve is increasing
throughout (,) since y 0 for all x.
y

(5) Special points : The curve is


concave downward in (, 0) and
concave upward in (0, ) since

8
6

y = 6x < 0 for x < 0

y =x3 +1

y = 6x > 0 for x > 0 and


y = 0 for x = 0 yields (0,1)
as the inflection point

-6 -4 -2

2
0
-2

-4
-6
-8

Fig. 6.2
Example 6.10 : Trace the cure y2 = 2x3.
Solution :
(1) Domain, extent, Intercept and Origin :
When x 0, y is well defined. As x , y ,
The curve exists in first and fourth quadrant only
The intercepts with the axes are given by :
x = 0, y = 0 and when y = 0, x = 0
Clearly the curve passes through origin.
(2) Symmetry : By symmetry test, we have, the curve is symmetric about
x axis only.
(3) Asymptotes : As x + , y , and vice versa.
the curve does not admit asymptotes.
(4) Monotonicity : For the branch y = 2 x3/2 of the curve is increasing since
dy
3/2
dx > 0 for x > 0 and the branch y = 2x of the curve is decreasing
dy
since dx < 0 for x > 0

76

(5) Special points : (0,0) is not a point of inflection.


y

This curve is called a semi cubical


parabola.
Note :
(0, 0) admits a pair of tangents
which coincide, resulting in a special
point, called cusp.

(0,0)

Fig. 6.3
Example 6.11 : Discuss the curve y2 ( 1 + x) = x2 (1 x)
for (i) existence (ii) symmetry (iii) asymptotes (iv) loops
Solution :
(i) Existence : The function is not well defined when x >1 and x 1 and the
curve lies between 1 < x 1.
(ii) Symmetry : The curve is symmetrical about the x axis only.
(iii) Asymptotes : x = 1 is a vertical asymptote to the curve parallel to
y axis.
(iv) Loops : (0,0) is a point through which the curve passes twice and hence a
loop is formed between x = 0 and x = 1.
x =-1

(0,0)
(-1,0)

(1,0)

Fig. 6.4
Example 6.12 : Discuss the curve a2 y2 = x2 (a2 x2), a > 0
for (i) existence (ii) symmetry (iii) asymptotes (iv) loops
Solution :
(i) Existence :
The curve is well defined for (a2 x2) 0 i.e., x2 a2 i.e., x a and x a

77

(ii) Symmetry : The curve is symmetrical about x-axis, y axis, and hence
about the origin.
(iii) Asymptotes : It has no asymptote.
(iv) Loops : For a < x < 0 and 0 < x < a, y2 > 0 y is positive and
negative a loop is formed between x = 0 and x = a and another loop is
formed between x = a and x = 0.
y

(0,0)

-a

Fig. 6.5
2

Example 6.13 : Discuss the curve y = (x 1) (x 2)2.


for (i) existence (ii) symmetry (iii) asymptotes (iv) loops
Solution :
(i) Existence :
The curve is not defined for x 1 < 0, ie., whenever x < 1, the R.H.S. is
negative y2 < 0 which is impossible. The curve is defined for x 1.
(ii) Symmetry : The curve is symmetrical about x-axis.
(iii) Asymptote : The curve does not admit asymptotes.
(iv) Loops : Clearly a loop is formed between (1, 0) and (2, 0).
y
2
1
0

Fig. 6.6
EXERCISE 6.2
(1) Trace the curve y = x3
Discuss the following curves for (i) existence
(ii) symmetry
(iii) asymptotes
(iv) loops
(2) y2 = x2 (1 x2)
(3) y2 (2 + x) = x2 (6 x)
(4) y2 = x2 (1 x)
(5) y2 = (x a) (x b)2 ; a, b > 0, a > b.

78

6.3 Partial Differentiation :


A nations economy (E) depends on many factors. An yield (Y) of a crop
also depends on various factors such as rain, soil, manure etc., Similarly the
character (C) of a child is formed by its parents characters, environment etc., In
plane geometry, area (A) and volume (V) also depend on the dimensions like
length, breadth and height. In all the above cases either economy or yield or
character or area or volume depends on more than one variable (factor). If any
small change is effected in any of the variables (factors), it becomes necessary
to know what changes will be caused in the respective dependent variable E or
Y or C or A or V. These small changes can take place in all the variables
(independent) simultaneously or in some of them while others are not subjected
to any change. The study of these changes in the dependent variable while a
corresponding change is made in one or more of the independent variables,
keeping the remaining independent variables fixed leads to what is known as
partial differentiation.
g
h
For clarity, let us consider the
x
area (A) of a rectangle of length x
f
c
d
and breadth y. Then A = xy = f(x,y).
x
Note that A depends on two
A
independent variables x and y.
a
b
e
y
A = xy = area of abcd
y
Fig. 6.7
Suppose a small change is made in y ie., y + y instead of y, then the new
area A = x(y + y). Note that x is fixed still there is change in the area A.
Similarly, if we interchange roles of x and y in the above we get
new area abgh = A = (x + x)y.
Note that change in both x and y will also cause change in area A. In this
case the area is (x +x) (y + y) = area of aeih.
But we shall restrict ourselves to the discussion of the change in one
variable fixing the rest. We may consider functions of two or three independent
variables only.
We can also discuss the continuity problems and the limit process for
functions depending on more than one variable similar to that of their
counterpart in single variable differential calculus.

79

Partial Derivatives :
Let (x0,y0) be any point in the domain of definition of f(x,y). Let u = f(x, y)
We define partial derivative of u with respect to x at the point (x0,y0) as the
ordinary derivative of f(x,y0) with respect to x at the point x = x0.
i.e.,

u
d
= dx f(x,y0)
x=x
x (x , y )
0 0
0
lim f(x0 + h, y0) f(x0,y0)
, (denoted by fx or ux at (x0, y0))
= h0
h

provided the limit exists.


Similarly, partial derivatives of u = f(x,y) with respect to y at the point
(x0,y0) is
d
u
= dy f(x0,y)

y=y
y (x , y )
0 0
0
lim f(x0, y0 + h) f(x0,y0)
(denoted by fy or uy at (x0, y0))
= h0
h
provided the limit exists.
A function is said to be differentiable at a point (at all points on a domain)
if its partial derivatives exist at that point (at all points of a domain). The
process of finding partial derivatives is called partial differentiation.
Remark :
Throughout we shall consider only continuous functions of two or three
variables possessing continuous first order partial derivatives.
Second Order Partial Derivatives : When we differentiate a function
u = f(x,y) twice we obtain its second order derivatives, defined by,
2f
f

2 = x
x
x

2f
f
and
2 = y
y
y

f
f
2f
2f
=
=
=
denoted respectively

x y
y x
x y
y x
as fxx or uxx, fyy or uyy and fxy = fyx or uxy = uyx
Note that since the function and its partial derivaties are continuous the
order of differentiation is immaterial (A result due to Euler)

80

Chain rule (function of a function rule) of two variables :


u = f (x,y) dependent variable

If u = f(x,y) is differentiable and


x and y are differentiable functions of
t, then u is a differentiable function
of t and

u / y

u / x
x

du
f dx
f dy
dt = x dt + y dt

y
dy / dt

dx / dt

Tree diagram to remember the


chain rule : (2 variables)

t independent variable

dx / dt

Tree diagram to remember the


chain rule : (3 variables)

dy / dt

u / y

Fig. 6.8
Chain rule (function of a function rule) of three variables :
If u = f(x,y, z) is differentiable
u dependent variable
and x, y, z are differentiable functions
of t, then u is a differentiable
u / z
u / x
function of t and
y
du f dx
f dy f dz
x
z
dt = x dt + y dt + z dt
dz / dt

t independent variable

Fig. 6.9
Chain rule for partial derivatives :
If w = f(u,v), u = g(x,y), ; v = h (x,y) then
w u
w v
w
=
+
u x
v x
x

w w u
w v
=
+
y
u y
v y
w = f (u,v)

w = f (u,v)
w / v

w / u
u

v
v / y

u / y

v / x

u / x

w / v

w / u

Fig. 6.10

81

Homogeneous functions :
A function of several variables is said to be homogeneous of degree n if
multiplying each variables by t (where t > 0) has the same effect as multiplying
the original function by tn. Thus, f(x,y) is homogeneous of degree
n if f(tx, ty) = tn f(x,y)
Eulers Theorem :
f
f
+y
= nf
If f(x,y) is a homogeneous function of degree n, then x
y
x
Remark : Eulers theorem can be extended to several variables.
Example 6.14 : Determine :

u u 2u 2u 2u
2u
,
, 2, 2,
and
x y x y x y
yx

if u(x,y) = x4 + y3 + 3x2 y2 + 3x2y


u
u
= 4x3 + 6xy2 + 6xy ;
= 3y2 + 6x2y + 3x2
Solution :
y
x
2u
2u
2
2
=
12x
+
6y
+
6y
;
= 6y + 6x2
x2
y2
2u
2 u
= 12 xy + 6x ;
= 12xy + 6x
x y
yx
Note that

2 u
2u
=
due to continuity of u and its first order partial
x y
yx

derivatives.
Example 6.15 : If u = log (tan x + tan y + tan z), prove that sin 2x

u
= 2
x

sec2x
u
= tanx + tany + tanz
x

Solution :

2 sin x cos x . sec2x


2 tan x
u
= tan x + tan y + tan z = tan x + tan y + tan z
x
u
2 tan y
similarly, sin 2y
= tan x + tan y + tan z
y
sin 2x

sin 2z
L.H.S. = sin 2x

2 tan z
u
= tan x + tan y + tan z
z
2 (tan x + tan y + tan z)
u
= tan x + tan y + tan z = 2 = R.H.S
x

82

Example 6.16 :
If U =(x y) (y z) (z x) then show that Ux + Uy + Uz = 0
Solution :

Ux = (y z) {(x y) ( 1) + (z x).1}

= (y z) [(z x) (x y)]
Similarly Uy = (z x) [(x y) (y z)]
Uz = (x y) [(y z) (z x)]
Ux + Uy + Uz = (y z) [(z x) (z x)] + (x y) [ (y z) + (y z)]
+ (z x) [(x y) (x y)]
=0
Example 6.17 : Suppose that z = ye
Solution :

x2

dz
where x = 2t and y = 1 t then find dt

z dx z dy
dz
dt = x dt + y dt
z
z
dy
x2
x2 dx
= ye 2x ;
= e ; dt = 2 ; dt = 1
x
y
dz
x2
x2
=
y
2x
e
(2)
+
e
(1)
dt

x2 x2
4t2
4t2
= 4 xy e e = e [(8t (1 t) 1)] = e (8t 8t2 1)
(Since x = 2t and y = 1 t)
x
w
w
and
Example 6.18 : If w = u2 ev where u = y and v = y log x, find
x
y
Solution : We know

w u w v
w w u w v
w
=
+
; and
=
+
u x v x
y
u y v y
x
w
w
= 2uev ;
= u2ev ;
v
u
1
u
x
u
=y ;
= 2
x
y
y
y
v
=x ;
x

v
y

= log x.

2uev
y
x
w
= y + u2ev x = xy 2 (2 + y)
x
y

83

x
w
= 2uev 2 + u2ev log x
y
y
x
x2
= 3 xy [ylog x 2], (since u = y and v = y log x)
y

dw
Example 6.19 : If w = x + 2y + z2 and x = cos t ; y = sin t ; z = t. Find dt
w dx w dy w dz
dw
+
+
Solution : We know dt =
x dt
y dt
z dt
dx
w
= 1 ; dt = sin t
x
dy
w
= 2 ; dt = cos t
y
dz
w
= 2 z ; dt = 1
z
dw
dt = 1 ( sin t) + 2 cos t + 2z = sin t + 2 cos t + 2 t
1

Example 6.20 : Verify Eulers theorem for f(x,y) =


Solution :

2 2

x + y2

1
= t f(x,y) = t1 f(x, y)

f(tx, ty) =

2 2

t x +t y

f is a homogenous function of degree 1 and by Eulers theorem,


x

f
f
+y
= f
y
x

Verification :

1
fx = 2

2x

(x2 +y2)

/2

(x2 +y2)

3
/2

Similarly, fy =

(x2 +y2)

3
/2

x2 + y2

xfx + yfy =

(x

3
2 /2

+y
Hence Eulers theorem is verified.

84

1
x2 + y2

= f.

Example 6.21 : If u is a homogenous function of x and y of degree n, prove that


2u
2u
u
+ y 2 = (n 1)
x y
y
y
Solution : Since U is a homogeneous function in x and y of degree n, Uy is
homogeneous function in x and y of degree n 1. Applying Eulers theorem for
Uy we have,
x

x(Uy)x + y (Uy)y = (n 1) Uy
i.e., xUyx + y Uyy = (n 1) Uy
i.e., x

2u
u
2u
+ y 2 = (n 1)
x y
y
y

Example 6.22 : Using Eulers theorem, prove that x

u 1
u
+ y
= tan u if
y 2
x

xy

x + y

u = sin1

Solution: R.H.S. is not homogeneous and hence


1
xy
f is homogeneous of degree 2 .
define f = sin u =
x+ y
f 1
f
By Eulers theorem, x + y = 2 f
y
x
1

i.e., x . (sin u) + y (sin u) = 2 sin u


x
y
u
u
1
x . cos u + y . cos u = 2 sin u
x
y
u
u 1
x + y = 2 tan u
x
y
EXERCISE 6.3
2u
2u
=
for the following functions :
x y
y x
x
y
(ii) u = 2 2
(i) u = x2 + 3xy + y2
y
x

(1) Verify

x
(iv) u = tan1 y .

(iii) u = sin 3x cos 4y

85

x2 + y2 , show that x

(2) (i) If u =
x
y

u
u
+y
=u
x
y

x
y
u
u
x
+y
= 0.
(ii) If u = e sin y + e cos x , show that x
x
y
dw
(3) Using chain rule find dt for each of the following :
(i) w = e xy where x = t2, y = t3
(ii) w = log (x2 + y2) where x = et, y = e t
x
(iii) w =
where x = cos t, y = sin t.
(x2 + y2)
(iv) w = xy + z where x = cos t, y = sin t, z = t
(4) (i) Find

w
w
and
if w = log (x2 + y2) where x = r cos , y = r sin
r

(ii) Find

w
w
and
if w = x2 + y2 where x = u2 v2, y = 2uv
v
u

(iii) Find

w
w
and
if w = sin1 xy where x = u + v, y = u v.
v
u

(5) Using Eulers theorem prove the following :


u
u
x3 + y3
prove that x
+y
= sin 2u.
x
y
x y

(i) If u = tan1

u
u
x
(ii) u = xy2 sin y , show that x
+y
= 3u.

x
y
(iii) If u is a homogeneous function of x and y of degree n, prove that
2u
u
2u
x 2 +y
= (n 1)
x
x
y
x
(iv) If V = zeax + by and z is a homogenous function of degree n in x and
V
V
+y
= (ax + by + n)V.
y prove that x
y
x

86

7. INTEGRAL CALCULUS AND ITS


APPLICATIONS
7.1. Introduction :
In class XI, we have studied the direct evaluation of definite integrals as
the limit of integral sums. Even when the integrands are very simple, direct
evaluation of definite integrals as the limit of integral sum involves great
difficulties. Sometimes this method involves cumbersome computations. There
is a formula called Second Fundamental Theorem on Calculus that yields a
practical and convenient method for computing definite integrals in case where
the anti-derivative of the integrand is known. This method which was
discovered by Newton and Leibnitz utilises the profound relationship that
exists between integration and differentiation. In this chapter we have the
following five sections dealing with the concept and applications of definite
integrals.
(i) To solve simple problems using second fundamental theorem of
calculus.
(ii) Properties of definite integral.
(iii) Reduction formulae
(iv) Area under the curve and volume of solid of revolution about an axis.
(v) Length of the curve and the surface area of a solid of revolution about
an axis.

7.2. Simple definite integrals :


First fundamental theorem of calculus :
x
Theorem 7.1 : If f(x) is a continuous function and F(x) =
f(t)dt, then we
a
have the equation F(x) = f(x).

Second fundamental theorem of calculus :


Theorem 7.2 : If f(x) is a continuous function with domain a x b, then
b

f(x)dx = F(b) F(a) where F is any anti-derivative of f.


a
/2 sin x
dx
Example 7.1 : Evaluate
1 + cos2x
0

87

/2 sin x
Let I =
dx
1 + cos2x
0

Solution:

t = cos x
0
/2
1
0

Let t = cos x
x
dt = sin x dx (or) sin x dx = dt
t
0 dt

0
I =
= [tan1 t] 1 = 0 4 = 4

1 + t2
1
1
Example 7.2 : Evaluate x ex dx

Solution:
Using the method of integration by parts

Here

udv = uv
v du

x
x e dx =

1
(xex)0

dv = ex dx

1
ex dx

v = ex

= e

1
(ex)0

= e (e 1)
=1
a
Example 7.3 : Evaluate

a2 x2 dx

0
a
Solution:

x
a2 x2 dx = 2

a2
x
a2 x2 + 2 sin1 a

a2

a
= 0 + 2 sin1 a (0 + 0)

a2 a2
a2
= 2 sin1(1) = 2 2 = 4

88

u=x
du = dx

/2
Example 7.4 : Evaluate e2x cos x dx

0
Solution:

eax
We know eax cos bx dx = 2
(a cos bx + b sin bx)
a + b 2
/2
e2x
/2
(2
cos
x
+
sin
x)
e2x cos x dx = 2

2 + 12
0
0
e
e0
= 5 (0 + 1) 5 (2 + 0)

e 2 1
= 5 5 = 5 (e 2)
EXERCISE 7.1
Evaluate the following problems using second fundamental theorem :
/2
/2
1
(2) cos3x dx
(3) 9 4x2 dx
(1) sin2x dx

0
/4
(4) 2 sin2x sin 2x dx

0
1

dx

(5)

4 x2
0
1 (sin1x)3

0
2

dx
(7) 2
x + 5x + 6
1
1
(10) x2 ex dx

(8)

1 x2

dx

0
/2
(11) e3x cos x dx

0
/2 sin x dx
(6)
9 + cos2x
0
/2
(9) sin 2x cos x dx

0
/2
(12) ex sin x dx

7.3 Properties of Definite Integrals :


b
b
Property (1) : f(x)dx = f(y) dy

a
a
Proof : Let F be any anti-derivative of f
b
f(x) dx = [F(b) F(a)]

89

(i)

f(y) dy = [F(b) F(a)]

a
b
From (i) and (ii)

(ii)

f(x) dx = f(y) dy

That is, integration is independent of change of variables provided the


limits of integration remain the same.
b
a
Property (2) : f(x)dx = f(x) dx

a
b
Proof : Let F be any anti-derivative of f
b
f(x) dx = [F(b) F(a)]

(i)

a
a

f(x) dx = [F(a) F(b)] = [F(b) F(a)]

(ii)

b
b

From (i) and (ii)

a
f(x)
dx
=

f(x) dx

a
b

That is, if the limits of definite integral are interchanged, then the value of
integral changes its sign only.
b
b
Property (3) : f(x)dx = f(a + b x) dx

Proof :

Let u = a + b x

u=a+bx

du = dx

or dx = du
u
b
b
a
b
b
f(a + b x)dx = f(u) du = f(u) du = f(x) dx

90

a
a
Property (4) : f(x)dx = f(a x) dx

Proof :

0
Let u = a x
u=ax
x
o
a
du = dx
u
a
o
or dx = du
a
a
a
o
f(a x)dx = f(u) du = f(u) du = f(x) dx

a
0
0
0
Property (5) (Without proof) : If f(x) is integrable on a closed interval
containing the three numbers a, b and c, then
b
c
b
f(x) dx = f(x) dx + f(x) dx

a
a
regardless of the order of a, b and c.

2a
a
a
Property (6) : f(x)dx = f(x) dx + f(2a x) dx

0
0
0
2a
2a
a
Proof : Consider f(x)dx = f(x) dx + f(x) dx

Put x = 2a u in the second integral on the R.H.S.,


and dx = du
2a
o
f(x)dx = f(2a u) du

(1)
u = 2a x
x
a
2a
u
a
o

a
a

= f(2a u) du

0
a
= f(2a x) dx

91

b
b

f(x) dx = f(y) dy

2a
a
a
Hence (1) becomes f(x) dx = f(x) dx + f(2a x) dx

0
0
0
2a
a
if f(2a x) = f(x)
Property (7) :
f(x)dx = 2 f(x) dx

=0
if f(2a x) = f(x)
Proof : We know that by property
2a
a
a
(1)
f(x)dx = f(x) dx + f(2a x) dx

0
0
0
If f(2a x) = f(x) then (1) becomes
2a
a
a
a
f(x)dx = f(x) dx + f(x) dx = 2 f(x) dx

0
0
0
0
If f(2a x) = f(x) then (1) becomes
a
a
2a
f(x)dx = f(x) dx f(x) dx = 0

Hence proved.
a
a
Property (8) : (i) f(x)dx = 2 f(x) dx,

a
a

if f is an even function.

(ii) f(x) dx = 0

if f is an odd function.

a
a
a
0
Proof : Consider f(x)dx = f(x) dx + f(x) dx

Let x = t in the first integral of the R.H.S.


Then dx = dt

92

(1)

x
t

x=t
0
a
a
0

a
o
f(x)
dx
=
f(
t)
(
dt)
+

f(x) dx

a
0
a
a
0
= f( t) dt + f(x) dx

(1) becomes

= f( t) dt + f(x) dx

0
0
a
a
a
f(x) dx = f( x) dx + f(x) dx

0
0
a
Case (ii) : If f is an even function, then (2) becomes
a
a
a
f(x) dx = f(x) dx + f(x) dx

a
= 2 f(x) dx

0
Case (iii) : If f is an odd function then (2) becomes
a
a
a
f(x)
dx
=
(
f(x)
dx
+

f(x) dx

0
a

a
= f(x) dx + f(x) dx = 0

0
0
Hence proved.
/4
Example 7.5 : Evaluate x3 sin2x dx.

Solution:

/4
Let f(x) = x3 sin2x = x3 (sin x)2
f( x) = ( x)3 (sin ( x))2
= ( x)3 ( sin x)2
= x3 sin2x
= f(x)

93

(2)

f( x) = f(x)

f(x) is an odd function.


/4 3 2
x sin x dx. = 0 (by property)

/4
Example 7.6 :
1
3x
Evaluate log 3 + x dx

1
3x
Let f(x) = log 3 + x

3
+
x
= log (3 + x) log (3 x)
f( x) = log
3 x
= [log (3 x) log (3 + x)]
3x
= log 3 + x = f(x)

Thus f( x) = f(x) f(x) is an odd function.


1
3x
log 3 + x dx = 0

1
Example 7.7 :
/2
Evaluate : x sin x dx

Solution:

/2
Let f(x) = x sin x
Solution:
f( x) = ( x) sin ( x)
= x sin x ( sin ( x) = sin x)
f(x) is an even function.
/2
/2
x sin x dx = 2 x sin x dx

/2

= 2 {x ( cos x)}

94

/2

( cos x) dx

/2

Using the method of integration by parts


/2
/2

= 2 0 + cos x dx = 2 [sin x]

0
= 2 [1 0] = 2

/2
Example 7.8 : Evaluate sin2x dx

/2

Let f(x) = sin2x = (sin x)2


f( x) = (sin ( x))2 = ( sin x)2 = sin2x = f(x)
Hence f(x) is an even function.
/2
/2
1 /2
sin2x dx = 2 sin2x dx = 2 2 (1 cos 2x) dx

Solution:

/2

sin 2x
= x 2

/2
0

=2

/2
f(sin x)
Example 7.9 : Evaluate f(sin x) + f(cos x) dx

0
/2
f(sin x)
Let I = f(sin x) + f(cos x) dx
Solution:

f sin 2 x
/2

dx
=

f sin x + f cos 2 x

0 2
/2
f (cos x)
I = f(cos x) + f (sin x) dx

0
/2 f(sin x) + f(cos x)
/2

(1) + (2) gives 2 I = f(cos x) + f(sin x) dx = dx = [x]/2


=2
0

o
o

I=4

95

(1)

(2)

1
Example 7.10 : Evaluate x(1 x)n dx

Solution:

1
Let I = x(1 x)n dx

0
1
n
= (1 x) [1 (1 x)] dx

0
1

a
a

f(x) dx = f(a x) dx

1
= (1 x) xn dx = (xn xn + 1) dx

0
0
1

xn + 1 xn + 2
1
1
n + 2 (n + 1)
= n + 1 n + 2 = n + 1 n + 2 = (n + 1) (n + 2)

x(1 x)n dx = (n + 1) (n + 2)

0
/2
Example 7.11 : Evaluate log (tan x)dx

0
Solution:

/2
Let I = log (tan x)dx

(1)

/2

= log tan 2 x dx

0
/2
I = log (cot x) dx

0
(1) + (2) gives

/2
2I = [log (tan x) + log (cot x)] dx

96

(2)

/2
/2
= [log (tan x) . (cot x)] dx = (log 1) dx = 0

0
0
( log 1 = 0)
I=0
/3
dx
Example 7.12 : Evaluate
1 + cot x
/6
Solution:

/3
dx
Let I =
cot x
1
+

/6
/3
I=

sin x
dx
sin x + cos x

(1)

/6


sin 3 + 6 x dx


sin 3 + 6 x +


cos 3 + 6 x

b
b

f(x) dx = f(a + b x) dx

3
=

sin 2 x

sin 2 x +

cos 2 x

/3
I=

cos x
dx
cos x + sin x

/3
2I =

sin x + cos x
dx
cos x + sin x

/6

(1) + (2) gives

/6

97

dx

(2)

/3
/3

2I = dx = [x]
=3 6 = 6

/6
/6

I = 12
EXERCISE 7.2
Evaluate the following problems using properties of integration.
/2
1
/4
(1) sin x cos4 x dx
(2) x3 cos3x dx
(3) sin3x cos x dx

1
/2

/4
/2

cos3x dx

(4)

(5) sin2x cosx dx

/2
1
1
(7) log x 1 dx

/2
3
(8)

0
/3

x dx
x+ 3x

0
/4

(6) x sin2x dx

/4
1
(9) x (1 x)10 dx

dx
(10)
1 + tan x
/6

7.4 Reduction formulae :


A formula which expresses (or reduces) the integral of the nth indexed
function interms of that of (n 1)th indexed (or lower indexed) function is
called a reduction formula.
Reduction formulae for sinnx dx. cosnx dx (n is a positive integer) :

1
n1
Result 1 : If In = sinnx dx then In = n sinn1x cos x + n In 2
1
n1
Result 2 : If In = cosnx dx then In = n cosn1x sin x + n In 2

Result 3 :
/2 n
/2 n
sin x dx = cos x dx =

n n 1 . nn 32 . nn 54 ... 23 . 1 when n is odd


n 1 . n 3 . n 5 ... 1 .
n n 2 n 4 2 2 when n is even
98

Note : For the proofs of these above three results, refer Solution Book.

sin5x dx
Example 7.13 : Evaluate :
Solution :

n
If In =
sin x dx, then we have

n1
1
In = n sinn1x cos x + n In2

(I)

sin5x dx = I5

1
4
= 5 sin4x cos x + 5 I3
(when n=5 in I)
1
4 1
2
= 5 sin4x cos x + 5 3 sin2x cosx + 3 I1 (when n=3 in I)

1 4
4
8
5
2

(II)
sin x dx = 5 sin x cos x 15 sin x cosx + 15 I1
1
I1 =
sin x dx = cos x + c

1 4
4
8
5
2

sin x dx = 5 sin x cos x 15 sin x cos x 15 cos x + c
6
Example 7.14 : Evaluate :
sin x dx
n
Solution : If In =
sin x dx, then we have

1
n1
In = n sinn 1x cos x + n In 2

(I)

sin x dx = I6

1
5
= 6 sin5x cos x + 6 I4
(when n=6 in I)
1
5 1
3
= 6 sin5x cos x + 6 4 sin3x cosx + 4 I2

(when n=4 in I)
1
5
5
sin6x dx = 6 sin5x cos x 24 sin3x cos x + 8 I2
(when n=2 in I)

1
1
5
5 1
= 6 sin5x cos x 24 sin3x cos x + 8 2 sin x cosx + 2 I0

5
1
5
5
6
5
3

sin x dx = 6 sin x cos x 24 sin x cos x 16 sin x cos x + 16 I0

99

0
I0 =
sin x dx =
dx = x
5
1
5
5
sin6x dx = 6 sin5x cos x 24 sin3x cos x 16 sin x cos x + 16 x

Example 7.15 : Evaluate :


/2
(i) sin7x dx

/2
(ii) cos8x dx

2
x
(iii) sin9 4 dx

/6
(iv) cos73x dx

0
Solution : (i)We have

(ii)

(iii)

/2 n
n 1 n 3 ... 2
sin x dx = n .

3 when n is odd
n2

0
/2 7
6 4 2 16
sin x dx = 7 . 5 . 3 = 35

0
/2 n
n 1 . n 3 . n 5 ... 1 .
cos x dx = n n 2 n 4 2 2 when n is even

0
/2
7 5 3 1 35
cos8x dx = 8 . 6 . 4 . 2 . 2 = 256

0
2 9 x
sin 4 dx

0
t=x/4
x
x
0
2
Put 4 = t
t
0
dx = 4dt
/2
2

/2

x
512
8 6 4 2
sin9 4 dx = 4 sin9t dt = 4. 9 . 7 . 5 . 3 . = 315

100

(iv)

/6 7
cos 3x dx

t = 3x
Put 3x = t
3dx = dt
dx = 1/3 dt
/6 7
1
cos 3x dx = 3

/6

/2

/2 7
16
1 6 4 2
cos t dt = 3 7 . 5 . 3 . = 105

/2
Example 7.16 : Evaluate : sin4x cos2x dx

0
Solution :
/2 4
/2 4
2
2
sin x cos x dx= sin x (1 sin x) dx

0
/2
/2
/2
= (sin4x sin6x) dx = sin4x dx sin6x dx

0
0
0
3 1 5 3 1
= 4 . 2 . 2 6 . 4 . 2 . 2 = 32
Two important results : The following two results are very useful in the
evaluation of certain types of integrals.
(1) If u and v are functions of x, then
udv = uv uv1 + uv2 uv3 + ... + ( 1)n unvn + ...
where u, u, u ... are successive derivatives of u and v1, v2, v3 ... are
repeated integrals of v
The above formula is well known as Bernoullis formula.
Bernoullis formula is advantageously applied when u = xn (n is a positive
integer).
n

(2) If n is a positive integer, then xneax dx = n+1

a
0

101

Note : The above formula is known as a particular case of Gamma Integral.


Example 7.17 : Evaluate :
1

(ii) x e 4x dx
(iii) x5e4x dx (iv) emxx7 dx
(i) x3e2x dx

0
dv = e2x dx

Solution :
(1) x3e2x dx
Using Bernoullis formula
udv = uv uv1 + uv2 ...
We get

u = x3

v = 1/2 e2x

u = 3x2

v1 = 1/4 e2x

u = 6x

v2 = 1/8 e2x

u = 6

v3 = 1/16 e2x

1
1
1
1
x3 e2x dx = (x3) 2 e2x (3x2) 4 e2x + (6x) 8 e2x (6) 16 e2x

1
3
3x 3
= 2 e2x x3 2x2 + 2 4

u=x

dv = e4x dx
1
v = 4 e4x

u = 1

1
v1 = 16 e4x

1
(ii) x e 4x dx

0
Using Bernoullis formula we get
1
xe

4x

1
1
dx = (x) 4 e4x (1) 16 e4x

0
1
1
= 4 e4 0 16 (e4 e0)

1
5
= 16 16 e4

(iii) x5e4x dx

Using Gamma Integral x5e4x dx = 6

4
0

(iv) emxx7 dx = 8 (Using Gamma Integral)

m
0

102

EXERCISE 7.3
(1) Evaluate :

(ii) cos5x dx

(i) sin x dx

/2
(2) Evaluate : (i) sin6x dx

0
/4 8
(3) Evaluate : (i) cos 2x dx

/2
(ii) cos9x dx

0
/6 7
(ii) sin 3x dx

0
1

0
6 x/2
(ii) x e
dx

(4) Evaluate : (i) x e2x dx

7.5 Area and Volume :

x =a

x =b

In this section, we apply the definite integral to compute measure of area,


length of arc and surface area. In our treatment it is understood that area,
volume etc. is a number without any unit of measurement attached to it.
7.5.1 Area of bounded regions :
Theorem : Let y = f(x) be a
y
continuous function defined on
B
[a, b], which is positive (f(x) lies on
y =f(x)
A
or above x-axis) on the interval
[a, b]. Then, the area bounded by the
curve y = f(x), the x-axis and the
ordinates x = a and x = b is given by
C

Area = f(x)dx or ydx

a
a

(i.e., The area below the x-axis


is negative)

103

x =b

b
b
Area = ( y) dx = ( f(x) dx)
a

Fig. 7.1

If f(x) 0 (f(x) lies on or below


x-axis) for all x in a x b then area
is given by

x =a

y =f(x)

Fig. 7.2

y=
(3/
2) (

x+

2)

Example 7.18 : Find the area of the region bounded by the line 3x 2y + 6 = 0,
x = 1, x = 3 and x-axis.
y
Since the line 3x 2y + 6 = 0 lies above
the x-axis in the interval [1, 3],
(i.e., y > 0 for x (1,3))
the required area
3
3 3
A = ydx = 2 (x + 2) dx

1
1
x
0
1 2 3
3
2

3 x
= 2 2 + 2x
Fig. 7.3
1
3
3 1
= 2 2 (9 1) + 2(3 1) = 2 [4 + 4]

Area = 12 sq. units


Example 7.19:
Find the area of the region bounded by the line 3x 5y 15 = 0, x = 1,
x = 4 and x-axis.
y
The line 3x 5y 15 = 0 lies
1
4
below the x-axis in the interval x = 1
x
and x = 4
5)
4
-1
x
3
Required area = ( y) dx
5 )(

(1/
y=

Fig. 7.4
4 1
3
= 5 (3x 15) dx = 5

x2
3

5x

(5

x)
dx
=

2
5
4

1
3
= 5 5(4 1) 2 (16 1)

15 9
3
= 5 15 2 = 2 sq. units.

Example 7.20: Find the area of the region bounded y = x2 5x + 4, x = 2, x = 3


and the x-axis.

104

For all x, 2 x 3 the curve lies


below the x-axis.
3
Required area = ( y) dx

2
3

-1

= (x2 5x + 4) dx

-2
2
3
x3

x2
Fig. 7.5
= 3 5 2 + 4x

2
8 20
45
13
13
= 9 2 + 12 3 2 + 8 = 6 = 6 sq. units

Area between a continuous curve and y-axis :


y
Let x = f(y) be a continuous
y =d
B
function of y on [c, d]. The area
D
bounded by the curve x = f(y) and the
x =f(y)
abscissae y = c, y = d to the right of
d
A
C
y-axis is given by xdy
y =c
x

Fig. 7.6
y
c
y =d

If the curve lies to the left of


y-axis between the lines y = c and
d
y = d, the area is given by ( x) dy.

x =f(y)

y =c

y=
2x
+1

Fig. 7.7

Example 7.21: Find the area of the


region bounded by y = 2x + 1, y = 3,
y = 5 and y axis.
Solution : The line y = 2x + 1 lies to
the right of y-axis between the lines
y = 3 and y = 5.
d
The required area A = xdy

y =5
y =3

Fig. 7.8

105

5 y1
1 5
= 2 dy = 2 (y 1)dy

3
3
5
1 y2
1 25 9
= 2 2 y = 2 2 2 (5 3)

3
1
= 2 [8 2] = 3 sq. units

y=
2x

+4

Example 7.22: Find the area of the region bounded y = 2x + 4, y = 1 and y = 3


and y-axis.
The curve lies to the left of y-axis
between the lines y = 1 and y = 3
Area is given by
3
y =3
A = ( x) dy

1
3

y =1

y4
= 2 dy

3 2

Fig. 7.9

3
1 3
y2
1
1
= 2 (4 y)dy = 2 4y 2 = 2 [8 4] = 2 sq. units.

1
Remark :
f (x)
If the continuous curve f crosses
b
the x-axis, then the integral f(x) dx x

a
gives the algebraic sum of the areas
between the curve and the axis,
counting area above as positive and
below as negative.
c
d
b
f(x) dx = f(x) dx + ( f(x)) dx +

1 0

above axis

Fig. 7.10
b
f(x) dx

below axis

106

above axis

f (x)

5
Example 7.23: (i) Evaluate the integral (x 3)dx

1
(ii) Find the area of the region bounded by the line y + 3 = x, x = 1 and x = 5
Solution :
5
x2
5 25
1

= 2 15 2 3 = 12 12 = 0 I
(i)

3x
(x

3)
dx
=

1
1
(ii) The line y = x 3 crosses x-axis at x = 3
From the diagram it is clear that A1
y
3
x
=
lies below x-axis.
y
3
A2
A1 = ( y) dx.

1
As A2 lies above the x-axis
5
A2 = ydx

A1

Fig. 7.11

3
5
3
5
Total area = (x 3)dx = (x 3) dx + (x 3) dx

1
1
3
= (6 4) + (8 6)
= 2+2
= 4 sq. units (II)
Note :
From I and II it is clear that the integral f(x) is not always imply an area.
The fundamental theorem asserts that the anti-derivative method works even
when the function f(x) is not always positive.
Example 7.24:
Find the area bounded by the curve y = sin 2x between the ordinates x = 0,
x = and x-axis.
Solution :
The points where the curve y = sin 2x meets the x-axis can be obtained by
putting y = 0.
sin 2x = 0 2x = n , n Z

107

n
x = 2 .


i.e., x = 0, 2, , 3 2

The values of x between x = 0 are x = are x = 0, 2,

The limits for the first arch are 0 and 2 and the curve lies above x-axis.

The limits for the second arch are 2 and and the curve lies below x-axis.
Required area

/2

A = sin 2x dx + ( sin 2x)dx

0
/2

/2

0
y =sin 2x

/2

cos2x
cos2x
= 2

0 + 2 /2

Fig. 7.12

1
= 2 [cos + cos 0 + cos 2 cos ]
1
= 2 [1 + 1 + 1 + 1] = 2 sq. units.
Example 7.25:
Find the area between the curves y = x2 x 2, x-axis and the lines
x = 2 and x = 4

A2 = y dx

1
Hence required area

-2

2
A2

Fig. 7.13

108

x =4

x-2

-2

A1

y= 2
x

The part A2 lies below x-axis.

x =-2

Solution : y = x2 x 2
= (x + 1) (x 2)
This curve intersects x-axis at x = 1 and
x=2
Required area = A1 + A2 + A3

A3
4

4
1
2
= y dx + ( y)dx + y dx

2
2
1
4
1
2
= (x2 x 2) dx + (x2 x 2)dx + (x2 x 2) dx

2
2
1
9
26
11
= 6 + 2 + 3 = 15 sq. units

(b, f (b))

x =b

(a, f (a))

x =a

General Area Principle :


Let f and g be two continuous
curves, with f lying above g. then the
area R between f and g, from
x = a to x = b, is given by
b
R = (f g)dx

A
bo

ve

:y

=
x+
1

a
g
(a, g (a))
(b, g (b))
No restriction on f and g where
they lie. Both may be lie above or
Fig. 7.14
below the x-axis or g lies below and
f lies above the x-axis.
Example 7.26: Find the area between the line y=x + 1 and the curve y = x2 1.
Solution : To get the points of intersection of the curves we should solve the
equations y = x + 1 and y = x2 1.
y
we get,
x2 1 = x + 1
x2 x 2 = 0
(x 2) (x + 1) = 0
x = 1 or x = 2
The line intersects the curve at
x = 1 and x = 2.
x
b f(x)
g(x)
-1
1 2

Required area = above below dx


Below : y =x2 - 1

a
Fig. 7.15
2
= [(x + 1) (x2 1)]dx

109

2
2

x2 x3
= [2 + x x2]dx = 2x + 2 3

1
1 1
8
9
= 4 + 2 3 2 + 2 + 3 = 2 sq. units

Example 7.27:
Find the area bounded by the curve y = x3 and the line y = x.
Solution : The line y = x lies above the curve y = x3 in the first quadrant and
y = x3 lies above the line y = x in the third quadrant. To get the points of
intersection, solve the curves y = x3, y = x x3 = x . We get x = {0, 1}
0
1
The required area = A1 + A2 = [g(x) f(x)]dx + [f(x) g(x)]dx

0
1
= (x3 x)dx + (x x3)dx

0
1
0
1
x4 x2
x2 x4
= 4 2 + 2 4

1
0
1
1
1 1
= 0 40 2+2 04 0

-1
y = x3

y = x3
x
1

(-1, -1)

1 1 1 1 1
= 4 + 2 + 2 4 = 2 sq. units.

y=

y=

(1, 1)

Fig. 7.16

Example 7.28: Find the area of the region enclosed by y2 = x and y = x 2


Solution : The points of intersection of the parabola y2 = x and the line
y = x 2 are (1, 1) and (4, 2)
y
To compute the region [shown in
figure (6.17)] by integrating with
x =y2
(4, 2)
respect to x, we would have to split
the region into two parts, because the
x
equation of the lower boundary
y =x - 2
changes at x = 1. However if we
(1, -1)
integrate with respect to y no
splitting is necessary.
Fig. 7.17

110

2
Required area = (f(y) g(y) dy

1
2
2
y3
y2
= [(y + 2) y2]dy = 2 + 2y 3

1
4 1
8 1
= 2 2 + (4 + 2) 3 + 3

9 9
3
= 2 + 6 3 = 2 sq. units.

y=
(

16

x2
)

Example 7.29: Find the area of the region common to the circle x2 + y2 = 16
and the parabola y2 = 6x
y
Solution : The points of intersection
2
2
2
y =6x
of x + y = 16 and y = 6x are
D
C
(2,23)
(2, 2 3) and (2, 2 3)
Required area is OABC
Due to symmetrical property, the
B
x
required area
O
2
OABC = 2 OBC
i.e., 2{[Area bounded by y2 = 6x,
x = 0, x = 2 and x-axis] + [Area
A
2
2
bounded by x + y = 16, x = 2, x = 4
and x-axis]}
Fig. 7.18

2
= 2

4
6x dx + 2

x3/22
x
= 2 6 3/2 + 2 2
0

16 x2 dx
42
x
42 x2 + 2 sin1 4

8
8 12
3 2 12 + 8 3
4
= 3 (4 + 3)
=

111

4
2

Example 7.30: Compute the area between the curve y = sin x and y = cosx and
the lines x = 0 and x =
y
Solution : To find the points of
y=
intersection solve the two equations.
sin
1
x

1
x=4
Sin x = cos x =
x
0
2
3/2
/4 /2
-1
y=
5
1
co s
x= 4
sin x = cos x =
x
2
Fig. 7.19

From the figure we see that cos x > sin x for 0 x < 4 and sin x > cos x for

4<x<

/4
Area A = (cos x sin x) dx + (sin x cos x)dx

0
/4

/4
= (sin x + cos x)
0 + ( cos x sin x) /4

= sin 4+ cos 4(sin 0 + cos0) + (cos sin ) cos 4sin 4

1
1
1
1
= + (0 + 1) + (1 0)

= 2 2 sq. units.
2
2
2
2

x2 y2
Example 7.31: Find the area of the region bounded by the ellipse 2 + 2 = 1
a b
Solution : The curve is symmetric
y
about both axes.
y =(b/a) (a2 x2)
Area of the ellipse = 4 Area
of the ellipse in the I quadrant.
a
I = 4 ydx
x

0
a b
= 4 a

a2 x2 dx

Fig. 7.20

112

4b a
= a

4b x
a2 x2 dx = a 2

a2
x
a2 x2 + 2 sin1 a

a
0

4b a2
a2
4b
= a 0 + 2 sin1(1) 0 = a 2 2


= ab sq. units.
a
/2
By using parametric form i.e., 4 y dx = 4 b sin ( a sin ) d, we

0
0
get the same area.
Example 7.32: Find the area of the curve y2 = (x 5)2 (x 6)
(i) between x = 5 and x = 6
(ii) between x = 6 and x = 7
Solution :
(i) y2 = (x 5)2 (x 6)
y = (x 5) x 6
This curve cuts the x-axis at x = 5 and at x = 6

y=
(x

a
7

= 2 (x 5) x 6 dx

6
(Since the curve is symmetrical
about x-axis)
7
= 2 (t + 1) t dt

6
1
= 2 (t3/2 + t1/2)dt

113

Fig. 7.21
Take t = x 6
dt = dx
t = x6
x
6
7
t
0
1

x =7

5)
(x

6)

When x takes any value between 5 and 6, y2 is negative.


The curve does not exist in the interval 5 < x < 6.
Hence the area between the curve at x = 5 and x = 6 is zero.
b
(ii) Required area = ydx

2 2
6 + 10 32
t5/2 t3/2
= 2 5 + 3 = 2 5 + 3 = 2 15 = 15 sq. units

2
2 0

Example 7.33: Find the area of the loop of the curve 3ay2 = x(x a)2
Solution :
y
Put y = 0 ; we get x = 0, a
It meets the x-axis at x = 0 and x = a
Here a loop is formed between
the points (0, 0) and (a, 0) about
x-axis. Since the curve is symmetrical
about x-axis, the area of the loop is
twice the area of the portion above the
x-axis.
a
Required area = 2 y dx

(a,0)

Fig. 7.22

2 a
x (x a)
[x3/2 a x]dx
dx =
3a
3a

0
0
a
2 2 5/2 2a 3/2
8a2 8 3 a2
x

x
=
=
3
0 15 3 = 45
3a 5

a
= 2

Required area =

8 3 a2
45 sq. units.

Example 7.34:
Find the area bounded by x-axis and an arch of the cycloid
x = a (2t sin 2t), y = a (1 cos 2t)
Solution : The curves crosses x-axis when y = 0.
a(1 cos 2t) = 0
cos 2t = 1

2t = 2n, n z

t = 0, , 2,
One arch of the curve lies between 0 and

114

b
Required area = y dx

= a(1 cos 2t) 2a (1 cos 2t) dt

y = a(1 cos2t)
x = a (2t sin 2t)
dx = 2a(1 cos 2t) dt

= 2a2 (1 cos 2t)2dt = 2a2 (2 sin2t)2dt = 8a2 sin4t dt

0
0
0
/2
= 2 8a2 sin4t dt

2a
f(x) dx = 2 f(2a x)dx

0
0

3 1
= 16a2 4 2 2 = 3a2 sq. units.

7.5.2 Volume of solids of revolution :


Let f be a non-negative and continuous curve on [a, b] and let R be the
region bounded above by the graph of f, below by the x-axis and on the sides by
the lines x = a and x = b [Fig 6.23 (a)].
f(x)
f(x)
x

Fig. 7.23 (b)

Fig. 7.23(a)

When this region is revolved about the x-axis, it generates a solid having
circular cross sections (Fig. 7.23(b)]. Since the cross section at x has radius f(x),
the cross-sectional area is A(x) = [f(x)]2 = y2
The volume of the solid is generated by moving the plane circular disc
[Fig.6.23(b)] along x-axis perpendicular to the disc.

115

b
b
Therefore volume of the solid is V = [f(x)]2dx = y2 dx

a
y

(ii) If the region bounded by the


graph of x = g(y), the y-axis and on
the sides by the lines y = c and y = d
(Fig. 7.24) then the volume of the
solid generated is given by

g(y)

d
d
V = [g(y)]2dx = x2 dy

c
Fig. 7.24

Example 7.35:
Find the volume of the solid that results when the ellipse
x2 y2
+ = 1 (a > b > 0) is revolved about the minor axis.
a2 b2
Solution :
Volume of the solid is obtained by
revolving the right side of the curve
x2 y2
+ = 1 about the y-axis.
a2 b2
Limits for y is obtained by putting
x = 0 y2 = b2 y = b
a2
From the given curve x2 = 2 (b2 y2)
b
Volume is given by

y
b
-a

a
-b

Fig. 7.25

b
a2 2 y3
a2 2 2

V = x dy = 2 (b y ) dy = 2 2 b y 3
0

b
b
c
b
d

= 2

a2 3 b3 4 2
b 3 = 3 a b cubic units

b2

Example 7.36:
Find the volume of the solid generated when the region enclosed by
y = x, y = 2 and x = 0 is revolved about the y-axis.

116

Solution : Since the solid is generated by


revolving about the y-axis, rewrite y = x
as x = y2.
Taking the limits for y, y = 0 and y = 2
(putting x = 0 in x = y2, we get y = 0)
d
Volume is given by V = x2dy

y =x
y =2

Fig. 7.26
y5 2 32
4
= y dy = 5 = 5 cubic units.
0

0
c
2

EXERCISE 7.4
(1) Find the area of the region bounded by the line x y = 1 and
(i) x-axis, x = 2 and x = 4
(ii) x-axis, x = 2 and x = 0
(2) Find the area of the region bounded by the line x 2y 12 = 0 and
(i) y-axis, y = 2 and y = 5 (ii) y-axis, y = 1 and y = 3
(3) Find the area of the region bounded by the line y = x 5 and the x-axis
between the ordinates x = 3 and x = 7.
(4) Find the area of the region bounded by the curve y = 3x2 x and the
x-axis between x = 1 and x = 1.
(5) Find the area of the region bounded by x2 = 36y, y-axis, y = 2 and y = 4.
(6) Find the area included between the parabola y2 = 4ax and its latus rectum.
x2 y2
(7) Find the area of the region bounded by the ellipse 9 + 5 = 1 between the
two latus rectums.
(8) Find the area of the region bounded by the parabola y2 = 4x and the line
2x y = 4.
(9) Find the common area enclosed by the parabolas 4y2 = 9x and 3x2 = 16y
(10) Find the area of the circle whose radius is a
Find the volume of the solid that results when the region enclosed by the given
curves : (11 to 14)
(11) y = 1 + x2, x = 1, x = 2, y = 0 is revolved about the x-axis.
(12) 2ay2 = x(x a)2 is revolved about x-axis, a > 0.
117

(13) y = x3, x = 0, y = 1 is revolved about the y-axis.


x2 y2
(14) 2 + 2 = 1 is revolved about major axis a > b > 0.
a b
(15) Derive the formula for the volume of a right circular cone with radius r
and height h.
(16) The area of the region bounded by the curve xy = 1, x-axis,
x = 1. Find the volume of the solid generated by revolving the area
mentioned about x-axis.

7.6. Length of the curve :

(i) If the function f(x) and its derivative f (x) are continuous on [a, b]
then the arc length L of the curve y = f(x) from x = a to x = b is defined
b
dy 2
to be L =
1 + dx dx

a
(ii) Similarly for a curve expressed in the form x = g(y), where g is
continuous on [c, d], the arc length L from y = c to y = d is given by
d
dx 2
L=
1 + dy dy

c
(iii) When the equation of the curve y = f(x) is represented in parametric
form x = (t), y = (t), t where (t) and (t) are continuous
function with continuous derivatives and (t) does not vanish in the

given interval then L = ((t))2 + ((t))2 dt

7.7 Surface area of a solid :


(i) If the function f(x) and its
derivatives f (x) are continuous on
[a, b], then the surface area of the solid
of revolution obtained by the
revolution about x-axis, the area
bounded by the curve y = f(x) the two
ordinates x = a, x = b and x-axis is
b
dy 2
1 + dx dx
S.A. = 2 y

118

f(x)
x

Fig. 7.27

(ii) Similarly for the curve expressed


in the form x = g(y) where g(y) is
continuous on [c, d], the surface area
of the solid of revolution obtained by
the revolution about y-axis, the area
bounded by the curve x = g(y) the two
abscissa y = c, y = d and y axis is

g(y)

dx 2
1 + dy dy

d
S.A. = 2 y

Fig. 7.28
(iii) When the equation of the curve y = f(x) is represented in parametric
form x = g(t), y = h(t), t where g(t) and h(t) are continuous
function with continuous derivatives and g(t) does not vanish in the
t=
interval, then S.A. = 2 y (g(t))2 + (h(t)) 2 dt.

t=
Example 7.37: Find the length of the curve 4y2 = x3 between x = 0 and x = 1
y
Solution :
2
3
4y2 =x3
4y = x
Differentiating with respect to x
dy
8y dx = 3x2
x =1
x
2
dy 3x
dx = 8y

dy 2
1 + dx =

1+

9x4
64y2

Fig. 7.29

9x
9x
9x
1+
1 + 16
2 =
3 =
16 4y
16x
The curve is symmetrical about x-axis.
The required length
1
1
dy 2
9x 1/2
1 + dx dx = 2 1 + 16 dx
L = 2
1+

119

9x
1 + 16
1 64 9x 3/21

= 2 9 3 = 27 1 + 16
0

16 2 0
3/2

61
64 125
= 27 64 1 = 27

x 2/3
y 2/3
Example 7.38: Find the length of the curve a + a = 1

Solution :
x = a cos3t, y = a sin3t is the
parametric form of the given astroid,
where 0 t 2
dx
2
dt = 3a cos t sin t ;

-a

dy
2
dt = 3a sin t cos t

-a

x2/3 + y2/3 =a2/3


Astroid

Fig. 7.30
2

dx + dy =
dt dt

9a2 cos4t sin2t + 9a2 sin4t cos2t = 3a sin t cos t

Since the curve is symmetrical about both axes, the total length of the
curve is 4 times the length in the first quadrant.

But t varies from 0 to 2 in the first quadrant.


/2
Length of the entire curve = 4

dx + dy dt
dt dt

/2
/2
= 4 3a sin t cos t dt = 6a sin 2t dt

0
0
= 6a .

cos 2t
2

/2

= 3a [cos cos 0]

= 3a [ 1 1] = 6a

120

Example 7.39: Show that the surface area of the solid obtained by revolving the
arc of the curve y = sin x from x = 0 to x = about x-axis is
2 [ 2 + log (1 + 2)]
Solution : y = sin x
dy
Differentiating with respect to x dx = cos x.

dy 2
1 + dx =

1 + cos2x

b
Surface area = 2y

dy 2
1 + dx dx

a
when the area is rotated about the x-axis.

S = 2 sin x

1 + cos2x dx

t = cos x

Put cos x = t
sin x dx = dt

0
1
= 2

1
t
= 4 2

1
1 + t2 ( dt) = 4

1 + t2 (dt)

0
1
1 + t2 + 2 log

(t +

1
1 + t2)
0

= 2 [ 2 + log (1 + 2)] 0
= 2 [ 2 + log (1 + 2)]
Example 7.40: Find the surface area of the solid generated by revolving the
cycloid x = a(t + sin t), y = a(1 + cos t) about its base (x-axis).
Solution : y = 0 1 + cos t = 0 cos t = 1 t = ,
x = a (t + sin t) ; y = a (1 + cos t)
dy
dx
dt = a (1 + cos t) dt = a sin t
2

dx + dy =
dt dt

t
a2 (1 + cos t)2 + a2 sin2t = 2a cos 2

121


t
Surface area = 2a (1 + cos t) 2a cos 2 dt

t
t
t
= 2 a . 2 cos2 2 . 2 a cos 2 dt = 16 a2 cos3 2 dt

/2
t
= 16a2 2cos3 x dx Take 2 = x

0
2
= 32a2I3 = 32a2 3
64
= 3 a2 sq. units.
EXERCISE 7.5
(1) Find the perimeter of the circle with radius a.
(2) Find the length of the curve x = a(t sin t), y = a(1 cos t) between t = 0
and .
(3) Find the surface area of the solid generated by revolving the arc of the
parabola y2 = 4ax, bounded by its latus rectum about x-axis.
(4) Prove that the curved surface area of a sphere of radius r intercepted
between two parallel planes at a distance a and b from the centre of the
sphere is 2r (b a) and hence deduct the surface area of the sphere.
(b > a).

122

9. DISCRETE MATHEMATICS
Discrete Mathematics deals with several selected topics in Mathematics
that are essential to the study of many Computer Science areas. Since it is very
difficult to cover all the topics, only two topics, namely Mathematical Logic,
and Groups have been introduced. These topics will be very much helpful to
the students in certain practical applications related to Computer Science.

9.1. Mathematical Logic : Introduction :


Logic deals with all types of reasonings. These reasonings may be legal
arguments or mathematical proofs or conclusions in a scientific theory. Aristotle
(384 322 BC) wrote the first treatise on logic. Gottfried Leibnitz framed the
idea of using symbols in logic and this idea was realised in the nineteenth
century by George Boole and Augustus DeMorgan.
Logic is widely used in many branches of sciences and social sciences. It
is the theoretical basis for many areas of Computer Science such as digital logic
circuit design, automata theory and artificial intelligence.
We express our thoughts through words. Since words have many
associations in every day life, there are chances of ambiguities to appear. In
order to avoid this, we use symbols which have been clearly defined. Symbols
are abstract and neutral. Also they are easy to write and manipulate. This is
because the mathematical logic which we shall study is also called symbolic
logic.

9.1.1 Logical statement or Proposition :


A statement or a proposition is a sentence which is either true or false but
not both.
A sentence which is both true and false simultaneously is not a statement,
rather it is a paradox.
Example 1 :
(a) Consider the following sentences :
(i)
Chennai is the capital of Tamilnadu.
(ii) The earth is a planet.
(iii) Rose is a flower.
Each of these sentences is true and so each of them is a statement.
(b) Consider the following sentences :
(iv) Every triangle is an isosceles triangle.

156

(v) Three plus four is eight


(vi) The sun is a planet.
Each of these sentences is false and so each of them is a statement.
Example 2 : Each of the sentences
(vii) Switch on the light.
(viii) Where are you going?
(ix) May God bless you with success.
(x) How beautiful Taj Mahal is!
cannot be assigned true or false and so none of them is a statement. In fact,
(vii) is a command, (viii) is a question (ix) is an optative and (x) is exclamatory.
Truth value of a statement :
The truth or falsity of a statement is called its truth value. If a statement is
true, we say that its truth value is TRUE or T and if it is false, we say that its
truth value is FALSE or F.
All the statements in Example 1(a) have the truth value T, while all the
statements in Example 1 (b) have the truth value F.
Simple statements :
A statement is said to be simple if it cannot be broken into two or more
statements. All the statements in (a) and (b) of Example 1 are simple statements.
Compound statements :
If a statement is the combination of two or more simple statements, then it
is said to be a compound statement.
Example : It is raining and it is cold.
This is a compound statement and it is a combination of two simple
statements It is raining, It is cold.
Simple statements which on combining form compound statements are
called sub-statements or component statements of the compound statement.
The fundamental property of a compound statement is that its truth value is
completely determined by the truth values of its sub-statements together with
the way in which they are combined to form the compound statement.
Basic logical connectives
The words which combine simple statements to form compound statements
are called connectives. We use the connectives and, or, etc., to form new
statements by combining two or more statements. But the use of these
connectives in English language is not always precise and unambiguous. Hence
it is necessary to define a set of connectives with definite meanings in the

157

language of logic, called object language. Three basic connectives are


conjunction which corresponds to the English word and, disjunction which
corresponds to the word or negation which corresponds to the word not.
We use the symbol to denote conjunction, to denote disjunction
and ~ to denote negation.
Conjunction :
If two simple statements p and q are connected by the word and, then the
resulting compound statement p and q is called the conjunction of p and q and
is written in the symbolic form as p q.
Example 1 : Form the conjunction of the following simple statements
p
: Ram is intelligent.
q
: Ravi is handsome.
p q : Ram intelligent and Ravi is handsome.
Example 2 : Convert the following statement into symbolic form :
Usha and Mala are going to school.
the given statement can be rewritten as :
Usha is going to school, and
Mala is going to school.
Let p : Usha is going to school.
q : Mala is going to school.
The given statement in symbolic form is p q.
Rule : (A1) The statement p q has the truth value T whenever both
p and q have the truth value T.
(A2) The statement p q has the truth value F whenever either
p or q or both have the truth value F.
Example : Write the truth value of each of the following statements :
(i)

Ooty is in Tamilnadu and 3 + 4 = 8

(ii)

Ooty is in Tamilnadu and 3 + 4 = 7

(iii) Ooty is in Kerala and 3 + 4 = 7


(iv) Ooty is in Kerala and 3 + 4 = 8
In (i) the truth value of the statement 3 + 4 = 8 is F. By (A2)
(i) has the truth value F.

158

In (ii) both the sub-statements have truth value T and hence by (A1). (ii)
has truth value T.
The truth values of (iii) and (iv) are F.
Disjunction :
If two simple statements p and q are connected by the word or, then the
resulting compound statement p or q is called the disjunction of p and q and is
written in symbolic form as p q.
Example : Form the disjunction of the following simple statements :
p
: John is playing cricket.
q
: There are thirty students in the class room.
p q : John is playing cricket or there are thirty students in the class
room.
Example : Convert the following statement into symbolic form.
5 is a positive integer or a square is a rectangle.
Let p : 5 is a positive integer.
q : A square is a rectangle.
The given statement in symbolic form is p w q.
Rule : (A3) The statement p q has the truth value F whenever both p
and q have the truth value F.
(A4) The statement p q has the truth value T whenever either p
or q or both have the truth value T.
Example :
(i) Chennai is in India or 2 is an integer.
(ii) Chennai is in India or 2 is an irrational number.
(iii) Chennai is in China or 2 is an integer.
(iv) Chennai is in China or 2 is an irrational number.
By (A4), we see that the truth values of (i), (ii) and (iv) are T and by (A3),
the truth value of (iii) is F.
Negation :
The negation of a statement is generally formed by introducing the word
not at some proper place in the statement or by prefixing the statement with It
is not the case that or It is false that.
If p denotes a statement, then the negation of p is written as p or p. We
use the symbol p to denote the negation of p.

159

Rule :

(A5) If the truth value of p is T then the truth value of p is F.


Also, if the truth value of p is F, then the truth value of p
is T.

Example :
p : All men are wise.
p : Not all men are wise. (or)
p : It is not the case that all men are wise (or)
p : It is false that all men are wise.
Note : Negation is called a connective although it does not combine two or
more statements. It only modifies a statement.
EXERCISE 9.1
Find out which of the following sentences are statements and which are not?
Justify your answer.
(1) All natural numbers are integers.
(2) A square has five sides.
(3) The sky is blue.
(4) How are you?
(5) 7 + 2 < 10.
(6) The set of rational numbers is finite.
(7) How beautiful you are!
(8) Wish you all success.
(9) Give me a cup of tea.
(10) 2 is the only even prime.
Write down the truth value (T or F) of the following statements :
(11) All the sides of a rhombus are equal in length.
(12) 1 + 8 is an irrational number.
(13) Milk is white.
(14) The number 30 has four prime factors.
(15) Paris is in France.
(16) Sin x is an even function.
(17) Every square matrix is non-singular.
(18) Jupiter is a planet.
(19) The product of a complex number and its conjugate is purely imaginary.
(20) Isosceles triangles are equilateral.

160

(21) Form the conjunction and the disjunction of


(i) p : Anand reads newspaper,
q : Anand plays cricket.
(ii) p : I like tea.
q : I like ice-cream.
(22) Let p be Kamala is going to school and q be There are twenty students
in the class . Give a simple verbal sentence which describes each of the
following statements :
(i) p q
(ii) p q
(iii) p
(iv) q
(v) p q
(23) Translate each of the following compound statements into symbolic
form :
(i) Rose is red and parrot is a bird.
(ii) Suresh reads Indian Express or The Hindu.
(iii) It is false that the mangoes are sweet.
(iv) 3 + 2 = 5 and Ganges is a river.
(v) It is false that sky is not blue.
(24) If p stands for the statement Sita likes reading and q for the statement
Sita likes playing what does p q stand for?
(25) Write negation of the each of the following statements :
5 is an irrational number.
(i)
(ii) Mani is sincere and hardworking.
(iii) This picture is good or beautiful.

9.1.2 Truth tables :


A table that shows the relation between the truth values of a compound
statement and the truth values of its sub-statements is called the truth table. A
truth table consists of rows and columns. The initial columns are filled with the
possible truth values of the sub-statements and the last column is filled with the
truth values of the compound statement on the basis of the truth values of the
sub-statements written in the initial columns. If the compound statement is
made up of n sub-statements, then its truth table will contain 2n rows.
Example 9.1 : Construct the truth table for p
Solution: The statement p consists of only one simple statement p. Therefore,
its truth table will contain 21(= 2) rows.
Also we know that if p has the truth value T then p has the truth value F
and if p has the truth value F, then p has the truth value T. Thus the truth table
for p is as given below :
Truth table for p
p
p
T
F
F
T

161

Example 9.2 : Construct the truth table for p (p).


Solution: The compound statement p (p) consists of only one single
statement p. Therefore its truth table will contain 21(= 2) rows.
In the first column, enter all possible truth values of p.
In the second column, enter the truth values of p based on the
corresponding truth values of p. Finally, in the last column, enter the truth
values of p (p), using (A4).
Truth table for p(p)
p
p
p(p)
T
F
T
F
T
T
Example 9.3 : Construct the truth table for p q.
Solution: The compound statement p q consists of two simple statements p
and q. Therefore, there must be 22(= 4) rows in the truth table of p q. Now
enter all possible truth values of statements p and q namely TT, TF, FT and FF
in the first two columns of the truth table.
Using (A1) and (A2), enter the truth values of p q in the final column
based on the corresponding truth values of p and q in the first two columns.
Truth table for p q
p
q
pq
T
T
T
T
F
F
F
T
F
F
F
F
Note : Similarly, by using (A3) and (A4) we can construct the truth table for
p q, as given below :
Truth table for p q
p
q
pq
T
T
T
T
F
T
F
T
T
F
F
F

162

Example 9.4 : Construct the truth table for the following statements :

(i) ((p) ( q))


(iii) (p q) ( q)
Solution:
(i)
p
T
T
F
F
(ii)
p
T
T
F
F
(iii)
p
T
T
F
F

(ii) (( p) q)
(iv) (( p) ( q))

Truth table for ((p) ( q))


q
p
q
((p) ( q))
T
F
F
F
F
F
T
T
T
T
F
T
F
T
T
T
Truth table for (( p) q)
q
p
( p) q
(( p) q)
T
F
F
T
F
F
F
T
T
T
T
F
F
T
F
T
Truth table for (p q) ( q)
q
pq
q
(p q) ( q)
T
T
F
F
F
T
T
T
T
T
F
F
F
F
T
F
Truth table for (( p) ( q))

(iv)
p
q
p
q
( p) ( q)
(( p) ( q))
T
T
F
F
F
T
T
F
F
T
F
T
F
T
T
F
F
T
F
F
T
T
T
F
Example 9.5 : Construct the truth table for (p q) ( r)
Solution: The compound statement (p q) ( r) consists of three simple
statements p, q and r. Therefore, there must be 23(= 8) rows in the truth table of
(p q) (r). The truth value of p remains at the same value of T or F for each
of four consecutive assignments of logical values. The truth value of q remains
at T or F for two assignments and that of r remains at T or F for one
assignment.
163

p
q
r
pq
r
(p q) (r)
T
T
T
T
F
T
T
T
F
T
T
T
T
F
T
F
F
F
T
F
F
F
T
T
F
T
T
F
F
F
F
T
F
F
T
T
F
F
T
F
F
F
F
F
F
F
T
T
Example 9.6 : Construct the truth table for (p q) r
Solution:
p
q
r
pq
(p q) r
T
T
T
T
T
T
T
F
T
F
T
F
T
T
T
T
F
F
T
F
F
T
T
T
T
F
T
F
T
F
F
F
T
F
F
F
F
F
F
F
EXERCISE 9.2
Construct the truth tables for the following statements :
(1) p ( q)
(2) ( p) ( q)
(3) (p q)
(4) (p q) ( p)
(5) (p q) ( q)
(6) (p ( q))
(7) (p q) [ (p q)]
(8) (p q) ( q)
(9) (p q) r
(10) (p q) r
Logical Equivalence :
Two compound statements A and B are said to be logically equivalent or
simply equivalent, if they have identical last columns in their truth tables.
In this case we write A B.
Example 9.7 : Show that (p q) ( p) ( q)

164

Solution:
Truth table for (p q)
p
q
pq
(p q)
T
T
T
F
T
F
T
F
F
T
T
F
F
F
F
T
Truth table for (( p) ( q))
p
q
p
q
(( p) ( q))
T
T
F
F
F
T
F
F
T
F
F
T
T
F
F
F
F
T
T
T
The last columns are identical. (p q) (( p) ( q))
Negation of a negation :
Negation of a negation of a statement is the statement itself. Equivalently
we write ( p) p
p
p
( p)
T
F
T
F
T
F
In the truth table, the columns corresponding to p and ( p) are identical.
Hence p and ( p) are logically equivalent.
Example 9.8 : Verify ( p) p for the statement p : the sky is blue.
Solution:
p
: The sky is blue
: The sky is not blue
p
( p) : It is not the case that the sky is not blue or
It is false that the sky is not blue or
The sky is blue
Conditional and bi-conditional statements :
In Mathematics, we frequently come across statements of the form If p
then q. Such statements are called conditional statements or implications. They
are denoted by p q, read as p implies q. The conditional p q is false only
if p is true and q is false. Accordingly, if p is false then p q is true regardless
of the truth value of q.

165

Truth table for p q


p
q
pq
T
T
T
T
F
F
F
T
T
F
F
T
If p and q are two statements, then the compound statement p q and
q p is called a bi-conditional statement and is denoted by p q, read as p if
and only if q. p q has the truth value T whenever p and q have the same truth
values; otherwise it is F.
Truth table for p q
p
q
pq
T
T
T
T
F
F
F
T
F
F
F
T

9.1.3 Tautologies :
A statement is said to be a tautology if the last column of its truth table
contains only T, i.e., it is true for all logical possibilities.
A statement is said to be a contradiction if the last column of its truth table
contains only F, i.e., it is false for all logical possibilities.
Example 9.9 : (i) p ( p) is a tautology. (ii) p ( p) is a contradiction
Solution:
(i)
Truth table for p ( p)
p
p
p ( p)
T
F
T
F
T
T
The last column contains only T. p (p) is a tautology.
(ii)
Truth table for p ( p)
p
p
p ( p)
T
F
F
F
T
F
The last column contains only F. p (p) is a contradiction.

166

Example 9.10 : (i) Show that (( p) ( q)) p is a tautology.


(ii) Show that (( q) p) q is a contradiction.
Solution:
(i)
Truth table for (( p) ( q)) p
p
q
p q ( p) ( q) (( p) ( q)) p
T
T
F
F
F
T
T
F
F
T
T
T
F
T
T
F
T
T
F
F
T
T
T
T
The last column contains only T. (( p) ( q)) p is a tautology.
(ii)
Truth table for (( q) p) q
p
q
q
( q) p
(( q) p) q
T
T
F
F
F
T
F
T
T
F
F
T
F
F
F
F
F
T
F
F
The last column contains only F. (( q) p) q is a contradiction.
Example 9.11 : Use the truth table to determine whether the statement
(( p) q) (p ( q)) is a tautology.
Solution:
Truth table for (( p) q) (p ( q))
p q p q ( p) q
p ( q) (( p) q) (p ( q)
T T
F
F
T
F
T
T F
F
T
F
T
T
F T
T
F
T
F
T
F F
T
T
T
F
T
The last column contains only T. The given statement is a tautology.
EXERCISE 9.3
(1) Use the truth table to establish which of the following statements are
tautologies and which are contradictions.
(i) (( p) q) p
(ii) (p q) ( (p q))
(iii) (p ( q)) (( p) q)
(iv) q (p ( q))
(v) (p ( p)) (( q) p)

167

(2)
(3)
(4)
(5)
(6)
(7)

Show that p q ( p) q
Show that p q (p q) (q p)
Show that p q (( p) q) (( q) p)
Show that (p q) (( p) ( q))
Show that p q and q p are not equivalent.
Show that (p q) (p q) is a tautology.

9.2 Groups :
9.2.1 Binary Operation :
We know that the addition of any two natural numbers is a natural number,
the product of any two natural numbers is also a natural number. Each of these
operations associates with the two given numbers, a third number, their sum in
the case of addition, and their product in the case of multiplication. In this
section we are going to deal with the notion of a binary operation or a binary
composition on a set which is nothing but a generalisation of the usual addition
and usual multiplication on the number systems.
Definition :
A binary operation * on a non-empty set S is a rule, which associates to
each ordered pair (a, b) of elements a, b in S an element a * b in S. Thus a
binary operation * on S is just a map, * : S S S by (a, b) a * b.
Where we denote by a * b, the image of (a, b) in S under *.
From the definition we see that, if * is a binary operation on S then
a, b S a * b S.
In this case, we also say that S is closed under *. This property is known as
the closure axiom or closure property.
List of symbols used in this chapter :
N
- The set of all natural numbers.
Z
- The set of all integers.
W
- The set of all non-negative integers (whole numbers).
E
- The set of all even integers.
O
- The set of all odd integers.
Q
- The set of all rational numbers.
R
- The set of all real numbers.
C
- The set of all complex numbers.
Q {0} - The set of all non-zero rational numbers.
R {0} - The set of all non-zero real numbers.
C - }0} - The set of all non-zero complex numbers.
- for every

- there exists

- such that
168

Illustrative examples :
The usual addition + is a binary operation on N.
Since a, b N a + b N. i.e., N is closed under +.
But the usual subtraction is not binary on N. Since 2, 5 N,
but 2 5 = 3 N.
N is not closed under subtraction.
At the same time, we see that is a binary operation on Z. From this we
see that, an operation becoming binary or not binary depends on the set. The
following table gives which number systems are closed under the usual
algebraic operations, namely addition, subtraction, multiplication and division
denoted by +, , . , respectively.
Number Systems

Q {0}

R {0}

C {0}

binary

binary

binary

binary

binary

not
binary

not
binary

not
binary

not
binary

binary

binary

binary

binary

not
binary

not
binary

not
binary

binary

binary

binary

binary

binary

binary

binary

binary

not
binary

not
binary

not
binary

not
binary

not
binary

binary

binary

binary

Operations

Apart from the usual algebraic operations, some new operations on the
number systems can also be defined. For example, consider the operation * on
N defined by a * b = ab.
It is clear that * is binary on N, a, b N a * b = ab N.
Some more facts about binary operations :
(1) Let the set S be R or any subset of real number system.
Define * as

(i)

a * b = minimum of {a, b}

(ii)

a * b = maximum of {a, b}

(iii) a * b = a
(iv) a * b = b
All the above operations (*) are binary operations on the corresponding
sets.

169

(2) (N, *)
* is defined as a * b = ab + 5. Since ab and 5 are natural numbers,
ab + 5 is also a natural number. * is a binary operation on N.
On the other hand, the operation * defined by a * b = ab 5 is not binary
on N because 2 * 1 = (2)(1) 5 = 3 N.
(3) (Z, *), where * is defined by, a * b = ab, is not a binary operation on z.
Since take a = 2, b = 1
1
ab = 21 = 2 Z
Note that * is also not a binary operaton on R {0}
1
ab = ( 1)1/2 R {0}
because take a = 1, b = 2
(4) (R, *)
Define a * b = a + b + ab
Clearly * is a binary operation on R since a + b and ab are real numbers
and their sum is also a real number.
(5) (O, +)
Addition is not a binary operation on the set of odd integers, since addition
of two odd integers is not odd.
(6) (O, .)
Multiplication is a binary operation on the set of odd integers. Since
product of two odd integers is an odd integer.
(7) Matrix addition is a binary operation on the set of m n matrices. Since
sum of two m n matrices is again an m n matrix.
(8) Matrix addition is not a binary operation on the set of n n singular
matrices as well as on the set of n n non-singular matrices. Because, sum
of two non-singular matrices need not be non-singular and sum of two
singular matrices need not be singular.
(9) Matrix multiplication is a binary operation on the set of singular matrices
as well as on the set of non-singular matrices.
(10) Cross product is a binary operation on the set of vectors, but dot product is
not a binary operation on the set of vectors.
Multiplication table for a binary operation
Any binary operation * on a finite set S = {a1, a2 ... an} can be described by
means of multiplication table. This table consists of n rows and n columns.
Place each element of S at the head of one row and one column, usually taking
them in the same order for columns as for rows. The operator * is placed at the
left hand top corner. The n n = n2 spaces can be filled by writing ai * aj in the
space common to the ith row and the jth column of the table.

170

a2
a1
aj
*
..................................
...
a1
.
.
.
.
ai
ai * aj
.
.
.
This table is also known as Cayleys table or composition table. In the
next section we will see that these composition tables are very much helpful in
exhibiting finite groups.

9.2.2 Groups :
Given any non-empty set S, the possibility of combining two of its
elements to get yet another element of S endows S with an algebraic structure. A
non-empty set S together with a binary operation * is called an algebraic
structure. Group is the simplest of all algebraic structures. It is the one
operational algebraic system. The study of groups was started in the nineteenth
century in connection with the solution of equations. The concept of group
arises not only in Mathematics but also in other fields like Physics, Chemistry
and Biology.
Definition :
A non-empty set G, together with an operation * i.e., (G, *) is said to be a
group if it satisfies the following axioms
(1) Closure axiom
: a, b G a * b G
(2) Associative axiom : a, b, c G, (a * b) * c = a * (b * c)
(3) Identity axiom
: There exists an element e G
such that a * e = e * a = a, a G.
(4) Inverse axiom

: a G there exists an element a1G such


that a1 * a = a * a1 = e.

e is called the identity element of G and a1 is called the inverse of a in G.


Definition (Commutative property) :
A binary operation * on a set S is said to be commutative
if a * b = b * a a, b S

171

Definition :
If a group satisfies the commutative property then it is called an abelian
group or a commutative group, otherwise it is called a non-abelian group.
Note (1) :
If the operation * is a binary operation, the closure axiom will be satisfied
automatically.
Note (2) :
We shall often use the same symbol G to denote the group and the
underlying set.
Order of a group :
The order of a group is defined as the number of distinct elements in the
underlying set.
If the number of elements is finite then the group is called a finite group
and if the number of elements is infinite then the group is called an infinite
group. The order of a group G is denoted by o(G).
Definition :
A non-empty set S with an operation * i.e., (S, *) is said to be a
semi-group if it satisfies the following axioms.
(1) Closure axiom
: a, b S a * b S
(2) Associative axiom : (a * b) * c = a * (b * c), a, b, c S.
Definition :
A non-empty set M with an operation * i.e., (M, *) is said to be a monoid if
it satisfies the following axioms :
(1) Closure axiom
: a, b M a * b M
(2) Associative axiom : (a * b) * c = a * (b * c) a, b, c M
(3) Identity axiom
: There exists an element e M
such that a * e = e * a = a, a M.
(N, +) is a semi-group but it is not a monoid, because the identity element
O N.
(N, *) where * is defined by a * b = ab is not a semi-group, because,
consider
(2 * 3) * 4 = 23 * 4 = 84 = 212 and
2 * (3 * 4) = 2 * 34 = 2 * 81 = 281
(2 * 3) * 4 2 * (3 * 4) i.e., associative axiom is not satisfied.
(Z, .) is a monoid. But it is not a group, because, the inverse axiom is not
1
satisfied. (5 Z, but 5 Z). (Z, +) and (Z, .) are semi-groups as well as
monoids. From the definitions, it is clear that every group is a monoid.
172

Example 9.12 : Prove that (Z, +) is an infinite abelian group.


Solution:
(i) Closure axiom
: We know that sum of two integers is again an
integer.
(ii) Associative axiom : Addition is always associative in Z
i.e., a, b, c Z, (a + b) + c = a + (b + c)
(iii) Identity axiom
: The identity element O Z and it satisfies
O + a = a + O = a, a Z
Identity axiom is true.
(iv) Inverse axiom
: For every a Z, an element a Z such
that a + a = a + ( a) = 0
Inverse axiom is true. (Z, +) is a group.
(v) a, b Z, a + b = b + a
addition is commutative. (Z, +) is an abelian group.
(vi) Since Z is an infinite set (Z, +) is infinite abelian group.
Example 9.13 : Show that (R {0}, .) is an infinite abelian group. Here .
denotes usual multiplication.
Solution:
(i) Closure axiom
: Since product of two non-zero real numbers is
again a non-zero a real number.
i.e., a, b R, a . b R.
(ii) Associative axiom : Multiplication is always associative in R {0}
i.e., a . (b . c) = (a . b) . c a, b, c R {0}
associative axiom is true.
(iii) Identity axiom
: The identity element is 1 R {0} under
multiplication and
1 . a = a . 1 = a, a R {0}
Identity axiom is true.
1
(iv) Inverse axiom
: a R {0}, a R {0} such that
1 1
a . a = a . a = 1 (identity element). Inverse
axiom is true. (R {0}, .) is a group.
(v) a, b R {0}, a . b = b . a
Commutative property is true. (R {0}, .) is an abelian group.

173

(vi) Further R {0} is an infinite set, (R {0}, .) is an infinite abelian


group.
Example 9.14 : Show that the cube roots of unity forms a finite abelian group
under multiplication.
Solution: Let G = {1, , 2}. The Cayleys table is
From the table, we see that,
1
.

2
(i) all the entries in the table are members of G.
1
1

2
So, the closure property is true.
(ii) multiplication is always associative.

2 1
(iii) the identity element is 1 and it satisfies the

identity axiom.
2 2 1
(iv) The inverse of 1 is 1
The inverse of is 2
the inverse of 2 is
and it satisfies the inverse axiom also. (G, .) is a group.
(v) the commutative property is also true.
(G, .) is an abelian group.
(vi) Since G is a finite set, (G, .) is a finite abelian group.
Example 9.15 : Prove that the set of all 4th roots of unity forms an abelian group
under multiplication.
Solution: We know that the fourth roots of unity are 1, i, 1, i.
Let G = {1, i, 1, i}. The Caylelys table is
1 1
i
.
i From the table,
1
1 1
i
i (i) the closure axiom is true.
(ii) multiplication is always associative in C and
i
1 1 1
i
hence in G.
i
i
i 1 1
(iii) the identity element is 1 G and it satisfies
i
1 1
i i
the identity axiom.
(iv) the inverse of 1 is 1 ; i is i ; 1 is 1 ; and i is i. Further it satisfies
the inverse axiom. hence (G, .) is a group.
(v) From the table, the commutative property is also true.
(G, .) is an abelian group.
Example 9.16 : Prove that (C, +) is an infinite abelian group.
Solution:
(i) Closure axiom : Sum of two complex numbers is always a complex number.

174

i.e., z1, z2 C z1 + z2 C
Closure axiom is true.
(ii) Associative axiom : Addition is always associative in C
i.e., (z1 + z2) + z3 = z1 + (z2 + z3) z1, z2, z3 C
Associative axiom is true.
(iii) Identity axiom :
The identity element o = o + io C and o + z = z + o = z z C
Identity axiom is true.
(iv) Inverse axiom : For every z C there exists a unique z C such that
z + ( z) = z + z = 0. Inverse is true. (C, +) is a group.
(v) Commutative property :
z1, z2 C , z1 + z2 = z2 + z1
the commutative property is true. Hence (C , +) is an abelian group.
Since C is an infinite set (C, +) is an infinite abelian group.
Example 9.17 : Show that the set of all non-zero complex numbers is an abelian
group under the usual multiplication of complex numbers.
Solution:
(i) Closure axiom : Let G = C {0} Product of two non-zero complex
numbers is again a non-zero complex number.
Closure axiom is true.
(ii) Associative axiom :
Multiplication is always associative.
Associative property is true.
(iii) Identity axiom :
1 = 1 + io G, 1 is the identity element and 1.z = z . 1 = z z G.
Identity axiom is true.
(iv) Inverse axiom :
Let z = x + iy G. Here z 0 x and y are not both zero.
x2 + y2 0
1
x iy
x iy
x
1
y
z = x + iy = (x + iy) (x iy) = x2 + y2 = x2 + y2 + i x2 + y2 G
1
1
1
Further z . z = z . z = 1 z has the inverse z G.
Thus inverse axiom is satisfied. (G, .) is a group.

175

(v) Commutative property :


z1 z2 = (a + ib) (c + id) = (ac bd) + i (ad + bc)
= (ca db) + i (da + cb) = z2 z1
It satisfies the commutative property.
G is an abelian group under the usual multiplication of complex
numbers.
Note : Here the number 0 is removed, because 0 has no inverse under
multiplication. We can also show that Q {0}, R {0} are abelian groups
under multiplication. But Z {0} is not a group under multiplication.
1
Q 7 Z {0} while its inverse 7 Z {0}
Note : While verifying the axioms, follow the order given in the definition. If one
axiom fails, stop the process at that stage. There is no use in continuing further.
The following table shows which number systems are satisfying the
axioms of a group in the order for a particular operation.
*

Q {0}

R {0}

C {0}

Semi

group

group

group

group

group

not closed

not closed

not closed

semi-group

monoid

monoid

monoid

monoid

group

group

group

not closed

not closed

not closed

not

not

group
.

monid
not

not

not

not

not

not

closed

associative

associative

associative

associative

associative

not

not closed

not closed

not closed

not closed

not closed

closed

not
associative

associative associative

Example 9.18 : Show that (Z, *) is an infinite abelian group where * is defined
as a * b = a + b + 2.
Solution:
(i) Closure axiom : Since a, b and 2 are integers a + b + 2 is also an integer.
a * b z a, b z
Thus closure axiom is true.
(ii) Associative axiom :
Let a, b, c G
(a * b) * c = (a + b + 2) * c = (a + b + 2) + c + 2 = a + b + c + 4
a * (b * c) = a * (b + c + 2) = a + (b + c + 2) + 2 = a + b + c + 4
(a * b) * c = a * (b * c)
Thus associative axiom is true.
176

(iii) Identity axiom :


Let e be the identity element.
By the definition of e, a * e = a
By the definition of *, a * e = a + e + 2
a+e+2=a
e=2
2 Z. Thus identity axiom is true.
(iv) Inverse axiom :
Let a G and a1 be the inverse element of a
By the definition of

a1, a * a1 = e = 2

By the definition of

*, a * a1 = a + a1 + 2
a + a1 + 2 = 2

a1 = a 4
Clearly a 4 Z. Inverse axiom is true. (Z, *) is a group.
(v) Commutative property :
Let a, b G
a * b = a + b + 2 = b + a + 2 = b * a * is commutative.
(Z, *) is an abelian group. further, Z is an infinite set. The group is an
infinite abelian group.
Example 9.19 : Show that the set of all 2 2 non-singular matrices forms a
non-abelian infinite group under matrix multiplication, (where the entries
belong to R).
Solution:
Let G be the set of all 2 2 non-singular matrices, where the entries belong
to R.
(i) Closure axiom : Since product of two non-singular matrices is again
non-singular and the order is 2 2, the closure axiom is satisfied.
i.e., A, B G AB G.
(ii) Associative axiom : Matrix multiplication is always associative and hence
associative axiom is true. i.e., A (BC) = (AB) C A, B, C G.
1 0
(iii) Identity axiom : The identity element is I2 =
G and it satisfies
0 1
the identity property.

177

(iv) Inverse axiom : the inverse of A G, exists i.e. A1 exists and is of order
2 2 and AA1 = A1A = I. Thus the inverse axiom is satisfied. Hence the
set of all 2 2 non-singular matrices forms a group under matrix
multiplication. Further, matrix multiplication is non-commutative (in
general) and the set contain infinitely many elements. The group is an
infinite non-abelian group.
Example 9.20 : Show that the set of four matrices

1 0 1 0 1 0 1 0

,
,
,
form an abelian group, under
0 1 0 1 0 1 0 1
multiplication of matrices.
Solution:
1 0
1 0
1 0
1 0
Let I =
and let
, B =
, C =
, A =
0 1
0 1
0 1
0 1
G = {I, A, B, C}
By computing the products of these matrices, taken in pairs, we can form
the multiplication table as given below :
I
A
B
C
.
I
I
A
B
C
A
A
I
C
B
B
B
C
I
A
C
C
B
A
I
(i) All the entries in the multiplication tables are members of G. So, G is
closed under . Closure axiom is true.
(ii) Matrix multiplication is always associative
(iii) Since the row headed by I coincides with the top row and the column
headed by I coincides with the extreme left column, I is the identity
element in G.
(iv)

I . I = I I is the inverse of I
A . A = I A is the inverse of A
B . B = I B is the inverse of B
C . C = I C is the inverse of C

From the table it is clear that . is commutative. G is an abelian group


under matrix multiplication.

178

x x
Example 9.21 : Show that the set G of all matrices of the form
, where
x x
x R {0}, is a group under matrix multiplication.
Solution:
x x

Let G =
/ x R {0} we shall show that G is a group under
x x

matrix multiplication.
(i) Closure axiom :
x x
y y
A=
G, B =
G
x x
y y
2xy 2xy
AB=
G , ( x 0, y 0 2xy 0)
2xy 2xy
i.e., G is closed under matrix multiplication.
(ii) Matrix multiplication is always associative.
e e
(iii) Let
E=
G be such that AE = A for every A G.
e e
x x e e x x
AE = A

=

x x e e x x
1
2xe 2xe x x

=
2xe = x e = 2 ( x 0)
2xe 2xe x x
1/2 1/2
G is such that AE = A, for every A G
1/2 1/2

Thus E =

We can similarly show that EA = A for every A G.


E is the identity element in G and hence identity axiom is true.
y y
(iv) Suppose A1 =
G is such that A1A = E
y y
1
1
2xy 2xy 1/2 1/2
Then we have
=
2xy = 2 y = 4x
2xy 2xy 1/2 1/2
1/4 x 1/4 x
A1 =
G is such that A1A = E
1/4 x 1/4 x
Similarly we can show that A A1 = E. A1 is the inverse of A.
G is a group under matrix multiplication.

179

Note : The above group is abelian since AB = BA. But in general matrix
multiplication is not commutative.
Example 9.22 : Show that the set G = {a + b 2 / a, b Q} is an infinite
abelian group with respect to addition.
Solution:
(i) Closure axiom :
Let x, y G. Then x = a + b 2, y = c + d 2 ; a, b, c, d Q.
x + y = (a + b 2) + (c + d 2) = (a + c) + (b + d) 2 G,
since (a + c) and (b + d) are rational numbers.
G is closed with respect to addition.
(ii) Associative axiom : Since the elements of G are all real numbers, addition
is associative.
(iii) Identity axiom :
There exists 0 = 0 + 0 2 G such that for all x = a + b 2 G,
x + 0 = (a + b 2) + (0 + 0 2)
= a+b 2=x
Similarly, we have 0 + x = x.
0 is the identity element of G and
satisfies the identity axiom.
(iv) Inverse axiom :
For each x = a + b 2 G, there exists x = ( a) + ( b) 2 G
such that
x + ( x) = (a + b 2) + (( a) + ( b) 2)
= (a + ( a)) + (b + ( b)) 2 = 0
Similarly we have ( x) + x = 0
( a) + ( b) 2 is the inverse of a + b 2 and satisfies the inverse
axiom. G is a group under addition.
(v) Commutative axiom :
x + y = (a + c) + (b + d) 2 = (c + a) + (d + b) 2
= (c + d 2) + (a + b 2)
= y + x, for all x, y G. The commutative property is true.
(G, +) is an abelian group. Since G is infinite, we see that (G, +) is an
infinite abelian group.
Example 9.23 : Let G be the set of all rational numbers except 1 and * be
defined on G by a * b = a + b ab for all a, b G. Show that (G, *) is an
infinite abelian group.
Solution: Let G = Q {1}
Let a, b G. Then a and b are rational numbers and a 1, b 1.

180

(i) Closure axiom : Clearly a * b = a + b ab is a rational number. But to


prove a * b G, we have to prove that a * b 1.
On the contrary, assume that a * b = 1 then
a + b ab = 1
b ab = 1 a
b(1 a) = 1 a
b = 1 ( a 1, 1 a 0)
This is impossible, because b 1. Our assumption is wrong.
a * b 1 and hence a * b G.
Closure axiom is true.
(ii) Associative axiom :
a * (b * c) = a * (b + c bc)
= a + (b + c bc) a (b + c bc)
= a + b + c bc ab ac + abc
(a * b) * c = (a + b ab) * c
= (a + b ab) + c (a + b ab) c
= a + b + c ab ac bc + abc
a * (b * c) = (a * b) * c a, b, c G
Associative axiom is true.
(iii) Identity axiom : Let e be the identity element.
By definition of e, a * e = a
By definition of *, a * e = a + e ae
a + e ae = a
e(1 a) = 0
e = 0 since a 1
e = 0G
Identity axiom is satisfied.
(iv) Inverse axiom :
Let a1 be the inverse of a G.
By the definition of inverse, a * a1 = e = 0
By the definition of *, a * a1 = a + a1 aa 1
a + a1 aa1 = 0
a1 (1 a) = a
a
G since a 1
a1 =
a1
Inverse axiom is satisfied. (G, *) is a group.
181

(v) Commutative axiom :


For any a, b G,

a * b = a + b ab
= b + a ba
= b*a
* is commutative in G and hence (G, *) is an abelian group. Since G is
infinite, (G, *) is an infinite abelian group.
Example 9.24 : Prove that the set of four functions f1, f2, f3, f4 on the set of nonzero complex numbers C {0} defined by
1
1
f1(z) = z, f2(z) = z, f3(z) = z and f4(z) = z z C {0} forms an
abelian group with respect to the composition of functions.
Let
G = {f1, f2, f3, f4}
Solution:
(f1 f1) (z) = f1(f1(z)) = f1(z)
f1f1 = f1
f2 f1 = f2 , f3f1 = f3, f4f1 = f4
Again

(f2f2) (z) = f2(f2(z)) = f2( z) = ( z) = z = f1(z)

f2f2 = f1
Similarly
f2f3 = f4, f2f4 = f3
1
(f3f2) (z) = f3 (f2 (z)) = f3( z) = z = f4(z)
f3f2 = f4
Similarly

f3f3 = f1, f3f4 = f2


1
1
= = f (z)
(f4f2) (z) = f4(f2(z)) = f4( z) =
z z 3

f4f2 = f3
Similarly
f4f3 = f2, f4f4 = f1
Using these results we have the composition table as given below :
f1
f2
f3
f4

f1
f2
f3
f4
f1
f2

f2

f1

f4

f3

f3

f3

f4

f1

f2

f4

f4

f3

f2

f1

182

From the table


(i) All the entries of the composition table are the elements of G .
Closure axiom is true.
(ii) Composition of functions is in general associative.
(iii) Clearly f1 is the identity element of G and satisfies the identity axiom.
(iv) From the table :
Inverse of f1 is f1 ;

Inverse of f2 is f2

Inverse of f3 is f3 ;

Inverse of f4 is f4

Inverse axiom is satisfied. (G, o) is a group.


(v) From the table the commutative property is also true.
(G, o) is an abelian group.

9.2.3 Modulo Operation


We shall now define new types of operations called Addition modulo n
and Multiplication modulo n, where n is a positive integer. To define these
operations we require the notion of Division Algorithm.
Let a, b Z with b 0. Then we can divide a by b to get a quotient q and a
non-negative remainder r which is smaller in size than b.
i.e., a = qb + r, where 0 r < | b |. This is called Division Algorithm.
For example, if a = 17, b = 5 then 17 = (3 5) + 2
Here q = 3 and r = 2
Addition modulo n (+ n) :
Let a, b Z and n be a fixed positive integer. We define addition modulo n
by a +n b = r ; 0 r < n where r is the least non-negative remainder when
a + b is divided by n.
For example, if a = 25, b = 8 and n = 7 then 25 +78 = 5
( 25 + 8 = 33 = (4 7) + 5)
Multiplication modulo n (. n)
As given above
a .n b = r ; 0 r < n, where r is the least non-negative remainder when ab
is divided by n.
For example, 2 .54 = 3
7 .9 8 = 2

183

Congruence modulo n :
Let a, b Z and n be a fixed positive integer.
We say that a is congruent to b modulo n (a b) is divisible by n
Symbolically,
a b (mod n) (a b) is divisible by n.
15 3 (mod 4) is true because 15 3 is divisible by 4.
17 4 (mod 3) is not true because 17 4 is not divisible by 3.
Congruence classes modulo n :
Let a Z and n be a fixed positive integer.
Collect all numbers which are congruent to a modulo n. This set will be
denoted as [a] and is called the congruence class modulo n or residue class
modulo n.
Thus
[a] = {x Z / x a (mod n)}
= {x Z / (x a) is divisible by n}
= {x Z / (x a) is a multiple of n}
= {x Z / (x a) = kn}, k Z
= {x Z / x = a + kn}, k Z
consider the congruence classes modulo 5.
[a] = {x Z / x = a + kn}
[0] = {x Z / x = 5k, k Z} = {... 10, 5, 0, 5, 10...}
[1] = {x Z / x = 5k + 1, k Z} = {... 9, 4, 1, 6, 11, ...}
[2] = {x Z / x = 5k + 2, k Z} = {... 8, 3, 2, 7, 12, ...}
[3] = {x Z / x = 5k + 3, k Z} = {... 7, 2, 3, 8, 13, ...}
[4] = {x Z / x = 5k + 4, k Z} = {... 6, 1, 4, 9, 14 ...}
[5] = {x Z / x = 5k + 5, k Z} = {... 5, 0, 5, 10, ...} = [0]
Similarly [6] = [1] ; [7] = [2] ; etc.
Note that, we have only 5 distinct classes whose union gives the entire Z.
Thus the set of congruence classes corresponding to 5 is
[0],
[1], [2], [3], [4]} and it will be deonoted by Z5.
{
i.e., Z5 = {[0], [1], [2], [3], [4]}
If we take the modulo 6, we have Z6 = {[0], [1] .... [5]}.
Thus for any positive integer n, we have Zn = {[0], [1] ... [n 1]}
Here [n] = [0] and the union of these classes gives Z.

184

Operations on congruence classes :


(1) Addition :
Let [a], [b] Zn
[a] + n [b] = [a + b] if a + b < n
= [r] if a + b n
Where r is the least non-negative remainder when a + b is divided by n.
For example,
In Z10 , [5] +10 [7] = [2]
In Z8 , [3] +8 [5] = [0]
(ii) Multiplication :
[ab] if ab < n
[a] .n [b] =
if ab n
[r]
where r is the least non-negative remainder when ab is divided by n
In Z5
[2] .5[2] = [4]
[3] .5 [4] = 2
In Z7,

[3] .7 [3] = [2]

In Z8 ,

[5] .8 [3] = [7]

Example 9.25 : Show that (Zn, +n) forms group.


Solution: Let Zn = {[0], [1], [2], ... [n 1]} be the set of all congruence
classes modulo n. and let [l], [m], Zn 0 l, m, < n
(i) Closure axiom : By definition
[l + m] if l + m < n
[l] + n [m] = [r]
if l + m n

where l + m = q . n + r 0 r < n
In both the cases, [l + m] Zn and [r] Zn
Closure axiom is true.
(ii) Addition modulo n is always associative in the set of congruence classes
modulo n.
(iii) The identity element [0] Zn and it satisfies the identity axiom.
(iv) The inverse of [l] Zn is [n l]
Clearly [n l] Zn and

185

[l] + n [n l] = [0]
[n l] + n [l] = [0]
The inverse axiom is also true. Hence (Zn, +n) is a group.
Note : (Zn, +n) is a finite abelian group of order n.
Example 9.26 : Show that (Z7 {[0]}, .7) forms a group.
Solution: Let G = [[1], [2], ... [6]]
The Cayleys table is
.7
[1] [2]
[1]
[2]
[3]
[4]
[5]
[6]

[1]
[2]
[3]
[4]
[5]
[6]

[2]
[4]
[6]
[1]
[3]
[5]

[3]

[4]

[5]

[6]

[3]
[6]
[2]
[5]
[1]
[4]

[4]
[1]
[5]
[2]
[6]
[3]

[5]
[3]
[1]
[6]
[4]
[2]

[6]
[5]
[4]
[3]
[2]
[1]

From the table :


(i) all the elements of the composition table are the elements of G.
The closure axiom is true.
(ii) multiplication modulo 7 is always associative.
(iii) the identity element is [1] G and satisfies the identity axiom.
(iv) the inverse of [1] is [1] ; [2] is [4] ; [3] is [5] ; [4] is [2] ; [5] is [3] and
[6] is [6] and it satisfies the inverse axiom.
the given set forms a group under multiplication modulo 7.
In general, it can be shown that (Zp {(0)}, . p) is a group for any prime p.
But the proof is beyond the scope of this book.
Note : Does the set of all non-zero congruence classes modulo n, a positive
integer, form a group under multiplication modulo n, ?
Example 9.27 : Show that the nth roots of unity form an abelian group of finite
order with usual multiplication.
Solution: We know that 1, , 2...... n 1 are the nth roots of unity, where
2
= cis n . Let G = {1, , 2 ... n 1}
(i) Closure axiom : Let l, m G, 0 l, m (n 1)
To prove l m = l + m G
186

Case (i) l + m < n


If l + m < n then clearly l + m G
Case (ii) l + m n By division algoritham,
l + m = (q . n) + r where 0 r < n, q is a positive integer.
q

l + m = qn + r = (n) . r = (1)qr = r G 0 r < n


Closure property is true.
(ii) Associative axiom : Multiplication is always associative in the set of
complex numbers and hence in G
l .(p.m) = l . (p + m) = l + (p + m) = ( l + p) + m = (l + p) . m
= (l . p) . m = l, m, p G
(iii) Identity axiom : The identity element 1 G and it satisfies
1.l = l .1 = l l G
(iv) Inverse axiom :
For any l G, n l G and l . n l = n l .l = n = 1
Thus inverse axiom is true.
(G, .) is a group.
(v) Commutative axiom :
l . m = l + m = m + l = m . l l , m G
(G, .) is an abelian group. Since G contains n elements, (G, .) is a finite
abelian group of order n.

9.2.4 Order of an element :


Let G be a group and a G. The order of a is defined as the least
positive integer n such that an = e, e is the identity element. If no such positive
integer exists, then a is said to be of infinite order. The order of a is denoted by
0(a).
Note : Here an = a * a * a ... *a (n times). If * is usual multiplication . then
an is a . a .a... (n times) i.e., an .
If * is usual addition then an is a + a + a + ... + a (n times) i.e., na. Thus an
is not a to the power n, it is a symbol to denote a * a * a ... * a (n times).
Clearly an G, if a G . (By the repeated application of closure axiom).
Theorem :
For any group G, the identity element is the only element of order 1.

187

Proof : If a ( e) is another element of order 1 then by the definition of order of


an element, we have (a)1 = e a = e which is a contradiction. e is the only
element of order 1.
Example 9.28 : Find the order of each element of the group (G, .)
where G = {1, 1, i, i}.
Solution: In the given group, the identity element is 1. 0(1) = 1.
0( 1) = 2 [Q we have to multiply 1 two times (minimum) to get 1 i.e.,
( 1) ( 1) = 1]
0(i) = 4 [Q we have to multiply i four times to get 1, i.e., (i) (i) (i) (i) = 1]
0( i) = 4 [Q we have to multiply i four times to get 1].
Example 9.29 : Find the order of each element in the group G = {1, , 2},
consisting of cube roots of unity with usual multiplication.
Solution: We know that the identity element is 1. 0(1) = 1.
0() = 3. Since . . = 3 = 1
0(2) = 3 since (2) (2) (2) = 6 = 1
Example 9.30 : Find the order of each element of the group (Z4, +4)
Solution: Z4 = {[0], [1], [2], [3]} is an abelian group under the addition
modulo 4. The identity element is [0] and note that [4] = [8] = [12] = [0]
0([0]) = 1
0([1]) = 4 [Q we have to add [1] four times to get [4] or [0]]
0 ([2]) = 2 [Q we have to add [2] two times to get [4] or [0]]
0 ([3]) = 4 Q we have to add [3] four times to get [12] or [0]

9.2.5 Properties of Groups :

Theorem :
The identity element of a group is unique.
Proof : Let G be a group. If possible let e1 and e2 be identity elements in G.
Treating e1 as an identity element we have e1 * e2 = e2
Treating e2 as an identity element, we have e1 * e2 = e1
From (1) and (2), e1 = e2
Identity element of a group is unique.
Theorem :
The inverse of each element of a group is unique.
Proof :
Let G be a group and let a G.
If possible, let a1 and a2 be two inverses of a.

188

(1)
(2)

Treating a1 as an inverse of a we have a * a1 = a1 * a = e.


Treating a2 as an inverse of a, we have a * a2 = a2 * a = e
a1 = a1 * e = a1 * (a * a2) = (a1 * a) * a2 = e * a2 = a2
Inverse of an element is unique.
Theorem : (Cancellation laws)
Let G be a group. Then for all a, b, c G,
(i) a * b = a * c b = c (Left Cancellation Law)
(ii) b * a = c * a b = c (Right Cancellation Law)
Now

Proof : (i)

a * b = a * c a1 * (a * b) = a1 * (a * c)
(a1 * a) * b = (a1 * a) * c
e*b=e*c
b=c

(ii)

b * a = c * a (b * a) * a1 = (c * a) * a1
b * (a * a 1) = c * (a * a1)
b*e=c*e
b=c

Theorem : In a group G, (a1)


Proof :

= a for every a G.
1

We know that a1 G and hence (a1)


a1 * (a1)

G. Clearly a * a1 = a1 * a = e

= (a1) * a1 = e

a * a1 = (a1)

a = (a1)
Theorem : (Reversal law)

* a1
(by Right Cancellation Law)

Let G be a group a, b G. Then (a * b) 1 = b1 * a1.


Proof : It is enough to prove b1 * a1 is the inverse of (a * b)
To prove (i) (a * b) * (b1 * a1) = e
(ii) (b1 * a1) * (a * b) = e
(i) (a * b) * (b1 * a1) = a * (b * b1) * a1
= a * (e) * a1
= a * a1 = e

189

(ii)

(b1 * a1) * (a * b) =
=
=

b1 * (a1 * a) * b
b1 * (e) * b
b1 * b = e

b1 * a1 is the inverse of a * b i.e., (a * b)1 = b1 * a1


EXERCISE 9.4
(1) Let S be a non-empty set and o be a binary operation on S defined by
xoy = x ; x, y S. Determine whether o is commutative and associative.
(2) Show that the set N of natural members is a semi-group under the
operation x * y = max {x, y}. Is it a monoid?
(3) Show that the set of all positive even integers forms a semi-group under
the usual addition and multiplication. Is it a monoid under each of the
above operations?
1 0 0 1
(4) Prove that the matrices
,
form a group under matrix
0 1 1 0
multiplication.
(5) Show that the set G of all positive rationals forms a group under the
ab
composition * defined by a * b = 3 for all a, b G.
1 0 0 2 0 0 1 0 2 0
(6) Show that
,
,
,

,
,
0 1 0 2 0 1 0 0 2 0
where 3 = 1, 1 form a group with respect to matrix multiplication.
(7) Show that the set M of complex numbers z with the condition | z | = 1
forms a group with respect to the operation of multiplication of complex
numbers.
(8) Show that the set G of all rational numbers except 1 forms an abelian
group with respect to the operation * given by a * b = a + b + ab for all a,
b G.
(9) Show that the set {[1], [3], [4], [5], [9]} forms an abelian group under
multiplication modulo 11.
(10) Find the order of each element in the group (Z5 {[0]}, .5)
a o
(11) Show that the set of all matrices of the form
, a R {0} forms
o o
an abelian group under matrix multiplication.
(12) Show that the set G = {2n / n Z} is an abelian group under
multiplication.

190

10. PROBABILITY DISTRIBUTIONS


10.1 Introduction :
In XI Standard we dealt with random experiments which can be described
by finite sample space. We studied the assignment and computation of
probabilities of events. In the Sciences one often deals with variables as a
quantity that may assume any one of a set of values. In Statistics we deal with
random variables - variables whose observed value is determined by chance.

10.2. Random Variable :


The outcomes of an experiment are represented by a random variable if
these outcomes are numerical or if real numbers can be assigned to them.
For example, in a die rolling experiment, the corresponding random
variable is represented by the set of outcomes {1, 2, 3, 4, 5, 6} ; while in the
coin tossing experiment the outcomes head (H) or tail (T) can be represented as
a random variable by assuming 0 to T and 1 to H. In this sense a random
variable is a real valued function that maps the sample space into the real line.
Let us consider the tossing of two fair coins at a time. The possible results
are {HH, TH, HT, TT}. Let us consider the variable X which is the number of
heads obtained while tossing two fair coins. We could assign the value X = 0
to the outcome of getting no heads, X = 1 to the outcome of getting only 1
head and X = 2 to the out come of getting 2 heads.
Therefore X (TT) = 0, X(TH) = 1, X (HT) = 1 and X (HH) = 2.
Therefore X takes the values 0,1,2. Thus we can assign a real number X(s) to
every element s of the sample space S.
Definition : If S is a sample space with a probability measure and X is a real
valued function defined over the elements of S, then X is called a random
variable.
A random variable is also called a chance variable or a stochastic variable.
Types of Random variables :
(1) Discrete Random variable (2) Continuous Random variable

10.2.1 Discrete Random Variable :


Definition : Discrete Random Variable
If a random variable takes only a finite or a countable number of values, it
is called a discrete random variable.
Note : Biased coins may have both sides marked as tails or both sides marked as
heads or may fall on one side only for every toss, whereas a fair or unbiased coin
means, it has equal chances of falling on heads and tails. Similarly biased dice may
have repeated numbers on several sides ; some numbers may be missing. For a fair die
the probability of getting any number from one to six will be 1/6.

191

Example :
1. The number of heads obtained when two coins are tossed is a discrete
random variable as X assumes the values 0, 1 or 2 which form a
countable set.
2. Number of Aces when ten cards are drawn from a well shuffled pack of
52 cards.
The random variable X assumes 0, 1, 2, 3 or 4 which is again a countable set.
i.e., X (No aces) = 0, X (one ace) = 1, X (two aces) = 2,
X (three aces) = 3, X (four aces) = 4
Probability Mass Function :
The Mathematical definition of discrete probability function p(x) is a
function that satisfies the following properties :
(1) The probability that X can take a specific value x is p(x)
ie., P(X = x) = p(x) = px.
(2) p(x) is non negative for all real x.
(3) The sum of p(x) over all possible values of X is one. That is
pi = 1 where j represents all possible values that X can have and pi
is the probability at X = xi
If a1, a2, . . . am, a, b1, b2, . . bn, b be the values of the discrete random
variable X in ascending order then
(i) P(X a) = 1 P(X < a)
(ii) P(X a) = 1 P(X > a)
(iii) P(a X b) = P(X = a) + P(X = b1) + P(X = b2) + . . .
. . . + P(X = bn) + P(X = b).
Distribution function : (Cumulative Distribution function)
The distribution function of a random variable X is defined as
F(x) = P(X x) = p(xi) : ( < x < ).
xi x

Properities of Distribution function :


1)

F(x) is a non-decreasing function of x

2)

0 F(x) 1, < x <

192

3)
4)
5)

Lt
F(x) = 0
x
Lt
F() = x
F(x) =1
+
P(X = xn) = F(xn) F(xn 1)

F( ) =

Illustration :
Find the probability mass function and cumulative distribution function for
getting number of heads when three coins are tossed once.
Solution : Let X be the random variable getting number of Heads. Sample
space when three coins are tossed is
S
= HHH HHT HTH THH HTT THT TTH TTT

R
:
3
2
2
2
1
1
1
0
(No.of Heads)

Since X is the random variable getting the number of heads, X takes the
values 0, 1,2 and 3. (X : S R).
1
P (getting no head)
= P (X = 0) = 8
3
P (getting one head)
= P (X = 1) = 8
3
P (getting two heads) = P (X = 2) = 8
1
P (getting three heads) = P (X = 3) = 8
P(x)
probability mass function is given by
1
1/8 if x = 0
3/8 if x = 1
OR
P (X= x) = 3/8 if x = 2
1/8 if x = 3
X
0
1
2
3
1/
P(X = x) 1/8 3/8
3/8
1/8
8
0

To find cumulative distribution function.

Fig. 10.1
x

We have F(x) =

P(X =

xi =

193

xi)

1
When X = 0, F(0) = P(X = 0) = 8
1

P(X =

When X = 1, F(1) =

xi)

i=

3
4
1
1
= P(X = 0) + P(X = 1) = 8 + 8 = 8 = 2
2

P(X =

When X = 2, F(2) =

xi)

i=

= P (X = 0) + P(X = 1) + P(X = 2)
1
3
3
7
= 8 + 8 + 8 =8
3

When X = 3, F(3) =

P(X =

xi)

i=

= P (X = 0) + P(X = 1) + P(X = 2) + P(X = 3)


3
3
1
1
=8 + 8 + 8 + 8 = 1
Cumulative distribution function is
0 if < x < 0
1/8 if 0 x < 1
1/2 if 1 x < 2
F(x) =
7/8 if 2 x < 3
1 if 3 x <

F(x)

1O
7/8 O
1/2 O
1/8 O
0

Fig. 10.2

Example 10.1 :
Find the probability mass function, and the cumulative distribution
function for getting 3s when two dice are thrown.
Solution :
Two dice are thrown. Let X be the random variable of getting number of
3s. Therefore X can take the values 0, 1, 2.

194

25
P(no 3) = P(X = 0) = 36

Sample Space
(1,4)
(1,3)
(2,4)
(2,3)

(1,5)
(2,5)

(1,6)
(2,6)

(3,4)

(3,5)

(3,6)

(4,1) (4,2)
(4,4)
(4,3)
(5,1) (5,2)
(5,4)
(5,3)
(6,1) (6,2)
(6,4)
(6,3)
probability mass function is given by
x
0
1
2
P(X = x) 25/36
10/36 1/36
Cumulative distribution function :

(4,5)
(5,5)
(6,5)

(4,6)
(5,6)
(6,6)

10
P(one 3) = P(X = 1) = 36
1
P(two 3s) = P(X = 2) = 36

(1,1)
(2,1)

(1,2)
(2,2)

(3,1)

(3,2)

(3,3)

We have F(x) =

P(X =

xi)

xi =

25
F(0) = P(X = 0) = 36
10
35
25
F(1) = P(X = 0) + P(X = 1) = 36 + 36 = 36
25 10 1 36
F(2) = P(X = 0 ) + P(X = 1) + P(X =2) = 36 + 36 + 36 = 36 = 1
x
0
1
2
F(x)
25/36
35/36
1
Example 10.2 A random variable X has the following probability mass function
x
0
1
2
3
4
5
6
5k
7k
9k
11k 13k
P(X = x)
k
3k
(1) Find k.
(2) Evaluate P(X < 4), P(X 5) and P(3< X 6)
1
(3) What is the smallest value of x for which P (X x) > 2 .
Solution :
6
(1) Since P(X = x) is a probability mass function P(X = x) = 1
x=0
ie.,P(X=0) + P(X = 1) +P(X = 2) +P(X = 3) +P(X = 4) +P(X = 5)+P(X = 6) = 1.
1
k + 3k + 5k + 7k + 9k + 11k + 13k = 1 49 k = 1 k = 49

195

(2)

P(X < 4) = P(X = 0) + P(X = 1 ) + P(X = 2) + P(X = 3)


1
3
5
7
16
= 49 + 49 + 49 + 49 = 49
11 13 24
P(X 5) = P(X = 5) + P(X = 6) = 49 + 49 = 49
9 11 13 33
P(3 < X 6) = P(X = 4) + P(X = 5) + P(X = 6) = 49 + 49 + 49 = 49
(3) The minimum value of x may be determined by trial and error method.
1
4 1
1
P(X 0) = 49 < 2 ; P(X 1) = 49 < 2
1
16 1
9
P(X 2) = 49 < 2 ; P(X 3) = 49 < 2
25
1
P(X 4) = 49 > 2
1

The smallest value of x for which P(X x) > 2 is 4.


Example 10.3 :An urn contains 4 white and 3 red balls. Find the probability
distribution of number of red balls in three draws one by one from the urn.
(i) with replacement (ii) without replacement
Solution : (i) with replacement
Let X be the random variable of drawing number of red balls in three draws.
X can take the values 0,1,2,3.
3
P(Red ball) = 7 = P(R)
4
P(Not Red ball) = 7 = P(W)
4 4 4 64
Therefore P(X = 0) = P(www) = 7 7 7 = 343
P(X = 1) = P(Rww) + P(wRw) + P(wwR)
3
4
4
4 4
3 4
4 3
= 7 7 7 + 7 7 7 + 7 7 7

144
48
= 3 343 = 343
P(X = 2) = P(RRw) + P(RwR) + P(wRR)
3 4
4 3
3 3
3
3
4
= 7 7 7 + 7 7 7 + 7 7 7

3
3
4
36 108
= 3 7 7 7 = 3 343 = 343

196

3 3 3
27
P(X = 3) = P(RRR) = 7 7 7 = 343
The required probability distribution is
X
0
1
2
P(X = x)
64/343 144/343
108/343
Clearly all pis are 0 and pi = 1.

3
27/343

2) Without replacement : It is also treated a simultaneous case.


Method 1 :
Method 2 :
Using combination
Using Conditional Probability
3
2
4
(i) P(no red ball)
(i)
P(www) = 7 6 5
4c 3c
3
0
4
P(X = 0) =
7c
= 35
3
41
4
= 35 = 35
(ii) P(1 red ball)
4c 3c
P(X = 1) =

7c

63
18
= 35 = 35
(iii) P(2 red ball)
4c 3c
P(X = 2) =

7c

43
12
= 35 = 35
(iv)P(3 red ball)
4c 3c
P(X = 3) =

7c

(ii) P(Rww) + P(wRw) + P(wwR)


4
3
3
3
3
4
= 7 6 5 + 7 6 5

3
3
4
+ 7 6 5

36 36 18
= 3 210 = 70 = 35
(iii) P(RRw) + P(RwR) + P(wRR)
2
4
4
2
3
3
= 7 6 5 + 7 6 5

3
2
4
+ 7 6 5

24
12
= 3 210 = 35
(iv)

11
1
= 35 = 35

197

3
2
1
P(RRR) = 7 6 5
1
= 35

P(X = x)

4
35

18
35

12
35

1
35

Clearly all pis are 0 and pi = 1

10.2.2 Continuous Random Variable :


Definition : A Random Variable X is said to be continuous if it can take all
possible values between certain given limits. i.e., X is said to be continuous if
its values cannot be put in 1 1 correspondence with N, the set of Natural
numbers.
Examples for Continuous Random Variable
The life length in hours of a certain light bulb.
Let X denote the ph value of a chemical compound which is randomly
selected. Then X is a continuous random variable because any ph value,
between 0 and 14 is possible.
If in the study of ecology of a lake, we make depth measurements at
randomly chosen locations then X = the depth at such location is a
continuous random variable. The limit will be between the maximum
and minimum depth in the region sampled.
Probability Density Function (p.d.f.) :
The mathematical definition of a continuous probability function f(x) is a
function that satisfies the following properties.
(i) The probability that X is between two points a and b is
b

P(a x b) =
f(x) dx
a

(ii) It is non-negative for all real X.

(iii) The integral of the probability function is 1 i.e.,


f(x) dx = 1

Continuous probability functions are referred to as p.d.f.


Since continuous probability function are defined for uncountable number of
points over an interval, the probability at a single point is always zero.
a
i.e., P(X = a) =
f(x) dx = 0.
a

198

The probabilities are measured over intervals and not at single points. That
is, the area under the curve between two distinct points defines the probability
for that interval.
P(a x b) = P(a X < b) = P(a < x b) = P(a < x < b)
Discrete Probability function are referred to as probability mass function and
continuous probability function are referred to as probability density function.
The term probability function covers both discrete and continuous distribution.
Cumulative Distribution Function :
If X is a continuous random variable, the function given by
x
F(x) = P(X x) = f(t)dt for < x < where f(t) is the value of the

probability density function of X at t is called the distribution function or


cumulative distribution of X.
Properties of Distribution function :
(i) F(x) is a non-decreasing function of x
(ii) 0 F(x) 1, < x < .
x

lt
f(x)
dx
=
(iii) F( ) = x

f(x) dx = 0

lt
f(x)
dx
=
(iv) F() = x

f(x) dx = 1

(v) For any real constant a and b and a b, P(a x b) = F(b) F(a)
d
(vi) f(x) = dx F(x)
i.e., F(x) = f(x)
Example 10.4 : A continuous random variable X follows the probability law,
k x (1 x )10, 0 < x < 1
f(x) =
elsewhere
0
Find k

Solution : Since f(x) is a p.d.f


f(x) dx = 1

199

By properties of definite
integral
a
a
f(x)
dx
=

f(a x)dx

10

kx(1 x) dx = 1

i.e.,

k(1 x) [1 (1 x)]10dx = 1

i.e.,

0
1

i.e.,

k (1 x)x10dx = 1

x11 x12
1
1
i.e., k (x x )dx = 1 k 11 12 = 1 k 11 12 = 1 k = 132

10

11

0
Example 10.5 : A continuous random variable X has p.d.f. f(x) = 3x2,
0 x 1, Find a and b such that.
(i) P(X a) = P(X > a) and (ii) P(X > b) = 0.05
Solution :
(i) Since the total probability is 1, [Given that P(X a) = P (X > a]
P(X a) + P(X > a) = 1
i.e., P(X a) + P(X a) = 1
1
P(X a) = 2
a

1

f(x) dx = 2

3x dx = 2
0

3x3
1 3
1
1
i.e., 3 = 2 a3 = 2 i.e., a = 2

0
(ii) P(X > b) = 0.05
1
1
2

3x dx = 0.05
f(x) dx = 0.05
b

b
1

3x3
3 = 0.05 1 b3 = 0.05
b
1

19 3
95
b = 1 0.05 = 0.95 = 100 b = 20

3

200

Example 10.6 : If the probability density function of a random variable is given


k (1 x2), 0 < x < 1
by f(x) =
elsewhere
0
find (i) k (ii) the distribution function of the random variable.

Solution: (i) Since f(x) is a p.d.f.


f(x) dx = 1

x3
1
k x 3 = 1 k 1 3 = 1

0
0
3
2
3 k = 1 or k = 2

x
(ii) The distribution function F(x) =
f(t) dt
2

k(1 x ) dx = 1

(a) When x ( , 0]
x
F(x) = f(t) dt = 0

(b) When x (0, 1)


x
F(x) = f(t) dt

x
x 3
0
3
= f(t) dt + f(t) dt = 0 + 2 (1 t2) dt = 2

0
0

x3
x 3

(c) When x [1, )


1
x
13
x
0
F(x) = f(t) dt = f(t) dt + f(t) dt + f(t) dt = 0 + 2 (1 t2) dt + 0

3 t3
= 2 t 3 = 1

<x0
0
F(x) = 3/2 (x x3/3) 0 < x < 1
1x<
1

201

1
+ tan1 x < x < is a distribution

2
function of a continuous variable X, find P(0 x 1)
1
F(x) = 2 + tan1 x
Solution:


P(0 x 1) = F(1) F(0)
1
1
+ tan1 1 2 + tan1 0
=

2
1

1
1
1
= 2 + 4 2 + 0 =

2 + 4 2 = 4

A, 1 < x < e3
is a probability density function of
Example 10.8 : If f(x) = x
0, elsewhere
a continuous random variable X, find p(x > e)

Solution: Since f(x) is a p.d.f.


f(x) dx = 1
Example 10.7 : If F(x) =

3
3

A dx = 1 A[log x] e = 1
1
x
1

A[log e3 log 1] = 1 A[3] = 1 A = 1/3


1 , 1 < x < e3
Therefore f(x)= 3x
0 elsewhere
e3

e
1
1
1
P(x > e) = 3 x dx =3 [log x]

e
e

1
2
1
= 3 [log e3 log e] = 3 [3 1] = 3
Example 10.9 :For the probability density function
2e2x, x > 0
f(x)=
, find F(2)
,x0
0
2
Solution : F(2) = P(X 2) =
f(x) dx

202

e2x
e4 1
2x
= [e4 1] = 1 e4 = 4
=
dx = 2 .
2e
2 0
e
0
Example 10.10 : The total life time (in year) of 5 year old dog of a certain
breed is a Random Variable whose distribution function is given by
, for x 5
0
F(x) = 1 25 , for x > 5 Find the probability that such a five year old dog

x2
will live (i) beyond 10 years (ii) less than 8 years (iii) anywhere between
12 to 15 years.
Solution : (i) P(dog living beyond 10 years)
P(X > 10) = 1 P(X 10)
25
= 1 1 2 when x = 10
x
25
3
1
= 1 1 100 = 1 4 = 4

(ii) P(dog living less than 8 years )


P(X < 8) = F(8) [since P(X < 8) = P(X 8) for a continuous distribution]
25
25
39
= 1 2 = 1 64 = 64

8
2

(iii) P(dog living any where between 12 and 15 years ) = P(12 < x < 15)
25
25
1
= F(15) F(12) = 1 2 1 2 = 16
15 12
EXERCISE 10.1
(1) Find the probability distribution of the number of sixes in throwing three
dice once.
(2) Two cards are drawn successively without replacement from a well
shuffled pack of 52 cards. Find the probability distribution of the number
of queens.
(3) Two bad oranges are accidentally mixed with ten good ones. Three
oranges are drawn at random without replacement from this lot. Obtain
the probability distribution for the number of bad oranges.
(4) A discrete random variable X has the following probability distributions.
X
0 1
2
3
4
5
6
7
8
P(x) a 3a 5 a 7 a 9 a 11 a 13 a 15 a 17 a
(i) Find the value of a (ii) Find P(x < 3) (iii) Find P(3 < x < 7)

203

(5) Verify that the following are probability density functions.


2x , 0 x 3
1
1
(a) f(x) = 9
, < x <
(b) f(x) =

(1 + x2)
0
elsewhere
3,

0<x<1
(6) For the p.d.f f(x) = cx (1 x)
elsewhere
0
1
find (i) the constant c
(ii) P x < 2

(7) The probability density function of a random variable x is

1 x
e
, x, , > 0 . Find (i) k (ii) P(X > 10)
f(x) = kx
0
, elsewhere
x<0
0
2
(8) For the distribution function given by F(x) = x 0 x 1
1
x>1
find the density function. Also evaluate
(i) P(0.5 < X < 0.75)
(ii) P(X 0.5)
(iii) P(X > 0.75)
(9) A continuous random variable x has the p.d.f defined by
ceax, 0 < x <
f(x) = 0
elsewhere . Find the value of c if a > 0.
(10) A random variable X has a probability density function
k , 0 < x < 2
f(x) = 0 elsewhere

Find (i) k (ii) P 0 < X < 2

(iii) P 2 < X < 2

10.3 Mathematical Expectation :


Expectation of a discrete random variable :
Definition : If X denotes a discrete random variable which can assume the
values x1, x2, . . . . . . xn with respective probabilities p1, p2, . . . pn then the
mathematical expectation of X, denoted by E(X) is defined by
E(X) = p1 x1 + p2 x2 + . . . . . + pnxn =

204

i=1

i=1

pi xi where pi = 1

Thus E(X) is the weighted arithmetic mean of the values xi with the

weights p(xi) X = E(X)


Hence the mathematical Expectation E(X) of a random variable is simply
the arithmetic mean.
Result : If (X) is a function of the random variable X,
then E[ (X)] = P(X = x) (x).
Properties :
Result (1) :

E(c) = c where c is a constant

Proof :

E(X) = pi xi
E(c) = pi c = c pi = c as pi = 1

Result (2) :
Proof :

Result (3) :
Proof :

E(c) = c
E(cX) = c E(X)
E(cX) = (cxi)pi = (c x1) p1 + (c x2) p2 + . . . (c xn) pn
= c( p1 x1 + p2x2 +. . . . pn xn)
=
E(aX + b) =
E(aX + b) =
=

c E(X)
a E(X) + b.
(a xi+ b) pi
(a x1+ b) p1 + (a x2 + b)p2 + (a xn + b) pn

= a( p1 x1 + p2x2 +. . . . pn xn) + b pi
= a E(X) + b. Similarly E(aX b) = aE(X) b
Moments : Expected values of a function of a random variable X is used for
calculating the moments. We will discuss about two types of moments.
(i) Moments about the origin
(ii) Moments about the mean which are called central moments.
Moments about the origin :
If X is a discrete random variable for each positive integer r (r = 1, ...) the
th
r moment
r = E(Xr) = pi xir
1 = E(X) = pi xi
First moment :
This is called the mean of the random variable X.
Second moment : 2 = E(X2) = pi xi2

205

Moments about the Mean : (Central Moments)


For each positive integer n, (n = 1, 2, ...) the nth central moment of the
discrete random variable X is

n = E(X X )n = (xi x )n pi

First moment about the Mean 1 = E(X X )1 = (xi x )1 pi


= x p x p = x p
x (1) as p = 1
1

i i

i i

= E(X) E(X) = 0
The algebraic sum of the deviations about the arithmetic mean is always
zero

2nd moment about the Mean 2 = E(X X )2


2

= E(X2 + X 2 X X ) = E(X2) + X 2 X E(X) ( X is a constant)


= E(X2) + [E(X)]2 2E(X) E(X)
2 = E(X2) [E(X)]2 = 2 (1)2
Second moment about the Mean is called the variance of the random
variable X

2 = Var (X) = E(X X )2 = E(X2) [E(X)]2

Result (4) :
Proof :

Var (X c) = Var X where c is a constant.

w.k.t. Var (X) = E(X X )2

Var (X + c) = E[(X + c) E (X + c)]2


= E[X + c E(X) c]2

= E[X X ]2 = Var X
Similarly Var (X c) = Var (X)
Variance is independent of change of origin.
Result (5) :

Var (aX) = a2 Var (X)

Proof :

Var (aX) = E[aX E(aX)]2 = E[aX aE(X)]2


= E[a {X E(X)}]2

= a2 E[X E(X)]2 =
Change of scale affects the variance

206

a2 Var X

Result (6) :

Var (c) = 0 where c is a constant.

Proof :
Var (c) = E[c E(c)]2 = E[c c]2 = E(0) = 0
Example 10.11 : Two unbiased dice are thrown together at random. Find the
expected value of the total number of points shown up.
Solution : Let X be the random variable which represents the sum of the
numbers shown in the two dice. If both show one then the sum total is 2. If both
show six then the sum is 12.
The random variable X can take values from 2 to 12.
(1, 1)
(1, 2) (2, 1)
(1, 3) (2, 2) (3, 1)
(1, 4) (2, 3) (3, 2) (4, 1)
(1, 5) (2, 4) (3, 3) (4, 2) (5, 1)
(1, 6) (2, 5) (3, 4) (4, 3) (5, 2) (6, 1)
(2, 6) (3, 5) (4, 4) (5, 3) (6, 2)
(3, 6) (4, 5) (5, 4) (6, 3)
(4, 6) (5, 5) (6, 4)
(5, 6) (6, 5)
(6, 6)
The probability distribution is given by.
X
2
3
4
5
6
7
8
9
10 11 12
2
3
4
5
6
5
4
3
2
1
1
P(X = x) 36 36 36 36 36 36 36 36 36 36 36
E(X) = pi xi = xi pi
1
2
3
1
252
= 2 36 + 3 26 + 4 36 + . . . .+ 12 36 = 36 = 7

Example 10.12 : The probability of success of an event is p and that of failure


is q. Find the expected number of trials to get a first success.
Solution: Let X be the random variable denoting Number of trials to get a first
success. The success can occur in the 1st trial. The probability of success in
the 1st trial is p. The success in the 2nd trial means failure in the 1st trial.
Probability is qp.
Success in the 3rd trial means failure in the first two trials. Probability of
success in the 3rd trial is q2p. As it goes on, the success may occur in the nth
trial which mean the first (n 1) trials are failures. probability = qn1p.

207

The probability distribution is as follows


X
1
2
3
...
P(x)

qp

q p

...

n ...
n1

p...
q
E(X) = pi xi
= 1 . p + 2qp + 3q2p + . . . + nqn1 p . .
= p[1 + 2q + 3q2 + . . .+ nqn1 + ... ]
p
1
= p[1 q]2 = p(p)2 = 2 = p
p
Example 10.13 : An urn contains 4 white and 3 Red balls. Find the probability
distribution of the number of red balls in three draws when a ball is drawn at
random with replacement. Also find its mean and variance.
Solution : The required probability distribution is [Refer Example 10.3]
X
0
1
2
3
144
108
27
64
P(X = x) 343 343
343
343
Mean E(X) = pi xi
64
144
108
27
9
= 0 343 + 1 343 + 2 343 + 3 343 = 7

Variance = E(X2) [E(X)]2


E(X2) = pi xi2
117
64
144
108
27
= 0 343 + 12343 + 22343 + 32 343 = 49

36
9 2
117
Variance= 49 7 = 49

Example 10.14 :A game is played with a single fair die, A player wins Rs. 20 if
a 2 turns up, Rs. 40 if a 4 turns up, loses Rs. 30 if a 6 turns up. While he neither
wins nor loses if any other face turns up. Find the expected sum of money he
can win.
Solution : Let X be the random variable denoting the amount he can win. The
possible values of X are 20,40, 30 and 0.
1
P[X = 20] = P(getting 2) = 6
1
P[X = 40] = P(getting 4) = 6
1
P[X = 30] = P(getting 6) = 6

208

1
The remaining probability is 2
X
20
40
0
30
P(x)
1/6
1/6
1/6
1/2
Mean E (X) = pi xi
1
1
1
1
= 20 6 + 40 6 + (30) 6 + 0 2 = 5




Expected sum of money he can win = Rs. 5
Expectation of a continuous Random Variable :
Definition : Let X be a continuous random variable with probability density
function f(x). Then the mathematical expectation of X is defined as

E(X) =
xf(x) dx

Note : If is function such that (X) is a random variable and E [ (X)] exists
then

E[ (X)] =
(x) f(x) dx

2
E(X ) =
x f(x) dx

Variance of X = E(X2) [E(X)]2


Results : (1) E(c) = c where c is a constant

c
f(x)
dx
=
c
E(c) =

f(x) dx = c

(2)

as
f(x) dx = 1

E(aX b) = a E(X) b

E(aX b) =
(ax b) f(x) dx =
ax f(x) dx


b f(x) dx

=a
x f(x) dx b
f(x) dx = a E(X) b

209

Example 10.15 : In a continuous distribution the p.d.f of X is


3 x (2 x) 0< x < 2
f(x)=4
, otherwise.

0
Find the mean and the variance of the distribution.
2

3
E(X) =
Solution :
x f(x) dx = x. 4 x(2 x) dx

0
2

3
3
2
2
3
= 4
x (2 x) dx = 4
(2x x ) dx
0

3
=4

x
x
3 2
16
2 3 4 = 4 3(8) 4 = 1

Mean = 1

2
x2f(x) dx =
E(X ) =

x2 3 x(2 x) dx
4
0

2
3 x4
x5
3 16 32
6
3
(2 x3 x4) dx = 4 2 4 5 = 4 2 5 = 5
= 4

0
0

6
1
Variance = E(X2) [E(X)]2 = 5 1 = 5
Example 10.16 : Find the mean and variance of the distribution
3e3x,0 < x <
f(x) =
,elsewhere
0
Solution :

E(X) =
x f(x) dx

xn e x dx = n + 1

1 1
3x
3x
=
dx = 3. 2 = 3
x (3e ) dx = 3
xe
3
0
0

0
When n is a positive
integer

2 2
2
2 3x
3x

x
E(X2) =
(3e
)
dx
=
3
dx = 3 . 3 = 9

x e
3
0
0

210

2 1 2 1
Var(X) = E[X2] E[X]2 = 9 3 = 9

1
Mean = 3

1
; Variance = 9
EXERCISE 10.2

(1) A die is tossed twice. A success is getting an odd number on a toss. Find
the mean and the variance of the probability distribution of the number of
successes.
(2) Find the expected value of the number on a die when thrown.
(3) In an entrance examination a student has to answer all the 120 questions.
Each question has four options and only one option is correct. A student
gets 1 mark for a correct answer and loses half mark for a wrong answer.
What is the expectation of the mark scored by a student if he chooses the
answer to each question at random?
(4) Two cards are drawn with replacement from a well shuffled deck of 52
cards. Find the mean and variance for the number of aces.
(5) In a gambling game a man wins Rs.10 if he gets all heads or all tails and
loses Rs.5 if he gets 1 or 2 heads when 3 coins are tossed once. Find his
expectation of gain.
(6) The probability distribution of a random variable X is given below :
X
P(X = x)

0
0.1

1
0.3

2
0.5

3
0.1

If Y = X2 + 2X find the mean and variance of Y.


(7) Find the Mean and Variance for the following probability density
functions

1 ,12 x 12
(i) f(x) = 24
0
,otherwise
xex

(iii) f(x) =

e x

(ii) f(x) =

, if x > 0
,otherwise

211

, if x > 0
,otherwise

10.4 Theoretical Distributions :


The values of random variables may be distributed according to some
definite probability law which can be expressed mathematically and the
corresponding probability distribution is called theoretical distribution.
Theoretical distributions are based on expectations on the basis of previous
experience.
In this section we shall study (1) Binomial distribution (2) Poisson
distribution (3) Normal distribution which figure most prominently in
statistical theory and in application. The first two distributions are discrete
probability distributions and the third is a continuous probability distribution.
Discrete Distributions :
Binomial Distribution :
This was discovered by a Swiss Mathematician James Bernoulli (16541705)
Bernoullis Trials :
Consider a random experiment that has only two possible outcomes. For
example when a coin is tossed we can take the falling of head as success and
falling of tail as failure. Assume that these outcomes have probabilities
p and q respectively such that p + q =1. If the experiment is repeated n times
independently with two possible outcomes they are called Bernoullis trials. A
Binomial distribution can be used under the following condition.
(i) any trial, result in a success or a failure
(ii) There are a finite number of trials which are independent.
(iii) The probability of success is the same in each trial.
Probability function of Binomial Distribution :
Let n be a given positive integer and p be a given real number such that
0 p 1. Also let q = 1 p. Consider the finite probability distribution
described by the following table.
xi
0
1
2
...
n
P(xi)

qn

nc1pqn1

nc2 p2qn2

...

pn

The table shown above is called the Binomial distribution. The 2nd row of
the table are the successive terms in the binomial expansion of (q + p)n.
Binomial probability function B(n,p,x) gives the probability of exactly
x successes in n Bernoullian trials, p being the probability of success in a trial.
The constants n and p are called the parameters of the distribution.

212

Definition of Binomial Distribution :


A random variable X is said to follow Binomial distribution if its
probability mass function is given by

n px qn x, x = 0, 1, . . .n
P(X = x) = p(x) = Cx
0
otherwise
Constants of Binomial Distribution :
Mean = np
Variance = npq
Standard deviation =

variance = npq

X B(n, p) denotes that the random variable X follows Binomial distribution


with parameters n and p.
Note : In a Binomial distribution mean is always greater than the variance.
Example 10.17 : Let X be a binomially distributed variable with mean 2 and
2
standard deviation . Find the corresponding probability function.
3

Solution :

np = 2
npq = 4/3
npq
q = np

npq

2
3

4 2
6 =3
2
1
p = 1q = 13 = 3
1
np = 2 n 3 = 2 n = 6

The probability function for the distribution is
1 x 2 6x
P[X = x] = 6C 3 3
, x = 0, 1, 2,

x
4/3

= 2

Example 10.18 : A pair of dice is thrown 10 times. If getting a doublet is


considered a success find the probability of (i) 4 success (ii) No success.
Solution : n = 10 . A doublet can be obtained when a pair of dice thrown is
{(1,1), (2,2) (3,3), (4,4), (5,5) (6,6)} ie., 6 ways.
Probability of success is getting a doublet
6
1
1
5
p = 36 = 6 ; q = 1 p = 1 6 = 6

213

Let X be the number of success.


We have P[X = x] = nC px qnx
x

1 4 5 6
(a) P(4 successes) = P[X = 4] = 10C 6 6

4
=
(b)

210 56
35 5 6
= 216 6
10

6

P (no success) = P(X = 0)


= 10C
0

5 10 = 5 10
6
6

Example 10.19 : In a Binomial distribution if n = 5and P(X = 3) = 2P(X = 2)


find p
Solution :

P(X = x) = nC px qnx
x

P(X = 3) = 5C p3q2 and P(X = 2) = 5C p2q3


3
2
5C p3q2 = 2 5C p2q3
3

p = 2q
2
3p = 2 ; p = 3
Example 10.20 : If the sum of mean and variance of a Binomial Distribution is
4.8 for 5 trials find the distribution.
np + npq = 4.8 np(1 + q) = 4.8
Solution :
5 p [1 + (1 p) = 4.8
p = 2 (1 p)

p2 2p + 0.96 = 0 p = 1.2 , 0.8


p = 0.8 ; q = 0.2 [p cannot be greater than 1]
The Binomial distribution is P[X = x] = 5C (0.8)x (0.2)5x, x = 0 to 5
x

Example 10.21 : The difference between the mean and the variance of a
Binomial distribution is 1 and the difference between their squares is 11.Find n.
Solution : Let the mean be (m + 1) and the variance be m from the given
data.[Since mean > variance in a binomial distribution]
(m +1)2 m2 = 11 m = 5
mean = m + 1 = 6
5
1
np = 6 ; npq = 5 q = 6 , p = 6

214

n = 36.

EXERCISE 10.3
(1) The mean of a binomial distribution is 6 and its standard deviation is 3. Is
this statement true or false? Comment.
(2) A die is thrown 120 times and getting 1 or 5 is considered a success. Find
the mean and variance of the number of successes.
(3) If on an average 1 ship out of 10 do not arrive safely to ports. Find the
mean and the standard deviation of ships returning safely out of a total of
500 ships
(4) Four coins are tossed simultaneously. What is the probability of getting
(a) exactly 2 heads (b) at least two heads (c) at most two heads.
(5) The overall percentage of passes in a certain examination is 80. If 6
candidates appear in the examination what is the probability that atleast 5
pass the examination.
(6) In a hurdle race a player has to cross 10 hurdles. The probability that he
5
will clear each hurdle is 6. What is the probability that he will knock
down less than 2 hurdles.

10.4.2 Poisson Distribution :


It is named after the French Mathematician Simeon Denis Poisson
(1781 1840) who discovered it. Poisson distribution is also a discrete
distribution.
Poisson distribution is a limiting case of Binomial distribution under the
following conditions.
(i) n the number of trials is indefinitely large ie., n .
(ii) p the constant probability of success in each trial is very small
ie., p 0.
(iii) np = is finite where is a positive real number. When an event
occurs rarely, the distribution of such an event may be assumed to
follow a Poisson distribution.
Definition : A random variable X is said to have a Poisson distribution if the
e x
probability mass function of X is P(X = x) =
, x = 0,1,2, for some > 0
x
The mean of the Poisson Distribution is , and the variance is also .
The parameter of the Poisson distribution is .

215

Examples of Poisson Distribution :


(1) The number of alpha particles emitted by a radio active source in a
given time interval.
(2) The number of telephone calls received at a telephone exchange in a
given time interval.
(3) The number of defective articles in a packet of 100, produced by a good
industry.
(4) The number of printing errors at each page of a book by a good
publication.
(5) The number of road accidents reported in a city at a particular junction
at a particular time.
Example 10.22 : Prove that the total probability is one.

Solution :

e x e 0 e 1 e 2
=
+
+
+ ...
x
0
1
2
x=0

p(x) =
x=0

2
+ . . . ] = e . e
= e [1 + +
2

= e0 = 1

Example 10.23 : If a publisher of non-technical books takes a great pain to


ensure that his books are free of typological errors, so that the probability of any
given page containing atleast one such error is 0.005 and errors are independent
from page to page (i) what is the probability that one of its 400 page novels
will contain exactly one page with error. (ii) atmost three pages with errors.
[e2 = 0.1353 ; e0.2. = 0.819].
n = 400 , p = 0.005
Solution :
np = 2 =
(i)
P(one page with error) = P(X = 1)
=

e 1 e221
=
= 0.1363 2 = 0.2726
1
1

(ii)P(atmost 3 pages with error) = P(X 3)

e x 3 e2(2)x
2
22 23
=
= e2 1 +
+
+
x
2
3
1
x=0 x
0

19
= e2 3 = 0.8569

3

216

Example 10.24 : Suppose that the probability of suffering a side effect from a
certain vaccine is 0.005. If 1000 persons are inoculated, find approximately the
probability that (i) atmost 1 person suffer. (ii) 4, 5 or 6 persons suffer.
[e5 = 0.0067]
Solution : Let the probability of suffering from side effect
n =
(i)

be p

= np = 5.

1000 , p = 0.005 ,

P(atmost 1 person suffer) = p(X 1)


= p(X = 0) + p(X = 1)
=

e 0 e1
+
= e [1 + ]
0
1

= e5 (1 + 5) = 6 e5
= 6 0.0067 = 0.0402
(ii) P(4, 5 or 6 persons suffer) = p(X = 4) + p(X = 5) + p(X = 6)
=

e 4 e 5 e 6 e 4

2
1 + 5 + 30
+
+
=

4
5
6
4

e5 54
5
25 e5 54 17
10625
1
+
+
24
5
30 = 24 6 = 144 0.0067

= 0.4944
Example 10.25 : In a Poisson distribution if P(X = 2) = P(X = 3) find P(X =5)
[given e3 = 0.050].
Solution : Given P(X = 2) = P(X = 3)

e 2 e 3
=
2
3
32 = 3

2 (3 ) = 0
P(X = 5) =

As

0. = 3

e3 (3)5 0.050 243


e 5
=
=
= 0.101
120
5
5

217

Example 10.26 : If the number of incoming buses per minute at a bus terminus
is a random variable having a Poisson distribution with =0.9, find the
probability that there will be
(i) Exactly 9 incoming buses during a period of 5 minutes
(ii) Fewer than 10 incoming buses during a period of 8 minutes.
(iii) Atleast 14 incoming buses during a period of 11 minutes.
Solution :
for number of incoming
(i)
= 0.9
buses per minute
for number of incoming
buses per 5 minutes = 0.9 5 = 4.5
P exactly 9 incoming buses e 9

during 5 minutes =
9
i.e., P(X = 9) =
(ii)

e4.5 (4.5)9
9

fewer than 10 incoming buses

during a period of 8 minutes = P(X <10)


Here = 0.9 8 = 7.2
e7.2 (7.2)x
x
x=0
9

Required probability =
(iii)

P atleast 14 incoming buses

during a period of 11 minutes = P(X 14) = 1 P(X < 14)


Here = 11 0.9 = 9.9
e9.9 (9.9)x
x
x=0
13

Required probability = 1

(The answer can be left at this stage).


EXERCISE 10.4
(1) Let X have a Poisson distribution with mean 4. Find (i) P(X 3)
(ii) P(2 X < 5) [e4 = 0.0183].
(2) If the probability of a defective fuse from a manufacturing unit is 2% in a
box of 200 fuses find the probability that
(i) exactly 4 fuses are defective (ii) more than 3 fuses are defective
[e4 = 0.0183].

218

(3) 20% of the bolts produced in a factory are found to be defective. Find the
probability that in a sample of 10 bolts chosen at random exactly 2 will
be defective using (i) Binomial distribution (ii) Poisson distribution. [e2
= 0.1353].
(4) Alpha particles are emitted by a radio active source at an average rate of
5 in a 20 minutes interval. Using Poisson distribution find the probability
that there will be (i) 2 emission (ii) at least 2 emission in a particular 20
minutes interval. [e5 = 0.0067].
(5) The number of accidents in a year involving taxi drivers in a city follows
a Poisson distribution with mean equal to 3. Out of 1000 taxi drivers find
approximately the number of drivers with (i) no accident in a year
(ii) more than 3 accidents in a year [e3 = 0.0498].

10.4.3 Normal Distribution :


The Binomial and the Poisson distribution described above are the most
useful theoretical distribution for discrete variables i.e., they relate to the
occurrence of distinct events. In order to have mathematical distribution
suitable for dealing with quantities whose magnitude is continuously varying, a
continuous distribution is needed. The normal distribution is also called the
normal probability distribution, happens to be the most useful theoretical
distribution for continuous variables. Many statistical data concerning business
and economic problems are displayed in the form of normal distribution. In fact
normal distribution is the corner stone of Modern statistics.
Like the Poisson distribution, the normal distribution may also be regarded
as a limiting case of binomial distribution. Indeed when n is large and neither
p nor q is close to zero the Binomial distribution is approximated by the normal
distribution inspite of the fact that the former is a discrete distribution, where as
the later is a continuous distribution. Examples include measurement errors in
scientific experiments, anthropometric measurements of fossils, reaction times
in psychological experiment, measurements of intelligence and aptitude, scores
on various tests and numerous economic measures and indication.
Definition : A continuous random variable X is said to follow a normal
distribution with parameter and (or and 2) if the probability function is
1 x 2

2
1

e
f(x) =
2

; < x < , < < , and > 0.

219

X N(, ) denotes that the random variable X follows normal distribution


with mean and standard deviation .
Note : Even we can write the normal distribution as X N(, 2) symbolically.
In this case the parameters are mean and variance.
The normal distribution is also called Gaussian Distribution. The normal
distribution was first discovered by De-Moivre (1667 1754) in 1733 as a
limiting case of Binomial distribution. It was also known to Laplace not later
than 1744 but through a historical error it has been credited to Gauss who first
made reference to it in 1809.
Constants of Normal distribution :
Mean =
Variance = 2
Standard deviation =

x =
The graph of the normal curve is -
z =0
shown above.
Fig. 10.3
Properties of Normal Distribution :
(1) The normal curve is bell shaped
(2) It is symmetrical about the line X = ie., about the mean line.
(3) Mean = Median = Mode =
1
(4) The height of the normal curve is maximum at X = and
is the
2
maximum height (probability).
(5) It has only one mode at X = . The normal curve is unimodal
(6) The normal curve is asymptotic to the base line.
(7) The points of inflection are at X =
(8) Since the curve is symmetrical about X = , the skewness is zero.
(9) Area property :
P( < X < + )
= 0.6826
P( 2 < X < + 2) = 0.9544
P( 3 < X < + 3) = 0.9973
(10) A normal distribution is a close approximation to the binomial
distribution when n, the number of trials is very large and p the
probability of success is close to 1/2 i.e., neither p nor q is so small.
(11) It is also a limiting form of Poisson distribution i.e., as Poisson
distribution tends to normal distribution.

220

Standard Normal Distribution :


A random variable X is called a standard normal variate if its mean is zero
and its standard deviation is unity.
A normal distribution with mean and standard deviation can be
converted into a standard normal distribution by performing change of scale and
origin.
The formula that enables us to change from the x scale to the z scale and
X
vice versa is Z =

The probability density function of the standard normal variate Z is given by


1

2 z2
1
e
; < z <
(z) =
2
The distribution does not contain any parameter. The standard normal
distribution is denoted by N(0,1).
The total area under the normal probability curve is unity.

i.e.,
f(x)
dx
=
(z)dz
=
1

(z)dz =

Area Property of Normal Distribution :

(z)dz = 0.5
0

The Probability that a random variable X lies in the interval


( , + ) is given by
+

P( < X < + ) =
f(x) dx

substituting X = and X = + in Z =

P(1< Z< 1)=


(z)dz
1

= 2
(z)dz (by symmetry)

-1

Fig. 10.4

= 2 0.3413, (from the area table)


= 0.6826

221

Also P( 2 < X < + 2)


+2

=
f(x) dx
2
2

-2

P(2 < Z < 2) =


(z)dz

Fig. 10.5

= 2
(z)dz , (by symmetry)
0

= 2 0.4772 = 0.9544
Similarly P( 3 < X < + 3)
=

+3

(z)dz
f(x) dx =

-3

= 2 0.49865 = 0.9973

Fig. 10.6

Therefore the probability that a normal variate X lies outside the range
3 is given by
P( | X | > 3) = P( | Z | >3) = 1 p(3 < Z < 3) = 1 0.9973 = 0.0027
Note : Since the areas under the normal probability curve have been tabulated
interms of the standard normal variate Z, for any problem first convert X to Z.
The entries in the table gives the areas under the normal curve between the
mean (z = 0) and the given value of z as shown below :
Therefore entries corresponding to
negative values are unnecessary because
the normal curve is symmetrical. For

-
z
0
example
P(0 Z 1.2) = P(1.2 Z 0)
Fig. 10.7
Example 10.27 : Let Z be a standard normal variate. Calculate the following
probabilities.
(i)

P(0 Z 1.2)

(ii) P(1.2 Z 0)

(iii) Area to the right of Z = 1.3

(iv) Area to the left of Z = 1.5

(v) P(1.2 Z 2.5) (vi) P(1.2 Z 0.5)

222

(vii) P(1.5 Z 2.5)

Solution :
(i) P(0 Z 1.2)
P(0 Z 1.2) =
=

area between
Z = 0 and Z = 1.2
0.3849

z =0 z =1.2

Fig. 10.8
(ii) P(1.2 Z 0)
P(1.2 Z 0)

= P(0 Z 1.2)
by symmetry
= 0.3849
(iii) Area to the right of Z = 1.3
P(Z > 1.3) = area between Z = 0 to Z =
area between Z = 0 to Z = 1.3
= P(0 < Z < ) P(0 Z <1.3)
= 0.5 0.4032 = 0.0968
(iv) Area of the left of Z = 1.5
= P(Z < 1.5)
= P( < Z< 0) + P(0 Z < 1.5)
= 0.5 + 0.4332
= 0.9332
(v) P(1.2 Z < 2.5)
= P( 1.2 < Z < 0) + P(0 < Z < 2.5)
= P(0 Z < 1.2) + P(0 Z 2.5)
[by symmetry]
= 0.3849 + 0.4938
= 0.8787
(vi) P(1.2 Z 0.5)
= P(1.2 < Z < 0) P(0.5 < Z < 0)
= P(0 < Z < 1.2) P(0 < Z < 0.5)
[due to symmetry]
= 0.3849 0.1915 = 0.1934

223

z =-1.2 z =0

Fig. 10.9

z =0 z =1.3

Fig. 10.10

z =0 z =1.5

Fig. 10.11

-1.2 z =0

2.5

Fig. 10.12

z =0
z =-1.2 z =-.5

Fig. 10.13

(vii) P(1.5 Z 2.5)


Required area
= P(0 Z 2.5) P(0 Z 1.5) = 0.4938 0.4332 = 0.0606
Example 10.28 : Let Z be a standard normal variate. Find the value of c in the
following problems.
(i) P(Z < c) = 0.05
(iii) P(Z > c) = 0.05

(ii) P(c < Z < c) = 0.94


(iv) P(c < Z < 0) = 0.31

Solution :
(i) P(Z < c) = 0.05 i.e., P( < Z < c) = 0.05
As area is < 0.5, c lies to the left of Z = 0.
From the area table Z value for the area
0.45 is 1.65. c = 1.65

. 45
-

Fig. 10.14

(ii) P(c < Z < c) = 0.94


As Z = c and Z = +c lie at equal distance from Z = 0,
0.94
We have P(0 < Z < c) = 2 = 0.47.
Z value for the area 0.47 from the table
is 1.88 c = 1.88 and c = 1.88

. 94
. 47

. 47
-

z =-c

z =c

z =0

Fig. 10.15
(iii) P(Z > c) = 0.05 P(c < Z < ) = 0.05
From the data it is clear that c lies to the right of Z = 0
The area to the right of Z = 0 is 0.5
P(0 < Z < ) P(0 < Z < c) = 0.05
0.5 P(0 < Z < c) = 0.05

. 45

0.5 0.05 = P(0 < Z < c)

0.45 = P(0 < Z < c)


From the area table Z value for the area 0.45 is 1.65

z =0

0.05
z =c

Fig. 10.16
c = 1.65

(iv) P(c < Z < 0) = 0.31


As c is less than zero, it lies to the left of Z = 0. From the area table the Z
value for the area 0.31 is 0.88. As it in to the left of Z = 0, c = 0.88
Example 10.29 : If X is normally distributed with mean 6 and standard
deviation 5 find. (i) P(0 X 8) (ii) P( | X 6 | < 10)

224

Solution : Given = 6, = 5
(i) P(0 X 8)
X
We know that Z =

06
6
-

When X = 0, Z = 5 = 5 = 1.2
z =-1.2 z =0 z =.4
2
86
Fig. 10.17
When X = 8, Z = 5 = 5 = 0.4
P(0 X 8) = P(1.2 < Z < 0.4)
= P(0< Z <1.2) + P(0 < Z < .4) (due to symmetry)
= 0.3849 + 0.1554
= 0.5403
(ii) P( | X 6| < 10) = P(10 < (X 6) < 10) P(4 < X < 16)
4 6
10
When X = 4, Z = 5
= 5 = 2
-

10
16 6
z =-2
z =2
z =0
= 5 = 2
When X = 16, Z = 5
Fig. 10.18
P( 4 < X < 16) = P(2 < Z < 2)
= 2 P(0 < Z < 2) (due to symmetry)
= 2 (0.4772) = 0.9544
Example 10.30 : The mean score of 1000 students for an examination is 34 and
S.D is 16. (i) How many candidates can be expected to obtain marks between
30 and 60 assuming the normality of the distribution and (ii) determine the limit
of the marks of the central 70% of the candidates.
Solution : = 34, = 16,
N = 1000
X
(i) P(30 < X < 60) ; Z =

30
30 34
=
X = 30, Z1 =
16

4
x =30
x =34 x =60
= 16 = 0.25
z1
z2
Z1 = 0.25
Fig. 10.19
60 34
26
Z2 = 16
= 16 = 1.625
Z2 1.63 (app.)
P(0.25 < Z < 1.63) =P(0 < Z< 0.25) + P(0 < Z < 1.63) (due to symmetry)

225

= 0.0987 + 0.4484 = 0.5471


No of students scoring

between 30 and 60 = 0.5471 1000 = 547.


(ii) limit of central 70% of Candidates :
Value of Z1 from the area table
= 1.04
for the area 0.35
[ as Z1 lies to left of Z = 0]
-

Similarly Z2 = 1.04

. 35
z1

. 35
z =0

z2

Fig. 10.20
X 34
= 1.04
Z2 = 16
X2 34 = 1.04 16 + 34
X2 = 16.64 + 34

X 34
= 1.04
16
X1 = 16 1.04 + 34
Z1 =

= 16.64 + 34
X1 = 50.64

X2 = 17.36

70% of the candidate score between 17.36 and 50.64.


Example 10.31 : Obtain k, and 2 of the normal distribution whose
probability distribution function is given by
2
f(x) = k e2x + 4x

< X <
Solution : Consider
2x2 + 4x = 2 (x2 2x) = 2 [(x 1)2 1] = 2 (x 1)2 + 2
2
2
e2x + 4x = e2. e2(x 1)
1 (x 1)
2
1/4

= e2. e
Comparing it with f(x) we get
2x2 + 4x

ke
2

ke

1 x 1 2
2 1
e /2

1 x 1 2
2 1/2

= e2. e

1 x
2
1
=
e
2

1 x 2

2
1
=
e
2

1
1
2e2 2 1
we get = 2 = = 1 and k = 1
, =4
. e2 =
2
2 2

226

Example 10.32 : The air pressure in a randomly selected tyre put on a certain
model new car is normally distributed with mean value 31 psi and standard
deviation 0.2 psi.
(i) What is the probability that the pressure for a randomly selected tyre
(a) between 30.5 and 31.5 psi (b) between 30 and 32 psi
(ii) What is the probability that the pressure for a randomly selected tyre
exceeds 30.5 psi ?
Solution : Given = 31 and = 0.2
(i) (a) P(30.5 < X < 31.5) ; Z =

When X = 30.5, Z =

30.5 31 0.5
= 0.2 = 2.5
0.2

When X = 31.5, Z =

31.5 31 0.5
= 0.2 = 2.5
0.2

x =30.5

=31

x =31.5

Fig. 10.21

Required probability
P(30.5 < X < 31.5) = P( 2.5 < Z < 2.5)
= 2 P(0< Z < 2.5)
[since due to symmetry]
= 2(0.4938) = 0.9876
(b) P(30 < X < 32)
When X = 30,

Z=

30 31 1
0.2 = 0.2 = 5

When X = 32,

Z=

32 31 1
0.2 = 0.2 = 5

=31

Fig. 10.22

P(30 < X < 32) = P(5 < Z < 5) = area under the whole curve = 1 (app.)
(ii)

When X = 30.5 , Z =

30.5 31 0.5
= 0.2 = 2.5
0.2

P(X > 30.5) = P(Z > 2.5)


= 0.5 + p(0 < Z < 2.5)
= 0.5 + 0.4938 = 0.9938

z =-2.5

Fig. 10.23

227

EXERCISE 10.5
(1) If X is a normal variate with mean 80 and standard deviation 10,
compute the following probabilities by standardizing.
(i) P(X 100)

(ii) P(X 80)

(iii) P(65 X 100)

(iv) P(70 < X)

(v) P(85 X 95)


(2) If Z is a standard normal variate, find the value of c for the following
(i) P(0 < Z < c) = 0.25

(ii) P(c < Z < c) = 0.40

(iii) P(Z > c) = 0.85


(3) Suppose that the amount of cosmic radiation to which a person is
exposed when flying by jet across the United States is a random
variable having a normal distribution with a mean of 4.35 m rem and a
standard deviation of 0.59 m rem. What is the probability that a person
will be exposed to more than 5.20 m rem of cosmic radiation of such a
flight.
(4) The life of army shoes is normally distributed with mean 8 months and
standard deviation 2 months. If 5000 pairs are issued, how many pairs
would be expected to need replacement within 12 months.
(5) The mean weight of 500 male students in a certain college in 151
pounds and the standard deviation is 15 pounds. Assuming the weights
are normally distributed, find how many students weigh (i) between
120 and 155 pounds (ii) more than 185 pounds.
(6) If the height of 300 students are normally distributed with mean 64.5
inches and standard deviation 3.3 inches, find the height below which
99% of the student lie.
(7) Marks in an aptitude test given to 800 students of a school was found to
be normally distributed. 10% of the students scored below 40 marks and
10% of the students scored above 90 marks. Find the number of
students scored between 40 and 90.
(8) Find c, and 2 of the normal distribution whose probability function
2
is given by f(x) = c ex + 3x , < X < .

228

You might also like